Acute - Module 9

Ace your homework & exams now with Quizwiz!

A patient arrives in the postanesthesia care unit (PACU). Which action does the nurse perform first? a. Assess for a patent airway and adequate gas exchange. b. Assess the patient's pain level using the 0-10 pain assessment scale. c. Position the patient in a supine position to prevent aspiration. d. Calculate the patient-controlled analgesia (PCA) pump maximum dose per hour to avoid an overdose.

A

The nurse is assessing a patient newly admitted with obstipation and failure to pass flatus. Which condition is the most likely cause of this patient's symptoms? a. Complete obstruction b. Partial obstruction c. Colorectal cancer d. Crohn's disease

A

Which antiviral drugs are given to patients with chronic hepatitis B virus? Select all that apply. a. Lamivudine b. Entecavir c. Tenofovir d. Oral ribavirin e. Adefovir f. Telaprevir

A,B,C,E

What manifestations are expected when a patient has esophageal diverticula? Select all that apply. a. Halitosis b. Dysphagia c. Swelling with difficulty breathing d. Nocturnal cough e. Regurgitation f. Pain radiating to the right arm

A,B,D,E

Which significant factors contribute to increased intraabdominal pressure (IAP)? Select all that apply. a. Obesity b. Daily exercise c. Pregnancy d. Heavy lifting e. Constipation f. Hernia formation

A,C,D

Which facial assessment finding in a client with a salivary gland tumor prompts the nurse to notify the health care provider? A) Loss of sensation in tongue B) Alternates smiling and grimacing C) Wrinkles brows on command D) Holds eyes shut as the nurse pulls gently upon the eyebrows

ANS: A. Impaired tongue movement and loss of sensation in the tongue can indicate tumor invasion of the hypoglossal nerve. Other findings are normal.

A client had an open partial colectomy and ascending colostomy 3 days ago. What assessment findings does the nurse expect? (Select all that apply.) A) Black, moist stoma B) Gas inside the pouch C) Pain controlled with analgesics D) Small amount of formed stool from the colostomy E) Serosanguineous fluid draining from 2 Jackson-Pratt drains.

ANS: B, C, E. Gas is expected to collect in the pouch following surgery. The client's pain should be controlled by analgesia at this time. It is expected that 2 drains will be in place, which will be draining serosanguineous fluid. The stoma should be pink and moist, and will drain liquid stool since it is an ascending colostomy.

The nurse is assessing the client in end-stage liver failure who is diagnosed with portal hypertension. Which intervention should the nurse include in the plan of care? 1. Assess the abdomen for a tympanic wave. 2. Monitor the client's blood pressure. 3. Percuss the liver for size and location. 4. Weigh the client twice each week.

ANSWER: 1. 1. A client who has been diagnosed with portal hypertension should be assessed for a tympanic (fluid) wave to check for ascites. 2. High blood pressure is not the etiology of portal hypertension. 3. In portal hypertension, percussion is difficult and will not provide pertinent information about the client's condition. 4. Weighing the client should be done daily, not twice each week.

The client two (2) hours postoperative laparoscopic cholecystectomy is complaining of severe pain in the right shoulder. Which nursing intervention should the nurse implement? 1. Apply a heating pad to the abdomen for 15 to 20 minutes. 2. Administer morphine sulfate intravenously after diluting with saline. 3. Contact the surgeon for an order to x-ray the right shoulder. 4. Apply a sling to the right arm, which was injured during surgery.

ANSWER: 1. 1. A heating pad should be applied for 15 to 20 minutes to assist the migration of the CO2 used to insufflate the abdomen. Shoulder pain is an expected occurrence. 2. Morphine sulfate does not affect the etiology of the pain. 3. The surgeon would not order an x-ray for this condition. 4. There is no indication an injury occurred during surgery. A sling would not benefit the migration of the CO2. Shoulder pain is expected. TEST-TAKING HINT: The test taker must understand laparoscopic surgery to be able to answer this question. Option "4" could be eliminated because of the phrase "injured during surgery."

The nurse and the unlicensed assistive personnel (UAP) are working on the surgical unit. Which task can the nurse delegate to the UAP? 1. Take routine vital signs on clients. 2. Check the Jackson Pratt insertion site. 3. Hang the client's next IV bag. 4. Ensure the client obtains pain relief.

ANSWER: 1. 1. Taking the vital signs of the stable client may be delegated to the UAP. 2. Assessments cannot be delegated; "check" is a word which means "to assess." 3. IVs cannot be hung by the UAP; this is considered administering a medication. 4. Evaluating the client's pain relief is a responsibility of the RN. TEST-TAKING HINT: The test taker cannot delegate assessment, teaching, or evaluating care of the client.

The nurse is caring for a client diagnosed with rule-out peptic ulcer disease. Which test confirms this diagnosis? 1. Esophagogastroduodenoscopy. 2. Magnetic resonance imaging (MRI). 3. Occult blood test. 4. Gastric acid stimulation.

ANSWER: 1. 1. The esophagogastroduodenoscopy (EGD) is an invasive diagnostic test that visualizes the esophagus, stomach, and duodenum to accurately diagnose an ulcer and evaluate the effectiveness of the client's treatment. 2. Magnetic resonance imaging (MRI) shows cross-sectional images of tissue or blood flow. 3. An occult blood test shows the presence of blood but not the source. 4. A gastric acid stimulation test is used to understand the pathophysiology of ulcer disease, but it has limited usefulness. TEST-TAKING HINT: If the test taker has no idea what the correct answer is, knowledge of anatomy can help identify the answer. A peptic ulcer is an ulcer in the stomach, and in option "1" the word "esophagogastroduodenoscopy" has "gastro," which refers to the stomach. Therefore, this would be the best option to select as the correct answer

Which intervention has priority for the nurse in the surgical holding area? 1. Verify the surgical checklist. 2. Prepare the client's surgical site. 3. Assist the client to the bathroom. 4. Restrain the client on the surgery table.

ANSWER: 1. 1. The surgical checklist is assessed when the client arrives in the surgery department holding area where clients wait for a short time before entering the operating room. 2. Preparing the surgical site is completed in the surgery suite but not until the client and surgery have been verified. 3. The client should have voided just prior to being transported to the surgery department. 4. Securing the client onto the surgical table would be important in the operating room, not in the holding area.

The client is diagnosed with esophageal diverticula. Which lifestyle modification should be taught by the nurse? 1. Raise the foot of the bed to 45 degrees to increase peristalsis. 2. Eat the evening meal at least two (2) hours prior to bed. 3. Eat a low-fat, low-cholesterol, high-fiber diet. 4. Wear an abdominal binder to strengthen the abdominal muscles.

ANSWER: 2. 1. The client should elevate the head, not the foot, of the bed to prevent the reflux of stomach contents. 2. The evening meal should be eaten at least two (2) hours prior to retiring. Small, frequent meals and semisoft foods ease the passage of food, which decreases signs and symptoms of the disease process. 3. This diet is recommended for a client with coronary artery disease, not for esophageal diverticula. 4. Restrictive clothing should be avoided, and abdominal binders do not strengthen muscles and would not benefit this client

The circulating nurse is positioning clients for surgery. Which client has the greatest potential for nerve damage? 1. The 16-year-old client in the dorsal recumbent position having an appendectomy. 2. The 68-year-old client in the Trendelenburg position having a cholecystectomy. 3. The 45-year-old client in the reverse Trendelenburg position having a biopsy. 4. The 22-year-old client in the lateral position having a nephrectomy

ANSWER: 2. 1. The young client is not a high risk for nerve damage secondary to positioning. 2. The client's age, along with positioning with increased weight and pressure on the shoulders, puts this client at higher risk. 3. This client is sitting in an upright position, which would not put this client at risk for nerve damage. 4. A younger client would not be at a high risk for nerve damage when lying on the side. TEST-TAKING HINT: Clients who are at highest risk for nerve damage are clients who are elderly, obese, or emaciated and clients placed in positions which increase pressure on bony prominences.

Which problem would be appropriate for the nurse to identify for the preoperative client having an open reduction and internal fixation of the right ankle? 1. Alteration in skin integrity. 2. Knowledge deficit of postoperative care. 3. Alteration in gas exchange and pattern. 4. Alteration in urinary elimination.

ANSWER: 2. 1. This client problem would not be written until after the surgery. 2. This would be an appropriate client problem for the preoperative client who is scheduled for ankle repair. Teaching is priority. 3. This would not be a problem for a client scheduled for surgery. 4. This would not be a problem for a client scheduled for surgery

The circulating nurse observes the surgeon tossing a bloody gauze sponge onto the sterile field. Which action should the circulating nurse implement first? 1. Include the sponge in the sponge count. 2. Obtain a new sterile instrument pack. 3. Tell the surgical technologist about the sponge. 4. Throw the sponge in the sterile trashcan

ANSWER: 3. 1. All sponges must be included in the sponge count, but it is not the first intervention. 2. The circulating nurse should obtain another sterile pack for the operation to continue, but it is not the first intervention. 3. The circulating nurse should inform the surgical technologist of any break in sterile technique or field. This is the first intervention because the field is now contaminated. 4. This action is below standards of the Association of Operating Room Nurses and violates the principles of sterility. The sponge is included in the count and will not be discarded until the end of the case and all sponges have been accounted for

The nurse is caring for a client diagnosed with ulcerative colitis. Which symptom(s) support this diagnosis? 1. Increased appetite and thirst. 2. Elevated hemoglobin. 3. Multiple bloody, liquid stools. 4. Exacerbations unrelated to stress

ANSWER: 3. 1. Clients suffering from ulcerative colitis experience anorexia, not an increased appetite. 2. The hemoglobin and hematocrit are decreased, not elevated, as a result of blood loss. 3. Clients report as many as 10 to 20 liquid, bloody stools in a day. 4. Stressful events have been linked to an increase in symptoms. The nurse needs to assess for perceived stress in the client's life producing symptoms

The client presents to the outpatient clinic complaining of diarrhea for two (2) days. Which laboratory data should the nurse monitor? 1. The sodium level. 2. The albumin level. 3. The potassium level. 4. The glucose level.

ANSWER: 3. 1. Sodium is retained by the body before potassium. The sodium level would be in the normal range after two (2) days of diarrhea. 2. Albumin is the protein level synthesized by the liver; the albumin level should not be affected. 3. Potassium is excreted through diarrhea; the nurse should assess the client's potassium level. 4. The glucose should not be affected. TEST-TAKING HINT: The nurse must be able to interpret common laboratory results and how they affect the body. Part of the assessment of a symptom requires determining what therapies can impact the result.

Which technique would be most appropriate for the nurse to implement when assessing a four (4)-year-old client in acute pain? 1. Use words a four (4)-year-old child can remember. 2. Explain the 0-to-10 pain scale to the child's parent. 3. Have the child point to the face which describes the pain. 4. Administer the medication every four (4) hours.

ANSWER: 3. 1. The nurse should use words a four (4)-yearold child understands and remembers, but this is not the best way to assess pain. 2. A four (4)-year-old child cannot be expected to use the numeric pain scale because of lack of cognitive abilities, and explaining it to the parents does not address the child's pain. 3. The Faces Scale is the best way to assess pain in a four (4)-year-old child. 4. This does not assess the child's pain, and administering the pain medication every four (4) hours may compromise the child's safety. TEST-TAKING HINT: When age is listed, it is an indication the question is asking for agespecific information. The test taker should consider developmental levels for that particular age.

The nurse is completing the preoperative checklist on a client going to surgery. Which information should the nurse report to the surgeon? 1. The client understands the purpose of the surgery. 2. The client stopped taking aspirin three (3) weeks ago. 3. The client uses the oral supplements licorice and garlic. 4. The client has mild levels of preoperative anxiety

ANSWER: 3. 1. The surgeon should be notified if the client does not understand the surgical procedure, not if the client understands. 2. Aspirin should be stopped before surgery to help prevent bleeding. 3. Licorice and garlic can interfere with coagulation; therefore, the surgeon should be notified. 4. Mild levels of anxiety and apprehension before surgery are normal.

The nurse identifies the client problem "alteration in gastrointestinal system" for the elderly client. Which statement reflects the most appropriate rationale for this problem? 1. Elderly clients have the ability to chew food more thoroughly with dentures. 2. Elderly clients have an increase in digestive enzymes, which helps with digestion. 3. Elderly clients have an increased need for laxatives because of a decrease in bile. 4. Elderly clients have an increase in bacteria in the GI system, resulting in diarrhea.

ANSWER: 4. 1. Dentures are not an improvement over the client's own teeth in mastication. 2. The secretion of digestive enzymes and bile is decreased in the elderly, resulting in an alteration in nutrition and elimination. 3. Bile does not affect motility of the intestines. The elderly client's perception of the need for laxatives is caused by the client's misunderstanding about normal bowel function. 4. When the motility of the gastrointestinal tract decreases, bacteria remain in the gut longer and multiply, which results in diarrhea.

The client diagnosed with end-stage liver failure is admitted with hepatic encephalopathy. Which dietary restriction should be implemented by the nurse to address this complication? 1. Restrict sodium intake to 2 g/day. 2. Limit oral fluids to 1,500 mL/day. 3. Decrease the daily fat intake. 4. Reduce protein intake to 60 to 80 g/day

ANSWER: 4. 1. Sodium is restricted to reduce ascites and generalized edema, not for hepatic encephalopathy. 2. Fluids are calculated based on diuretic therapy, urine output, and serum electrolyte values; fluids do not affect hepatic encephalopathy. 3. A diet high in calories and moderate in fat intake is recommended to promote healing. 4. Ammonia is a by-product of protein metabolism and contributes to hepatic encephalopathy. Reducing protein intake should decrease ammonia levels. TEST-TAKING HINT: The test taker could eliminate options "1" and "2" based on the knowledge sodium and water work together and address edema, not encephalopathy. The test taker's knowledge of biochemistry— protein breaks down to ammonia, carbohydrates break down to glucose, and fat breaks down to ketones—may be helpful in selecting the correct answer.

The nurse is teaching the client diagnosed with diverticulosis. Which instruction should the nurse include in the teaching session? 1. Discuss the importance of drinking 1,000 mL of water daily. 2. Instruct the client to exercise at least three (3) times a week. 3. Teach the client about eating a low-residue diet. 4. Explain the need to have daily bowel movements.

ANSWER: 4. 1. The client should drink at least 3,000 mL of water daily to help prevent constipation. 2. The client should exercise daily to help prevent constipation. 3. The client should eat a high-fiber diet to help prevent constipation. 4. The client should have regular bowel movements, preferably daily. Constipation may cause diverticulitis, which is a potentially life-threatening complication of diverticulosis. TEST-TAKING HINT: The test taker must be careful to distinguish between -osis and -itis. Diverticulosis is the condition of having small pouches in the colon, and preventing constipation is the most important action the client can take to prevent diverticulitis (inflammation of the diverticulum).

Which statement explains the nurse's responsibility when obtaining informed consent for the client undergoing a surgical procedure? 1. The nurse should provide detailed information about the procedure. 2. The nurse should inform the client of any legal consultation needed. 3. The nurse should write a list of the risks for postoperative complications. 4. The nurse should ensure the client is voluntarily giving consent.

ANSWER: 4. 1. The nurse is not responsible for explaining details of surgery. This is the surgeon's responsibility. 2. The nurse is not responsible for informing the client of any need for legal representation. 3. The surgeon is responsible for informing the client of the risks and hazards of the surgery. 4. The nurse is responsible for ensuring the client voluntarily signs the surgical consent form giving permission for the surgery without coercion.

_____: From the American Society of Anesthesiologists (ASA), a system that assesses the fitness of patients for surgery.

ASA Physical Status Classification System

_____: A complication after abdominal trauma that occurs when the intraabdominal pressure is sustained at greater than 200 mm Hg.

Abdominal acute compartment syndrome (AACS)

_____: The absence of hydrochloric acid from gastric secretions.

Achlorhydria

_____: Inflammation of the gastric mucosa or submucosa after exposure to local irritants. Various degrees of mucosal necrosis and inflammatory reaction occur in acute disease. Complete regeneration and healing usually occur within a few days.

Acute gastritis

_____: A serious inflammation of the pancreas characterized by a sudden onset of abdominal pain, nausea, and vomiting. It is caused by premature activation of pancreatic enzymes that destroy ductal tissue and pancreatic cells and results in autodigestion and fibrosis of the pancreas.

Acute pancreatitis

_____: A complication of gastric surgery in which the pylorus is bypassed or removed. Endoscopic examination reveals regurgitated bile in the stomach and mucosal hyperemia. Symptoms include early satiety, abdominal discomfort, and vomiting. Also called bile reflux gastropathy.

Alkaline reflux gastropathy

_____: Pain relief or pain suppression.

Analgesia

When caring for a client with Laennec's cirrhosis, which of these does the nurse expect to find on assessment? (Select all that apply). A) Prolonged partial thromboplastin time B) Icterus of skin C) Swollen abdomen D) Elevated magnesium E) Currant jelly stool F) Elevated amylase level

Answer: A, B, C. Clients with Laennec's cirrhosis have damaged clotting factors, so prolonged coagulation times and bleeding may result. Icterus, or jaundice, results from cirrhosis. The client with cirrhosis may develop ascites, or fluid in the abdominal cavity.Elevated magnesium is not related to cirrhosis. Amylase is typically elevated in pancreatitis. Currant jelly stool is consistent with intussusception, a type of bowel obstruction. The client with cirrhosis may develop hypocalcemia and/or hypokalemia. It is also consistent with elevations of aspartate aminotransferase (AST), alanine aminotransferase (ALT), and alkaline phosphatase.

A client has undergone the Whipple procedure (radical pancreaticoduodenectomy) for pancreatic cancer. Which precautionary measures does the nurse implement to prevent potential complications? (Select all that apply). A) Check blood glucose often. B) Check bowel sounds and stools. C) Ensure that drainage color is clear. D) Monitor mental status. E) Place the client in the supine position.

Answer: A, B, D. To prevent potential complications after a Whipple procedure, the nurse would check the client's glucose often to monitor for diabetes mellitus. Bowels sounds and stools would be checked to monitor for bowel obstruction. A change in mental status or level of consciousness could be indicative of hemorrhage.Clear, colorless, bile-tinged drainage or frank blood with increased output may indicate disruption or leakage of a site of anastomosis but is not a precautionary action for the nurse to implement. The client should be placed in semi-Fowler's and not supine position to reduce tension on the suture line and the anastomosis site and to optimize lung expansion.

When caring for a client with portal hypertension, the nurse assesses for which potential complications? (Select all that apply). A) Esophageal varices. B) Hematuria. C) Fever. D) Ascites. E) Hemorrhoids.

Answer: A, D, E. Potential complications of portal hypertension include esophageal varices, ascites, and hemorrhoids. Portal hypertension results from increased resistance to or obstruction (blockage) of the flow of blood through the portal vein and its branches. The blood meets resistance to flow and seeks collateral (alternative) venous channels around the high-pressure area. Veins become dilated in the esophagus (esophageal varices), rectum (hemorrhoids), and abdomen (ascites due to excessive abdominal [peritoneal] fluid).Hematuria may indicate insufficient production of clotting factors in the liver and decreased absorption of vitamin K. Fever indicates an inflammatory process.

A patient who has colorectal cancer is scheduled for a colostomy. Which referral is initially of greatest value to this patient? A) Certified Wound, Ostomy, and Continence Nurse (CWOCN) B) Home health nursing agency C) Hospice. D) Hospital chaplain

Answer: A. A CWOCN (or an enterostomal therapist) will be of greatest value to the patient with colorectal cancer because the patient is scheduled to receive a colostomy.The patient is newly diagnosed, so it is not yet known whether home health nursing will be needed. A referral to hospice may be helpful for a terminally ill patient. Referral to a chaplain may be helpful later in the process of adjusting to the disease.

A nurse is caring for a client who has bradycardia following a surgical procedure using spinal anesthesia. The nurse should plan to administer which of the following medications to the client? A) Epinephrine. B) Propranolol. C) Amiodarone. D) Methyldopa.

Answer: A. A) The nurse should plan to administer epinephrine, a vasopressor, to increase the client's heart rate and prevent cardiac arrest. B)Propranolol is an antiarrhythmic medication and has the adverse effect of bradycardia. Therefore, this medication is contraindicated for this client. C) Amiodarone is an antiarrhythmic medication and has the adverse effect of bradycardia. Therefore, this medication is contraindicated for this client. D) Methyldopa is an antihypertensive medication and has the adverse effect of bradycardia. Therefore, this medication is contraindicated for this client.

A patient is experiencing bleeding related to peptic ulcer disease (PUD). Which nursing intervention is the highest priority? A) Starting a large-bore IV B) Administering IV pain medication C) Preparing equipment for intubation D) Monitoring the patient's anxiety level

Answer: A. The nursing intervention that has the highest priority for a patient with a bleeding peptic ulcer is to start a large-bore IV. A large-bore IV is inserted so that blood products can be administered.IV pain medication is not a recommended treatment for gastrointestinal bleeding. Intubation is also not recommended. The mental status of the patient would be monitored, but it is not necessary to monitor the anxiety level of the patient.

The nurse is assessing a patient with gastroesophageal reflux disease (GERD). Which findings does the nurse expect to observe? (Select all that apply). A) Bood-tinged sputum. B) Dyspepsia. C) Excessive salivation. D) Flatulence. E) Regurgitation.

Answer: B, C, D, E. When assessing a patient for GERD, the nurse expects to find dyspepsia (heartburn), excessive salivation, flatulence which is common after eating, and regurgitation (backward flow of food and fluid into the throat).Blood-tinged sputum and excessive salivation are not symptoms of GERD.

A patient with a family history of colorectal cancer (CRC) regularly sees a primary health care provider for early detection of any signs of cancer. Which laboratory result may be an indication of CRC in this patient? A) Decrease in liver function test results B) Elevated carcinoembryonic antigen C) Elevated hemoglobin levels D) Negative test for occult blood

Answer: B. Carcinoembryonic antigen may be elevated in many patients diagnosed with CRC.Liver involvement may or may not occur in CRC. Hemoglobin will likely be decreased with CRC, not increased. An occult blood test is not reliable to affirm or rule out CRC.

A 67-year-old male patient, with no surgical history, reports pain in the inguinal area that occurs when he coughs. A bulge that can be pushed back into the abdomen is found in his inguinal area. What type of hernia does he have? A) Femoral. B) Reducible. C) Strangulated. D) Incarcerated.

Answer: B. The hernia is reducible because its contents can be pushed back into the abdominal cavity.Femoral hernias tend to occur more frequently in obese and pregnant women. A hernia is considered to be strangulated when the blood supply to the herniated segment of the bowel is cut off. An incarcerated or irreducible hernia cannot be reduced or placed back into the abdominal cavity. Any hernia that is not reducible requires immediate surgical evaluation.

A client who is preparing to undergo a vaginal hysterectomy is concerned about being exposed. How does the nurse ensure that this client's privacy will be maintained? A) Remind the client that she will be asleep. B) Ensure that drapes will minimize perianal exposure. C) Explain postoperative expectations. D) Restrict the number of technicians in the procedure.

Answer: B. Using drapes is the best action to take to ensure the client's privacy.Telling the client that she will be asleep or explaining the procedure will not alleviate the client's anxiety. The number of people involved in the procedure is not something the nurse can necessarily control.

The nurse and the dietitian are planning sample diet menus for a patient who is experiencing dumping syndrome. Which sample meal is best for this patient? A) Chicken salad on whole wheat bread B) Liver and onions C) Chicken and rice. D) Cobb salad with buttermilk ranch dressing

Answer: C. Chicken and rice is the best sample meal for this patient. It is the only selection suitable for the patient who is experiencing dumping syndrome because it contains high protein without the addition of milk or wheat products.The patient with dumping syndrome would not be allowed to have mayonnaise, onions, or buttermilk ranch dressing. Buttermilk dressing is made from milk products. The patient can have whole wheat bread only in very limited amounts.

An obese patient is discharged 10 days after being hospitalized for peritonitis, which resulted in an exploratory laparotomy. Which assessment finding by the patient's home health nurse requires immediate action? A) Pain when coughing B) States, "I am too tired to walk very much" C) States, "I feel like the incision is splitting open" D) Temperature of 100.8°F (38.2°C).

Answer: C. The assessment finding of a patient who had an exploratory laparotomy that requires immediate action by the home health nurse is the patient stating, "I feel like the incision is splitting open." The patient feeling like the incision is splitting open is at risk for poor wound healing and possible wound dehiscence. The nurse must immediately assess the wound and notify the primary health care provider.Reports of pain when coughing, being too tired to ambulate, and a temperature of 100.8°F (38.2°C) all require further assessment or intervention but are not as great a concern as the possibility of wound dehiscence for this patient.

The nurse is preparing to instruct a client with chronic pancreatitis who is to begin taking pancrelipase (Cotazym). Which instruction does the nurse include when teaching the client about this medication? A) Administer pancrelipase before taking an antacid. B) Chew tablets before swallowing. C) Take pancrelipase before meals. D) Wipe your lips after taking pancrelipase.

Answer: D. The nurse will instruct the client to wipe the lips after taking pancrelipase. Pancrelipase is a pancreatic enzyme used for enzyme replacement for clients with chronic pancreatitis. To avoid skin irritation and breakdown from residual enzymes, the lips should be wiped.Pancrelipase should be administered after, and not before, antacids or histamine2 blockers are taken. It should not be chewed to minimize oral irritation and allow the drug to be released more slowly. It should be taken with meals and snacks, and not before, and followed with a glass of water.

Which patient assessment information is correlated with a diagnosis of chronic gastritis? A) Anorexia, nausea, and vomiting B) Frequent use of corticosteroids C) Hematemesis and anorexia D) Radiation therapy, smoking, and excessive alcohol use

Answer: D. Treatment with radiation therapy, smoking, and alcohol use are known to be associated with the development of chronic gastritis.Anorexia, nausea, and vomiting are all signs and symptoms of acute gastritis. Corticosteroid use and hematemesis are also more likely to be signs and symptoms of acute gastritis.

A client arrives at the emergency department with acute abdominal pain in the left lower quadrant. In which order does the nurse examine and assess the client's abdomen? (left lower quadrant (LLQ), left upper quadrant (LUQ), right lower quadrant (RLQ), and right upper quadrant (RUQ))?

Answer: RUQ, LUQ, RLQ, LLQ. The LLQ would be the last area assessed for this client. Abdominal examination usually begins at the client's right side and proceeds in a systematic fashion: RUQ, LUQ, RLQ, LLQ. However, if the client is experiencing pain in a specific quadrant, that area should be assessed last in the examination sequence. This action prevents the client from tensing abdominal muscles because of the pain, which would make the examination difficult.The options that do not assess the quadrant where the pain presents last are incorrect.

What is the major source of hepatitis B transmission to health care workers? a. Improper handwashing b. Needle sticks c. Touching contaminated surfaces d. Contact with infected stool

B

Which procedure would the health care provider recommend for immediate relief of dysphagia? a. Photodynamic therapy b. Esophageal dilation c. Targeted therapy d. Chemoradiation therapy

B

Which statement by a patient indicates an understanding of surgical management of hemorrhoids? a. "It will take 10-14 days for the rubber band used on the hemorrhoid to fall off." b. "My first bowel movement after the surgery may be very painful." c. "After surgery, I will need to eat a low-fiber, low-fluid diet." d. "Stool softeners and laxatives are avoided after hemorrhoid surgery."

B

Which surgical procedure is not appropriate for a patient with morbid obesity? a. Gastric restriction b. Liposuction c. Roux-en-Y gastric bypass d. Panniculectomy

B

Which are manifestations of pancreatic cancer? Select all that apply. a. Light-colored urine and dark-colored stools b. Anorexia and weight loss c. Splenomegaly d. Ascites e. Leg or calf pain f. Weakness and fatigue

B,C,D,E,F

Which are examples of mechanical bowel obstructions? Select all that apply. a. Paralytic ileus b. Adhesions c. Tumors d. Absent peristalsis e. Fecal impaction f. Hernias

B,C,E,F

Which are criteria used by the health care team to determine when a patient is ready to be discharged from the PACU? Select all that apply. a. Recovery rating score of 7 to 10 on rating scale b. Stable vital signs with normal body temperature c. Ability to swallow but remains NPO for at least 4 hours d. Intact cough and swallow reflexes e. Adequate urine output f. Return of gag reflex

B,D,E,F

The postanesthesia care unit (PACU) nurse is assessing a patient transferred in from the OR. Which assessment findings apply to assessment of the cardiovascular system? Select all that apply. a. Opens eyes on command. B. Absent dorsalis pedis pulse in left foot. C. Foley catheter in place with clear yellow drainage. D. Monitor shows normal sinus rhythm. E. States name correctly when asked. F. Apical pulse 85 beats/minute.

B,D,F

_____: Swishing sound in the larger arteries (carotid, aortic, femoral, and popliteal) that can be heard with a stethoscope or Doppler probe; may indicate narrowing of the artery and is usually associated with atherosclerotic disease.

Bruit

_____: An eating disorder that is characterized by episodes of binge eating in which the patient ingests a large amount of food in a short time, followed by purging behavior such as self-induced vomiting or excessive use of laxatives and diuretics.

Bulimia nervosa

The nurse is assessing a patient who has had a total gastrectomy today and notes bright-red blood in the nasogastric (NG) tube and abdom- inal distention. What does the nurse do next? a. Irrigate the NG tube. b. Reposition the NG tube. c. Inform the surgeon of these findings. d. Remove the NG tube.

C

What is the cause of late dumping syndrome? a. Rapid emptying of food into the small intestine b. Shift of fluids into the gut leading to abdominal distention c. Release of an excessive amount of insulin d. Rapid entry of high-protein foods into the jejunum

C

_____: Carbon monoxide on oxygen-binding sites of the hemoglobin molecule.

Carboxyhemoglobin

_____: Alcohol-induced chronic pancreatitis that is characterized by protein precipitates that plug the ducts and lead to ductal obstruction, atrophy, and dilation. The epithelium of the ducts undergoes histologic changes, resulting in metaplasia (cell replacement) and ulceration. This inflammatory process causes fibrosis of the pancreatic tissue.

Chronic calcifying pancreatitis (CCP)

_____: A patchy, diffuse inflammation of the mucosal lining of the stomach. Chronic gastritis usually heals without scarring but can progress to hemorrhage and ulcer formation.

Chronic gastritis

_____: Liver disease that is characterized by extensive scarring of the liver and that is usually caused by a chronic irreversible reaction to hepatic inflammation and necrosis; disease typically develops insidiously and has a prolonged, destructive course.

Cirrhosis

_____: Surgical removal of part or all of the colon.

Colectomy

_____: Acute ulcerative gastroduodenal disease that may develop as a result of increased intracranial pressure.

Cushing's ulcer

A female patient is having a biopsy of a nodule found in the right breast. Which classification identifies this surgery? a. Urgent b. Minor c. Cosmetic d. Diagnostic

D

Administration of which drug has greatly improved the success of organ transplants? A. Telaprevir b. Entecavir c. Tenofovir d. Cyclosporine

D

The nurse is supervising a student during assessment of the abdomen. When must the nurse intervene with the student? a. The nursing student inspects the patient's abdomen for symmetry. b. The nursing student auscultates for bowel sounds in an organized manner. c. The nursing student performs light palpation for areas of discomfort. d. The nursing student performs deep palpation for a pulsing midline mass.

D

Which gastrointestinal problem is related to anorexia? a. Heartburn b. Constipation c. Steatorrhea d. Loss of appetite

D

Which peptic ulcer disease drug is useful to protect patients against NSAID-induced ulcers? a. Magnesium hydroxide b. Omeprazole c. Esomeprazole d. Misoprostol

D

_____: The visual and radiographic examination of the liver, gallbladder, bile ducts, and pancreas by means of an endoscope and the injection of radiopaque dye to identify the cause and location of obstruction.

Endoscopic retrograde cholangiopancreatography (ERCP)

_____: The direct visualization of the gastrointestinal tract by means of a flexible fiberoptic endoscope.

Endoscopy

_____: Surgical anastomosis of the stomach to the jejunum.

Gastrojejunostomy

_____: Abnormal enlargement of the breasts in men.

Gynecomastia

_____: The vomiting of blood.

Hematemesis

_____: The passage of red blood via the rectum.

Hematochezia

_____: Unnaturally swollen or distended vein in the anorectal region.

Hemorrhoid

_____: The excision of a hemorrhoid.

Hemorrhoidectomy

_____: The widespread inflammation of liver cells.

Hepatitis

_____: Enlargement of the liver.

Hepatomegaly

_____: A state of progressive oliguric renal failure associated with hepatic failure, resulting in functional impairment of kidneys with normal anatomic and morphologic features. It indicates a poor prognosis for the patient with hepatic failure and is often the cause of death in patients with cirrhosis.

Hepatorenal syndrome (HRS)

_____: Pancreatitis that may be associated with SPINK1 and CFTR gene mutations.

Hereditary chronic pancreatitis

_____: A weakness in the abdominal muscle wall through which a segment of the bowel or other abdominal structure protrudes.

Hernia

_____: Surgical repair of a hernia in which the surgeon reinforces the weakened outside muscle wall with a mesh patch.

Hernioplasty

_____: Protrusion of the stomach through the esophageal hiatus of the diaphragm and into the thorax. Also called diaphragmatic hernia.

Hiatial hernia

_____: A substance normally secreted by the gastric mucosa and needed for intestinal absorption of vitamin B12. A deficiency of intrinsic factor and the resulting failure to absorb vitamin B12 lead to pernicious anemia.

Intrinsic factor

_____: A chronic gastrointestinal disorder characterized by chronic or recurrent diarrhea, constipation, and/or abdominal pain and bloating. Also called spastic colon, mucous colon, or nervous colon.

Irritable bowel syndrome (IBS)

_____: Intestinal obstruction that does not involve a physical obstruction in or outside the intestine. Instead, decreased or absent peristalsis results in a slowing of the movement or a backup of intestinal contents. This is also known as paralytic ileus or adynamic ileus because it is a result of neuromuscular disturbance.

Nonmechanical obstruction

_____: Acquired in an inpatient health care setting; for example, infections that were not present at hospital admission. Also called hospital-acquired infections and health care-associated infections.

Nosocomial (infection)

_____: A mnemonic (memory device) that may help in the current problem assessment of patients with gastrointestinal tract disorders. The letters represent these areas: P, precipitating or palliative (What brings it on? What makes it better or worse?); Q, quality or quantity (How does it look, feel, or sound?); R, region or radiation (Where is it? Does it spread anywhere?); S, severity scale (How bad is it [on a scale of 0 to 10]? Is it getting better, worse, or staying the same?); T, timing (Onset, duration, and frequency?).

PQRST

_____: A collection of purulent material that results from extensive inflammatory necrosis of the pancreas after infection by organisms such as Escherichia coli; the most serious complication of pancreatitis. It is fatal if left untreated.

Pancreatic abscesses

_____: A false cyst, so named because, unlike a true cyst, it does not have an epithelial lining. It is an encapsulated saclike structure that forms on or surrounds the pancreas and develops as a complication of acute or chronic pancreatitis. It may contain up to several liters of straw-colored or dark-brown viscous fluid, the enzymatic exudate of the pancreas.

Pancreatic pseudocyst

_____: The surgical removal of any panniculus, most often the abdominal apron; usually done as a follow-up to bariatric surgery in an obese patient.

Panniculectomy

_____: A procedure in which the physician inserts a trocar catheter into the abdomen to remove and drain ascitic fluid from the peritoneal cavity.

Paracentesis

_____: Life-threatening metabolic complication that can occur when nutrition is restarted for a patient who is in a starvation state.

Refeeding syndrome

_____: An excessive amount of fat in the stool

Steatorrhea

_____: The surgical creation of an opening; usually refers to an opening in the abdominal wall.

Stoma

_____: Inflammation of the oral mucosa; characterized by painful single or multiple ulcerations that impair the protective lining of the mouth. The ulcerations are commonly referred to as "canker sores."

Stomatitis

_____: A tightly constricted hernia that compromises the blood supply to the herniated segment of the bowel as a result of pressure from the hernial ring (the band of muscle around the hernia); leads to ischemia and obstruction of the bowel loop, with necrosis of the bowel and possibly bowel perforation.

Strangulated hernia

_____: Intestinal obstruction with compromised blood flow.

Strangulated obstruction

_____: Multiple shallow erosions of the proximal stomach and occasionally the duodenum.

Stress ulcer

_____: Narrowing.

Stricture

_____: Acute enlargement of the colon along with fever, leukocytosis, and tachycardia; usually associated with ulcerative colitis.

Toxic megacolon

_____: An iron-transport protein that can be measured directly or calculated as an indirect measurement of total iron-binding capacity.

Transferrin

_____: The radiographic visualization of the gastrointestinal tract from the oral part of the pharynx to the duodenojejunal junction; used to detect disorders of structure or function of the esophagus (barium swallow), stomach, or duodenum.

Upper GI radiographic series

_____: The ringlike band of muscle fibers at the upper end of the esophagus. When at rest, the sphincter is closed to prevent air from entering into the esophagus during respiration.

Upper esophageal sphincter (UES)

_____: A surgical treatment for cancer of the head of the pancreas. The procedure entails removal of the proximal head of the pancreas, the duodenum, a portion of the jejunum, the stomach (partial or total gastrectomy), and the gallbladder, with anastomosis of the pancreatic duct (pancreaticojejunostomy), the common bile duct (choledochojejunostomy), and the stomach (gastrojejunostomy) to the jejunum.

Whipple procedure (radical pancreaticoduodenectomy)

The nurse is taking a GI health history from a newly admitted patient. Which questions would the nurse include in the interview? Select all that apply. a. "Have you lost or gained weight recently?" b. "Have you had any recent cardiac or respiratory surgeries?" c. "Do you wear dentures, and if so, how do they fit you?" d. "Do you have difficulty chewing or swallowing?" e. "Have you traveled in the USA recently, and where?" f. "What is your usual bowel elimination pattern?"

A,C,D,F

The nurse is teaching a group of older adults how to prevent fecal impaction. Which are key teaching points? Select all that apply. a. Include plenty of raw fruits and vegetables and whole-grain products in your diet. b. Take your prescribed laxative every morning as directed. c. Walking every day is an excellent exercise for increasing bowel motility. d. Take bulk-forming products, such as Metamucil, to provide fiber. e. Use a bedpan instead of a commode or toilet for bowel movements. f. Drink lots of fluids, especially water.

A,C,D,F

The nurse is teaching a patient who will have a radical neck dissection. The nurse will teach the patient that what structures will be removed during this procedure? Select all that apply. A. Sternocleidomastoid muscle b. Removal of the jaw c. Excision of cervical lymph nodes on the affected side d. Cranial nerve XI e. Excision of the tongue f. Internal jugular vein

A,C,D,F

Which are the focus areas for the Surgical Care Improvement Project (SCIP)? Select all that apply. a. Prevention of infection b. Prevention of respiratory complications c. Prevention of serious cardiac events d. Prevention of venous thromboembolism e. Prevention of acute kidney injury f. Maintenance of normothermia

A,C,D,F

A patient with severe gastroesophageal reflux disease (GERD) tells the nurse that she has pain after each meal that lasts for 45 minutes and is worse when she lies down. What interventions should the nurse teach this patient? Select all that apply. a. Drink fluids. b. When you lie down, try lying on your side. c. Take an antacid as prescribed by your health care provider. d. Eat something bland such as a slice of white bread. e. Maintain an upright position for at least 1 hour after you eat. f. Try pressing over your abdomen to mobilize the food in your stomach.

A,C,E

Which statements about intrinsic factor are correct? Select all that apply. a. It is produced by the parietal cells. b. It is essential to fat emulsification. c. It aids in the absorption of vitamin B12. d. It forms and secretes bile. e. Its absence causes pernicious anemia. f. It causes the stomach to secrete hydrochloric acid.

A,C,E

In performing an assessment on a patient with liver trauma, what does the nurse expect to find? Select all that apply. a. Right upper quadrant pain b. Increased blood pressure c. Guarding of the abdomen d. Bradypnea e. Kehr's sign f. Abdominal rigidity

A,C,E,F

Which laboratory results are expected with mal- absorption syndrome resulting in hypochromic microcytic anemia? Select all that apply. a. Low mean corpuscular hemoglobin (MCH) b. High serum vitamin A level c. Elevated fecal fat content d. Increased mean corpuscular volume (MCV) e. Decreased serum cholesterol level f. Low mean corpuscular hemoglobin concentration (MCHC)

A,F

An appendectomy is being performed on a patient with appendicitis. What is the correct classification for this surgery? a. Curative b. Diagnostic c. Urgent d. Radical

A

Which substances predispose a patient to peptic ulcer disease and gastrointestinal (GI) bleeding? Select all that apply. a. Nonsteroidal anti-inflammatory drugs b. Anticoagulants c. Aspirin d. Lasix e. Digitalis f. Caffeine

A,B,C,F

Why does the nurse place a patient with a bowel obstruction in semi-Fowler's position? Select all that apply. a. To promote increased peristalsis b. To alleviate the pressure of abdominal distention on the chest c. To decrease the likelihood of nausea and vomiting d. To facilitate breathing e. To prevent aspiration f. To relieve pressure on the back

A,B,D

Which interventions are useful in preventing spread of gastroenteritis? Select all that apply. a. Careful handwashing b. Sanitizing all surfaces that may be contaminated c. Prophylactic use of antibiotics d. Easily accessible hand sanitizers e. Testing all food preparation employees f. Proper food and beverage preparation

A,B,D,F

An underweight client is receiving nutritional supplements to restore nutritional status. What does the nurse do to assess the effectiveness of the supplements for the client? A) Keeps an accurate and precise food and fluid intake record daily B) Makes certain the client is weighed daily at the same time C) Monitors vital signs every 4 hours and as needed D) Assesses the client's skin for evidence of breakdown weekly

Answer: B. To assess the effectiveness of supplements for this client, the nurse would perform daily weigh-ins. This will best show the effects of nutritional supplements since the primary client outcome is weight gain.Identifying everything that the client is taking in orally, monitoring vital signs, and assessing for any evidence of skin breakdown do not help determine the effects of nutritional supplements for the client.

A nurse is assessing a client who has Crohn's disease. Which of the following findings should the nurse expect? A) Weight gain. B) Sharp epigastric pain. C) Fatty diarrheal stools. D) Hyperkalemia.

Answer: C. A) Weight loss is an expected finding in a client who has Crohn's disease. B) Abdominal pain in the right lower quadrant is an expected finding in a client who has Crohn's disease. C) Steatorrhea, or fatty stool, is an expected finding in a client who has Crohn's disease. D) Hypokalemia is an expected finding in a client who has Crohn's disease.

The nurse is monitoring a patient with gastric cancer for signs and symptoms of upper gastrointestinal bleeding. Which change in vital signs is most indicative of bleeding? A) Respiratory rate from 24 to 20 breaths/min B) Apical pulse from 80 to 72 beats/min C) Temperature from 97.9° F to 98.9° F (36.6°C to 37.2°C) D) Blood pressure from 140/90 to 110/70 mm Hg

Answer: D. A decrease in blood pressure from 140/90 to 110/70 is the most indicative sign of bleeding.A slight decrease in respiratory rate, apical pulse, and temperature is not the primary indication of bleeding.

The nurse prepares a teaching session regarding lifestyle changes needed to decrease the discomfort associated with a patient's gastroesophageal reflux disease (GERD). Which change does the nurse recommend to this patient? A) Eat only two or three meals daily. B) Sleep flat in a left side-lying position. C) Drink tea instead of coffee. D) Avoid working while bent over the computer.

Answer: D. The patient should avoid working while bent over because this position presses on the diaphragm, causing discomfort.The patient with GERD needs to eat four to six meals a day. The head of the patient's bed would be elevated approximately 6 inches (15 cm). Both tea and coffee need to be eliminated from this patient's diet because of the caffeine content.

The nurse is attempting to position a client having an acute attack of pancreatitis in the most comfortable position possible. In which position does the nurse place this client? A) Supine, with a pillow supporting the abdomen B) Up in a chair between frequent periods of ambulation C) High-Fowler's position, with pillows used as needed D) Side-lying position, with knees drawn up to the chest

Answer: D. The side-lying position with the knees drawn up has been found to be the most comfortable possible position to relieve abdominal discomfort related to acute pancreatitis.No evidence suggests that supine position, sitting up in a chair, or high-Fowler's position has any effect on abdominal discomfort related to acute pancreatitis.

_____: Noninfectious stomatitis.

Aphthous stomatitis

_____: One of several regulators of lipoprotein metabolism.

Apolipoprotein E

_____: Surgical removal of the inflamed appendix.

Appendectomy

_____: Collapse of alveoli.

Atelectasis

_____: A type of gastritis that involves all layers of the stomach and includes diffuse inflammation and destruction of deeply located glands.

Atrophic gastritis

_____: Self-digestion. Specifically, the process of the stomach digesting itself if there is a break in its protective mucosal barrier.

Autodigestion

_____: A chronic inflammatory form of pancreatitis that can also affect the bile ducts, kidneys, and other major connective tissues.

Autoimmune pancreatitis

_____: Reinfusing the patient's own blood during surgery.

Autologous blood transfusion

A 75-year-old patient is having an exploratory laparotomy tomorrow. The wife tells the nurse that at night the patient gets up and walks around his room. What priority action does the nurse take after hearing this information? A. Notifies the provider. B. Develops a plan to keep the patient safe. C. Obtains an order for sleep medication. D. Tells the patient not to get out of bed at night.

B

In the postanesthesia care unit (PACU), the nurse assesses that a patient is bleeding profusely from an abdominal incision. What is the nurse's best first action? a. Notify the surgeon. b. Apply pressure to the wound dressing. c. Instruct the unlicensed assistive personnel (UAP) to get additional dressing supplies. d. Request and draw a complete blood count.

B

The nurse auscultates a patient's abdomen and hears high-pitched, loud, musical sounds in an air-filled abdomen. How does the nurse best describe this finding? a. Bruits b. Tympanic c. Dull d. Medium-pitched

B

The patient is to continue pancreatic enzyme replacement therapy (PERT) after discharge. Which statement indicates that the patient understands teaching about this therapy? a. "I will take the enzymes before meals with a full glass of water." b. "I will take the enzymes after I take my ranitidine." c. "I will mix the enzymes with chopped meat." d. "I will chew the capsules before swallowing the enzymes."

B

What is the most reliable indicator of fluid status? a. Intake and output b. Trends in weight c. Skin turgor d. Edema

B

What is the most reliable method to confirm initial placement of nasoduodenal or nasogastric (NG) tube placement? a. Auscultation b. X-ray c. Capnometry d. Testing pH of gastric contents

B

The nurse hears the unlicensed assistive personnel (UAP) instructing a new UAP about obtaining patients' weights. Which statements by the UAP indicate a need for clarification? Select all that apply. A. "It is best to weigh the patients before breakfast." B. "Just ask the patients how much they weigh." C. "You can weigh the patients whenever you have time during your shift." D. "Weigh the patients while they are in minimal clothing and no shoes." E. "Ambulatory patients can be weighed on the upright scales." F. "Bedridden patients are weighed using a bed scale."

B,C

Which simple noninvasive tests can be used to detect H. pylori in a patient with peptic ulcer disease (PUD)? Select all that apply. a. Echocardiogram b. Serologic testing for antibodies c. Urea breath test d. Stool antigen test e. CT scan f. MRI

B,C,D

Which types of ulcers are included in peptic ulcer disease? Select all that apply. a. Esophageal ulcers b. Gastric ulcers c. Pressure ulcers d. Duodenal ulcers e. Stress ulcers f. Colon ulcers

B,D,E

How is xerostomia characterized? a. Reduction of taste sensation b. Inflammation of a salivary gland c. Excessive mouth dryness d. Inflammation of the mouth

C

The definitive diagnosis for esophageal cancer is made with which procedure? a. Barium swallow b. Esophageal manometry c. Esophageal ultrasound with fine needle aspiration d. Esophagogastroduodenoscopy (EGD)

C

The nurse understands that a patient loses weight under which circumstance? a. The patient takes in more calories than are used. b. The patient takes in as many calories as are needed. c. The patient takes in fewer calories than are used. d. The patient takes in enough calories for daily activity

C

Which observation of a patient with an intestinal obstruction does the nurse report immediately? a. Urinary output of 1000 mL in an 8-hour period b. The patient's request for something to drink c. Abdominal pain changing from colicky to constant discomfort d. The patient is changing positions frequently

C

_____: An infection caused by the fungus Candida albicans.

Candidiasis

_____: Inflammation of the gallbladder.

Cholecystitis

_____: A hormone that stimulates digestive juices and that may work with leptin to increase or decrease appetite.

Cholecystokinin

_____: The endoscopic examination of the entire large bowel.

Colonoscopy

_____: The surgical creation of an opening between the colon and the surface of the abdomen.

Colostomy

_____: A form of cirrhosis in which the liver has significant scarring but is still able to perform essential functions without causing significant symptoms

Compensated cirrhosis

_____: (1) A way of decreasing muscle pain by "cooling down" the area with a local, short-acting gel or cream, such as after physical therapy; (2) in ophthalmologic surgery, use of a freezing probe to repair retinal detachment.

Cryotherapy

When examining the abdomen, which technique for abdominal assessment is used second? a. Inspection b. Percussion c. Palpation d. Auscultation

D

Which member of the operating room team is responsible for setting up the sterile field? a. Nurse anesthetist (CRNA) b. Surgical assistant c. Circulating nurse d. Scrub nurse

D

Which structure is involved in the protective function of the liver? a. Sphincter of Oddi b. Gallbladder c. Pancreas d. Kupffer cells

D

The nurse is caring for an elderly client diagnosed with acute gastritis. Which client problem is priority for this client? 1. Fluid volume deficit. 2. Altered nutrition: less than body requirements. 3. Impaired tissue perfusion. 4. Alteration in comfort

ANSWER: 1. 1. Pediatric and geriatric clients have an increased risk for fluid volume and electrolyte imbalances. The nurse should always be alert to this possible complication. 2. Altered nutrition may be appropriate, depending on how long the client has been unable to eat, but it is not priority over fluid volume deficit. 3. Impaired tissue perfusion may be appropriate if the mucosal lining of the stomach is unable to heal, but it is not priority over fluid volume deficit. 4. Alteration in comfort may be appropriate, but it is not priority over fluid volume deficit.

Which intervention should the nurse include when discussing ways to prevent food poisoning? 1. Wash hands for 10 seconds after handling raw meat. 2. Clean all cutting boards between meats and fruits. 3. Maintain food temperatures at 1408F during extended servings. 4. Explain fruits do not require washing prior to eating or preparing.

ANSWER: 3. 1. Hand washing for 10 seconds is not long enough to remove any bacteria. Hands should be washed for at least 30 seconds before handling food or eating. 2. Cutting surfaces used for meats should be different from those used for fruits and vegetables to prevent contamination. 3. Foods being served for an extended time should be kept at 140°F because food sitting at less than this temperature allows for bacterial growth. 4. All fruits and vegetables should be washed before eating or preparing.

The client is complaining of painful swallowing secondary to mouth ulcers. Which statement indicates the nurse's teaching is effective? 1. "I will brush my teeth with a soft-bristle toothbrush." 2. "I will rinse my mouth with Listerine mouthwash." 3. "I will swish with antifungal solution and then swallow." 4. "I will avoid spicy foods, tobacco, and alcohol."

ANSWER: 4. 1. A soft-bristle toothbrush will not affect painful swallowing. 2. An alcohol-based mouthwash (Listerine) is irritating to the oral cavity and can increase pain. 3. An antifungal medication should be used with candidiasis and is not effective treatment for plain mouth ulcers. 4. Irritating substances should be avoided during the outbreaks of ulcers in the mouth. Spicy foods, alcohol, and tobacco are common irritants the client should avoid.

Which problems should the nurse include in the plan of care for the client diagnosed with peptic ulcer disease to observe for physiological complications? 1. Alteration in bowel elimination patterns. 2. Knowledge deficit in the causes of ulcers. 3. Inability to cope with changing family roles. 4. Potential for alteration in gastric emptying

ANSWER: 4. 1. There is no indication from the question there is a problem or potential problem with bowel elimination. 2. Knowledge deficit does not address physiological complications. 3. This client may have problems from changing roles within the family, but the question asks for potential physiological complications, not psychosocial problems. 4. Potential for alteration in gastric emptying is caused by edema or scarring associated with an ulcer, which may cause a feeling of "fullness," vomiting of undigested food, or abdominal distention. TEST-TAKING HINT: This question asks the test taker to identify a physiological problem identifying a complication of the disease process. Therefore, options "2" and "3" could be eliminated because they do not address physiological problems.

The client weighs 160 pounds and is 5'1'' tall. Calculate the body mass index (BMI) using the following formula: (BMI 703 × weight in pounds) / (height in inches)2

ANSWER: BMI of 30.2.

_____: Fatty

Adipose

An unidentified client from the emergency department requires immediate surgery, but he is not conscious and no one is with him. What must the nurse, who is verifying the informed consent, do? A) Ensure written consultation of two noninvolved physicians. B) Read the surgeon's consult to determine whether the client's condition is life-threatening. C) Sign the operative permit. D) Withhold surgery until the next of kin is notified

Answer: A. In a life-threatening situation in which every effort has been made to contact the person with medical power of attorney, consent is desired but not essential. In place of written or oral consent, written consultation by at least two physicians who are not associated with the case may be requested by the health care provider.It is not within the nurse's role to make a judgment about the client's life-threatening status based on the surgeon's consult. Signing documents on the client's behalf is not legal. Withholding surgery is not in this client's best interests.

_____: The surgical removal of all or part of the esophagus.

Esophagectomy

_____: A hernia that protrudes through the femoral ring.

Femoral hernia

_____: The telescoping of a segment of the intestine within itself.

Intussusception

_____: The inside cavity of a tube or tubular organ, such as a blood vessel or airway.

Lumen

_____: A weight that has a severely negative effect on health; usually more than 100% above ideal body weight or a body mass index greater than 40.

Morbid obesity

_____: An illness or an abnormal condition or quality.

Morbidity

_____: Death.

Mortality

_____: Muscle aches/muscle pain.

Myalgia

_____: The release of muscle myoglobulin into the urine.

Myoglobinuria

_____: The need to urinate excessively at night.

Nocturia (nocturnal polyuria)

_____: No eating, drinking (including water), or smoking.

Nothing by mouth (NPO)

_____: Blockage.

Obstruction

_____: Absence of peristalsis.

Paralytic ileus

_____: Cells lining the wall of the stomach that secrete hydrochloric acid and produce intrinsic factor.

Parietal cells

_____: The surgical removal of the parotid glands.

Parotidectomy

_____: A mucosal lesion of the stomach or duodenum.

Peptic ulcer

_____: Acute inflammation of the visceral/parietal peritoneum and endothelial lining of the abdominal cavity, or peritoneum.

Peritonitis

_____: After surgery.

Postoperative period

_____: The difference between the apical and peripheral pulses.

Pulse deficit

_____: An incision through the serosa and muscularis of the pylorus, down to the mucosa; created to prevent gastric motility disturbances in patients who have undergone esophagectomy.

Pyloromyotomy

_____: A hernia that can be placed back into the abdominal cavity by gentle pressure.

Reducible hernia

_____: A type of local anesthesia that blocks multiple peripheral nerves in a specific body region.

Regional anesthesia

_____: Vitamin D deficiency in children.

Rickets

_____: Folds, as of a mucous membrane.

Rugae

_____: Tube inserted through the nose and placed into the stomach that is attached to low continuous suction. It has a vent ("pigtail") that prevents the stomach mucosa from being pulled away during suctioning.

Salem sump tube

_____: Liver inflammation resulting from exposure to hepatotoxins (e.g., industrial toxins, alcohol, and medications).

Toxic and drug-induced hepatitis

_____: Abnormal dryness of the mouth caused by a severe reduction in the flow of saliva.

Xerostomia

_____: The most accurate testing method of diagnosing GERD, accomplished by placing a small catheter into the distal esophagus or esophageal wall (depending on the specific technique). The patient then records a diary of activities and symptoms over a 24- to 48-hour period while pH is continuously monitored.

pH monitoring examination

A blind patient is to have a surgical procedure. She asks the nurse whether she will be able to sign her own consent form. What is the nurse's best response? A. "Yes, but your signature will need to have two witnesses." B. "No, but your next of kin can sign the consent form for you." C. "Yes, but you will need to make an X instead of signing your name." d. "No, but you can give instructions to sign for you to any responsible adult."

A

A colostomy is scheduled to be done on a patient who has severe Crohn's disease. What is the correct classification for this surgery? a. Palliative b. Minor c. Restorative d. Curative

A

A patient with chronic gastritis is being admitted. Which sign/symptom does the nurse identify as being associated with this patient's condition? a. Pernicious anemia b. Gastric hemorrhage c. Hematemesis d. Dyspepsia

A

During preoperative screening, the nurse dis- covers that the patient is allergic to shellfish. What is the nurse's best first action? a. Notifies the surgeon b. Develops a plan to keep the patient safe c. Obtains an order for a shellfish-free diet d. Asks the patient if any other family members have the same allergy

A

The nurse is caring for a patient receiving a continuous feeding through an NG tube. Which position is best to prevent aspiration? a. Semi-Fowler's b. Trendelenburg c. Supine d. Sims'

A

The nurse is teaching a patient with dumping syndrome about diet. Which statement by the patient indicates that teaching has been effective? A. "I will use sugar-free gelatin with caution." B. "I will avoid rice in my diet." C. "Meat in my diet will consist of a total of 8 ounces a day." D. "I will limit fluids with my meals to 8 ounces."

A

Which actions will help prevent viral hepatitis in health care workers? Select all that apply. a. Wash hands before and after each patient. b. Use needleless systems. c. Use contact and respiratory precautions. d. After exposure to hepatitis A, get immunoglobulin (Ig). e. Report all cases of hepatitis to the health department. f. Wear gloves during all patient contacts.

A,B,D,E

While performing an abdominal assessment on a patient with abdominal pain, the nurse performs inspection of the abdomen. Which inspection findings should the nurse be sure to document? Select all that apply. a. Overall shape of the abdomen b. Presence of discoloration or scarring c. Size of percussed abdominal organs d. Symmetry or asymmetry of the abdominal contour e. Family history of abdominal illnesses f. Distention of the abdomen

A,B,D,F

Which are potential complications of polyps? Select all that apply. a. Gross rectal bleeding b. Colorectal cancer c. Intestinal obstruction d. Septic shock e. Intussusception f. Bowel strangulation

A,C,E

The nurse is providing teaching about ways to reduce the risk for colorectal cancer. Which dietary suggestions will the nurse be sure to include in the teaching? Select all that apply. a. Low fat b. Low protein c. High fiber d. High in red meat e. Low in refined carbohydrates f. High in brassica vegetables

A,C,E,F

The nurse is teaching a patient with dumping syndrome. Which foods should the patient be instructed are permitted and encouraged? Select all that apply. a. White bread, rolls, and crackers b. Sweetened juice or fruit c. Cooked vegetables d. Carbonated drinks e. Fish, poultry, beef, or pork f. Butter and salad dressings

A,C,E,F

An older adult is admitted to the hospital. The client's height is 5 feet 10 inches (1.78 meters) and weight is 286 lbs (129.7 kg). The nurse calculates the client's current body mass index (BMI) as ______. (Round your answer to the nearest whole number.)

ANS: 41. BMI is calculated by dividing weight in kg by height squared in meters. The client's height (1.78 meters) squared is 3.17 (1.78 × 1.78 = 3.17). The client's weight is 129.7 kg. 129.7 kg divided by 3.17 = 40.9, which is 41 rounded to the nearest whole number.

The emergency department nurse is assessing a client with a known inguinal hernia. Which assessment finding indicates that the hernia may have strangulated? (Select all that apply.) A) Fever B) Tachycardia C) Abdominal distention D) Mild abdominal pain E) Nausea and vomiting

ANS: A, B, C, E. Strangulation of a hernia is accompanied by fever, tachycardia, abdominal distention, severe pain, nausea, and vomiting.

The nurse is teaching a client about nutrition and diverticulosis. Which food will the nurse teach the client to avoid? A) Popcorn B) Oatmeal C) Bran D) Lettuce

ANS: A. Foods such as nuts, corn, popcorn, cucumbers, tomatoes, figs, and strawberries—all of which contain seeds or indigestible material—may block a diverticulum and should be avoided. Oatmeal, bran, and lettuce are acceptable for patients with diverticulosis to eat.

The PACU nurse caring for a client with a nasogastric (NG) tube notes 300 mL of bright red blood has collected. What is the appropriate nursing action? A) Document as a normal finding. B) Immediately remove the NG tube. C) Place the client in Trendelenburg position. D) Call the client's surgeon to report the drainage.

ANS: D. The presence of bright red blood reflects active bleeding. The nurse must report this to the surgeon immediately.

The nurse received a report the elderly postoperative client became confused during the previous shift. Which client problem would the nurse include in the plan of care? 1. Risk for injury. 2. Altered comfort level. 3. Impaired circulation. 4. Impaired skin integrity

ANSWER: 1. 1. Anytime the nurse has a client who is disoriented, the nurse must initiate fall/ safety precautions. 2. Confusion would not indicate a change in comfort level. 3. Sudden confusion is usually not a circulation problem. 4. Impaired skin integrity would not cause confusion.

The client with ulcerative colitis is scheduled for an ileostomy. The nurse is aware the client's stoma will be located in which area of the abdomen? 1. RLQ. 2. LLQ. 3. LUQ. 4. RUQ.

ANSWER: 1. 1. The cure for ulcerative colitis is a total colectomy, which is removing the entire large colon and bringing the terminal end of the ileum up to the abdomen in the right lower quadrant. This is an ileostomy. 2. This site is the left lower quadrant. 3. This site is the transverse colon. 4. This site is the right upper quadrant. TEST-TAKING HINT: The test taker must identify the area by using the computer mouse. These are called "hot spots" on the NCLEX-RN

Which client would the nurse identify as having the highest risk for developing postoperative complications? 1. The 67-year-old client who is obese, has diabetes, and takes insulin. 2. The 50-year-old client with arthritis taking nonsteroidal anti-inflammatory drugs. 3. The 45-year-old client having abdominal surgery to remove the gallbladder. 4. The 60-year-old client with anemia who smokes one (1) pack of cigarettes per day.

ANSWER: 1. 1. This client has comorbid conditions— advanced age, obesity, and diabetes—which put this client at a higher risk for postoperative complications. 2. This client's risk factor of arthritis can make positioning in surgery and movement in the postoperative period more difficult, but this does not put the client at greater risk for postoperative complications. The NSAIDs can be held a few days prior to surgery to decrease the problems associated with NSAIDs. 3. The client with abdominal surgery may have respiratory complications, but the client is not at as high risk as the older client with diabetes and obesity. 4. The client's smoking increases the risk of pulmonary complications and increases the blood level of carboxyhemoglobin, but this client does not have the problems of delayed healing and age the 67-year-old client with diabetes has.

The nurse is assessing the client diagnosed with chronic gastritis. Which symptom(s) support this diagnosis? 1. Rapid onset of midsternal discomfort. 2. Epigastric pain relieved by eating food. 3. Dyspepsia and hematemesis. 4. Nausea and projectile vomiting

ANSWER: 2. 1. Acute gastritis is characterized by sudden epigastric pain or discomfort, not midsternal chest pain. 2. Chronic pain in the epigastric area relieved by ingesting food is a sign of chronic gastritis. 3. Dyspepsia (heartburn) and hematemesis (vomiting blood) are frequent symptoms of acute gastritis. 4. Projectile vomiting is not a sign of chronic gastritis.

The PACU nurse administers Narcan, an opioid antagonist, to a postoperative client. Which client problem should the nurse include to the plan of care based on this medication? 1. Alteration in comfort. 2. Risk for depressed respiratory pattern. 3. Potential for infection. 4. Fluid and electrolyte imbalance.

ANSWER: 2. 1. Narcan does not cause pain for the client. 2. A client with respiratory depression treated with Narcan can have another episode within 15 minutes after receiving the drug as a result of the short half-life of the medication. 3. Infection would not be a concern immediately after surgery. 4. Although the client may experience an imbalance in fluid or electrolytes, this problem would not be of concern as a result of the administration of Narcan. TEST-TAKING HINT: The test taker should be knowledgeable about medications commonly used in the postoperative period. If the test taker had no idea of the answer, selecting an option addressing the airway is an appropriate action

The male client diagnosed with a peptic ulcer who is being admitted to the surgical unit admits to using multiple over-the-counter medications to control his symptoms, including omeprazole (Prilosec, a proton pump inhibitor) daily; ranitidine (Zantac, an H2 receptor blocker) once or twice a day; and several different brands of antacids at least every 2 hours. Which ABG results should the nurse expect to find on the chart? 1. pH 7.34, O2 sat 96%, PaCO2 36, HCO3 21. 2. pH 7.4, O2 sat 98%, PaCO2 40, HCO3 24. 3. pH 7.49, O2 sat 95%, PaCO2 44, HCO3 28. 4. pH 7.25, O2 sat 100%, PaCO2 32, HCO3 23

ANSWER: 3. 1. This is metabolic acidosis; the client taking many antacids would be in metabolic alkalosis. 2. These are normal blood gas results. 3. This is metabolic alkalosis, a result of taking many antacids. 4. This is respiratory alkalosis, a result of hyperventilation.

_____: Loss of memory.

Amnesia

_____: A painful ulcer at the margin of the anus.

Anal fissure

During surgery, who is most responsible for monitoring for possible breaks in sterile technique? A) Circulating nurse. B) Holding nurse. C) Anesthesiologist. D) Surgeon.

Answer: A. All operating room team members are responsible, but the circulating nurse moves around the room and can see more of what is happening.The holding nurse is not in the operating room. The anesthesiologist is focused on providing sedation to the client. The surgeon is concentrating on the surgery and usually cannot monitor all staff.

Aside from chemotherapeutic agents, what other medications does the nurse expect to administer to a patient with advanced colorectal cancer for relief of symptoms? A) Analgesics and antiemetics B) Analgesics and benzodiazepines C) Steroids and analgesics D) Steroids and anti-inflammatory medications

Answer: A. Besides chemotherapeutic agents, the nurse expects to administer analgesics and antiemetics to a patient with advanced colorectal cancer for relief of symptoms related to pain and nausea.Benzodiazepines, steroids, and anti-inflammatory medications are not routinely requested for these patients.

What does the nurse advice a patient diagnosed with irritable bowel syndrome (IBS) to take during periods of constipation? A) Bulk-forming laxatives. B) Saline laxatives. C) Stimulant laxatives. D) Stool-softening agents.

Answer: A. The nurse advises the patient diagnosed with IBS to take bulk-forming laxatives during periods of constipation. For treatment of constipation-predominant IBS, bulk-forming laxatives are generally taken at mealtimes with a glass of water.Saline and stimulant laxatives are not used for the treatment of constipation-predominant IBS. Stool-softening agents are not effective.

The nurse is instructing a patient with recently diagnosed diverticular disease about diet. What food does the nurse suggest the patient include? A) A slice of 5-grain bread B) Chuck steak patty (6 ounces [170 grams]) C) Strawberries (1 cup [160 grams]) D) Tomato (1 medium)

Answer: A. The nurse suggests to the patient with recently diagnosed diverticular disease to include a slice of 5-grain bread in the diet. Whole-grain breads are recommended to be included in the diet of patients with diverticular disease because cellulose and hemicellulose types of fiber are found in them. Dietary fat would be reduced in patients with diverticular disease.If the patient wants to eat beef, it must be of a leaner cut. Foods containing seeds, such as strawberries, must be avoided. Tomatoes would also be avoided unless the seeds are removed. The seeds may block diverticula in the patient and present problems leading to diverticulitis.

_____: A type of contrast radiography (x-rays) in which the patient's colon and rectum are visualized after a liquid containing barium, and then air, is placed into the colon.

Double-contrast barium enema

_____: Difficulty in swallowing.

Dysphagia

_____: Painful urination.

Dysuria

_____: Large purple, blue, or yellow bruises of the skin resulting from small hemorrhages; these bruises are larger than petechiae.

Ecchymoses

_____: Without teeth.

Edentulous

_____: Drainage

Effluent

_____: The total separation of all layers of a wound and the protrusion of internal organs through the open wound.

Evisceration

_____: A surgical opening of the abdominal cavity to investigate the cause of an obstruction or peritonitis.

Exploratory laparotomy

_____: A hemorrhoid that lies below the anal sphincter and can be seen on inspection of the anal region.

External hemorrhoid

_____: A diagnostic test that measures the presence of blood in the stool from gastrointestinal bleeding; this is a common finding associated with colorectal cancer.

Fecal occult blood test (FOBT)

_____: Liver cell.

Hepatocyte

_____: A vegetarian diet pattern in which milk, cheese, eggs, and dairy foods are eaten but meat, fish, and poultry are avoided.

Lacto-ovo-vegetarian

_____: An open surgical approach in which a large abdominal incision is made.

Laparotomy

_____: Any feeding tube that is inserted nasally and then advanced into the gastrointestinal tract.

Nasoenteric tube (NET)

_____: Scant urine output. Usually less than 400 mL per day.

Oliguria

_____: Neuropeptide that is an appetite stimulant.

Orexin

_____: A patient with an ostomy.

Ostomate

_____: Gum disease in which mandibular bone loss has occurred.

Periodontal disease

_____: The operative experience consisting of the preoperative, intraoperative, and postoperative time periods.

Perioperative

_____: A form of megaloblastic anemia caused by failure to absorb vitamin B12 because of a deficiency of intrinsic factor (normally secreted by the gastric mucosa) needed for intestinal absorption of vitamin B12.

Pernicious anemia

_____: Damage to the esophageal mucosa, often with erosion and ulceration, in patients with gastroesophageal reflux disease.

Reflux esophagitis

_____: A vegetarian diet pattern in which only foods of plant origin are eaten.

Vegan

_____: Inflammation of the liver that results from an infection caused by one of five major categories of viruses (hepatitis A, B, C, D, or E). Viral hepatitis is the most common type and can be either acute or chronic.

Viral hepatitis

_____: Obstruction of the bowel caused by twisting of the bowel.

Volvulus

Which serum albumin level does the nurse expect to see in a healthy, ambulatory adult client? A) 2.3 g/dL (23 g/L) B) 3.7 g/dL (37 g/L) C) 5.1 g/dL (51 g/L) D) 5.8 g/dL (58 g/L)

Answer: B. The normal serum albumin level for men and women is 3.5 to 5.0 g/dL (35 to 50 g/L).The other options given are incorrect.

The nurse is caring for a client diagnosed with aphthous ulcers. The nurse instructs the client to avoid which foods? (Select all that apply). A) Apples. B) Bananas. C) Cheese. D) Nuts. E) Potatoes.

Answer: C, D, E. Aphthous ulcers (canker sores) are small, shallow lesions that develop on the soft tissues in the mouth or at the base of the gums. The nurse tells the client that certain foods such as cheese, nuts, and potatoes may trigger allergic responses that cause aphthous ulcers and should be avoided.Apples and bananas are not acidic and do not trigger allergic responses that cause aphthous ulcers.

A nurse is caring for a client who is receiving moderate (conscious) sedation with midazolam. The client's respiratory rate decreases from 16/min to 6/min, and the oxygen saturation decreases from 92% to 85%. Which of the following medications should the nurse administer? A) Protamine sulfate. B) Atropine. C) Flumazenil. D) Acetylcysteine.

Answer: C. A) Protamine sulfate is used to treat heparin overdose. B) Atropine is an anticholinergic that is used to treat overdose of cholinesterase inhibitors. C) The client's respiratory rate and oxygen saturation level indicate increased sedation caused from a benzodiazepine. The nurse should administer flumazenil, a benzodiazepine agonist, to reverse the sedative effects of the medication. D) Acetylcysteine is an antidote that is used to treat acetaminophen overdose.

After colostomy surgery, which intervention does the nurse employ? a. Cover the stoma with a dry, sterile dressing. b. Apply a pouch system as soon as possible. c. Make a hole in the pouch for gas to escape. d. Watch for the colostomy to start functioning on day 1.

B

What is the primary purpose of a postanesthe- sia care unit (PACU)? a. Follow-through on the surgeon's postoper- ative orders b. Ongoing critical evaluation and stabiliza- tion of the patient c. Prevention of lengthened hospital stay d. Arousal of patient following the use of conscious sedation

B

What is the priority nursing concern for a patient with gastroenteritis? a. Nutrition therapy b. Fluid replacement c. Skin care d. Drug therapy

B

Which parasitic infection is manifested by diarrhea and occurs most commonly in immunosuppressed patients, especially those with human immunodeficiency virus (HIV)? a. Entamoeba histolytica b. Cryptosporidium c. Giardia lamblia d. Escherichia coli

B

The nurse screens a preoperative patient for conditions that may increase the risk for complications during the perioperative period. Which conditions are possible risk factors? Select all that apply. A. Emotionally stable. B. Age 67. C. Obesity. D. Marathon runner. E. Pulmonary disease. F. Hypertension

B,C,E,F

When assessing the hydration status of an older postoperative patient, where must the nurse assess for tenting of the skin? Select all that apply. a. On the back of the hand b. On the forehead c. On the forearm d. On the sternum e. On the abdomen f. On the thigh

B,D

_____: Branch of medicine that manages obesity and its related diseases.

Bariatrics

_____: Columnar epithelium (instead of the normal squamous cell epithelium) that develops in the lower esophagus during the process of healing from gastroesophageal reflux disease. It is considered premalignant and is associated with an increased risk of cancer in patients with prolonged disease.

Barrett's epithelium

_____: Ulceration of the lower esophagus caused by exposure to acid and pepsin, leading to the replacement of normal distal squamous mucosa with columnar epithelium as a response to tissue injury.

Barrett's esophagus

_____: Intense pain due to obstruction of the cystic duct of the gallbladder from a stone moving through or lodged within the duct. Tissue spasm occurs in an effort to mobilize the stone through the small duct.

Biliary colic

_____: A plastic or metal device that is placed percutaneously to keep a duct of the biliary system open in patients experiencing biliary obstruction.

Biliary stent

A female patient is to have her gallbladder removed by natural orifice transluminal endoscopic surgery. What does the nurse teach about this surgery? a. The surgeon will use powerful shock waves to break up the gallstones. b. The surgeon will insert a transhepatic biliary catheter to open blocked bile ducts. c. The surgeon will use a vaginal approach to remove the gallbladder. d. The surgeon will inject ursodeoxycholic acid to dissolve any remaining gallstone fragments.

C

A male patient is having revision of a scar on his forehead from a third-degree burn. What is the correct classification for this surgery? A. Major. B. Restorative. C. Cosmetic. D. Curative.

C

After an esophagectomy, what is the nurse's priority for patient care? a. Wound care b. Nutrition care c. Respiratory care d. Hydration care

C

The nurse is providing care for a patient after an esophagogastroduodenoscopy (EGD). What is the first priority action after this diagnostic study? a. Monitor vital signs. b. Auscultate breath sounds. c. Keep patient NPO until gag reflex returns. d. Keep accurate intake and output.

C

The patient comes to the emergency department (ED) with right lower quadrant pain. What does the ED nurse suspect? A. Gastroenteritis b. Ulcerative colitis c. Appendicitis d. Crohn's disease

C

The patient is recovering in a postanesthesia care unit (PACU) environment that advances the patient quickly from a Phase I care level to a Phase III care level, preparing for discharge to home. What type of surgery is this patient most likely having? a. Elective surgery b. Emergency surgery c. Same-day surgery d. Urgent surgery

C

To avoid electrical safety problems during surgery, what does the nurse do? a. Observes for breaks in sterile technique b. Continuously assists the anesthesia provider c. Ensures proper placement of the grounding pads d. Monitors the operating room with available cameras

C

Which statement about intraabdominal pressure (IAP) is correct? a. The normal IAP for adults is 15-20 mm Hg. b. Patients with high IAP have bradycardia. c. High IAP leads to elevated systemic and portal venous pressure. d. Patients with high IAP are hypertensive.

C

Which treatment offers the patient with liver cancer the possibility of long-term survival? a. Chemotherapy b. Selective internal radiation therapy c. Liver transplantation d. Hepatic arterial embolization

C

_____: A method of tube feeding in which small amounts of enteral product are continuously infused (by gravity drip or by a pump or controller device) over a specified time.

Continuous feeding

_____: Acute ulcerative gastroduodenal disease, which may develop within 24 hours of a severe burn injury because of reduced gastrointestinal blood flow and mucosal damage.

Curling's ulcer

For which patient is radiation therapy contraindicated? a. Patient with lung cancer b. Patient with esophageal tumor c. Patient with sliding hernia d. Patient with tracheoesophageal fistula

D

How is neomycin sulfate used to treat patients with cirrhosis? a. It treats the current infection the patient has. b. It prevents future infections of the liver. C. It restores normal function to the liver cells. D. It decreases the rate of ammonia production.

D

The patient describes experiencing a dull and steady substernal pain, especially after drinking cold liquids. Which manifestation of esophageal cancer does the nurse recognize? a. Angina b. Aspiration c. Dysphagia d. Odynophagia

D

What is the most reliable method of assessing the return of peristalsis? a. Auscultate and count the number of bowel sounds over 30 seconds. b. Administer a laxative as ordered by the health care provider. c. Monitor and document all oral intake of food and fluids. d. Assess whether the patient has passed flatus or a stool.

D

A nurse is caring for a client who has colorectal cancer and is receiving chemothearpy. The client asks the nurse why his blood is being drawn for a carcinoembryonic antigen (CEA) level. Which of the following responses should the nurse make? A) "The CEA determines if the neutrophil count is below the expected reference range." B) "The CEA determines the current stage of your colon cancer." C) "The CEA determines the efficacy of your chemotherapy." D) "The CEA determines if you are experiencing occult bleeding from the gastrointestinal tract."

Answer: C. A) A provider uses a CBC to determine if the neutrophil count is within the expected reference range. B) A provider uses a colonoscopy to determine the diagnosis of colorectal cancer. C) A provider uses the CEA level to determine the efficacy of the chemotherapy. The client's CEA levels will decrease if the chemotherapy is effective. D) A provider uses a fecal occult blood test to determine if there is bleeding in the gastrointestinal tract.

A patient is admitted with obstructive jaundice. Which sign/symptom does the nurse expect to find upon assessment of the patient? a. Pruritus b. Pale urine in increased amounts c. Pink discoloration of sclera d. Dark, tarry stools

A

A patient is given midazolam hydrochloride before receiving a colonoscopy procedure. What is the priority assessment for the nurse during this procedure? a. Monitor rate and depth of respirations. b. Auscultate for bowel sounds in all 4 quadrants. c. Place on a cardiac monitor and watch for dysrhythmias. d. Suction secretions to prevent aspiration.

A

A patient who has been prescribed famotidine is being discharged home. Which statement by the patient indicates a need for further discharge teaching by the nurse? a. "I will double up on the dose if I begin to feel increased heartburn." b. "I will avoid all alcohol." c. "I will call the health care provider if I continue to have heartburn." d. "This drug is available over the counter."

A

A patient will undergo an abdominal paracentesis. Which factor provides an additional safety measure? a. The procedure is performed using ultrasound. b. The procedure is performed at the bedside. c. A trocar is inserted into the peritoneal cavity. d. General anesthesia is administered.

A

The nurse is assisting a vegan patient to complete a lunch menu. Which food would the nurse suggest? a. Black beans over rice b. Baked beans with bacon c. Tuna fish casserole d. Green salad with olive oil dressing

A

The older overweight patient is being placed on the operating table for an appendectomy. What is the priority medical-surgical concept for this patient? A. Safety. B. Tissue integrity. C. Immunity. D. Gas exchange.

A

Which adipokine hormone is known as the hunger hormone? a. Ghrelin b. Cholecystokinin c. Leptin d. Resistin

A

Which is a key feature of pancreatic cancer? a. Anorexia b. Weight gain c. Pale urine d. Dark-colored stools

A

Which type of gastric ulcer does the nurse expect may occur when caring for a patient with extensive burns? a. Curling's ulcer b. Cushing's ulcer c. Stress ulcer d. Ischemic ulcer

A

Which are most likely laboratory values for a patient with liver disease? Select all that apply. a. Increased prothrombin time b. Increased aspartate transaminase (AST) c. Increased albumin levels d. Decreased ammonia levels e. Increased unconjugated bilirubin f. Decreased alanine aminotransferase (ALT)

A,B,E

Which characteristics are appropriate to moderate sedation drugs? Select all that apply. a. Reduce sensory perception. b. Require placement of an artificial airway. c. Amnesia action is short. d. Return to normal function is rapid. e. Increase level of consciousness. f. May be administered only by a physician.

A,C,D

Which patients are commonly affected by oral cavity disorders? Select all that apply. a. Patients who are homeless or live in institutions b. Patients who work in coal mines c. Patients who are developmentally disabled d. Patients who regularly use tobacco or alcohol e. Patients who have sexually transmitted diseases f. Patients who eat too much of an unhealthy diet

A,C,D,F

Which findings for a patient with a new colostomy will the nurse report to the surgeon? Select all that apply. a. A dark-red, dry stoma b. Stoma protruding about 2 cm from the abdominal wall c. Mucocutaneous separation d. A slight amount of edema in the initial postoperative period e. Large amount of bleeding f. Moist pink stoma

A,C,E

The nurse is assisting a patient who follows a lactovegetarian diet to fill out a menu. Based on this diet, which foods could the patient select for breakfast? Select all that apply. A. Milk b. Scrambled eggs c. Toast d. Sausage e. Cereal f. String cheese

A,C,E,F

The nurse is providing teaching about the risk factors for malnutrition for a group of older adults. What factors does the nurse emphasize in the teaching plan? Select all that apply. a. Poor dental health b. Hypersecretion of saliva c. Depression d. Chronic diarrhea e. Lack of transportation f. Acute or chronic pain

A,C,E,F

A patient has a Kaposi's sarcoma lesion on his hard palate. How is this lesion described? Select all that apply. a. Small raised lesion b. Dark-yellow nodule c. Painful lesion d. Purplish-brown nodule e. Usually not painful f. Appears off-white and raised

A,D,E

Which assessment is most important for the nurse to perform for the client admitted to the postanesthesia care unit (PACU) after surgery under general anesthesia? A) Determining the client's level of consciousness B) Checking for pain on dorsi and plantar flexion of the foot C) Assessing the response to pin prick stimulation from feet to mid chest level D) Comparing blood pressure taken in the right arm to blood pressure taken in the left arm

ANS: A. After general anesthesia, which affects the entire body, the priority assessment is to determine that the client's level of consciousness has returned. All other assessment can be performed subsequently.

A client reports ongoing episodes of heartburn. The nurse educates the client on prevention and control of reflux by recommending dietary elimination of which food item? A) Lean steak B) Carrot sticks C) Chocolate candy D) Air-popped popcorn

ANS: C. Foods that decrease esophageal sphincter pressure, such as fatty food, caffeine, and chocolate, should be avoided.

Which data should the nurse expect to assess in the client who had an upper gastrointestinal (UGI) series? 1. Chalky white stools. 2. Increased heart rate. 3. A firm, hard abdomen. 4. Hyperactive bowel sounds

ANSWER: 1. 1. A UGI series requires the client to swallow barium, which passes through the intestines, making the stools a chalky white color. 2. Increased heart rate is abnormal data and would be cause for further assessment. 3. A firm, hard abdomen is not expected from the UGI series. 4. Hyperactive bowel sounds is not an expected sequela of a UGI series. TEST-TAKING HINT: Option "2" could be eliminated because it does not have anything to do with the gastrointestinal system. A firm, hard abdomen is seldom ever expected, so option "3" could be eliminated.

The nurse at the scene of a knife fight is caring for a young man who has a knife in his abdomen. Which action should the nurse implement? 1. Stabilize the knife. 2. Remove the knife gently. 3. Turn the client on the side. 4. Apply pressure to the insertion site

ANSWER: 1. 1. The nurse should not remove any penetrating object in the abdomen; removal could cause further internal damage. 2. Removal of the knife could cause further internal damage. 3. The client should be kept on the back and the knife should be stabilized. 4. The nurse should stabilize the knife and notify Emergency Medical Services as quickly as possible.

When preparing a client to undergo paracentesis, which action is necessary to reduce potential injury as a result of the procedure? A) Encourage the client to take deep breaths and cough B) Ask the client to void prior to the procedure C) Position the client with the head of the bed flat D) Assist the physician to insert a trocar catheter into the abdomen

Answer: B. To avoid injury to the bladder during a paracentesis, the client would be asked to void prior to the procedure (Chart 58-1).Taking deep breaths and coughing does not prevent complications or injury as a result of paracentesis. Clients would be positioned with the head of the bed elevated. The trocar catheter is used to drain the ascetic fluid and does not reduce the risk of damage to the bladder.

The nurse is completing the preoperative checklist. Which laboratory value should be reported to the health-care provider immediately? 1. Hemoglobin 13.1 g/dL. 2. Glucose 60 mg/dL. 3. White blood cells 6 (× 103 )/mm3 . 4. Potassium 3.8 mEq/L.

ANSWER: 2. 1. This hemoglobin is within normal limits and would not warrant immediate action. 2. This glucose level indicates hypoglycemia, which requires medical intervention. 3. This white blood cell value is within normal range and would not be reported. 4. This potassium level is within normal limits and would not require intervention.

The nurse is admitting a client to a medical floor with a diagnosis of adenocarcinoma of the rectosigmoid colon. Which assessment data support this diagnosis? 1. The client reports up to 20 bloody stools per day. 2. The client has a feeling of fullness after a heavy meal. 3. The client has diarrhea alternating with constipation. 4. The client complains of right lower quadrant pain

ANSWER: 3. 1. Frequent bloody stools are a symptom of inflammatory bowel disease (IBD). IBD is a risk factor for cancer of the colon, but the symptoms are different when the colon becomes cancerous. 2. Most people have a feeling of fullness after a heavy meal; this does not indicate cancer. 3. The most common symptom of colon cancer is a change in bowel habits, specifically diarrhea alternating with constipation. 4. Lower right quadrant pain with rebound tenderness would indicate appendicitis. TEST-TAKING HINT: The test taker could eliminate option "4" based on anatomical position. The rectosigmoid colon is in the left lower quadrant.

Which statement made by the client who is postoperative abdominal surgery indicates the discharge teaching has been effective? 1. "I will take my temperature each week and report any elevation." 2. "I will not need any pain medication when I go home." 3. "I will take all of my antibiotics until they are gone." 4. "I will not take a shower until my three (3)-month checkup."

ANSWER: 3. 1. The client should check the temperature twice a day. 2. It is not realistic to expect the client to experience no pain after surgery. 3. This statement about taking all the antibiotics ordered indicates the teaching is effective. 4. Clients may shower after surgery, but not taking a tub bath for three (3) months after surgery is too long a time.

The nurse identifies the problem of "fluid volume deficit" for a client diagnosed with gastritis. Which intervention should be included in the plan of care? 1. Obtain permission for a blood transfusion. 2. Prepare the client for total parenteral nutrition. 3. Monitor the client's lung sounds every shift. 4. Assess the client's intravenous site

ANSWER: 4. 1. There are no data to suggest the client needs a blood transfusion. 2. TPN is not a treatment for a client with fluid volume deficit. TPN provides calories for nutritional deficits, not fluid deficits. 3. If the client's problem were fluid volume excess, assessing lung sounds would be appropriate. 4. Fluid administration is the medical treatment for dehydration, so the nurse must monitor and ensure the IV site is patent.

_____: The uptake from the intestinal lumen of nutrients produced by digestion.

Absorption

_____: Tumor that arises from the glandular epithelial tissue.

Adenocarcinoma

_____: An anti-inflammatory and insulin-sensitizing hormone.

Adiponectin

_____: Drugs that suppress appetite, which reduces food intake and, over time, may result in weight loss; may be prescribed for obese patients in a comprehensive weight reduction program.

Anorectic drugs

_____: The loss of appetite for food.

Anorexia

_____: An eating disorder of self-induced starvation resulting from a fear of fatness, even though the patient is underweight.

Anorexia nervosa

_____: Neuropeptide that decreases appetite.

Anorexin

The nurse is teaching a patient with a newly created colostomy about foods to limit or avoid because of flatulence or odors. Which foods are included? (Select all that apply). A) Broccoli. B) Buttermilk. C) Mushrooms. D) Onions. E) Peas. F) Yogurt.

Answer: A, C, D, E. Foods the patient with a newly created colostomy needs to limit or avoid because of flatulence or odors include: Broccoli, mushrooms, onions, and peas.Buttermilk will help prevent odors. Yogurt can help prevent flatus.

The nurse is caring for an older adult male patient who reports stomach pain and heartburn. Which sign/symptom is most significant suggesting the patient's ulceration is duodenal in origin and not gastric? A) Pain occurs 1½ to 3 hours after a meal, usually at night. B) Pain is worsened by the ingestion of food. C) The patient has a malnourished appearance. D) The patient is a man older than 50 years.

Answer: A. A key symptom of duodenal ulcers is that pain usually awakens the patient between 1:00 a.m. and 2:00 a.m. and occurs 1½ to 3 hours after a meal.Pain that is worsened with ingestion of food and a malnourished appearance are key features of gastric ulcers. A male over 50 years is a finding that could apply to either type of ulcer.

In conducting a postoperative assessment of a client, what is important for the nurse to examine first? A) Breathing pattern. B) Level of consciousness. C) Oxygen saturation. D) Surgical site.

Answer: A. Respiratory assessment is the first and most important.Assessing level of consciousness, oxygen saturation, and the surgical site are important, but not the priority.

The nurse administers lactulose (Evalose) to a client with cirrhosis for which purpose? A) To aid in digestion of dairy products B) To reduce portal pressure C) To promote gastrointestinal (GI) excretion of ammonia D) To reduce the risk of GI bleeding

Answer: C. In a client with cirrhosis, the administration of lactulose reduces serum ammonia levels by causing the client to excrete ammonia through the GI tract.Lactase, not lactulose, is the enzyme that aids in the digestion of dairy products. The mechanism of action of lactulose is not to reduce portal pressure. Lactulose does not affect bleeding.

The nurse is working with the dietitian to plan a menu for a patient who has persistent difficulty swallowing. What is a suitable breakfast selection for this patient? A) Scrambled eggs and toast B) Oatmeal and orange juice C) Puréed fruit and English muffin D) Cream of wheat and applesauce

Answer: D. A breakfast selection of both cream of wheat and applesauce are foods of semisolid consistency and are appropriate for this patient. The patient who is having difficulty swallowing would be given semisolid foods and thickened liquids.Toast would not be appropriate, and orange juice would have to be thickened before it is given to this patient. An English muffin would be inappropriate for this patient because it is not a semisolid food.

_____: Acute inflammation of the vermiform appendix, which is the blind pouch attached to the cecum of the colon, usually located in the right iliac region just below the ileocecal valve.

Appendicitis

_____: The accumulation of free fluid within the peritoneal cavity. Increased hydrostatic pressure from portal hypertension causes this fluid to leak into the peritoneal cavity.

Ascites

_____: A coarse tremor characterized by rapid, nonrhythmic extensions and flexions in the wrists and fingers; a motor disturbance seen in portal-systemic encephalopathy. Also called a liver flap or flapping tremor.

Asterixis

A patient is undergoing a workup for carcinoma of the esophagus. What are the two primary risk factors associated with the development of this carcinoma? a. High-fat, low-fiber diet and tobacco use b. Tobacco use and obesity c. Sedentary lifestyle and family history of squamous cell carcinoma d. Heavy alcohol intake and high-fat, low-fiber diet

B

A patient who can barely ambulate with a walker at home is having a left total knee replacement. What is the most appropriate category for this surgery? a. Urgent b. Restorative c. Simple d. Palliative

B

Mild gas pain and flatulence may be experi- enced because of air instilled into the rectum during the examination, and if a biopsy speci- men is obtained, a small amount of bleeding may be observed. Which endoscopic proce- dure is this follow-up care describing? a. Colonoscopy b. Proctosigmoidoscopy c. Enteroscopy d. Endoscopic retrograde cholangiopancreatography (ERCP)

B

The nurse is caring for a patient with acute viral hepatitis. What is the major care concern at this time? A. Providing three small meals a day. B. Alternating periods of activity with periods of rest. C. Monitoring for the development of jaundiced skin. D. Teaching the patient the importance of avoiding alcohol intake.

B

The nurse is supervising a senior nursing student in the care of a patient after esophageal surgery. For which action by the student must the nurse intervene? a. Student secures the NG tube to prevent dislodgment. b. Student prepares to irrigate NG tube. c. Student provides mouth care every 2-4 hours. d. Student elevates the head of the patient's bed.

B

The nurse is teaching a patient about ranitidine (Zantac) prescribed for gastritis. Which statement by the patient indicates effective teaching by the nurse? a. "The drug will heal the areas of my stomach that are sore." b. "This drug will block the secretions of my stomach." c. "Zantac will coat the inside of my stomach to protect it from acid." d. "This pill kills the bacterial infection I have in my stomach."

B

The patient with acute necrotizing pancreatitis experiences a temperature spike to 104°F (40°C). What does the nurse suspect? a. Pancreatic pseudocyst b. Pancreatic abscess c. Chronic pancreatitis d. Pancreatic cancer

B

Which is an example of an environmental factor that can lead to IBS? a. Pseudomonas aeruginosa b. Dairy products c. Pro-inflammatory interleukins d. Mental stress

B

Which conditions place a patient at high risk for the development of fatty liver (steatosis)? Select all that apply. a. Hypertension b. Diabetes mellitus c. Obesity d. Elevated lipid profile e. Alcohol abuse f. Hepatitis A

B,C,D,E

The nurse is preparing the patient for surgery. Which common laboratory tests does the nurse anticipate being ordered? Select all that apply. a. Total cholesterol b. Urinalysis c. Electrolyte levels d. Uric acid e. Clotting studies f. Serum creatinine

B,C,E,F

_____: A measure of nutritional status that does not depend on frame size; indirectly estimates total fat stores within the body by the relationship of weight to height.

Body mass index (BMI)

_____: A method of tube feeding that involves intermittent feeding of a specified amount of enteral product at specified times during a 24-hour period, typically every 4 hours.

Bolus feeding

_____: An oncofetal antigen that may be elevated in 70% of people with colorectal cancer. CEA is not specifically associated with the colorectal cancer and may be elevated in the presence of other benign or malignant diseases and in smokers. CEA is often used to monitor the effectiveness of treatment and to identify disease recurrence.

Carcinoembryonic antigen (CEA)

_____: A progressive, destructive disease of the pancreas characterized by remissions and exacerbations. Inflammation and fibrosis of the tissue contribute to pancreatic insufficiency and diminished function of the organ.

Chronic pancreatitis

_____: A method of tube feeding similar to continuous feeding (see definition of continuous feeding) except the infusion is stopped for a specified time in each 24-hour period ("down time"); the down time typically occurs in the morning to allow bathing, treatments, and other activities.

Cyclic feeding

A patient is prescribed pantoprazole. What does the nurse tell the patient is the major action of this medication? a. It produces a coating on the stomach lining. b. It neutralizes gastric acid. c. It heals esophageal irritation. d. It inhibits gastric acid secretion.

D

The nurse is taking a history from a patient with peptic ulcer disease. Which factor indicates the largest risk for stomach cancer? a. History of GERD for 6 weeks b. History of hiatal hernia c. History of gastritis d. History of infection with H. pylori

D

The patient who had bariatric surgery and is to be discharged asks the nurse when to expect the panniculectomy surgery. What is the nurse's best response? A. Usually in 6-8 months b. Usually in 12-18 months c. Usually in 18-24 months d. When your weight stabilizes

D

Which is the most accurate way to obtain a height measurement for a patient who cannot stand? a. Review the patient's old chart. b. Ask the patient's family member. c. Estimate height based on the patient's position in bed. d. Use a sliding blade knee height caliper

D

_____: Recommendations made by the USDA and U.S. Department of Health and Human Services to help people maintain nutritional health; updated every 5 years.

Dietary Guidelines for Americans

_____: Nutrition guide developed by the Institute of Medicine of the National Academies that provides a scientific basis for food guidelines in the United States and Canada.

Dietry Reference Intakes (DRI's)

_____: The mechanical and chemical process in which complex foodstuffs are broken down into simpler forms that can be used by the body.

Digestion

_____: A sac formed from the peritoneum that contains a portion of the intestine and passes through a weak point in the abdominal wall.

Direct inguinal hernia

_____: Sacs resulting from the herniation of the mucosa and submucosa of a tubular organ into surrounding tissue.

Diverticula

_____: The inflammation of one or more diverticula.

Diverticulitis

_____: The presence of many abnormal pouchlike herniations (diverticula) in the wall of the intestine.

Diverticulosis

_____: The least common type of colostomy, which is created by dividing the bowel and bringing both the proximal and distal portions to the abdominal surface to create two stomas.

Double-barrel stoma

_____: Indigestion or heartburn following meals.

Dyspepsia

_____: Visualization of the small intestine.

Enteroscopy

_____: A tube used for patients who need long-term enteral feeding; the physician directly accesses the gastrointestinal tract using surgical, endoscopic, or laparoscopic techniques.

Enterostomal feeding tubes

_____: The act of belching.

Eructation

_____: A velvety red mucosal lesion, most often occurring in the oral cavity.

Erythroplakia

_____: Narrowing of the esophageal opening.

Esophageal stricture

_____: The distention of fragile, thin-walled esophageal veins due to increased pressure; the increased pressure is a result of portal hypertension, in which the blood backs up from the liver and enters the esophageal and gastric vessels that carry it into the systemic circulation.

Esophageal varices

_____: The visual examination of the esophagus, stomach, and duodenum by means of a fiberoptic endoscope.

Esophagogastroduodenoscopy (EGD)

_____: The surgical creation of a communication between the stomach and the esophagus; it involves the removal of part of the esophagus and proximal stomach.

Esophagogastrostomy

_____: The distinctive fruity or musty breath odor of chronic liver disease and portal-systemic encephalopathy.

Fetor hepaticus

_____: The partial or total surgical removal of the tongue.

Glossectomy

_____: The hepatitis D virus (HDV) co-infects with hepatitis B virus (HBV) and needs the presence of HBV for viral replication. HDV can co-infect a patient with HBV or can occur as a superinfection in a patient with chronic HBV. Superinfection usually develops into chronic HDV infection. The incubation period is 14 to 56 days. As with HBV, the disease is transmitted primarily by parenteral routes.

Hepatitis D

_____: Person who has had hepatitis B but has not developed immunity. Hepatitis carriers can infect others even though they are not sick and demonstrate no obvious signs of disease. Chronic carriers are at high risk for cirrhosis and liver cancer.

Hepatitis carrier

_____: An elevated level of potassium in the blood.

Hyperkalemia

_____: Elevated body temperature; fever.

Hyperthermia

_____: A decreased serum potassium level; a common electrolyte imbalance.

Hypokalemia

_____: A decrease in serum proteins.

Hypoproteinemia

_____: Yellow discoloration of the sclerae.

Icterus

_____: Pancreatitis that may be associated with SPINK1 and CFTR gene mutations.

Idiopathic pancreatitis

_____: Protrusion of the intestine at the site of a previous surgical incision resulting from inadequate healing. Most often caused by postoperative wound infections, inadequate nutrition, and obesity

Incisional (ventral) hernia

_____: A syndrome characterized by excessive circulating bilirubin levels. Liver cells cannot effectively excrete bilirubin, and skin and mucous membranes become characterized by a yellow coloration.

Jaundice

_____: The surgical creation of an opening between the jejunum and the surface of the abdominal wall.

Jejunostomy

_____: Device that uses the distance between the patella and heel to estimate height.

Knee height caliper

_____: Phagocytic cells that are part of the body's reticuloendothelial system and that are involved in the protective function of the liver. Kupffer cells engulf harmful bacteria and anemic red blood cells.

Kupffer cells

_____: Lack of protein quantity and quality in the presence of adequate calories. Body weight is somewhat normal, and serum proteins are low.

Kwashiorkor

_____: A hormone that is released by fat cells and possibly by gastric cells; it also acts on the hypothalamus to control appetite.

Leptin

_____: A cosmetic procedure to reduce the amount of adipose tissue in selected areas of the body.

Liposuction

_____: Anesthesia that is delivered by applying it to the skin or mucous membranes of the area to be anesthetized or by injecting it directly into the tissue around an incision, wound, or lesion.

Local anesthesia

_____: A gastrostomy device that uses a firm or balloon-style internal bumper or retention disk; an antireflux valve keeps gastric contents from leaking onto the skin.

Low-profile gastrostomy device (LPGD)

_____: The portion of the esophagus proximal to the gastroesophageal junction; when at rest, the sphincter is closed to prevent reflux of gastric contents into the esophagus.

Lower esophageal sphincter (LES)

_____: A noninvasive diagnostic procedure that provides information about tissue stiffness by assessing qualities of shear waves generated into the tissue.

MR Elastography

_____: A syndrome associated with a variety of disorders and intestinal surgical procedures and characterized by impaired intestinal absorption of nutrients.

Malabsorption

_____: Surgical removal of the jaw.

Mandibulectomy

_____: A combined protein and energy malnutrition that often presents clinically when metabolic stress is imposed on a chronically starved patient.

Marasmik-kwashiorkor

_____: A calorie malnutrition in which body fat and protein are wasted but serum proteins are often preserved.

Marasmus

_____: The physical obstruction of the bowel by disorders outside the intestine (e.g., adhesions or hernias) or by blockages in the lumen of the intestine (e.g., tumors, inflammation, strictures, or fecal impactions).

Mechanical obstruction

_____: Blood in the stool, with the appearance of black tarry stools.

Melena

_____: To spread cancer from the main tumor site to many other body sites.

Metastasize

_____: A laparoscopic surgical procedure to remove part of the esophagus; may be performed in patients with early-stage cancer.

Minimally invasive esophagectomy (MIE)

_____: Surgical repair of an inguinal hernia through a laparoscope, which is the treatment of choice.

Minimally invasive inguinal hernia repair (MIIHR)

_____: A general term for any surgery performed using laparoscopic technique.

Minimally invasive surgery (MIS)

_____: A factor in disease development that can be altered or controlled by the patient. Examples include elevated serum cholesterol levels, cigarette smoking, hypertension, impaired glucose tolerance, obesity, physical inactivity, and stress.

Modifiable risk factor

_____: A low-molecular-weight heme protein found in cardiac and skeletal muscle; an early marker of myocardial infarction.

Myoglobin

_____: The surgical creation of an opening, usually referring to an opening in the abdominal wall; stoma.

Ostomy

_____: A patient who goes to the hospital for treatment and returns home on the same day.

Outpatient

_____: Surgical anastomosis of the pancreatic duct with the jejunum.

Pancreaticojejunostomy

_____: Infection of the panniculus.

Panniculitis

_____: Performed through the skin and other tissues

Percutaneous

_____: An abnormal outgrowth from a mucous membrane

Polyp

_____: Recovery room.

Postanesthesia care unit (PACU)

_____: A protein secreted by the liver that binds thyroxine.

Prealbumin (PAB)

_____: Before surgery.

Preoperative

_____: A disorder of nutrition that may present in three forms: marasmus, kwashiorkor, and marasmic-kwashiorkor.

Protein-calorie malnutrition (PCM)(Protein-energy malnutrition, PEM)

_____: A group of drugs that inhibit the proton pump in the stomach to decrease gastric acid production.

Proton pump inhibitors (PPIs)

_____: Measurement that estimates body fat.

Skinfold measurements

_____: A type of gastritis that refers to an inflammation of the glands, fundus, and body of the stomach.

Type A gastritis

_____: A chronic inflammatory process that affects the mucosal lining of the colon or rectum; one of a group of bowel diseases of unknown etiology characterized by remissions and exacerbations. It can result in loose stools containing blood and mucus, poor absorption of vital nutrients, and thickening of the colon wall.

Ulcerative colitis (UC)

The client is diagnosed with Crohn's disease, also known as regional enteritis. Which statement by the client supports this diagnosis? 1. "My pain goes away when I have a bowel movement." 2. "I have bright red blood in my stool all the time." 3. "I have episodes of diarrhea and constipation." 4. "My abdomen is hard and rigid and I have a fever."

ANSWER: 1. 1. The terminal ileum is the most common site for regional enteritis, which causes right lower quadrant pain that is relieved by defecation. 2. Stools are liquid or semiformed and usually do not contain blood. 3. Episodes of diarrhea and constipation may be a sign/symptom of colon cancer, not Crohn's disease. 4. A fever and hard, rigid abdomen are signs/ symptoms of peritonitis, a complication of Crohn's disease. TEST-TAKING HINT: The test taker should eliminate option "2" because of the word "all," which is an absolute. There are very few absolutes in the health-care arena.

The client presents to the emergency department experiencing frequent watery, bloody stools after eating some undercooked meat at a fast-food restaurant. Which intervention should be implemented first? 1. Obtain a stool sample from the client. 2. Initiate antibiotic therapy intravenously. 3. Have the laboratory draw a complete blood count. 4. Administer the antidiarrheal medication Lomotil.

ANSWER: 1. 1. This client may have developed an infection from the undercooked meat. The nurse should obtain a stool specimen for the laboratory to analyze. 2. Antibiotic therapy is initiated in only the most serious cases of infectious diarrhea; the diarrhea must be assessed first. A specimen for culture should be obtained before beginning medication. 3. A complete blood count will provide an estimate of blood loss, but it is not the first intervention. 4. An antidiarrheal medication would be administered after the specimen collection. TEST-TAKING HINT: All options in a prioritysetting question may be interventions the nurse could implement, but the right answer will be the one (1) implemented first. Collecting a stool sample is assessment, which is the first step in the nursing process.

Which statement should the nurse identify as the expected outcome for a client experiencing acute pain? 1. The client will have decreased use of medication. 2. The client will participate in self-care activities. 3. The client will use relaxation techniques. 4. The client will repeat instructions about medications.

ANSWER: 2. 1. A decrease in use of pain medication does not mean the client's pain is managed; the client may be concerned about possible addiction to the pain medication. 2. Clients experiencing acute pain will not be involved in self-care because of their reluctance to move, which increases the pain; therefore, participation indicates the client's pain is tolerable. 3. Using relaxation techniques does not indicate the client's pain is under control. 4. This would be an expected outcome of a knowledge-deficit problem. TEST-TAKING HINT: The test taker must first determine what is the expected outcome, which should be "relief of pain," and then determine which option addresses the client's relief of pain.

The nurse is caring for the client diagnosed with hepatic encephalopathy. Which sign and symptom indicate the disease is progressing? 1. The client has a decrease in serum ammonia level. 2. The client is not able to circle choices on the menu. 3. The client is able to take deep breaths as directed. 4. The client is able to eat previously restricted food items.

ANSWER: 2. 1. An increase in serum ammonia levels is seen in clients diagnosed with hepatic encephalopathy and coma. 2. The inability to circle food items on the menu may indicate deterioration in the client's cognitive status. The client's neurological status is impaired with hepatic encephalopathy; the nurse should investigate this behavior. 3. The client being able to follow commands indicates the client's neurological status is intact. 4. Consuming foods providing adequate nutrition indicates the client is getting better and able to follow client teaching.

The client with a new colostomy is being discharged. Which statement made by the client indicates the need for further teaching? 1. "If I notice any skin breakdown, I will call the HCP." 2. "I should drink only liquids until the colostomy starts to work." 3. "I should not take a tub bath until the HCP okays it." 4. "I should not drive or lift more than five (5) pounds."

ANSWER: 2. 1. If the tissue around the stoma becomes excoriated, the client will be unable to pouch the stoma adequately, resulting in discomfort and leakage. The client understands the teaching. 2. The client should be on a regular diet, and the colostomy will have been working for several days prior to discharge. The client's statement indicates the need for further teaching. 3. Until the incision is completely healed, the client should not sit in bath water because of the potential contamination of the wound by the bath water. The client understands the teaching. 4. The client has had major surgery and should limit lifting to minimal weight. The client understands the teaching. TEST-TAKING HINT: This is an abdominal surgery and all instructions for major surgery apply. This is an "except" question; therefore, three (3) options would indicate the client understands the teaching.

Which priority teaching information should the nurse discuss with the client to help prevent contracting hepatitis B? 1. Explain the importance of good hand washing. 2. Recommend the client take the hepatitis B vaccine. 3. Tell the client not to ingest unsanitary food or water. 4. Discuss how to implement Standard Precautions.

ANSWER: 2. 1. This intervention would be appropriate for prevention of hepatitis A. 2. The hepatitis B vaccine will prevent the client from contracting this disease. 3. This intervention would be appropriate for prevention of hepatitis A. 4. The nurse uses Standard Precautions, not the client.

The clinic nurse is returning client calls. Which client should the nurse call first? 1. The 39-year-old client complaining of headache pain with a 3 on the pain scale. 2. The 45-year-old client who needs a prescription refill for warfarin. 3. The 54-year-old client diagnosed with diabetes type 1 who has been vomiting. 4. The 60-year-old client who cannot afford to buy food and needs assistance.

ANSWER: 3. 1. A three (3) on the pain scale indicates mild pain. This would not be the first client for the nurse to return the call. 2. A prescription refill would not be a reason for the nurse to make this client first. 3. A client who has diabetes type 1 and is vomiting is at risk for diabetes ketoacidosis. The nurse should have the client come in immediately. 4. This client needs a referral to a social worker. The client is not the first one to receive a return call. TEST-TAKING HINT: The nurse must be able to interpret implications of disease processes and comorbid conditions (vomiting). Part of the assessment of a symptom requires determining what other diseases can impact the result.

The nurse is caring for the client diagnosed with hemorrhoids. Which statement indicates further teaching is needed? 1. "I should increase fruits, bran, and fluids in my diet." 2. "I will use warm compresses and take sitz baths daily." 3. "I must take a laxative every night and have a stool daily." 4. "I can use an analgesic

ANSWER: 3. 1. Clients with hemorrhoids need to eat highfiber diets and increase fluid intake to keep the stools soft and prevent constipation; therefore, the teaching is effective. 2. Warm compresses or sitz baths decrease pain; therefore, the teaching is effective. 3. Laxatives can be harsh to the bowel and are habit forming; they should not be taken daily. Stool softeners soften stool and can be taken daily. 4. Analgesic ointments, suppositories, and astringents can be used to decrease pain and decrease edema; therefore, the teaching has been effective.

The male client in the day surgery unit complains of difficulty urinating postoperatively. Which intervention should the nurse implement? 1. Insert an indwelling catheter. 2. Increase the intravenous fluid rate. 3. Assist the client to stand to void. 4. Encourage the client to increase fluids.

ANSWER: 3. 1. This intervention is invasive, increases the client's risk for infection, and should be the last resort. 2. Increasing the IV fluids might increase the amount of urine in the bladder and cause further discomfort. 3. Helping the male client to stand can offer the assistance needed to void. The safety of the client should be ensured. 4. Drinking more fluids helps to increase urinary output but will not assist the client to empty the bladder

Based on a client's body mass index (BMI) of 30.2, which category should the nurse document in the client's medical records? 1. Underweight. 2. Overweight. 3. Ideal weight. 4. Obese.

ANSWER: 4. 1. A client with less than a 19 BMI is underweight. 2. A BMI between 25 and 30 is considered overweight. 3. The ideal weight for a client is a BMI between 19 and 24.9. 4. This client is obese, with a BMI greater than 30.

The client has been experiencing difficulty and straining when expelling feces. Which intervention should the nurse discuss with the client? 1. Explain some blood in the stool will be normal for the client. 2. Instruct the client in manual removal of feces. 3. Encourage the client to use a cathartic laxative on a daily basis. 4. Place the client on a high-fiber diet.

ANSWER: 4. 1. Blood may indicate a hemorrhoid, but it is not normal to expel blood when having a bowel movement. 2. Nurses manually remove feces; it is not a self-care activity. 3. Cathartic use on a daily basis creates dependence and a narrowing of the lumen of the colon, creating a much more serious problem. 4. A high-fiber (residue) diet provides bulk for the colon to use in removing the waste products of metabolism. Bulk laxatives and fiber from vegetables and bran assist the colon to work more effectively. TEST-TAKING HINT: Blood is not normal in any circumstance. It may be expected but is not "normal" unless inside a vessel.

The client is in the preicteric phase of hepatitis. Which signs/symptoms should the nurse expect the client to exhibit during this phase? 1. Clay-colored stools and jaundice. 2. Normal appetite and pruritus. 3. Being afebrile and left upper quadrant pain. 4. Complaints of fatigue and diarrhea

ANSWER: 4. 1. Clay-colored stools and jaundice occur in the icteric phase of hepatitis. 2. Normal appetite and itching occur in the icteric phase of hepatitis. 3. Fever subsides in the icteric phase, and the pain is in the right upper quadrant. 4. "Flu-like" symptoms are the first complaints of the client in the preicteric phase of hepatitis, which is the initial phase and may begin abruptly or insidiously. TEST-TAKING HINT: The test taker must use anatomy knowledge in ruling out incorrect answers; option "3" could be ruled out because the liver is in the right upper quadrant.

The nurse writes a psychosocial problem of "risk for altered sexual functioning related to new colostomy." Which intervention should the nurse implement? 1. Tell the client there should be no intimacy for at least three (3) months. 2. Ensure the client and significant other are able to change the ostomy pouch. 3. Demonstrate with charts possible sexual positions for the client to assume. 4. Teach the client to protect the pouch from becoming dislodged during sex.

ANSWER: 4. 1. Intimacy involves more than sexual intercourse. The client can be sexually active whenever the wounds are healed sufficiently to not cause pain. 2. This is an appropriate nursing intervention for home care, but it has nothing to do with sexual activity. 3. The nurse is not a sexual counselor who would have these types of charts. The nurse should address sexuality with the client but would not be considered an expert capable of explaining the advantages and disadvantages of sexual positioning. 4. A pouch that becomes dislodged during the sexual act would cause embarrassment for the client, whose body image has already been dealt a blow. TEST-TAKING HINT: Option "2" does not address the issue and option "3" is outside of the nurse's professional expertise. Option "1" could be eliminated because of the word "no," which is an absolute word.

The client is diagnosed with an acute exacerbation of inflammatory bowel disease (IBD). Which food selection would be the best choice for a meal? 1. Roast beef on wheat bread and a milk shake. 2. Hamburger, french fries, and a cola. 3. Pepper steak, brown rice, and iced tea. 4. Roasted turkey, instant mashed potatoes, and water.

ANSWER: 4. 1. Wheat bread and whole grains should be avoided, and most clients cannot tolerate milk products. 2. Fried foods such as hamburger and french fries should be avoided. Raw fruits and vegetables such as lettuce and tomatoes are usually not tolerated. 3. Whole grains such as brown rice should be avoided. White rice can be eaten. Spicy meats and foods should be avoided. 4. Meats can be eaten if prepared by roasting, baking, or broiling. Vegetables should be cooked, not raw, and skins should be removed. Instant mashed potatoes do not have the skin. A low-residue diet should be eaten.

The nurse is observing a coworker who is caring for a patient with a nasogastric tube following esophageal surgery. Which actions by the coworker require the nurse to intervene? (Select all that apply). A) Checking tube placement every 12 hours B) Keeping the bed flat C) Placing the patient upright when taking sips of water D) Providing mouth care every 8 hours E) Securing the tube

Answer: A, B, D. The nurse would intervene to make sure the nasogastric tube is checked every 4 to 8 hours and not every 12 hours. Also, the head of the bed needs to be elevated at least 30 degrees and not kept flat. Oral hygiene would be provided every 2 to 4 hours and not every 8 hours.The patient should be placed upright when taking sips or small amounts of water to prevent choking and to allow observation of the patient for dysphagia. The tube should be secured to prevent dislodgment.

The nurse is teaching a group of patients with irritable bowel syndrome (IBS) about complementary and alternative therapies. What does the nurse suggest as possible treatment modalities? (Select all that apply). A) Acupuncture. B) Decreasing physical activities. C) Meditation. D) Peppermint oil capsules. E) Yoga

Answer: A, C, D, E. Possible treatment modalities the nurse suggests for a patient with IBS include: acupuncture, meditation, peppermint oil capsules, and yoga. Acupuncture is recommended as a complementary therapy for IBS. Meditation, yoga, and other relaxation techniques help many patients manage stress and their IBS symptoms. Research has shown that peppermint oil capsules may be effective in reducing symptoms of IBS. Regular exercise is important for managing stress and promoting bowel elimination.The nurse would not suggest decreasing physical activity. Regular exercise is important for managing stress and promoting bowel elimination.

The nurse is teaching a patient who recently began taking sulfasalazine (Azulfidine) about the drug. What side effects does the nurse tell the patient to report to the primary health care provider? (Select all that apply). A) Anorexia. B) Depression. C) Drowsiness. D) Frequent urination. E) Headache. F) VOmiting

Answer: A, E, F. Anorexia, headache, and nausea/vomiting are side effects of sulfasalazine that must be reported to the primary health care provider.Depression, drowsiness, and urinary problems are not side effects of sulfasalazine.

The nurse is teaching a patient with peptic ulcer disease (PUD) about the prescribed drug regimen. Which statement made by the patient indicates a need for further teaching before discharge? A) "Nizatidine (Axid) needs to be taken three times a day to be effective." B) "Taking ranitidine (Zantac) at bedtime should decrease acid production at night." C) "Sucralfate (Carafate) should be taken 1 hour before and 2 hours after meals." D) "Omeprazole (Prilosec) should be swallowed whole and not crushed."

Answer: A. Further discharge teaching is needed when the patient says that Nizatidine works best when taken three times a day. Nizatidine is most effective if administered once daily.A dose of ranitidine at bedtime would decrease acid production throughout the night. Sucralfate is taken 1 hour before and 2 hours after meals. Because omeprazole is a delayed-release capsule, it needs to be swallowed whole and not crushed.

The nurse is monitoring a client who is receiving an intravenous fat emulsion (IVFE) nutritional supplement. What action does the nurse take in the event that the client develops fever, increased triglycerides, and clotting problems? A) Discontinues the IVFE infusion and notifies the health care provider (HCP) B) Documents the findings and continues to monitor C) Slows the rate of flow of the IVFE infusion D) Switches to total parenteral nutrition (TPN)

Answer: A. If a client receiving an IVFE nutritional supplement develops fever, increased triglycerides and clotting problems, the nurse must discontinue the IVFE and notify the HCP. For clients receiving fat emulsions, the nurse would monitor for manifestations of fat overload syndrome, especially in those who are critically ill. These manifestations include fever, increased triglycerides, clotting problems, and multisystem organ failure. The IVFE infusion must be discontinued, and the nurse must report any of these changes to the HCP immediately if this complication is suspected.Documenting the findings and continuing to monitor will have serious repercussions for this client. Slowing the rate of flow of the IVFE infusion will present a serious safety risk. Nurses do not request IV parenteral therapies or change them unless ordered by the HCP.

How does the nurse accurately calculate a client's body mass index (BMI)? A) BMI = weight (kg)/height (in meters)2 B) BMI = weight (lb)/height (in inches)2 C) BMI = weight (kg)/height (in meters) D) BMI = weight (lb)/height (in meters)

Answer: A. The correct formula to accurately calculate a client's body mass index (BMI) is: BMI = weight (kg)/height (in meters)2

As a result of being treated with radiation for oral cancer, a client is experiencing xerostomia. What collaborative resource does the nurse suggest for this client's care? A) Dentist. B) Occupational therapist. C) Psychiatrist. D) Speech therapist.

Answer: A. Xerostomia is the subjective feeling of oral dryness, which is often (but not always) associated with hypofunction of the salivary glands. It is a long-term effect of radiation therapy and requires ongoing oral care such as the use of saliva substitutes and follow-up dental visits.Occupational therapists, psychiatrists, and speech therapists are not the appropriate resource for a client with xerostomia.

A 24-year-old male is scheduled for a minimally invasive inguinal hernia repair (MIIHR). Which patient statement indicates a need for further teaching about this procedure? A) "I may have trouble urinating immediately after the surgery." B) "I will need to stay in the hospital overnight." C) "I will not eat after midnight the day of the surgery." D) "My chances of having complications after this procedure are slim."

Answer: B. A need for further teaching about MIIHR is when the patient says, "I will need to stay in the hospital overnight." Usually, the patient is discharged 3 to 5 hours after MIIHR surgery.Male patients who have difficulty urinating after the procedure would be encouraged to force fluids and to assume a natural position when voiding. Patients undergoing MIIHR surgery must be NPO after midnight before the surgery. Most patients who have MIIHR surgery have an uneventful recovery.

An older client is at risk for malnutrition. Which nursing intervention is most appropriate to ensure optimum nutritional intake? A) Administering antiemetics and analgesics after meals B) Assisting the client with toileting and oral care prior to meals C) Turning on the television during meals to provide distraction D) Reminding UAPs to allow the client to remain in bed during meals

Answer: B. The most appropriate intervention to ensure optimum nutritional intake in an older client at risk for malnutrition is to assist the client. Clients need to be assisted with toileting and oral care prior to meals for comfort and to prevent these from distracting clients from meals.Antiemetics and analgesics should be provided prior to meals. Clients need to be free from distractions while eating. When possible, clients would be placed in chairs for eating.

When assessing a client with hepatitis B, the nurse anticipates which assessment findings? (Select all that apply). A) Recent influenza infection. B) Brown stool. C) Tea-colored urine. D) Right upper quadrant tenderness. E) Itching

Answer: C, D, E. Assessment findings the nurse expects to find in a client with Hepatitis B include brown, tea-, or cola-colored urine, right upper quadrant pain due to inflammation of the liver, and itching, irritating skin caused by deposits of bilirubin on the skin secondary to high bilirubin levels and jaundice.Hepatitis B virus, not the influenza virus, causes hepatitis B, which is spread by blood and body fluids. The stool in hepatitis may be tan or clay-colored, and not typically brown.

A client is placed on orlistat (Xenical) as part of a treatment regimen for morbid obesity. What side effects does the nurse tell the client to expect from using this drug? A) Dry mouth, constipation, and insomnia B) Insomnia, dry mouth, and blurred vision C) Loose stools, abdominal cramps, and nausea D) Palpitations, constipation, and restlessness

Answer: C. The nurse tells the client to expect loose stools, abdominal cramps, and nausea. These are side effects unique to orlistat (Xenical).Dry mouth, constipation, and insomnia are not side effects of orlistat. Insomnia, dry mouth, blurred vision, palpitations, constipation, and restlessness are all side effects of short-term therapy drugs such as phentermine (Adipex-P), diethylpropion (Tenuate), and phendimetrazine (Bontril).

An obese client with a body mass index of 30 and hypertension has been taking prescription orlistat for 4 weeks and reports loose stools, abdominal cramps, and nausea. What does the nurse recommend for this client? A) Asking the provider to change the medication to phendimetrazine (Bontril). B) Changing to the lower dose, over-the-counter form of orlistat to reduce these effects. C) Increasing the daily activity level to improve overall metabolism. D) Reducing nutritional fat intake to less than 30% of the client's daily food intake.

Answer: D. The nurse recommends reducing nutritional fat intake to less than 30% of the client's daily food intake. Loose stools, abdominal cramps, and nausea are common side effects of orlistat and can be reduced by decreasing fat intake.Unless side effects persist or become more severe, it is not necessary to change the medication. Reducing the dose of orlistat does not affect these symptoms, since they are dependent on fat intake. Increasing the daily activity level helps with weight loss, but does not reduce side effects of Orlistat.

An older client's adult child tells the nurse that the client does not want life support. What does the nurse do first? A) Call the legal department to draft the paperwork. B) Document this in the chart. C) Thank the person and do nothing else. D) Talk to the client.

Answer: D. The nurse would first talk to the client in order to determine the client's wishes and state of mind.The nurse should not call the legal department or document in the client's chart before speaking with the client. Doing nothing is not appropriate.

"A patient is being discharged, 8 days postoperatively following a total esophagectomy." Which teaching point does the nurse consider to be of the highest priority during the transition to home? A) Instruct the patient to eat three meals daily. B) Emphasize the importance of lying down after meals. C) Encourage the patient to ask his or her health care provider for antidepressant medication. D) Stress the importance of notifying the primary health care provider if leaking is noted at the incision site.

Answer: D. The teaching point with the highest priority is to notify the primary health care provider (PHCP) immediately if leaking is noted at the incision site. Leakage from the site of anastomosis is a dreaded complication that can appear 2 to 10 days after surgery. Wound management and prevention of infection are major concerns because the patient who has had an esophagectomy typically has multiple drains and incisions.The patient should eat six to eight small meals daily, and should sit up after meals to encourage satisfactory swallowing. The patient's coping skills should be assessed, as well as his or her level of anxiety and/or depression, before antidepressant medication is prescribed.

A patient with a colostomy may safely include which food item in the diet? a. Burritos b. Yogurt c. Cabbage d. Carbonated beverages

B

A patient with an abdominal aortic aneurysm is having surgical repair. What is the correct classification for this surgery? a. Restorative b. Emergent c. Urgent d. Minor

B

A patient with gastroesophageal reflux disease (GERD) is on a medication that raises the pH of gastric contents. Which drug does the nurse expect to administer? a. Ranitidine b. Mylanta c. Gaviscon d. Omeprazole

B

The nurse has provided postoperative teaching for a patient who underwent a laparoscopic Nissen fundoplication. Which statement by the patient indicates a need for additional teaching? a. "I will walk every day." b. "I will no longer need the anti-reflux drugs after the surgery." c. "I will report a fever above 101°F." d. "I'll remove the gauze dressing 2 days after surgery and shower."

B

The nurse is evaluating electrolyte values for a patient with acute pancreatitis and notes that the serum calcium is 6.8 mEq/L. How does the nurse interpret this finding? a. Within normal limits considering the diagnosis of acute pancreatitis b. A result of the body not being able to use bound calcium c. A protective measure that will reduce the risk of complications d. Full compensation of the parathyroid gland

B

The nurse is performing an oral assessment on a patient and notes white plaque-like lesions on the tongue, palate, pharynx, and buccal mucosa. When the patches are wiped away, the underlying surface is red and sore. What disorder does the nurse suspect the patient has? a. Leukoplakia b. Candida albicans c. Erythroplakia d. Kaposi's sarcoma

B

The nurse is teaching a patient and family how to prevent exacerbations of chronic pancreatitis. Which teaching point does the nurse include? a. Moderation in the use of caffeinated beverages b. Avoidance of alcohol and nicotine c. Consume a bland, high-fat, low-protein diet d. Regular exercise, emphasizing aerobic activities

B

The nurse is caring for a patient with gastroenteritis who has frequent stools. Which task is best to delegate to the unlicensed assistive personnel (UAP)? a. Teach the patient to avoid use of toilet paper and harsh soaps. b. Instruct the patient on how to take a sitz bath. c. Use a warm washcloth to remove stool from the skin. d. Dry the skin with absorbent cotton.

C

The nursing student is caring for a patient with chronic pancreatitis who is receiving pancreatic enzyme replacement therapy. Which statement by the student indicates the need for further study concerning this therapy? a. "The enzymes will be administered with meals." b. "The patient will take the drugs with a glass of water." c. "If the patient has difficulty swallowing the enzyme preparation, I will crush it and mix it with foods." d. "The effectiveness of pancreatic enzyme treatment is monitored by the frequency and fat content of stools.

C

The patient develops abdominal bloating along with nausea and vomiting. Which complication of peptic ulcer disease (PUD) does the nurse recognize? a. Perforation b. Hemorrhage c. Pyloric obstruction d. Intractable PUD

C

The patient is scheduled to have minimally invasive surgery (MIS) for a laparoscopic cholecystectomy. Part of this surgery is the injection of air (insufflation) into the abdomen to separate and better see the organs. What patient teaching must the nurse do about the insufflation? a. "Your surgeon will make several small incisions instead of a large one." b. "You will be able to go home once your surgery is completed and you are awake." c. "You may experience some abdominal discomfort from the air injected with the surgery." d. "You will have a tube for drainage for a few days after your surgery is completed.

C

The patient is transferred to the operating room for right foot surgery. How does the nurse safely assure the patient's identification? a. Ask the patient what type of surgery is scheduled. b. State the patient's name and ask the patient if he or she is that person. c. Check the patient's armband and ask the patient to state his or her name and birth date. d. Check the patient's chart and armband to assure that these match.

C

The patient received moderate sedation (conscious sedation) intravenously prior to a bronchoscopy procedure. Before allowing the patient to have oral liquids, what must the nurse assess in this patient? a. The patient is arousable. b. The patient is able to speak. c. The patient's gag reflex is working. d. The patient is able to rotate his head.

C

The patient recovering from surgery for peritonitis tells the nurse that he is experiencing abdominal pain and has devel- oped foul-smelling drainage from his wound, and his incision is red and swollen. What is the nurse's best first action? a. Clean and dress the incision. b. Measure the patient's abdominal girth. c. Notify the health care provider. d. Place the patient on bedrest in semi-Fowler's position.

C

What is the name of the first 12 inches of the small intestine? a. Jejunum b. Ileum c. Duodenum d. Esophagus

C

What key assessment data would the nurse expect to find in a patient with peritonitis? a. Fever and headache b. Dizziness with nausea and vomiting c. Abdominal pain, distention, and tenderness d. Nausea and loss of appetite

C

When a patient has an oral cavity disorder, what is the patient's priority medical-surgical concept? a. Nutrition b. Gas exchange c. Tissue integrity d. Comfort

C

Which activity of a nutritional screening can be delegated to the unlicensed assistive personnel (UAP)? a. Review of the patient's nutritional history b. Review of the patient's laboratory data c. Obtaining the patient's height and weight d. Psychosocial assessment of the patient

C

Which drug would the health care provider prescribe to treat H. pylori infection? a. Ranitidine b. Omeprazole c. Clarithromycin d. Pantoprazole

C

Which medical condition increases a patient's risk for surgical wound infection? a. Anxiety b. Hiatal hernia c. Diabetes mellitus d. Amnesia

C

Which oral cavity tumor appears as a raised scab, primarily on the lips, and evolves to an ulcer with a raised pearly border? a. Leukoplakia b. Erythroplakia c. Basal cell carcinoma d. Kaposi's sarcoma

C

Which patient is at lowest risk for develop- ment of acute sialadenitis? a. Patient with Sjögren's syndrome b. Patient with HIV infection c. Patient with anemia d. Patient prescribed phenothiazine drugs

C

Which patient would the nurse assess as low risk for the development of gallbladder disorders? a. Patient with sickle cell anemia b. Patient who is Mexican American c. Patient who is 20 years old and male d. Patient with a history of prolonged parenteral nutrition

C

Which statement about esophageal trauma caused by chemical injury is accurate? a. Alkaline substances tend to affect the superficial mucosal lining. b. Acid burns cause deep penetrating injuries. c. Strong alkalis can cause full perforation of the esophagus within 1 minute. d. Chemical injuries damage the mouth and esophagus over a period of several hours.

C

After removal of the gallbladder, a patient experiences abdominal pain with vomiting for several weeks. What does the nurse recognize? a. Chronic cholecystitis b. Recurrence of acute cholecystitis c. Unremoved gallstones d. Postcholecystectomy syndrome

D

During an assessment of a patient with an oral tumor, the nurse notes that the patient devel- ops stridor. What functional assessment is the least important for the nurse to complete at this time? a. Ability to speak b. Gag reflex c. Quality of respirations d. Pain rating

D

The nurse has provided teaching to a patient with gastroesophageal reflux disease (GERD). Which statement by the patient indicates the teaching has been effective? a. "I will eat three meals a day." b. "I will not snack 1 hour before I go to bed." c. "I will stay up for at least 15-30 minutes after eating dinner before going to bed." d. "I won't lift heavy objects."

D

The nurse is assessing a patient with viral gastroenteritis. Which data is the nurse most concerned about? a. Orthostatic blood pressure changes b. Poor skin turgor c. Dry mucous membranes d. Rebound tenderness

D

What test result is the tumor marker for cancers of the liver? a. Decreased alkaline phosphatase b. Increased serum ammonia c. Decreased serum total bilirubin d. Increased alpha-fetoprotein (AFP)

D

_____: A severe acute and often fatal form of hepatitis caused by failure of the liver cells to regenerate, with progression to necrosis.

Fulminant hepatitis

_____: The process of chewing.

Mastication

_____: A layer of membrane; also used to refer to skinfold areas in the obese patient.

Panniculus

_____: An infection in the normally sterile urinary system. The unobstructed and complete passage of urine from the renal and urinary systems is critical in maintaining a sterile urinary tract. When any structural abnormality is present, the risk for damage as a result of infection is greatly increased.

Urinary tract infection (UTI)

The nurse is caring for a patient with abdomi- nal pain. While assessing the patient, which questions will the nurse ask the patient? Select all that apply. a. "Is the pain burning, gnawing, or stabbing?" b. "Can you point to where you feel the pain?" c. "Do you have a family history of cancer?" d. "When did you first notice the pain?" e. "Have you gained weight since the pain began?" f. "Does the pain spread anywhere?"

A,B,D,F

Which body structures are located in the RUQ of the abdomen? Select all that apply. a. Duodenum b. Liver c. Stomach d. Spleen e. Gallbladder f. Pancreas head

A,B,E,F

During surgery, what things do anesthesia personnel monitor, measure, and assess? Select all that apply. a. Intake and output b. Room temperature c. Cardiopulmonary function d. Level of anesthesia e. Family concerns f. Vital signs

A,C,D,F

Which are the most common signs of colorectal cancer (CRC)? Select all that apply. a. Change in stool consistency b. Absent bowel sounds c. Abdominal cramping d. Anemia e. Rectal bleeding f. Gas pains

A,D,E

A client is scheduled for surgery at noon. The surgeon is delayed and the surgery is now scheduled for 3:00 PM. How will the nurse plan to administer the preoperative prophylactic antibiotic? A) Give at noon as originally prescribed. B) Cancel orders; preoperative prophylactic antibiotics are given optionally. C) Adjust the administration time to be given within one hour prior to surgery. D) Hold the preoperative antibiotic so it can be administered immediately following surgery.

ANS: C. According to the Surgical Care Improvement Project (SCIP) guidelines, prophylactic antibiotics should be given within one hour before the surgical incision.

The nurse writes the problem "risk for impaired skin integrity" for a client with a sigmoid colostomy. Which expected outcome would be appropriate for this client? 1. The client will have intact skin around the stoma. 2. The client will be able to change the ostomy bag. 3. The client will express anxiety about the body changes. 4. The client will maintain fluid balance.

ANSWER: 1. 1. Intact skin around the stoma is the most appropriate outcome for the problem of "impaired skin integrity." 2. The client's ability to change the ostomy bag is a goal for a knowledge-deficit problem or self-care. 3. Expressing anxiety about the body changes is a goal for an alteration in body image. 4. Maintaining a balance in fluid is a goal for a nursing diagnosis of risk for fluid deficit.

Which information should the nurse teach the client post-barium enema procedure? 1. The client should not eat or drink anything for four (4) hours. 2. The client should remain on bedrest until the sedative wears off. 3. The client should take a mild laxative to help expel the barium. 4. The client will have normal elimination color and pattern.

ANSWER: 3. 1. The client may resume the regular diet. 2. The client will not be sedated for this procedure; therefore, the client does not need to be on bedrest. 3. The nurse needs to teach the client to take a mild laxative to help evacuate the barium and return to the client's normal bowel routine. Failure to pass the barium could cause constipation when the barium hardens. 4. The client can expect to pass white- or lightcolored stools until the barium has completely been evacuated.

A client is experiencing an attack of acute pancreatitis. Which nursing intervention is the highest priority for this client? A) Measure intake and output every shift. B) Do not administer food or fluids by mouth. C) Administer opioid analgesic medication. D) Assist the client to assume a position of comfort.

Answer: C. Pain relief is the highest priority for the client with acute pancreatitis.Although measuring intake and output, NPO status, and positioning for comfort are all important, they are not the highest priority.

The nursing team consists of an RN, an LPN/LVN, and a nursing assistant. Which client should be assigned to the RN? A) Client who is taking lactulose and has diarrhea B) Client with hepatitis C who requires a dressing change C) Client with end-stage cirrhosis who needs teaching about a low-sodium diet D) Obtunded client with alcoholic encephalopathy who needs a blood draw

Answer: C. The client with end-stage cirrhosis would be assigned to the RN. The RN is responsible for client teaching.Assisting a client with toileting and recording stool number and amount can be accomplished by nonprofessional staff. The LPN/LVN can provide dressing changes. Ancillary staff can perform venipuncture.

What pain management does a client who has been admitted to the post-anesthesia care unit typically receive? A) Intramuscular nonopioid analgesics. B) Intramuscular opioid analgesics. C) Intravenous nonopioid analgesics. D) Intravenous opioid analgesics.

Answer: D. Intravenous (IV) opioid analgesics are given in small doses to provide pain relief, but not to mask an anesthetic reaction.Intramuscular nonopioid analgesics and opioid analgesics are too long-acting. IV nonopioid analgesics usually are not given within the first 48 hours after surgery.

As the unit nurse is about to give a preoperative medication to a client going into surgery, it is discovered that the surgical consent form is not signed. What does the nurse do after verifying the procedure with the client? A) Calls the surgeon. B) Calls the anesthesiologist. C) Gives the medication as ordered. D) Asks the client to sign the consent form.

Answer: D. The unit nurse will ask the client to sign the consent form, after which the medication can be administered.Calling the surgeon or the anesthesiologist is not necessary. It is illegal for the client to sign the permit after being sedated.

The nurse is using capnometry testing to check NG tube placement prior to medication administration and the capnometry test is positive for carbon dioxide. What action does the nurse take? a. Administer the medication orally. b. Remove the NG tube. c. Administer the medication through the NG tube. d. Verify placement by auscultation.

B

The nurse would include which intervention in the plan of care for a male patient who has had an inguinal herniorrhaphy? a. Apply a warm pack to the scrotum. b. Elevate the scrotum on a pillow. c. Encourage use of a bedpan to void. d. Decrease fluid intake to decrease bladder emptying.

B

Which nursing intervention is part of nonsurgi- cal management for a patient with peritonitis? a. Monitor weekly weight and intake and output. b. Insert a nasogastric tube to decompress the stomach. c. Order a breakfast tray when the patient is hungry. d. Administer NSAIDs for pain.

B

Which physiologic factor contributes to gastroesophageal reflux disease (GERD)? a. Accelerated gastric emptying b. Irritation from reflux of stomach contents c. Competent lower esophageal sphincter d. Increased esophageal clearance

B

Which statement by a patient indicates the need for additional teaching about oral care? A. "I will use a soft-bristled toothbrush to clean my teeth and gums." B. "I will brush my teeth with a toothpaste with sodium lauryl sulfate." C. "I will rinse my mouth with a sodium bicarbonate solution." D. "I will read the labels of mouthwash bottles for alcohol content."

B

Which statement is true about drug therapy for Crohn's disease (CD) or ulcerative colitis (UC)? a. Infliximab is used to manage episodes of diarrhea with CD. b. Sulfasalazine is the first aminosalicylate approved for UC. c. Metronidazole has been helpful in patients with fistulas and UC. d. Adalimumab is a glucocorticoid approved for the treatment of CD.

B

Why is patient compliance higher with the fecal immunochemical test (FIT) than with the traditional fecal occult blood test (FOBT)? a. Anticoagulants, such as warfarin, and NSAIDs should be discontinued for 7 days before FIT testing begins. b. Drugs and food do not interfere with the FIT test results. c. Raw fruits and vegetables and red meat must be avoided before the FIT test. d. Vitamin C-rich foods, juices, and tablets must also be avoided before the FIT test

B

A patient reports decreased appetite over the past month along with simultaneous decreased intake or oral nutrition as well as episodes of nausea. What priority information should the nurse inquire about next? a. Favorite foods b. Bowel pattern c. Patient weight d. Vital signs

C

How many vaccine injections does a health care worker usually need to be protected with the hepatitis B vaccine? a. 1 b. 2 c. 3 d. 4

C

If a patient experiences a wound dehiscence, which description best characterizes what is happening with the wound? a. Purulent drainage is present at incision site because of infection. b. Extreme pain is present at incision site. c. A partial or complete separation of outer layers is present at incision site. d. The inner and outer layers of the incision are separated.

C

The nurse is instructing a patient about home care after an exploratory laparotomy for peritonitis. Which statement by the patient indicates that teaching has been effective? a. "It is normal for the incision site to be warm." b. "I will stop taking the antibiotics if diar- rhea develops." c. "I will call the health care provider for a temperature greater than 101°F (38.3°C)." d. "I will resume activity with my bowling league this week for exercise."

C

The patient who received moderate sedation with midazolam appears to be overly sedated and has respiratory depression. Which drug does the nurse prepare to administer to this patient? a. Lorazepam b. Naloxone c. Flumazenil d. Butorphanol tartrate

C

Which group of drugs is the main treatment for severe gastroesophageal reflux disease (GERD)? a. Antacids b. Histamine receptor agonists c. Proton pump inhibitors d. Gaviscon preparations

C

_____: The presence of an excessive amount of gas in the stomach or intestines.

Flatulence

_____: The surgical repair of a hernia.

Herniorrhaphy

After a patient is prepared for surgery and before preoperative drugs are given and the patient is transported to surgery, which essential intervention can the nurse delegate to the unlicensed assistive personnel (UAP) at this time? A. Assist the patient to empty his or her bladder. B. Help the patient to remove all clothing. C. Ask the patient if he or she wants to brush teeth. D. Recheck the patient's identity.

A

The nurse has provided teaching to a patient on ways to prevent the recurrence of aphthous ulcers. Which statement by the patient indi- cates teaching has been effective? a. "I will rinse with the tetracycline syrup for 2 minutes and then swallow the syrup." b. "Potatoes have nothing to do with the development of the ulcers." c. "I will continue to eat peanut butter and jelly sandwiches." d. "It doesn't matter what types of foods I eat as long as I brush my teeth after every meal."

A

The nurse is administering ketorolac to a 78-year-old patient for mild to moderate biliary pain management. Which assessment finding indicates the patient is experiencing a side effect of this drug? a. Gastrointestinal upset b. Ventricular cardiac dysrhythmias c. Decreased urinary output d. Jaundice

A

The nurse is caring for a patient with esophageal cancer who is scheduled to undergo an esopha- gogastrostomy with a section of the jejunum to replace the esophagus. Which procedure does the nurse expect to perform preoperatively? a. Complete bowel preparation b. Abdominal shave c. Urinary catheter placement d. Nasogastric tube placement for feeding

A

The nurse is teaching a patient with cirrhosis about lactulose therapy. Which statement by the patient indicates the teaching has been effective? a. "This therapy will promote the removal of ammonia in my stool." b. "Constipation is a frequent side effect of this therapy." c. "I will know the therapy is working when I am less itchy." d. "The drug tastes bitter and is watery."

A

The nurse is teaching a young woman about cirrhosis prevention by limiting alcohol intake. What is the nurse's best advice? a. "As few as two or three drinks per day over 10 years can lead to cirrhosis." b. "You should be all right as long as you drink less than five drinks per day." c. "Binge drinking, rather than drinking every day, reduces your risk for hepatitis or fatty liver." d. "The amount of alcohol that causes cirrhosis does not vary by gender."

A

The patient has alopecia. Which nutritional deficiency is the likely cause? a. Zinc b. Vitamin A c. Riboflavin d. Vitamin C

A

The patient is receiving intermittent feedings of a specified amount at specified times through a feeding tube. Which type of feeding is the patient receiving? a. Bolus feeding tube b. Continuous feeding tube c. Cycle feeding tube d. Gravity tube feeding

A

The patient is scheduled for a plain abdominal x-ray in the morning. What preprocedure teaching does the nurse provide? a. "Wear a hospital gown and remove any jewelry or belts." b. "You will receive nothing by mouth from midnight until after the procedure." c. "We will place a nasogastric tube before the procedure to decompress your stomach." d. "A laxative will be given to move stool out of your bowel before the procedure."

A

What is the priority nursing intervention for a patient who is malnourished? a. Determine the patient's food preferences. b. Provide the patient with high-calorie, high-protein food. c. Weigh the patient. d. Offer the patient snacks.

A

Which NG tubes can be connected to low continuous suction? A. Salem sump b. Levin c. Anderson d. Carney

A

Which description best defines an anal fissure? a. Perianal tear that can be very painful b. Duct obstruction and infection c. Communicating tract d. Localized area of induration with pus

A

Which factor renders a patient the least likely to benefit from extracorporeal shock wave lithotripsy (ESWL) for the treatment of gallstones? a. Height 5 feet 10 inches, 325 lbs b. Cholesterol-based stones c. Height 5 feet 7 inches, 138 lbs d. Small gallstones

A

Which indicator of return to consciousness occurs first as a patient recovers from general anesthesia? a. Muscular irritability b. Restlessness and delirium c. Recognition of pain. d. Ability to reason and control behavior

A

Which intervention by the nurse will help a postoperative patient with compliance in getting up to ambulate? a. Offer the patient pain medication 30-45 minutes before ambulation. b. Assist the patient to turn from side to side every 2 hours. c. Remind the patient to perform extremity exercises every 4 hours. d. Teach the patient that activity helps prevent postoperative complications.

A

Which intervention is contraindicated in the nonsurgical management of hemorrhoids? a. Diets low in fiber and fluids b. Dibucaine ointment c. Warm sitz baths three or four times a day d. Cleansing the anal area with moistened cleaning tissues

A

Which intervention should the nurse delegate to the unlicensed assistive personnel (UAP) when caring for a patient with cirrhosis experiencing pruritus? a. Apply lotion to soothe the patient's skin. b. Use lots of soap and hot water to cleanse the skin. c. Assess the patient for signs of skin infection. d. Encourage the patient to use distraction to avoid scratching.

A

Which is a priority concept for a patient with a noninflammatory intestinal disorder? a. Elimination b. Nutrition c. Fluid and electrolyte balance d. Comfort

A

Which key feature does the nurse most likely find when performing a physical assessment on a patient with a small bowel obstruction? a. Visible peristaltic waves in the upper and middle abdomen b. Minimal or no vomiting c. No major fluid and electrolyte imbalances d. Metabolic acidosis

A

Which lab value is usually low in patients with malabsorption, liver disease, pernicious anemia, terminal cancer, and sepsis? a. Cholesterol b. Hematocrit c. Hemoglobin d. Albumin

A

Which laboratory test result indicates permanent immunity to hepatitis A? a. Immunoglobulin G (IgG) antibodies b. Immunoglobulin M (IgM) antibodies c. A positive enzyme-linked immunosorbent assay (ELISA) d. The presence of anti-HAV antibodies

A

Which medication is the drug of choice for the treatment of IBS when pain is the predominant symptom? a. Amitriptyline b. Fesoterodine c. Loperamide d. Psyllium hydrophilic mucilloid

A

Which members of the surgical team usually accompany a postoperative patient to the postanesthesia care unit (PACU)? a. Anesthesia provider and circulating nurse b. Circulating nurse and surgeon c. Surgeon and anesthesia provider d. Surgical assistant and surgeon

A

Which nursing intervention is most appropriate for the patient in the operative setting? a. Provide a climate of privacy, comfort, and confidentiality when caring for the patient. b. Instruct the patient that after the preoperative medication has taken effect, he or she will be drowsy. c. Avoid discussing the activities taking place around the patient while in the holding area. d. Assist members of the surgical team readying the operating room suite.

A

Which nursing interventions will prevent the potential intraoperative complication of radial nerve complications (wrist drop)? a. Support the wrist with padding; do not overtighten wrist straps. b. Place pillows or foam padding under bony prominences, maintain good body alignment, and slightly flex joints and sup- port with pillows and pads. c. Pad the elbow, avoid excessive abduction, and secure the arm firmly on an arm board positioned at shoulder level. d. Place a safety strap above or below the area. Place a pillow or padding under the knees.

A

Which question would the nurse be sure to ask a patient with suspected leukoplakia? a. "Do you smoke, dip, or chew tobacco products?" b. "How much alcohol do you drink each day?" c. "Do you eat a lot of fast food meals?" d. "How often do you have dental checkups?"

A

Which statement about diverticular disease is true? a. Most diverticula occur in the sigmoid colon. b. Diverticula are uncomfortable even when not inflamed. c. High-fiber diets contribute to diverticula occurrence. d. Diverticula form where intestinal wall muscles are weak.

A

The nurse is assisting a patient who follows a lacto-ovovegetarian diet to fill out a menu. Based on this diet, which foods could the patient select for breakfast? Select all that apply. a. Milk b. Scrambled eggs c. Toast d. Sausage e. Cereal f. Tuna fish

A,B,C,E

A patient with decompensated cirrhosis is at risk for which complications? Select all that apply. a. Jaundice b. Esophageal varices c. Coagulation defects d. Hepatitis A virus (HAV) e. Spontaneous bacterial peritonitis f. Ascites

A,B,C,E,F

The nurse is caring for a patient who received a barium swallow with a small bowel follow- through. What key points must the nurse include in teaching this patient after the procedure? Select all that apply. a. "Drink lots of fluids." b. "Depending on the results, you may need a colonoscopy." c. "You will be on bedrest for about 6-8 hours." d. "A laxative will be provided to help remove the barium." e. "Your stools will be chalky white for 1-3 days." f. "Be sure to tell the health care provider if you are allergic to seafood or iodine."

A,B,D,E

The postanesthesia care unit (PACU) nurse is receiving the "handoff " report for a patient transferred in from the OR. Which statements about this report are accurate? Select all that apply. a. A handoff report requires clear, concise language. b. A handoff report is a two-way verbal interaction between the health care professional giving the report and the nurse receiving it. c. A handoff report should be individualized based on the patient and his or her surgery. d. The receiving nurse takes the time to restate (report back) the information to verify what was said. e. The receiving nurse takes the time to ask questions, and the reporting professional must respond to the questions until a common understanding is established. f. The receiving nurse continues assessing other patients while the handoff report is being given.

A,B,D,E

Which statements about a patient with cirrhosis and esophageal varices are accurate? Select all that apply. a. All patients with cirrhosis should be screened for esophageal varices to detect them before they bleed. b. Bleeding esophageal varices are a medical emergency. C. Esophageal balloon tamponade is often used to control bleeding esophageal varices. D. A nonselective beta blocker such as propranolol is prescribed to prevent varices from bleeding. E. Bleeding esophageal varices can be managed by use of endoscopic variceal ligation. F. The bleeding appears as dark coffee grounds in emesis or stool

A,B,D,E

Which interventions are necessary to provide safe, quality care to a patient receiving enteral tube feeding? Select all that apply. a. Check the residual volume every 4-6 hours. b. Change the feeding bag and tubing every 12 hours. c. Keep the head of the bed elevated at least 30 degrees. d. Use clean technique when changing the feeding system. e. Allow closed system containers to hang for 24 hours. f. Monitor for complications especially constipation.

A,C,D,E

Which statements about Barrett's esophagus are accurate? Select all that apply. a. It is considered to be a premalignant condition. b. It is associated with excessive intake of fresh fruits and vegetables. c. It results from exposure to acid and pepsin. d. It is associated with pickled and fermented foods. e. Normal cells undergo dysplasia to become cancerous. f. It is more common in younger adults.

A,C,D,E

When preparing a patient for paracentesis, what does the nurse do? Select all that apply. a. Ask the patient to void before the procedure. b. Place the patient in the supine position. c. Weigh the patient before the procedure. d. Obtain the patient's heart rate. e. Assess the patient's respiratory rate. f. Obtain the patient's blood pressure.

A,C,D,E,F

Which prescribed drugs can contribute to weight gain when they are taken on a long-term basis? Select all that apply. a. Estrogens b. Acetaminophen c. Corticosteroids d. Nonsteroidal antiinflammatory drugs (NSAIDs) e. Antiepileptics f. Antidepressants

A,C,D,E,F

Which are implied with informed consent? Select all that apply. a. The patient understands the nature of and the reason for surgery. b. The patient is informed of what type of anesthesia drugs will be used. c. The patient understands who will do the surgery and who will be present during surgery. d. The patient understands the risks associated with the surgical procedure and its potential outcomes. e. The patient understands that blood and blood products must be available during surgery. f. The patient is informed of all available options and the benefits and risks associated with each option.

A,C,D,F

Which factors may lead to hepatic encephalopathy in patients with cirrhosis? Select all that apply. a. High-protein diet b. Hypervolemia c. Infection d. Constipation e. Hyperkalemia

A,C,D,F

The public health nurse is teaching day-care workers. Which type of hepatitis is transmitted by the fecal-oral route via contaminated food, water, or direct contact with an infected person? 1. Hepatitis A. 2. Hepatitis B. 3. Hepatitis C. 4. Hepatitis D.

ANSWER: 1. 1. The hepatitis A virus is in the stool of infected people and takes up to two (2) weeks before symptoms develop. 2. Hepatitis B virus is spread through contact with infected blood and body fluids. 3. Hepatitis C virus is transmitted through infected blood and body fluids. 4. Hepatitis D virus only causes infection in people who are also infected with hepatitis B or C. TEST-TAKING HINT: This is a knowledge question; the nurse must be aware of how the various types of hepatitis virus are transmitted.

The respiratory problems that may accompany peritonitis are a result of which factor? a. Associated pain interfering with ventilation b. Decreased pressure against the diaphragm c. Fluid shifts to the thoracic cavity d. Decreased oxygen demands related to the infectious process

A

The student nurse is caring for a patient with cirrhosis. Which action by the student nurse causes the supervising nurse to intervene? a. Uses a straight-edge razor to shave the patient. b. Monitors for orthostatic changes of blood pressure. c. Avoids intramuscular injections. d. Uses a toothette for oral care.

A

The student nurse is performing a gastric lavage on a patient with an active upper GI bleed. Which action by the student requires intervention by the supervising nurse? a. Using an ice-cold solution to perform lavage of the stomach b. Instilling the lavage solution in volumes of 200-300 mL c. Continuing the lavage until the solution re- turned is clear or light pink without clots d. Positioning the patient on his left side during the procedure

A

The nurse is assessing a patient's mouth for lesions. Which actions will the nurse implement? Select all that apply. A. Wear clean gloves. B. Assure adequate lighting with a penlight. C. Ask the patient to say "ahh." D. Use a tongue blade. E. Instruct the patient to perform the Valsalva maneuver. F. Assess for lesions, coatings, and cracking.

A,B,D,F

When assessing a patient with a salivary gland tumor, the nurse pays particular attention to the facial nerve. Which requests by the nurse are likely to determine if the tumor has affected the facial nerve? Select all that apply. a. "Puff out your cheeks." b. "Wrinkle your nose." c. "Cough." d. "Raise your eyebrows." e. "Turn your head back and forth." f. "Pucker your lips."

A,B,D,F

Which abnormal laboratory findings are cardinal findings in acute pancreatitis? Select all that apply. a. Elevated serum lipase b. Increased serum amylase c. Decreased serum trypsin d. Elevated serum elastase e. Decreased serum glucose f. Elevated bilirubin

A,B,D,F

Which are pathologic changes associated with acute gastritis? Select all that apply. a. Vascular congestion b. Severe mucosal damage and ruptured vessels. C. Autodigestion. D. Acute inflammatory cell infiltration. E. Increased cell production in the superficial epithelium of the stomach lining. F. Edema.

A,B,D,F

Which are recommended prevention strategies for oral cancer? Select all that apply. a. Minimize sun exposure. b. Stop using tobacco. c. Avoid intake of fatty foods. d. Decrease alcohol intake. e. Exercise 3-5 days per week. f. Avoid use of tanning beds.

A,B,D,F

Which findings does the nurse expect for a postoperative colostomy patient? Select all that apply. a. Reddish-pink, moist stoma b. Small amount of bleeding c. Large amount of stoma swelling d. Mucocutaneous separation e. Smooth, intact peristomal skin f. Excoriation immediately around the stoma

A,B,E

The nurse is teaching a patient with cirrhosis about nutrition therapy. Which key points must the nurse include? Select all that apply. a. Do not use table salt. b. Adding salt when cooking is acceptable. c. Eat small frequent meals. d. Drink supplemental liquids such as Ensure. e. Be sure to take a multivitamin every day. f. Avoid foods that are rich in vitamin K.

A,C,D,E

The nurse is caring for four clients who will undergo surgery today. Which client does the nurse recognize as at highest risk for surgical complication? A) 52-year-old who takes aspirin daily B) 58-year-old who has well-controlled Type II diabetes C) 64-year-old who has just received pre-surgical prophylactic antibiotics D) 69-year-old who will be discharged after surgery to an extended care facility

ANS: A. Aspirin and NSAIDs taken before surgery may increase clotting time and risk for hemorrhage.

Upon assessment of a client with GERD, which statement requires nursing intervention? A) "I quit smoking several years ago." B) "Sometimes I wake up gasping for air in the middle of the night." C) "My family likes to eat small meals every 3 to 4 hours throughout the day." D) "When I buy meat, I ask for the leanest cut that is available."

ANS: B. Gasping for air upon waking in the middle of the night can be a sign of sleep apnea. Often patients who have one condition (sleep apnea or GERD) also experience the other.

The nurse is caring for a client who has just been prescribed a glucocorticoid to treat an exacerbation of ulcerative colitis. What teaching will the nurse provide? A) Decrease the drug dose during the next exacerbation. B) Report fever to healthcare provider immediately. C) Determine if the client's insurance covers payment for this medication. D) This drug will act as an antidiarrheal.

ANS: B. Glucocorticoid medication, a form of steroid therapy, can cause immunosuppression (not act as an antidiarrheal); therefore, any sign of infection should be immediately reported to the healthcare provider. The drug dose may be increased by the health care provider during the next exacerbation. The client will determine with the insurance provider (and possibly a social worker) whether the drug is covered for payment.

The nurse is teaching a client recovering from a laparoscopic cholecystectomy. Which statement indicates the discharge teaching is effective? 1. "I will take my lipid-lowering medicine at the same time each night." 2. "I may experience some discomfort when I eat a high-fat meal." 3. "I need someone to stay with me for about a week after surgery." 4. "I should not splint my incision when I deep breathe and cough."

ANSWER: 2. 1 This surgery does not require lipid-lowering medications, but eating high-fat meals may cause discomfort. 2. After removal of the gallbladder, some clients experience abdominal discomfort when eating fatty foods. 3. Laparoscopic cholecystectomy surgeries are performed in day surgery, and clients usually do not need assistance for a week. 4. Using a pillow to splint the abdomen provides support for the incision and should be continued after discharge. TEST-TAKING HINT: When answering questions stating "teaching is effective," the test taker should look for the correct information. Basic concepts should help the test taker answer questions, and because pain often occurs after surgeries, option "2" would probably be a correct answer.

The nurse is admitting a client diagnosed with protein calorie malnutrition. Which interventions should the nurse implement? Select all that apply. 1. Place the client on a 72-hour calorie count. 2. Ask the client to describe the stools. 3. Have the UAP weigh the client. 4. Obtain a list of current medications. 5. Make a referral to the dietitian.

ANSWER: 1, 2, 3, 4, 5. 1. The nurse should assess the client's intake; a 72-hour calorie count will allow the nurse to do this. 2. Protein calorie malnutrition can result from several different diseases. Diarrhea can impart the ability to absorb calories and nutrition from food. 3. Daily weights will monitor the client's weight loss or gain. 4. The nurse should assess medications for drug and food interactions. 5. The dietitian can be invaluable in assisting this client to gain or at least maintain weight TEST-TAKING HINT: To answer "select all that apply" questions, the test taker must read each option as a true/false question. One option does not rule out another one.

The client in end-stage liver failure has vitamin K deficiency. Which interventions should the nurse implement? Select all that apply. 1. Avoid rectal temperatures. 2. Use only a soft toothbrush. 3. Monitor the platelet count. 4. Use small-gauge needles. 5. Assess for asterixis.

ANSWER: 1, 2, 3, 4. 1. Vitamin K deficiency causes impaired coagulation; therefore, rectal thermometers should be avoided to prevent bleeding. 2. Soft-bristle toothbrushes will help prevent bleeding of the gums. 3. Platelet count, partial thromboplastin time/prothrombin time (PTT/PT), and international normalized ratio (INR) should be monitored to assess coagulation status. 4. Injections should be avoided, if at all possible, because the client is unable to clot, but if they are absolutely necessary, the nurse should use small-gauge needles. 5. Asterixis is a flapping tremor of the hands when the arms are extended and indicates an elevated ammonia level not associated with vitamin K deficiency. TEST-TAKING HINT: The test taker must know the function of specific vitamins. Vitamin K is responsible for blood clotting. This is an alternate-type question, which requires the test taker to select all applicable interventions; the test taker should select interventions addressing bleeding.

The 70-year-old client is admitted to the medical unit diagnosed with acute diverticulitis. Which interventions should the nurse implement? Select all that apply. 1. Tell the client not to eat or drink. 2. Start an intravenous line. 3. Assess the client for abdominal tenderness. 4. Have the dietitian consult for a low-residue diet. 5. Place the client on bedrest with bathroom privileges.

ANSWER: 1, 2, 3, 5. 1. The client should remain NPO until the inflammation in the colon resolves. 2. The client should have an IV to maintain hydration while being NPO. 3. The nurse should assess the client for complications of a ruptured diverticulum. 4. The client will be NPO to rest the bowel. 5. The client is kept on bedrest with bathroom privileges to decrease colon activity. Ambulation increases peristalsis. TEST-TAKING HINT: The test taker must make decisions based on basic principles. If the client has a diagnosis of a gastrointestinal disease, the treatment will depend on allowing the bowel to rest. When choosing options for a "select all that apply" question, each option is treated as a true/false question.

Which complaint is significant for the nurse to assess in the adolescent male client who uses oral tobacco? 1. The client complains of clear to white sputum. 2. The client has an episodic blister on the upper lip. 3. The client complains of a nonhealing sore in the mouth. 4. The client has bilateral ducts at the second molars.

ANSWER: 1, 3. 1. Clear to white sputum is not significant in the client using oral tobacco. 2. Episodic blisters on the lips are herpes simplex 1 and are not specific to this client. 3. Presence of any nonhealing sore on the lips or mouth may be oral cancer. Oral cancer risk increases by using oral tobacco. 4. Bilateral Stensen's ducts visible at the site of the second molars are normal assessment data

Which oral medication should the nurse question before administering to the client with peptic ulcer disease? 1. E-mycin, an antibiotic. 2. Prilosec, a proton pump inhibitor. 3. Flagyl, an antimicrobial agent. 4. Tylenol, a nonnarcotic analgesic

ANSWER: 1. 1. E-mycin is irritating to the stomach, and its use in a client with peptic ulcer disease should be questioned. 2. Prilosec, a proton pump inhibitor, decreases gastric acid production, and its use should not be questioned by the nurse. 3. Flagyl, an antimicrobial, is administered to treat peptic ulcer disease secondary to H. pylori bacteria. 4. Tylenol can be safely administered to a client with peptic ulcer disease. TEST-TAKING HINT: The test taker needs to understand how medications work, adverse effects of medications, when to question administering a specific medication, and how to administer the medication safely. By learning classifications, the test taker should be able to make a knowledgeable selection in most cases.

The client has dark, watery, and shiny-appearing stool. Which intervention should the nurse implement first? 1. Check for a fecal impaction. 2. Encourage the client to drink fluids. 3. Check the chart for sodium and potassium levels. 4. Apply a protective barrier cream to the perianal area.

ANSWER: 1. 1. This is a symptom of diarrhea moving around an impaction higher up in the colon. The nurse should assess for an impaction when observing this finding. 2. Encouraging the client to drink fluids should be done, but it is not the first intervention. 3. The sodium level is usually not a problem for clients experiencing diarrhea, but the potassium level may be checked. However, again, this is not the first intervention. 4. A protective cream can be applied to an excoriated perineum, but first the nurse should assess the situation. TEST-TAKING HINT: The first step of the nursing process is assessment, after which a nursing diagnosis and interventions follow. The nurse should assess first.

Which client assessment data are priority for the postanesthesia care nurse? 1. Bowel sounds. 2. Vital signs. 3. IV fluid rate. 4. Surgical site

ANSWER: 2. 1. Bowel sounds should be assessed, but it is not priority for the surgical client in the PACU. 2. The postanesthesia care unit nurse should follow the ABCs format described by the American Heart Association. "A" is for airway, "B" is for breathing, and "C" is for circulation. Vital signs assess for hemodynamic stability; this is priority in the PACU. 3. Intravenous fluids should be assessed after breathing and circulation have been assessed. 4. The surgical site should be assessed after the intravenous fluid rate is assessed.

The client is diagnosed with an acute exacerbation of ulcerative colitis. Which intervention should the nurse implement? 1. Provide a low-residue diet. 2. Rest the client's bowel. 3. Assess vital signs daily. 4. Administer antacids orally

ANSWER: 2. 1. The client's bowel should be placed on rest and no foods or fluids should be introduced into the bowel. 2. Whenever a client has an acute exacerbation of a gastrointestinal disorder, the first intervention is to place the bowel on rest. The client should be NPO with intravenous fluids to prevent dehydration. 3. The vital signs must be taken more often than daily in a client who is having an acute exacerbation of ulcerative colitis. 4. The client will receive anti-inflammatory and antidiarrheal medications, not antacids, which are used for gastroenteritis. TEST-TAKING HINT: "Acute exacerbation" is the key phrase in the stem of the question. The word "acute" should cause the test taker to eliminate any daily intervention.

The client being admitted from the emergency department is diagnosed with a fecal impaction. Which nursing intervention should be implemented? 1. Administer an antidiarrheal medication every day and prn. 2. Perform bowel training every two (2) hours. 3. Administer an oil retention enema. 4. Prepare for an upper gastrointestinal (UGI) series x-ray.

ANSWER: 3. 1. An antidiarrheal medication would slow down the peristalsis in the colon, worsening the problem. 2. The client has an immediate need to evacuate the bowel, not a need for bowel training. 3. Oil retention enemas will help to soften the feces and evacuate the stool. 4. A UGI series adds barium to the already hardened stool in the colon. Barium enemas x-ray the colon; a UGI series x-rays the stomach and jejunum. TEST-TAKING HINT: If the test taker understands fecal impaction is the opposite of diarrhea, then answer option "1" can be eliminated. Knowledge of anatomy and physiology eliminates option "4" because stool is formed in the colon and transported to the anus, part of the lower gastrointestinal tract.

Which gastrointestinal assessment data should the nurse expect to find when assessing the client in end-stage liver failure? 1. Hypoalbuminemia and muscle wasting. 2. Oligomenorrhea and decreased body hair. 3. Clay-colored stools and hemorrhoids. 4. Dyspnea and caput medusae

ANSWER: 3. 1. Hypoalbuminemia (decreased albumin) and muscle wasting are metabolic effects, not gastrointestinal effects. 2. Oligomenorrhea is no menses, which is a reproductive effect, and decreased body hair is an integumentary effect. 3. Clay-colored stools and hemorrhoids are gastrointestinal effects of liver failure. 4. Dyspnea is a respiratory effect, and caput medusae (dilated veins around the umbilicus) is an integumentary effect, although it is on the abdomen. TEST-TAKING HINT: The adjective "gastrointestinal" is the key word guiding the test taker to select the correct answer. The test taker must rule out options not addressing gastrointestinal symptoms. Although liver failure affects every body system, the question asks for a gastrointestinal effect.

Which laboratory result would require immediate intervention by the nurse for the client scheduled for surgery? 1. Calcium 9.2 mg/dL. 2. Bleeding time two (2) minutes. 3. Hemoglobin 15 g/dL. 4. Potassium 2.4 mEq/L.

ANSWER: 4. 1. This laboratory value is within normal limits. 2. This laboratory value is within normal limits. 3. This laboratory value is within normal limits. 4. This potassium level is low and should be reported to the health-care provider because potassium is important for muscle function, including the cardiac muscle. TEST-TAKING HINT: There are some items, such as normal laboratory values, the test taker must memorize. Laboratory values may differ slightly between laboratories, but the test taker must know them.

The clinic nurse is talking on the phone to a client who has diarrhea. Which intervention should the nurse discuss with the client? 1. Tell the client to measure the amount of stool. 2. Recommend the client come to the clinic immediately. 3. Explain the client should follow the BRAT diet. 4. Discuss taking an over-the-counter histamine-2 blocker

ANSWER: :3. 1. The clinic nurse should not ask the client to measure stool at home; this is done in the acute care setting. 2. Unless the client has had diarrhea for longer than 48 hours, the client does not need to be seen in the clinic. 3. The BRAT (bananas, rice, applesauce, and toast) diet is recommended for a client with diarrhea because it is low residue and produces nutrition while not irritating the GI system. 4. Histamine-2 blockers decrease gastric acid production and would not be prescribed for a client with diarrhea. TEST-TAKING HINT: The test taker should realize diarrhea involves the gastrointestinal system and selecting an intervention addressing the GI system would be an appropriate choice

A nurse is providing discharge teaching for a client wh ohas a new colostomy and is concerned about flatus and odor. Which of the following foods should the nurse recommend to the client? A) Yogurt. B) Broccoli. C) Eggs. D) Fish.

Answer: A. A) The nurse should recommend yogurt, crackers, and toast, which can prevent flatus and stool odor. B) The nurse should inform the client that broccoli, beans, and spicy foods can cause flatus. C) The nurse should inform the client that eggs, asparagus, and cabbage can contribute to odor when the colostomy pouch is open. D) The nurse should inform the client that fish can contribute to odor when the colostomy pouch is open.

What is a common gastrointestinal problem that older adults experience more frequently as they age? A) Decreased hydrochloric acid levels B) Excess lipase production C) Increased liver size D) Increased peristalsis.

Answer: A. In older adults, decreased hydrochloric acid levels (hypochlorhydria) results from atrophy of the gastric mucosa.A decrease in lipase production results from calcification of pancreatic vessels. A decrease in the number and size of hepatic cells leads to decreased liver weight and mass. Peristalsis decreases, and nerve impulses are dulled.

Which client does the charge nurse assign to an experienced LPN/LVN working on the adult medical unit? A) A 32-year-old who needs a nasogastric tube inserted for gastric acid analysis B) A 36-year-old who needs teaching about an endoscopic retrograde cholangiopancreatography C) A 40-year-old who will need administration of IV midazolam hydrochloride (Versed) during an upper endoscopy D) A 46-year-old who was recently admitted with abdominal cramping and diarrhea of unknown causes

Answer: A. Nasogastric tube insertion is included in LPN/LVN education and is an appropriate task for an experienced LPN/LVN.Assessment and client teaching would be done by an RN. IV hypnotic medications would be administered by an RN.

Which morbidly obese client is the least likely candidate for bariatric surgery? A) A 34-year-old woman experiencing mental confusion B) A 44-year-old man with a history of hypertension C) A 50-year-old woman with a history of sleep apnea D) A 52-year-old man with a history of type 1 diabetes mellitus

Answer: A. The least likely candidate is the client who is experiencing mental confusion. This client may have difficulty complying with the postoperative treatment regimen.The client with hypertension, the client with sleep apnea, and the client with diabetes are all candidates for bariatric surgery despite having these complications.

The nurse is caring for a client who has cirrhosis of the liver. The client has exhibited hand flapping and mental confusion for several weeks. Although the mental confusion is worsening, the client has stopped exhibiting hand flapping movements. How will the nurse interpret these findings? A) The client's symptoms are progressing and getting worse. B) The client's serum ammonia levels are decreasing. C) The client probably has a decrease in serum proteins. D) The client is showing signs of improvement.

Answer: A. The nurse interprets these findings as an indication that the client's is getting worse. Clients with cirrhosis who exhibit asterixis or hand flapping, may eventually stop exhibiting this sign as they worsen. The fact that the client's mental confusion is worsening indicates that this is the case.Increased mental confusion is related to elevated, not decreased, ammonia levels, as well as other serum proteins. The client is worsening, not improving.

Which electrolyte laboratory result does the nurse report immediately to the anesthesiologist? A) Creatinine, 1.9 mg/dL (168 mcmol/L). B) Fasting glucose, 80 mg/dL (4.4 mmol/L). C) Potassium, 3.9 mEq/L (3.9 mmol/L). D) Sodium, 140 mEq/L (140 mmol/L)

Answer: A. The nurse will immediately report a creatinine of 1.9 mg/dL (168 mcmol/L) to the anesthesiologist. A creatinine of 1.9 mg/dL (168 mcmol/L) is outside the normal range and may indicate renal problems.A fasting glucose of 80 mg/dL (4.4 mmol/L), a potassium level of 3.9 mEq/L (3.9 mmol/L), and sodium level of 140 mEq/L (140 mmol/L) are normal laboratory values.

A client has developed acute pancreatitis after also developing gallstones. Which is the highest priority nursing instruction for this client to avoid further attacks of pancreatitis? A) "You may need a surgical consult for removal of your gallbladder." B) "See your health care provider (HCP) immediately when experiencing symptoms of a gallbladder attack." C) "If you have a gallbladder attack and pain does not resolve within a few days, call your health care provider." D) "You'll need to drastically modify your alcohol intake."

Answer: B. The highest priority nursing instruction for the client to avoid more attacks of pancreatitis is to report symptoms of gallbladder attacks immediately to the HCP.The client may not require removal of the gallbladder. That decision is made by the HCP. The client must see the provider promptly when experiencing gallbladder disease and should not wait. Because this client's acute pancreatitis is likely related to gallstones, alcohol consumption need not be restricted.

A patient has been newly diagnosed with ulcerative colitis (UC). What does the nurse teach the patient about diet and lifestyle choices? A) "Drinking carbonated beverages will help with your abdominal distress." B) "It's OK to smoke cigarettes, but you should limit them to ½ pack per day." C) "Lactose-containing foods should be reduced or eliminated from your diet." D) "Raw vegetables and high-fiber foods may help to diminish your symptoms."

Answer: C. The nurse teaches the newly diagnosed patient with ulcerative colitis that lactose-containing foods are often poorly tolerated and need to be reduced or eliminated from the diet.Carbonated beverages are GI stimulants that can cause discomfort and must be used rarely or completely eliminated from the diet. Cigarette smoking is a stimulant that can cause GI distress symptoms. Nurses would never advise patients that any amount of cigarette smoking is "OK." Raw vegetables and high-fiber foods can cause GI symptoms in patients with UC.

A client with a history of esophageal varices has just been admitted to the emergency department after vomiting a large quantity of blood. Which action does the nurse take first? A) Obtain the charts from the previous admission. B) Listen for bowel sounds in all quadrants. C) Obtain pulse and blood pressure. D) Ask about abdominal pain.

Answer: C. When caring for a newly admitted client with esophageal varices and vomiting of blood, the nurse would first assess vital signs to detect hypovolemic shock caused by hemorrhage. Assessment for adequate perfusion is the highest priority at this time.Obtaining charts from the previous admission, assessing bowel sounds, and pain assessment can be delayed until the client has stabilized.

A nurse is assessing a client who has cirrhosis. Which of the following findings is the priority for the nurse to report to the provider? A) Peripheral edema. B) Jaundice. C) Spider angiomas. D) Bloody stools.

Answer: D. A) Peripheral edema is an expected finding for a client who has cirrhosis. Therefore, there is another finding that is the priority for the nurse to report to the provider. B) Jaundice is an expected finding for a client who has cirrhosis. Therefore, there is another finding that is the priority for the nurse to report to the provider. C) Spider angiomas are an expected finding for a client who has cirrhosis. Therefore, there is another finding that is the priority for the nurse to report to the provider. D) The greatest risk to the client who has cirrhosis of the liver is hemorrhagic shock due to bleeding in the esophageal varices. Therefore, bloody stools is the priority finding to report to the provider.

Colostomy surgery is categorized as what type of surgery? A) Cosmetic. B) Curative. C) Diagnostic. D) Palliative.

Answer: D. Colostomy surgery is categorized as palliative. Palliative surgery is performed to relieve symptoms of a disease process, but does not cure the disease.Cosmetic surgery is performed primarily to alter or enhance personal appearance. Curative surgery is performed to resolve a health problem by repairing or removing the cause. Diagnostic surgery is performed to determine the origin and cause of a disorder or the cell type for cancer.

A patient with an intestinal obstruction has pain that changes from a "colicky" intermittent type to constant discomfort. What does the nurse do first? A) Administers medication for pain B) Changes the nasogastric suction level from "intermittent" to "constant" C) Positions the patient in high-Fowler's position D) Prepares the patient for emergency surgery

Answer: D. The first action the nurse takes for a patient with intestinal obstruction whose pain changes from "colicky" intermittent type to constant discomfort is to prepare the patient for emergency surgery. The change in pain type could be indicative of perforation or peritonitis and will require immediate surgical intervention.Pain medication may mask the patient's symptoms but will not address the root cause. A change in the nasogastric suction rate will not resolve the cause of the patient's pain and could be particularly ineffective if a nonvented tube is in use. A high-Fowler's position will have no effect on an intestinal perforation or peritonitis, which this patient is likely experiencing.

A patient has an anal fissure. Which intervention most effectively promotes perineal comfort for the patient? A) Administering a Fleet's enema when needed B) Applying heat to acute inflammation for pain relief C) Avoiding the use of bulk-forming agents D) Using hydrocortisone cream to relieve pain

Answer: D. The intervention that most effectively promotes perineal comfort in a patient with anal fissure is using hydrocortisone skin cream to relieve perineal pain.Enemas would be avoided when an anal fissure is present. Cold packs would be applied to acute inflammation to diminish discomfort. Bulk-forming agents would be used to decrease pain associated with defecation.

The nurse is providing preoperative care for a client who will have an arthroscopy of the left knee. As part of the Joint Commission National Patient Safety Goals (NPSG), what will the nurse be required to do? A) Ensure that the correct procedure is noted in the client's history. B) Remind the surgeon that the client will have a left knee arthroscopy. C) Verify with the client that a left knee arthroscopy will be performed. D) Mark the left knee site with the client awake and the surgeon present.

Answer: D. The nurse will be required to mark the left knee site with the client awake and the surgeon present. The Joint Commission NSPG requires that the surgical site be marked by an independent licensed professional and should, when possible, involve the client. The surgeon is accountable and should be present.The EMR should identify the correct procedure, but is not a specific JCAHO requirement. The nurse will verify the procedure with the client when possible, but this is not a requirement. Communication with the surgeon is ideal, but is not specifically required.

The patient's potassium level is 3.1 mEq/L. Which condition would cause this value? A. Malabsorption. B. Gastric suctioning. C. Acute pancreatitis. D. Kidney failure.

B

When caloric energy is inadequate, what does the body use for energy? a. Carbohydrates b. Proteins c. Glucose d. Fats

B

A patient is suspected to have ulcerative colitis (UC). Which diagnostic tests does the nurse expect the patient to undergo to confirm the diagnosis? Select all that apply. a. Sigmoidoscopy b. C-reactive protein c. Albumin levels d. Erythrocyte sedimentation rate e. Magnetic resonance enterography (MRE) f. Clotting studies

B,C,D,E

Which are considered acute effects of radia- tion therapy? Select all that apply. a. Excessive drooling b. Stomatitis c. Herpes simplex d. Treatment-related mucositis e. Alteration in taste f. Xerostomia

B,D,E

A patient being seen the emergency depart- ment (ED) has been vomiting blood for the past 12 hours. What test will likely be ordered for the patient? a. Endoscopic retrograde cholangiopancrea- tography (ERCP) b. Upper GI radiographic series c. Esophagogastroduodenoscopy (EGD) d. Barium enema

C

A patient is in the clinic for a nonhealing sore on the lower left corner of her bottom lip and right side of her tongue. The lesions are red, raised, and have erosions. After taking a history, the nurse suspects the patient may have which type of oral cancer? A. Basal cell carcinoma. B. Kaposi's sarcoma. C. Squamous cell carcinoma. D. Erythroplakia.

C

The medical-surgical nurse is caring for a postoperative patient whose lab values reveal an increase in band cells (immature neutro- phils). What is the nurse's best interpretation of this value? a. The patient may need a transfusion. b. The patient is using up clotting factors. c. The patient is developing an infection. d. The patient's result is expected postoperatively.

C

The nurse assesses a patient with a hernia and finds that the patient's symptoms include abdominal distention, nausea, vomiting, and pain. The patient's heart rate is 118/minute and temperature is 101oF (38.3oC). Which type of hernia does the nurse suspect? a. Incisional b. Incarcerated c. Strangulated d. Umbilical

C

The nurse is assessing a postoperative patient's gastrointestinal system. What is the best indicator that peristaltic activity has resumed? a. Presence of bowel sounds b. Patient states he is hungry c. Passing of flatus or stool d. Presence of abdominal cramping

C

The patient is to receive regional anesthesia for injured knee repair surgery. Which type of regional anesthesia is this patient likely to receive? A. Field block. B. Nerve block. C. Spinal anesthesia. D. Epidural anesthesia.

C

Which statement best describes the collaborative roles of the nurse and surgeon when obtaining the informed consent? a. The nurse is responsible for having the informed consent form on the chart for the health care provider (HCP) to witness. b. The nurse may serve as a witness that the patient has been informed by the HCP before surgery is performed. c. The nurse may serve as witness to the patient's signature after the HCP has the consent form signed before preoperative sedation is given and before surgery is performed. d. The nurse has no duties regarding the consent form if the patient has signed the informed consent form for the HCP, even if the patient then asks additional questions about the surgery.

C

Which test is definitive for the diagnosis of colorectal cancer (CRC)? a. Carcinoembryonic antigen (CEA) b. Barium swallow c. Colonoscopy with biopsy d. Fecal occult blood test (FOBT)

C

Which test may be used in diagnosing IBS? a. Erythrocyte sedimentation rate b. Stool sample for ova and parasites c. Hydrogen breath test d. Blood cultures for infection

C

_____: Chronic liver inflammation that usually occurs as a result of hepatitis B or C. Superimposed infection with hepatitis D virus (HDV) in patients with chronic hepatitis B may also result in chronic hepatitis. Can lead to cirrhosis and liver cancer.

Chronic hepatits

_____: Pancreatitis that develops from inflammation, spasm, and obstruction of the sphincter of Oddi. Inflammatory and sclerotic lesions occur in the head of the pancreas and around the ducts, causing obstruction and backflow of pancreatic secretions.

Chronic obstructive pancreatitis

A 47-year-old patient is having surgery to remove kidney stones. What is the correct classification for this surgery? a. Restorative b. Emergent c. Palliative d. Urgent

D

A patient has returned to the unit after a Stretta procedure for gastroesophageal reflux disease (GERD). Which action by the student nurse requires the supervising nurse to intervene? a. The patient is offered clear liquids in the early postprocedure period. b. The patient's routine 81 mg of aspirin is held. c. A proton pump inhibitor is administered. d. A nasogastric tube is prepared for insertion.

D

A patient in a starvation state has been started on enteral feedings. The nurse assesses the patient and finds shallow respirations, weakness, acute confusion, and oozing from the IV site. What does the nurse suspect is happening in this patient? A. Septicemia b. Hypoglycemia c. Aspiration d. Refeeding syndrome

D

What is the classic symptom of malabsorption syndrome? a. Unintentional weight loss b. Decreased libido c. Bloating with flatus d. Chronic diarrhea

D

When beginning an abdominal assessment, the nurse would begin in which quadrant? a. Right lower quadrant (RLQ) b. Left lower quadrant (LLQ) c. Left upper quadrant (LUQ) d. Right upper quadrant (RUQ)

D

Which acid-base abnormality results from a bowel obstruction high in the small intestine? a. Respiratory acidosis b. Respiratory alkalosis c. Metabolic acidosis d. Metabolic alkalosis

D

Which condition is most likely to be treated with antibiotics? a. Cancer of the gallbladder b. Acute cholelithiasis c. Chronic pancreatitis d. Acute necrotizing pancreatitis

D

Which drug is often used in older patients for pain management of moderate to severe diverticulitis? a. An NSAID drug b. An acetaminophen-based drug c. An aspirin-based drug d. An opioid analgesic drug

D

_____: Procedure of irrigating the stomach in which a large-bore nasogastric tube is inserted into the stomach and room-temperature solution is instilled in volumes of 200 to 300 mL. The solution and blood are repeatedly withdrawn manually until returns are clear or light pink and without clots.

Gastric lavage

_____: An inflammation of the gastric mucosa (stomach lining).

Gastritis

_____: Hepatitis that is caused by the hepatitis A virus (HAV) and is characterized by a mild course similar to that of a typical viral syndrome and often goes unrecognized. It is spread via the fecal-oral route by oral ingestion of fecal contaminants. Sources of infection include contaminated water, shellfish caught in contaminated water, and food contaminated by infected food handlers. The virus may also be spread by oral-anal sexual activity. The incubation period is usually 15 to 50 days. The disease is usually not life threatening but may be more severe in people older than 40 years. It can also complicate pre-existing liver disease.

Hepatitis A

_____: Enteral products taken by patients who cannot consume enough nutrients in their usual diet (e.g., Ensure, Boost).

Medical nutrition supplements (MNSs)

_____: An incision of a papilla, a small nipple-shaped projection or structure.

Papillotomy

_____: An abnormal persistent increase in pressure within the portal vein; a major complication of cirrhosis.

Portal hypertension

A patient has an malignant hyperthermia (MH) incident during surgery. To whom does the nurse report this incident? a. North American Malignant Hyperthermia Registry b. The Joint Commission c. Centers for Disease Control d. Occupational Safety and Health Administration

A

A patient is prescribed sulfasalazine for the treatment of ulcerative colitis (UC). Which pa- tient statement indicates the patient is experi- encing a side effect of this drug? a. "My skin is covered with a rash." b. "My knees hurt." c. "My appetite has increased." d. "I wake up at night sweating sometimes."

A

An older adult is scheduled for an elective surgical procedure. On assessment the nurse notes brittle nails, dry flaky skin, muscle wasting, and dry sparse hair. The patient's BP is 82/48 and heart rate is 112/minute. How does the nurse interpret this assessment data? A. Poor fluid and nutrition status b. Improper care in the home c. Expected physiological changes of aging d. Depression related to aging processes

A

An older adult with gastroesophageal reflux disease (GERD) is prescribed omeprazole. What priority teaching point must the nurse instruct the patient about while taking this drug? a. Older adults taking this drug may be at increased risk for hip fracture because it interferes with calcium absorption. b. Because of this drug's effect of decreasing potassium, the patient may be prescribed a potassium supplement. c. This drug causes sodium retention so the patient may be prescribed a sodium restriction. d. A heart monitor may be needed because of changes in magnesium that can lead to life-threatening dysrhythmias.

A

In the postoperative period, following an open fundoplication repair for a paraesophageal (rolling) hernia, the nurse notes that with oral feedings, the patient has continuous dysphagia. What is the nurse's best interpretation of this finding? a. The fundoplication is too tight, and dilation may be required. b. The patient is not ready for any nutrition other than clear liquids. c. The health care provider will need to prescribe a stool softener. d. The patient needs a nasogastric tube placed to decompress the stomach

A

The emergency department nurse is assessing a patient admitted with frequent, liquid, foul-smelling stools containing mucus and blood. Assessment findings include tempera- ture 103.8° F (39.9°C), tenesmus, abdominal tenderness, and vomiting. Which additional laboratory test(s) does the nurse expect to collect? A. Serial stool samples b. Urine culture c. Throat culture d. Sputum culture

A

The nurse has taught a patient with acute sial- adenitis to use sialagogues to stimulate saliva. The patient demonstrates teaching has been effective when the patient states he will eat which food? a. Lemon slices b. Apple slices c. Bananas d. Bread

A

The nurse is assessing a patient after bariatric surgery. The patient has increased back pain, is restless, has a heart rate of 126/minute, and has only 15 mL of urine output for the past hour. What does the nurse suspect? A. Anastomotic leak b. Hypovolemic shock c. Bowel obstruction d. Hemorrhage

A

The nurse is caring for a patient who has had abdominal surgery. After a hard sneeze, the patient reports pain in the surgical area, and the nurse immediately sees that the patient has a wound evisceration. What priority action must the nurse do first? a. Call for help and stay with the patient. b. Leave the patient to immediately call the surgeon. c. Cover the wound with a nonadherent dressing moistened with normal saline. d. Take the patient's vital signs.

A

What action of darifenacin makes it suitable for treatment of IBS? A. Inhibits intestinal motility b. Decreases abdominal distention c. Eliminates constipation d. Increases fluid in the intestines

A

What is the drug of choice for the treatment of a fungal mouth infection? A. Nystatin. B. Acyclovir. C. Minocycline. D. Benzocaine.

A

Which statement about general principles of diet therapy for patients with dumping syndrome is true? a. Patients with dumping syndrome should have liquids between meals only. b. Patients with dumping syndrome should be encouraged to eat a diet high in roughage. c. Patients with dumping syndrome should eat a high-carbohydrate diet. d. The diet for a patient with dumping syn- drome must be low in fat and protein.

A

Which statement best describes phase I care after surgery? a. Phase I care occurs immediately after surgery, most often in a postanesthesia care unit (PACU). b. Phase I care focuses on preparing the patient for care in an extended care environment. c. Phase I care discharge occurs when presurgery level of consciousness has returned, oxygen saturation is at baseline, and vital signs are stable. d. Phase I care most often occurs on a hospital unit, in an extended care facility, or in the home.

A

Nonsurgical treatment options for cancer of the esophagus can include which therapies? Select all that apply. a. Swallowing therapy b. Chemoradiation c. Targeted therapies d. Smoking cessation programs e. Photodynamic therapy f. Endoscopic therapies

A,B,C,E,F

When performing an assessment on a patient with an active upper GI bleed, which conditions does the nurse identify as common causes of upper GI bleeding? Select all that apply. A. Esophageal cancer b. Esophageal varices c. Gastroesophageal reflux disease d. Duodenal ulcer e. Gastritis f. Gastric cancer

A,B,D,E,F

The nurse assesses for which potential complications in a patient who is malnour- ished? Select all that apply. a. Poor wound healing b. Intolerance to heat c. Infection d. Lethargy e. Edema f. Increased activity tolerance

A,C,D,E

Which are common manifestations of acute cholecystitis? Select all that apply. a. Anorexia b. Ascites c. Eructation d. Steatorrhea e. Jaundice f. Rebound tenderness

A,C,E,F

The nurse is caring for several patients with gastric and duodenal ulcers. Which differential features of gastric ulcers compared to duodenal ulcers does the nurse identify? Select all that apply. A. In gastric ulcers, there is normal secretion or hyposecretion. B. Gastric ulcers are relieved by ingestion of food. C. Hematemesis is more common than melena. D. No gastritis is present. E. Most often, the patient has type O blood. F. Pain occurs 30-60 minutes after a meal and at night.

A,C,F

Dyspepsia is characterized by which factors? Select all that apply. a. Indigestion associated with eating b. Loss of appetite for food c. Malabsorption of fats d. Heartburn associated with eating e. Vomiting that occurs after eating f. Unintentional weight loss

A,D

A nurse is preparing a health teaching session about early detection of colorectal cancer. Which test should the nurse include? (Select all that apply.) A) Colonoscopy every 10 years B) Single sample fecal immunochemical test (FIT). C) Flexible sigmoidoscopy every 5 years D) Stool DNA test (sDNA) every 3 years E) Double contrast barium enema every 5 years F) Take home yearly guaiac fecal occult blood test (gFOBT)

ANS: A, C, D, E, F. The nurse should teach about all methods of early detection except the FIT test, which should use a multi-sample approach.

The nurse is caring for a client who has celiac disease. Which food will the nurse remove from the client's dietary tray? (Select all that apply.) A) Rice B) Graham crackers C) Croissant D) Fresh peaches E) Chicken breast

ANS: B, C. Clients with celiac disease cannot eat gluten. Foods containing gluten, such as graham crackers and toast, must be removed from the client's tray. The other foods listed do not contain gluten.

The nurse is performing a physical assessment on a client's abdomen. The nurse inspects the abdomen and finds the abdomen asymmetrical, with a non-pulsating mass in the RUQ. What is the appropriate priority nursing intervention? A) Document the findings in the electronic health record. B) Auscultate for bowel sounds and bruits. C) Lightly palpate the mass. D) Notify the health care provider of the findings.

ANS: B. The nurse will eventually document the findings, palpate the mass, and notify the health care provider of the findings but should finish the physical assessment first.

Which client statement regarding appropriate pain control requires nursing intervention? A) "I will listen to music when I feel pain." B) "Before exercise, I will be sure I have taken my medication." C) "If the prescribed dose of medication doesn't help my pain, I'll take an extra dose." D) "I plan to keep a pain diary so that I can see trends about when my pain worsens."

ANS: C. The nurse should intervene to discuss appropriate doses of medication, and risks associated with taking more medication than has been prescribed. The nurse should also educate the client to call the health care provider if pain is not being adequately managed with the prescribed pain reliever.

Which teaching will the nurse provide when discharging a client with chronic pancreatitis? A) Weight reduction and daily exercise regimen B) Constipation precautions including daily laxative use C) Dietary adjustments to include avoiding high-fat foods, caffeine, and alcohol D) Relaxation techniques and stress management

ANS: C. The nurse will teach the client about a diet low in fat and avoiding caffeine and alcohol, which is recommended for those with chronic pancreatitis. Weight reduction, exercise programs, bowel retraining program, relaxation techniques, and stress management are not interventions pertinent to pancreatitis

Which client statement regarding treatment for gastric cancer requires the nurse to immediately intervene? A) "I understand my treatment regimen." B) "My prognosis is frightening to me and my partner." C) "Life just does not seem to be worth living anymore." D) "There is a list of community resources stored in my computer for when I need them."

ANS: C. This client statement requires immediate nursing intervention as it indicates that the client may be experiencing hopelessness. The nurse should assess for suicidal thoughts, provide therapeutic care and listening, and suggest referrals as needed.

The nurse will include what post-operative teaching when caring for the client who is preparing to undergo endoscopic cholecystectomy? (Select all that apply.) A) "You will have a small, midline abdominal incision." B) "You cannot eat or drink for a few days after the procedure." C) "You will not be able to return to regular activity for several weeks." D) "Generally, the pain associated with this procedure is minimal." E) "This procedure has a low incidence of infection." F) "The hospital stay after this procedure is typically 1 to 2 days."

ANS: D, E, F. The laparoscopic cholecystectomy is a minor procedure with few associated complications. There is decreased wound infection than associated with a traditional open cholecystectomy, minimal pain and a 1 to 2 day stay in the hospital. An abdominal incision, nasogastric tube, nothing by mouth, low fat diet, Jackson-Pratt drain, and restricted activity are associated with an open cholecystectomy procedure.

A scrub person is discussing artificial nail use with the nurse. The scrub person states, "I do not use artificial nails; I am wearing gel polish to strengthen my nails." What is the appropriate nursing response? A) "I understand. That is my nail treatment of choice, also." B) "Hand hygiene is enhanced by covering natural nails." C) "Wear double gloves to prevent puncture or contamination." D) "Gel polish is a type of artificial nail which alters skin flora and impedes hand hygiene."

ANS: D. Hand hygiene and skin flora are affected by any type of artificial nail. Gel polish, also known as "Shellac," is much thinner than traditional acrylic, but is still considered to be a type of artificial nail.

The client in the surgery holding area identifies the left arm as the correct surgical site, but the operative permit designates surgery to be performed on the right arm. Which interventions should the nurse implement? Select all that apply. 1. Review the client's chart. 2. Notify the surgeon. 3. Immediately call a "time-out." 4. Correct the surgical permit. 5. Request the client mark the left arm.

ANSWER: 1, 2, 3, 5. 1. When the client in the holding area states the surgery site differs from the scheduled surgery, the nurse should identify the client and review the client's chart. 2. If there is a discrepancy, the nurse should notify the surgeon to explain the situation and resolve the issue. 3. The Joint Commission surgical standards state a "time-out" period is called and everything stops until the discrepancy is resolved. 4. The nurse should not change a permit. If an error is discovered, the nurse should correct the situation within legal and ethical guidelines. 5. Clients are encouraged to mark the correct side or site with indelible ink

The public health nurse is discussing hepatitis B with a group in the community. Which health promotion activities should the nurse discuss with the group? Select all that apply. 1. Do not share needles or equipment. 2. Use barrier protection during sex. 3. Get the hepatitis B vaccine. 4. Obtain immune globulin injections. 5. Avoid any type of hepatotoxic medications

ANSWER: 1, 2, 3. 1. Hepatitis B can be transmitted by sharing any type of needles, especially those used by drug abusers. 2. Hepatitis B can be transmitted through sexual activity; therefore, the nurse should recommend abstinence, mutual monogamy, or barrier protection. 3. Three doses of hepatitis B vaccine provide immunity in 90% of healthy adults. 4. Immune globulin injections are administered as postexposure prophylaxis (after being exposed to hepatitis B), but encouraging these injections is not a health promotion activity. 5. Hepatotoxic medications should be avoided in clients who have hepatitis or who have had hepatitis. The health-care provider prescribes medications, and the layperson does not know which medications are hepatotoxic. TEST-TAKING HINT: In this "Select all that apply" type of question, there may be only one (1) correct answer, there may be several, or all five options may be correct answers.

The nurse is administering an opioid narcotic to the client. Which interventions should the nurse implement for client safety? Select all that apply. 1. Compare the hospital number on the MAR to the client's bracelet. 2. Have a witness verify the wasted portion of the narcotic. 3. Assess the client's vital signs prior to administration. 4. Determine if the client has any allergies to medications. 5. Clarify all pain medication orders with the health-care provider.

ANSWER: 1, 3, 4. 1. This procedure ensures client safety by preventing medication from being given to the wrong client. 2. This is a legal requirement, not a safety issue. 3. This intervention would prevent giving a narcotic to a client who is unstable or compromised. 4. Determining allergies addresses client safety. 5. It would not be realistic to recheck all orders. TEST-TAKING HINT: This question specifically asks the test taker to identify interventions for safely administering medication to the client. Therefore, options "2" and "5" could be eliminated because they do not address the client's safety. This is an alternate-type question requiring the test taker to select more than one (1) option as the correct answer

The client diagnosed with AIDS is experiencing voluminous diarrhea. Which interventions should the nurse implement? Select all that apply. 1. Monitor diarrhea, charting amount, character, and consistency. 2. Assess the client's tissue turgor every day. 3. Encourage the client to drink carbonated soft drinks. 4. Weigh the client daily in the same clothes and at the same time. 5. Assist the client with a warm sitz bath PRN

ANSWER: 1, 4, 5. 1. It is important to keep track of the amounts, color, and other characteristics of body fluids excreted. 2. Skin turgor should be assessed at least every six (6) to eight (8) hours, not daily. 3. Carbonated soft drinks increase flatus in the GI tract, and the increased sugar will act as an osmotic laxative and increase diarrhea. 4. Daily weights are the best method of determining fluid loss and gain. 5. Sitz baths will assist in keeping the client's perianal area clean without having to rub. The warm water is soothing, providing comfort. TEST-TAKING HINT: The test taker should note the time frame for any answer option. "Every day" is not often enough to assess for dehydration in a client who is experiencing massive ("voluminous") fluid loss. If the test taker were not aware of the definition, then an associated word, "volume," would be a hint.

The nurse is planning the care of the surgical client having procedural sedation. Which intervention has highest priority? 1. Assess the client's respiratory status. 2. Monitor the client's urinary output. 3. Take a 12-lead ECG prior to injection. 4. Attempt to keep the client focused.

ANSWER: 1. 1. Assessing the respiratory rate, rhythm, and depth is the most important action. 2. The nurse needs to monitor all systems, but monitoring the urine output would not be priority over monitoring breathing. 3. Monitoring the client's ECG is appropriate, but it is not priority. 4. The client needs to be relaxed, not focused, but this is not priority over respiratory status. TEST-TAKING HINT: When the test taker must prioritize nursing care, assessment is usually first. Using Maslow's hierarchy of needs to prioritize, assessment of respiration is always first. All sedation agents can depress respirations.

Which sign/symptom should the nurse expect to find in a client diagnosed with ulcerative colitis? 1. Twenty bloody stools a day. 2. Oral temperature of 102°F. 3. Hard, rigid abdomen. 4. Urinary stress incontinence

ANSWER: 1. 1. The colon is ulcerated and unable to absorb water, resulting in bloody diarrhea. Ten (10) to 20 bloody diarrhea stools is the most common symptom of ulcerative colitis. 2. Inflammation usually causes an elevated temperature but is not expected in the client with ulcerative colitis. 3. A hard, rigid abdomen indicates peritonitis, which is a complication of ulcerative colitis but not an expected symptom. 4. Stress incontinence is not a symptom of colitis. TEST-TAKING HINT: If the test taker is not sure of the answer, the test taker should use knowledge of anatomy and physiology to help identify the correct answer. The colon is responsible for absorbing water, and if the colon can't do its job, then water will not be absorbed, causing diarrhea (option "1"). Colitis is inflammation of the colon; therefore, option "4" referring to the urinary system can be eliminated.

The circulating nurse assesses tachycardia and hypotension in the client. Which interventions should the nurse implement? 1. Prepare ice packs and mix dantrolene sodium. 2. Request the defibrillator be brought into the OR. 3. Draw a PTT and prepare a heparin drip. 4. Obtain finger stick blood glucose immediately

ANSWER: 1. 1. Unexplained tachycardia, hypotension, and elevated temperature are signs of malignant hyperthermia, which is treated with ice packs and dantrolene sodium. 2. A defibrillator would be needed if the client were in ventricular tachycardia or ventricular fibrillation. 3. These interventions would not be appropriate for malignant hyperthermia. 4. This would be important if the client had diabetes, but it does not address malignant hyperthermia. TEST-TAKING HINT: The test taker could eliminate options based on basic concepts. Option "2" could be eliminated because the client is not coding. Option "3" would increase bleeding, which would not be appropriate for a client having surgery

The unlicensed assistive personnel (UAP) can be overheard talking loudly to the scrub technologist discussing a problem which occurred during one (1) of the surgeries. Which intervention should the nurse in the surgical holding area with a female client implement? 1. Close the curtains around the client's stretcher. 2. Instruct the UAP and scrub tech to stop the discussion. 3. Tell the surgeon on the case what the nurse overheard. 4. Inform the client the discussion was not about her surgeon.

ANSWER: 2. 1. Closing curtains will not keep loud conversations from being overheard. 2. The UAP and scrub tech are violating HIPAA and should be told to stop the conversation immediately. 3. This is a nursing problem and the nurse should handle the situation. The surgeon is not in the chain of command of the UAP or the scrub tech. 4. Telling the client the situation does not involve her surgeon is involving the client even more in the overheard conversation

Which statement made by the client indicates to the nurse the client may be experiencing GERD? 1. "My chest hurts when I walk up the stairs in my home." 2. "I take antacid tablets with me wherever I go." 3. "My spouse tells me I snore very loudly at night." 4. "I drink six (6) to seven (7) soft drinks every day

ANSWER: 2. 1. Pain in the chest when walking up stairs indicates angina. 2. Frequent use of antacids indicates an acid reflux problem. 3. Snoring loudly could indicate sleep apnea but not GERD. 4. Carbonated beverages increase stomach pressure. Six (6) to seven (7) soft drinks a day would not be tolerated by a client with GERD. TEST-TAKING HINT: The stem of the question indicates an acid problem. The drug classification of antacid, or "against acid," provides the test taker a hint as to the correct answer.

Which instruction should the nurse discuss with the client who is in the icteric phase of hepatitis C? 1. Decrease alcohol intake. 2. Encourage rest periods. 3. Eat a large evening meal. 4. Drink diet drinks and juices.

ANSWER: 2. 1. The client must avoid alcohol altogether, not decrease intake, to prevent further liver damage and promote healing. 2. Adequate rest is needed for maintaining optimal immune function. 3. Clients are more often anorexic and nauseated in the afternoon and evening; therefore, the main meal should be in the morning. 4. Diet drinks and juices provide few calories, and the client needs an increased-calorie diet for healing. TEST-TAKING HINT: The test taker must be aware of key words in both the stem and answer options. The "icteric" phase means the acute phase. The word "decrease" should cause the test taker to eliminate option "1" as a possible correct answer, and "large" should cause the test taker to eliminate "3" as a possible correct answer.

Which nursing intervention has the highest priority when preparing the client for a surgical procedure? 1. Pad the client's elbows and knees. 2. Apply soft restraint straps to the extremities. 3. Prepare the client's incision site. 4. Document the temperature of the room.

ANSWER: 2. 1. This intervention prevents nerve damage from positioning, but it is not a higher priority than preventing the client from falling off the OR table. 2. This action would prevent the client from falling off the table, which is the highest priority. 3. Preparing the incision site is not a higher priority than preventing the client from falling off the OR table. 4. The temperature of the room does not have a higher priority than safety. TEST-TAKING HINT: Client safety should always be a high priority. In order, the priorities would be preventing a fall, preventing nerve damage, preventing infection, and then documenting.

The nurse is caring for the client diagnosed with ascites secondary to hepatic cirrhosis. Which information should the nurse report to the health-care provider? 1. A decrease in the client's daily weight of one (1) pound. 2. An increase in urine output after administration of a diuretic. 3. An increase in abdominal girth of two (2) inches. 4. A decrease in the serum direct bilirubin to 0.6 mg/dL.

ANSWER: 3. 1. A decrease in weight indicates a loss in fluid and is not data necessary to report to the health-care provider. 2. An increase in urine output indicates the diuretic was effective. 3. An increase in abdominal girth indicates the ascites is increasing, meaning the client's condition is becoming more serious and should be reported to the health-care provider. 4. The normal direct bilirubin value is 0.1 to 0.4 mg/dL; therefore, a decrease in the value, although it is still elevated, would not be reported.

Which medication should the nurse expect the HCP to order to treat the client diagnosed with botulism secondary to eating contaminated canned goods? 1. An antidiarrheal medication. 2. An aminoglycoside antibiotic. 3. An antitoxin medication. 4. An ACE inhibitor medication.

ANSWER: 3. 1. Antidiarrheal medications are contraindicated with botulism because the toxin needs to be expelled from the body. 2. Aminoglycoside antibiotics will not be ordered because there is no bacterium with botulism; it is caused by a neurotoxin. 3. A botulism antitoxin neutralizes the circulating toxin and is prescribed for a client with botulism. 4. An angiotensin-converting enzyme (ACE) inhibitor is prescribed for a client diagnosed with cardiovascular disease. TEST-TAKING HINT: The key word in this question is "treat." Because botulism does not end in -itis, and thus is not an infection, the use of an antibiotic can be eliminated.

The client in the surgical holding area tells the nurse "I am so scared. I have never had surgery before." Which statement would be the nurse's most appropriate response? 1. "Why are you afraid of the surgery?" 2. "This is the best hospital in the city." 3. "Does having surgery make you afraid?" 4. "There is no reason to be afraid."

ANSWER: 3. 1. The nurse should never ask the client "why." The client does not owe the nurse an explanation. 2. This response defends the hospital and does not address the client's feelings. 3. This response is therapeutic and promotes communication of feelings. 4. This statement is closed-ended and will not encourage the continued discussion of "fear."

Which nursing intervention is the highest priority when administering pain medication to a client experiencing acute pain? 1. Monitor the client's vital signs. 2. Verify the time of the last dose. 3. Check for the client's allergies. 4. Discuss the pain with the client.

ANSWER: 4. 1. It is important to monitor vital signs, but it is not the priority intervention prior to administering the medication. 2. The nurse should verify the time the last dose was administered to determine the time the next dose could be administered, but this is not the priority intervention. 3. Prior to giving any medication, the nurse should assess any allergies, but it is not the priority intervention. 4. The nurse should question the client to rule out complications and to determine which medication and amount would be most appropriate for the client. This is assessment. TEST-TAKING HINT: When questions require a priority answer, the test taker should look for an option which addresses assessment, but the test taker should remember there are many words which reflect assessment, such as "discuss," "determine," "monitor," or "obtain," to name a few.

The nurse is administering morning medications at 0730. Which medication should have priority? 1. A proton pump inhibitor. 2. A nonnarcotic analgesic. 3. A histamine receptor antagonist. 4. A mucosal barrier agent

ANSWER: 4. 1. Proton pump inhibitors can be administered at routine dosing times, usually 0900 or after breakfast. 2. Pain medication is important, but a nonnarcotic medication, such as Tylenol, can be administered after a medication, which must be timed. 3. A histamine receptor antagonist can be administered at routine dosing times. 4. A mucosal barrier agent must be administered on an empty stomach for the medication to coat the stomach. TEST-TAKING HINT: Basic knowledge of how medications work is required to administer medications for peak effectiveness. There are very few medications requiring a specific time. The test taker should memorize these specific medications.

The 36-year-old female client diagnosed with anorexia nervosa tells the nurse "I am so fat. I won't be able to eat today." Which response by the nurse is most appropriate? 1. "Can you tell me why you think you are fat?" 2. "You are skinny. Many women wish they had your problem." 3. "If you don't eat, we will have to restrain you and feed you." 4. "Not eating might cause physical problems."

ANSWER: 4. 1. The client does not have to explain her actions to the nurse; the nurse should not ask "why." 2. Telling the client she is skinny is belittling the client. 3. The client is 36 years old and has the right to refuse to eat, even to the detriment of her body. Restraining the client could be considered assault unless the psychiatric team had a court order. 4. "Might cause physical problems" is a factual statement to the client about the possible results if the client refuses nourishment. TEST-TAKING HINT: The test taker could eliminate answer option "1" based on "why." Option "2" is not therapeutic or factual information

Which outcome should the nurse identify for the client scheduled to have a cholecystectomy? 1. Decreased pain management. 2. Ambulate first day postoperative. 3. No break in skin integrity. 4. Knowledge of postoperative care

ANSWER: 4. 1. The expected outcome is pain control for both preoperative and postoperative care. 2. Postoperative care includes ambulation. 3. Prevention of an additional impaired skin integrity is a desired postoperative outcome. The incision would be a break in skin integrity. 4. This would be an expected outcome for the client scheduled for surgery. This indicates preoperative teaching has been effective. TEST-TAKING HINT: The time element is important in this question. The expected outcome is required for the preoperative period. Option "1" is incorrect because of the adjective "decreased." Adjectives commonly determine the accuracy of the options

The nurse is caring for the client recovering from intestinal surgery. Which assessment finding requires immediate intervention? 1. Presence of thin, pink drainage in the Jackson Pratt. 2. Guarding when the nurse touches the abdomen. 3. Tenderness around the surgical site during palpation. 4. Complaints of chills and feeling feverish.

ANSWER: 4. 1. Thin pink drainage is expected in the Jackson Pratt (JP) bulb. 2. Guarding is a normal occurrence when touching a tender area on the abdomen and does not require immediate intervention. 3. Tenderness around the surgical site is a normal finding and does not require intervention. 4. Complaints of chills, sudden onset of fever, tachycardia, nausea, and hiccups are symptoms of peritonitis, which is a lifethreatening complication

_____: Liver inflammation caused by the toxic effect of alcohol on hepatocytes. The liver becomes enlarged, with cellular degeneration and infiltration by fat, leukocytes, and lymphocytes.

Alcoholic hepatitis

Which diagnostic results lead the nurse to suspect that a client may have gallbladder disease? A) Increased white blood cell (WBC) count, visualization of calcified gallstones, edema of the gallbladder wall B) Decreased WBC count, visualization of calcified gallstones, increased alkaline phosphatase C) Increased WBC count, visualization of noncalcified gallstones, edema of the gallbladder wall D) Decreased WBC count, visualization of noncalcified gallstones, increased alkaline phosphatase

Answer: A. An increased WBC count, calcified gallstones visualized on the abdominal X-ray, and edema of the gallbladder wall are the best diagnostic results to indicate gallbladder disease. Acute cholecystitis is seen as edema of the gallbladder wall and pericholecystic fluid. Ultrasonography of the right upper quadrant is the best diagnostic test for cholecystitis.An increased WBC count, not decreased, is evidence of inflammation. Only calcified gallstones, not noncalcified gallstones, will be visualized on abdominal X-ray. Alkaline phosphatase will be elevated if liver function is abnormal; this is not common in gallbladder disease.

A client who was awaiting liver transplantation is excluded from the procedure after the presence of which condition is discovered? A) Colon cancer with metastasis to the liver. B) Hypertension. C) Hepatic encephalopathy. D) Ascites and shortness of breath.

Answer: A. Clients with metastatic cancers are not candidates for liver transplant. Transplantation is performed for hepatitis and primary (not secondary) liver cancers.Hypertension is a controllable factor and would not preclude the client from a liver transplant. Encephalopathy is a consequence of advanced liver disease, consistent with the condition of a client awaiting transplantation. It can be treated with lactulose and nonabsorbable antibiotics. Ascites and resulting shortness of breath are also consequences of advanced liver disease, consistent with the client awaiting transplantation. They can be managed with diuretics and paracentesis.

Which is a correct statement differentiating Crohn's disease (CD) from ulcerative colitis (UC)? A) Patients with CD experience about 20 loose, bloody stools daily. B) Patients with UC may experience hemorrhage. C) The peak incidence of UC is between 15 and 40 years of age. D) Very few complications are associated with CD.

Answer: B. A correct statement about differentiating Crohn's disease (CD) from ulcerative colitis (UC) is that patients with UC may experience hemorrhage. Patients with CD can have 5-6 soft, loose stools per day, but they are nonbloody.Five to six stools daily is common with CD, not 20 loose, bloody stools. The peak incidences of UC are between 30 and 40 years and again at 55 to 65 years of age, and not just 15 to 40 years of age. Fistulas commonly occur as a complication of CD.

The nurse is teaching a patient about dietary choices to prevent dumping syndrome after gastric bypass surgery. Which statement by the patient indicates a need for further teaching? A) "I will need to avoid sweetened fruit juice beverages." B) "I can eat ice cream in moderation." C) "I cannot drink alcohol at all." D) "It is okay to have a serving of sugar-free pudding."

Answer: B. A need for further teaching about dietary changes related to dumping syndrome is indicated when the patient says that ice cream can be eaten in moderation. Milk products such as ice cream must be eliminated from the diet of a patient with dumping syndrome.The patient with dumping syndrome can no longer consume sweetened drinks. Alcohol must also be eliminated from the diet. The patient can eat sugar-free pudding, custard, and gelatin but with caution.

A nurse is reviewing the laboratory results of a client who has acute pancreatitis. Which of the following findings should the nurse expect? A) WBC 9,000/mm^3. B) Increased serum amylase C) Blood glucose 110 mg/dL D) Decreased bilirubin

Answer: B. A) A WBC of 9,000/mm3 is within the expected reference range. An elevated WBC is an expected finding in a client who has acute pancreatitis. B) Serum amylase levels are increased in a client who has acute pancreatitis due to pancreatic cell injury. C) A blood glucose level of 110 mg/dL is within the expected reference range. Elevated blood glucose is an expected finding for a client who has acute pancreatitis. D) Increased bilirubin is an expected finding in a client who has acute pancreatitis due to the hepatobiliary obstructive process.

Which action does the nurse implement for a client with wound evisceration? A) Apply direct pressure to the wound. B) Cover the wound with a sterile, warm, moist dressing. C) Irrigate the wound with warm, sterile saline. D) Replace tissue protruding into the opening.

Answer: B. Covering the wound with a sterile, warm, moist dressing protects the organs until the surgeon can repair the wound. Evisceration occurs when a wound opens up and body organs are exposed.Applying direct pressure to a wound traumatizes the organs. Irrigating the wound is not necessary. Replacing protruding tissue could induce infection.

Which client is at greatest risk for slow wound healing? A) A 12-year-old healthy girl B) A 47-year-old obese man with diabetes C) A 48-year-old woman who smokes D) A 98-year-old healthy man

Answer: B. Obesity and diabetes would significantly put a client at greatest risk for slow wound healing.The healthy 12-year-old would likely heal quickly. The 48-year-old smoker will experience delayed wound healing, but is not as high a risk as an obese client who is diabetic. The healthy 98-year-old is not at risk for delayed wound healing.

A patient with colorectal cancer was started on 5-fluorouracil (5-FU) and is experiencing fatigue, diarrhea, and mouth ulcers. What does the nurse tell the patient about the cause of diarrhea and mouth ulcers? A) "A combination of chemotherapeutic agents has caused them." B) "GI problems are symptoms of the advanced stage of your disease." C) "5-FU cannot discriminate between your cancer and your healthy cells and is causing your ulcers and diarrhea." D) "You have these as a result of the radiation treatment."

Answer: C. "5-FU cannot discriminate between your cancer and your healthy cells and is causing your ulcers and diarrhea."

A nurse is assessing a client who is postoperative following a gastrectomy. The nurse should identify which of the following findings as an indication of abdominal distension? A) Bradycardia. B) Hypertension. C) Hiccups. D) Chest pain.

Answer: C. A) The nurse should identify tachycardia as an indication of abdominal distension. B) The nurse should identify hypotension as an indication of abdominal distension. C) Following surgery, hiccups can be caused by irritation of the phrenic nerve due to abdominal distension. If the hiccups are intractable, the nurse should anticipate a prescription for chlorpromazine because persistent hiccups are distressful to the client and can lead to complications, such as vomiting. D) The nurse should identify abdominal pain as an indication of abdominal distension.

A nurse is providing discharge teaching for a client who is postoperative following a rhinoplasty using general anesthesia. Which of the following instructions should the nurse include? A) "Lie on your side when resting for the first week after surgery." B) "Close your mouth when you are about to sneeze." C) "Use cool compresses on your eyes, nose, and face." D) "Limit intake to clear liquids for the first 24 hours after surgery."

Answer: C. A) The nurse should instruct the client to maintain a semi-Fowler's positon when resting at home. B) The nurse should instruct the client to avoid sneezing with his mouth closed until after the surgical packing is removed. C) The nurse should instruct the client to place cool compresses on his face to reduce swelling and ecchymosis. D) The nurse should instruct the client that he can eat soft foods after his gag reflex has returned.

How does the nurse position a client with postoperative nausea and vomiting? A) Flat in bed, with the head in alignment with the body B) Prone, with the head of the bed flat. C) Side-lying, with the head in a neutral position. D) Supine in bed, with the neck flexed.

Answer: C. The side-lying position with the client's head in a neutral position helps reduce postoperative nausea and vomiting.The flat-in-bed position with the head in alignment is not a neutral position. The prone position with the head of the bed flat is unnatural, as is the supine position with the neck flexed.

It is essential that the nurse monitor the client returning from hepatic artery embolization for hepatic cancer for which potential complication? A) Right shoulder pain. B) Polyuria. C) Bone marrow suppression. D) Bleeding.

Answer: D. A potential complication of hepatic artery embolization for hepatic cancer is bleeding. Prompt detection of hemorrhage is the priority.Discomfort such as right shoulder pain may be present, but the priority is to assess for hemorrhage. The nurse must assess for signs of shock, not polyuria. Embolization does not suppress the bone marrow. If chemotherapy or immune modulators are used, the nurse then assesses for bone marrow suppression.

A nurse is assessing a client who has upper gastrointestinal bleeding. Which of the following findings should the nurse expect? A) Bounding peripheral pulses. B) Bradycardia C) Increased hematocrit levels D) Hypotension

Answer: D. A) Weak peripheral pulses are a manifestation of hemorrhagic shock. B) Tachycardia is a manifestation of hemorrhagic shock. C) Decreased hematocrit and hemoglobin levels are manifestations of hemorrhagic shock. D) A client who has upper gastrointestinal bleeding is at risk for hemorrhagic shock. Hypotension is a manifestation of hemorrhagic shock.

A client is being prepared for gastrointestinal surgery and undergoes a bowel preparation. This preoperative procedure is done to: A) decrease expected blood loss during surgery. B) eliminate any risk of infection. C) ensure that the bowel is sterile. D) reduce the number of intestinal bacteria.

Answer: D. Bowel or intestinal preparations are performed to empty the bowel to minimize the leaking of bowel contents, prevent injury to the colon, and reduce the number of intestinal bacteria.Decreasing expected blood loss and sterilizing the bowel are not the goals of a bowel preparation. While the bowel prep may reduce the number of intestinal bacteria, it will not completely eliminate the risk of infection.

A patient who had surgery for inflammatory bowel disease is being discharged. The case manager will arrange for home health care follow-up. The patient tells the nurse that family members will also be helping with care. What information is critically important for the nurse to provide to these collaborating members? A) A list of medical supply facilities where wound care supplies may be purchased B) Proper handwashing techniques to avoid cross-contamination of the patient's wound C) The amount of pain medication that the patient is allowed to take in each dose D) Written and oral instructions regarding signs/symptoms to report to the primary health care provider

Answer: D. It is critically important to provide the patient and case manager with both written and oral instructions on reportable signs/symptoms to avoid the development of complications.It will be the home health nurse's responsibility to bring supplies to the patient's home. Although instruction on proper handwashing and the patient's medication regimen are important, they are not the highest priority.

A client who is receiving total enteral nutrition exhibits acute confusion and shallow breathing and says, "I feel weak." As the client begins to have a generalized seizure, how does the nurse interpret this client's signs and symptoms? A) The enteral tube is dislodged. B) Abdominal distention is present. C) Severe hyperglycemia is present. D) Refeeding syndrome is occurring.

Answer: D. Refeeding syndrome is a syndrome consisting of metabolic disturbances that occur as a result of reinstitution of nutrition to clients who are starved, severely malnourished or metabolically stressed due to severe illness. Symptoms of refeeding syndrome include shallow respirations, weakness, acute confusion, seizures, and increased bleeding tendency.If the enteral tube becomes misplaced or dislodged, the client may develop aspiration pneumonia displayed by increased temperature, increased pulse, dehydration, diminished breath sounds, and shortness of breath. Abdominal distention is most frequently accompanied by nausea and vomiting. In refeeding syndrome, insulin secretion decreases in response to the physiologic changes in the body, so hyperglycemia is not present. When refeeding begins, insulin production resumes and the cells take up glucose and electrolytes from the bloodstream, thus depleting serum levels, resulting in hypoglycemia.

A diabetic client who is scheduled for vascular surgery is admitted on the day of surgery with several orders. Which order does the nurse accomplish first? A) Use electric clippers to cut hair at the surgical site. B) Start an infusion of lactated Ringer's solution at 75 mL/hr. C) Administer one-half of the client's usual lispro insulin dose. D) Draw blood for glucose, electrolyte, and complete blood count values.

Answer: D. The blood sample needs to be drawn and sent to the laboratory first to confirm that results are within normal limits. If blood work is abnormal, the surgery may be rescheduled.Removal of hair can be accomplished in the operating room directly before the start of surgery. The IV infusion can be accomplished after the laboratory orders have been completed. The nurse should check blood glucose with the laboratory orders before administration of lispro.

An intensive care unit (ICU) RN is "floated" to the medical-surgical unit. Which patient does the charge nurse assign to the float nurse? A) A 28-year-old with an exacerbation of Crohn's disease (CD) who has a draining enterocutaneous fistula B) A 32-year-old with ulcerative colitis (UC) who needs discharge teaching about the use of hydrocortisone enemas C) A 34-year-old who has questions about how to care for a newly created ileo-anal reservoir D) A 36-year-old with peritonitis who just returned from surgery with multiple drains in place

Answer: D. The charge nurse assigns to the ICU nurse who was floated to the medical-surgical unit a 36-year-old patient with peritonitis who just returned from surgery with multiple drains in place. The ICU nurse is familiar with the care of a patient with peritonitis, including monitoring for complications such as sepsis and kidney failure.The patient with CD who has a draining enterocutaneous fistula, the patient with UC who needs discharge teaching, and the patient with questions about an ileo-anal reservoir are best assigned to a medical-surgical nurse who is more familiar with the care and teaching needed for patients with their respective disorders.

The RN is caring for a client with end-stage liver disease who has ascites. Which action does the RN delegate to unlicensed assistive personnel (UAP)? A) Assessing skin integrity and abdominal distention B) Drawing blood from a central venous line for electrolyte studies C) Evaluating laboratory study results for the presence of hypokalemia D) Placing the client in a semi-Fowler's position

Answer: D. The nurse delegates the client who needs to be placed in a semi-Fowler's position to the UAP. Positioning the client in this position is included within UAP education and scope of practice, although the RN will need to supervise the UAP in providing care and will evaluate the effect of the semi-Fowler's position on the client's comfort and breathing.Assessment of skin integrity and abdominal distention, obtaining blood from a central line, and evaluation of laboratory results must be done by the RN.

A patient newly diagnosed with ulcerative colitis (UC) is started on sulfasalazine (Azulfidine). What does the nurse tell the patient about why this therapy has been prescribed? A) "It is to stop the diarrhea and bloody stools." B) "This will minimize your GI discomfort." C) "With this medication, your cramping will be relieved." D) "Your intestinal inflammation will be reduced."

Answer: D. The nurse tells the newly diagnosed patient with UC who is started on sulfasalazine that, "Your intestinal inflammation will be reduced." Sulfasalazine (Azulfidine) is one of the primary treatments for UC. It is thought to inhibit prostaglandin synthesis and thereby reduce inflammation.Although it is hoped that reduction of inflammation will cause the diarrhea and bloody stools to stop, this is not the way that the drug works. Antidiarrheal drugs "stop" diarrhea. The drug's action as an anti-inflammatory will diminish the patient's pain as the inflammation subsides, but this is not the purpose of the drug. Sulfasalazine is an anti-inflammatory medication, not an analgesic.

The nurse is caring for an older adult patient with acute biliary pain. Which drug order does the nurse question? a. Ketorolac b. Meperidine c. Morphine d. Hydromorphone

B

The patient with acute cholecystitis has a pace- maker. Which diagnostic test is contraindicated? a. Extracorporeal shock wave lithotripsy (ERCP) b. Magnetic resonance cholangiopancreatog- raphy (MRCP) c. Ultrasonography of the right upper quadrant d. Hepatobiliary (HIDA) scan

B

Which diagnostic test is a noninvasive imag- ing procedure that can get multidimensional views of the entire colon? a. Abdominal ultrasound b. Virtual colonography c. Colonoscopy d. Sigmoidoscopy

B

A patient scheduled for surgery has a history of myocardial infarction 6 weeks ago. Which classification will this patient meet preoperatively based on the ASA Physical Status Classification system? a. ASA class I b. ASA class II c. ASA class III d. ASA class IV

D

_____: The surgical removal of part or all of the stomach.

Gastrectomy

_____: A type of gastric restriction surgery in which gastric resection is combined with malabsorption surgery. The patient's stomach, duodenum, and part of the jejunum are bypassed so that fewer calories can be absorbed. Also known as a Roux-en-Y gastric bypass, or RNYGB.

Gastric bypass

_____: Condition that occurs as a result of backward flow of stomach contents into the esophagus.

Gastroesophageal reflux (GER)

_____: An upper gastrointestinal disease caused by the backward flow (reflux) of gastrointestinal contents into the esophagus.

Gastroesophageal reflux disease (GERD)

_____: Flowing in the opposite direction from normal, as the occurrence of warm fluid traveling up the throat, unaccompanied by nausea, in the patient with gastroesophageal reflux disease.

Regurgitation

The nurse is assessing a patient with acquired immune deficiency syndrome (AIDS) who has muscle wasting related to poor nutrition. How does the nurse interpret this finding? a. Cachexia b. Candidiasis c. Protein catabolism d. Positive nitrogen balance

A

The nurse is assessing a patient's nasogastric drainage following a conventional fundoplication procedure. How does the nurse expect the drainage to appear in the first 8 hours after surgery? a. Dark brown b. Bright red mixed with brown c. Yellowish to green d. Green to clear

A

The nurse is caring for a patient who underwent gastric resection. On assessment, the nurse notes that the patient's tongue is smooth and shiny and appears "beefy." What does the nurse suspect has occurred? a. Vitamin B12 deficiency b. Anemia c. Hypovolemia d. Inadequate nutrition

A

The nurse is teaching a patient about how to control gas and odor from a colostomy. Which information does the nurse include? a. Placing a breath mint in the pouch can help. b. Place an aspirin in the colostomy. c. Do not consume buttermilk. d. Do not eat parsley.

A

The nurse is teaching a patient about what kind of stool to expect after a descending colon colostomy. The nurse tells the patient to expect to have what kind of stool? a. Similar to stool expelled from the rectum b. Thick and paste-like c. Thin and gelatin-like d. Watery

A

The nurse is to administer alvimopan to a patient with postoperative ileus. What is the action of this drug? a. Increases gastrointestinal (GI) motility b. Laxative promotes bowel movement c. Antibiotic prevents infection d. Prevents nausea and vomiting

A

The nurse on a medical-surgical unit is caring for several patients with acute cholecystitis. Which task is best to delegate to the unli- censed assistive personnel (UAP)? A. Obtain the patients' vital signs. B. Determine if any foods are not tolerated. C. Assess what measures relieve the abdominal pain. D. Ask the patients to describe their daily activity or exercise routines

A

The nursing student caring for a patient with a duodenal ulcer is about to administer a proton pump inhibitor (PPI). Which statement about this medication is true? a. These drugs should not be used for a prolonged period because they may con- tribute to osteoporotic-related fractures. b. PPIs may not be given via feeding tube. c. These drugs help prevent stress-induced ulcers. d. PPIs work by coating the stomach with a protective barrier.

A

The patient presents with abdominal discom- fort and a history of gastrointestinal cancer. Which cancer-specific laboratory studies does the nurse expect will be ordered by the health care provider? a. Oncofetal antigens (CA19-9 and CEA) b. Conjugated (direct) bilirubin and unconjugated (indirect) bilirubin c. Aspartate aminotransferase (AST) and alanine aminotransferase (ALT) d. Serum amylase and lipase

A

The patient reports episodes of diarrhea for the past 2 days. What types of bowel sounds does the nurse expect to auscultate when assessing this patient? a. Increased loud and gurgling sounds b. Decreased soft and diminished sounds c. Increased in the left lower quadrant only d. Decreased in the right upper quadrant only

A

The patient tells the nurse that she is experi- encing emotional stress related to concerns about her children and husband and whether she will be able to return to her job. Which GI condition is she at increased risk for? a. Exacerbation of irritable bowel syndrome b. Nausea accompanied with vomiting c. Esophageal ulcers d. Hiatal hernia

A

The patient with a gastric ulcer presents with a rigid, tender, and painful abdomen. He prefers lying in a knee-chest (fetal) position. What is the nurse's priority action at this time? a. Notify the health care provider. b. Administer opioid pain medication. c. Reposition the patient supine. d. Measure the abdominal circumference.

A

The patient with acute pancreatitis experiences abdominal pain. What is the best intervention to begin managing this pain? a. IV opioids by means of patient-controlled analgesia (PCA) b. Oral opioids such as morphine sulfate given as needed c. Intramuscular opioids given every 6 hours d. Oral hydromorphone (Dilaudid) given twice a day

A

The patient with esophageal cancer has an excess of HER2 protein on the cell surface. What therapy does the nurse expect will be ordered for this patient? a. Targeted therapy with IV trastuzumab b. Chemoradiation with chemotherapy during the first and fifth weeks c. Radiation therapy alone to shrink the tumor d. Nutrition and swallowing therapy to prevent malnutrition

A

The unlicensed assistive personnel (UAP) is providing oral care for a patient after resection of an oral tumor. Which would the nurse instruct the UAP to report immediately? a. There was unusual odor from the patient's mouth. b. Oral care was provided every 4 hours. c. The suture site appears to be intact. d. The patient's secretions were thick before oral care was given.

A

What is the nature of pain associated with diverticulitis? a. Intermittent becoming progressively steady b. Sharp and continuous c. Localized to the right upper quadrant d. Severe and incapacitating

A

What is the primary focus of care after conventional surgery for hiatal hernia? a. Prevention of respiratory complications b. Pain management c. Management of fluid balance d. Teaching the patient self-care activities

A

What is the priority focus in caring for a patient with advanced liver cancer? a. Hospice and end-of-life care b. Getting placed on the liver transplant list c. Hepatic arterial infusion of chemotherapy d. Cryotherapy to freeze and destroy liver tumors

A

When admitting the patient with cirrhosis, the nurse assesses for which conditions related to splenomegaly as possible complications of the disease? A. Thrombocytopenia b. Bleeding esophageal varices c. Hepatorenal syndrome d. Portal hypertensive gastropathy

A

When caring for a patient with stomatitis, what is the most important priority for the nurse to assess? a. Nutritional status b. Level of pain c. Self-care abilities d. Airway status

A

Which action should the nurse delegate to the unlicensed assistive personnel (UAP) when providing care for a patient with Crohn's disease? a. Check the patient's daily weight. b. Instruct the patient about the importance of adequate nutrition. c. Assess the patient's skin for areas of breakdown. d. Provide the patient with information about the disease process.

A

Which definition best describes morbid obesity? A. Weight that has a severely negative effect on health. B. Excessive amount of body fat when compared to lean body mass. C. Increase in body weight for height as compared to a standard. D. Excessive amount of body weight requiring surgical intervention.

A

Which diagnostic test is the gold standard for diagnosing gastritis? a. Esophagogastroduodenoscopy (EGD) with biopsy b. Computed tomography (CT) scan c. Upper gastrointestinal (GI) series d. Cholangiogram

A

Which discharge information does the nurse include for the patient who has had an intestinal obstruction caused by fecal impaction? A. Encourage the patient to report abdominal distention, nausea or vomiting, and constipation. B. Provide the patient a written description of a low-fiber diet. C. Remind the patient to limit activity. D. Remind the patient to decrease fluid intake.

A

Which discharge instruction does the nurse include for a patient after abdominoperitoneal (AP) resection? a. "Use a soft pillow to sit on whenever you sit down." b. "Lie on your back when you are resting inbed." c. "Use a rubber doughnut device for sitting on when in the car." d. "Sit in a chair for at least 4 consecutive hours a day."

A

Which intervention does the nurse delegate to the unlicensed assistive personnel (UAP) when caring for a postoperative patient with peritonitis? a. Measure intake and output. b. Assess wound drainage. c. Administer IV antibiotics. d. Teach patient about wound care.

A

Which patient is most at risk for postoperative nausea and vomiting (PONV)? a. The patient with a history of motion sickness b. The patient with a nasogastric tube c. The patient who recently experienced a weight loss of 50 pounds d. The patient who had minimally invasive surgery (MIS)

A

Which statement about a patient with a tube feeding indicates best practice for patient safety and quality care? a. If the tube becomes clogged, use 30 mL of water for flushing, while applying gentle pressure with a 50-mL piston syringe. b. Use cranberry juice to flush the tube if it is clogged. c. When administering medications, use cold water to dissolve the drug before administering it. d. Administer drugs down the feeding tube without flushing first, but flush the feeding tube after the drug is given.

A

Which statement is true about Barrett's epithelium in the patient with gastroesophageal reflux disease (GERD)? a. It is considered premalignant and is associated with higher risk for cancer. b. This new tissue is less resistant to acid so it must be removed. c. Barrett's epithelium is resistant to the development of cancer. d. Esophageal strictures are less likely to occur with this type of epithelium.

A

Which statement is true regarding the patient who has given consent for a surgical procedure? a. Information necessary to understand the nature of and reason for the surgery has been provided. b. The length of stay in the hospital has been preapproved by the managed care provider. c. Information about the surgeon's experience has been provided. d. The nurse has provided detailed informa- tion about the surgical procedure.

A

In caring for a patient with Crohn's disease (CD), the nurse observes for which complications? Select all that apply. a. Peritonitis b. Small bowel obstruction c. Nutritional and fluid imbalances d. Presence of fistulas e. Appendicitis f. Severe nausea and vomiting

A,B,C,D

Which statements does the nurse include while providing discharge instructions for a patient with giardiasis? Select all that apply. a. "Avoid contact with stool from dogs and beavers." b. "All household and sexual partners should have stool examinations for parasites." c. "Treatment will most likely consist of metronidazole (Flagyl)." d. "The infection can be transmitted to others until the amebicides kill the parasites." e. "Stools are examined 6 days after treatment to assess for eradication." f. "Be sure to bathe or shower at least every other day."

A,B,C,D

After the unlicensed assistive personnel (UAP) tells the nurse that an older patient will not eat dinner, the nurse enters the patient's room to assess the situation. Which factors likely contribute to the patient's lack of desire to eat? Select all that apply. a. An emesis basin is on the bedside table. b. The volume of the television is loud. c. The food is cold. d. Housekeeping staff is in the room disinfecting the bathroom. e. Cartons and packages are opened, and food has been cut into bite-sized pieces. F. The patient's roommate has two adults and three children visiting.

A,B,C,D,F

After initial placement of nasoduodenal and NG tubes is confirmed, how often must the placement be checked? Select all that apply. a. Before intermittent feeding b. Before medication administration c. Every 4-8 hours during feeding d. It is not necessary to recheck placement. e. According to facility policy f. Check every 12 hours after flush with saline.

A,B,C,E

When teaching a group of adult patients about measures for preventing hepatitis A (HAV), which information does the nurse include? Select all that apply. A. Perform proper handwashing, especially after handling shellfish. B. Receive immune globulin within 14 days if exposed to the virus. C. Receive the HAV vaccine before traveling to Mexico or the Caribbean. D. After exposure, HAV symptoms always let the patient know something is wrong. E. Receive the vaccine if working in a long-term care facility. F. Avoid unprotected sex with a person who has HAV.

A,B,C,E

A patient develops an active upper GI bleed. Which are the priority actions the nurse takes in caring for this patient? Select all that apply. a. Provide oxygen. b. Start 1-2 large-bore IV lines. c. Prepare to infuse 0.9% normal saline 30. solution. d. Monitor serum electrolytes. e. Prepare for nasogastric (NG) tube insertion. f. Monitor hematocrit and hemoglobin.

A,B,C,E,F

Uncontrolled gastroesophageal reflux disease (GERD) can be a cause of which adult-onset disorders? Select all that apply. a. Dental decay b. Aspiration pneumonia c. Laryngitis d. Diverticulitis e. Asthma f. Cardiac disease

A,B,C,E,F

Which clinical features are found in a malignant hyperthermia (MH) crisis? Select all that apply. a. Sinus tachycardia b. Jaw muscle rigidity c. Hypotension d. A decrease in end-tidal carbon dioxide level e. Skin mottling and cyanosis f. An extremely elevated temperature at onset

A,B,C,E,F

Which statements will the nurse include when providing health teaching for a patient with hiatal hernia? Select all that apply. a. "Elevate the head of your bed at least 6 inches for sleeping at night." b. "Remain in the upright position for several hours after eating." c. "Avoid straining or excessive vigorous exercise." d. "After surgery, you will have no dietary restriction." e. "Avoid wearing clothing that is tight around the abdomen." f. "Avoid eating in the late evening."

A,B,C,E,F

The health care provider prescribes "magic mouthwash" for a patient experiencing oral pain from cancer treatments. What is the nurse's best response when the patient asks, "What ingredients are in this concoction?" Select all that apply. a. Lidocaine b. Benadryl c. Maalox d. Tetracycline e. Nystatin f. Carafate

A,B,C,F

The nurse has received a patient in the holding area who is scheduled for a left femoral-popliteal bypass. What are the priority safety measures for this patient before surgery? Select all that apply. a. The operative limb is marked by the surgeon. b. The patient is positively identified by checking the name and date of birth. c. The patient is asked to confirm the marked operative limb. d. The patient is identified by checking the name and room number. e. The patient is instructed to verify any family members waiting. F. The patient is kept on NPO status

A,B,C,F

Which elevated laboratory test results indicate hepatic cell destruction? Select all that apply. a. Elevated serum aspartate aminotransferase (AST) b. Elevated serum alanine aminotransferase (ALT) c. Elevated lactate dehydrogenase (LDH) d. Decreased serum total bilirubin e. Increased fecal urobilinogen f. Increased International Normalized Ratio (INR)

A,B,C,F

The nurse is assessing a male patient with cirrhosis. Which male-specific characteristics does the nurse expect to find? Select all that apply. a. Gynecomastia b. Testicular atrophy c. Ascites d. Impotence e. Spider angiomas f. Petechiae

A,B,D

Which drugs are available for long-term treatment of obesity? Select all that apply. a. Phentermine-topiramate b. Lorcaserin c. Sibutramine d. Orlistat e. Diethylpropion f. Phentermine hydrochloride

A,B,D

The 79-year-old patient with type 2 diabetes is scheduled for surgery to remove his left great toe. Which risk factors for complications of surgery does the nurse assess in this patient? Select all that apply. a. Presence of chronic illnesses b. Problems with healing c. Absence of smoking history d. Dehydration e. Electrolyte imbalances f. Daily exercise routine

A,B,D,E

The nurse is collaborating with the speech- language pathologist to assist the patient with oral exercises to improve swallowing (swallowing therapy) and with the occupa- tional therapist for feeding therapy. Which key points must be included when teaching this patient? Select all that apply. a. Ask the patient to suck on a lollipop to enhance tongue strength. b. Teach the patient to reach for food parti- cles on the lips or chin using the tongue. c. Instruct the patient to place food at the front of the mouth. d. In preparation for swallowing, remind the patient to position the head in forward flexion (chin tuck). e. Monitor the patient for sealing of the lips and for tongue movements while eating. f. Check for pocketing of food under the tongue after swallowing

A,B,D,E

The nurse is providing care for a patient with cirrhosis who has massive ascites and has developed hepatopulmonary syndrome. Which elements of nursing care are appropriate for this patient? Select all that apply. a. Auscultate lungs every 4-8 hours for crackles. b. Monitor the patient's oxygen saturation. c. Elevate the head of the bed 15 degrees. d. Apply oxygen therapy. e. Weigh the patient every day. f. Lower the patient's legs and feet

A,B,D,E

The patient has a diagnosis of irritable bowel syndrome (IBS). Which forms can IBS take? Select all that apply. a. Diarrhea (IBS-D) b. Constipation (IBS-C) c. Bloating (IBS-B) d. Alternating diarrhea and constipation (IBS-A) e. Mix of constipation and diarrhea (IBS-M) f. Uncertain (IBS-U)

A,B,D,E

The postanesthesia care unit (PACU) nurse is assessing an older adult patient for postopera- tive pain. Which nonverbal manifestations by the patient suggest pain to the nurse? Select all that apply. a. Restlessness b. Profuse sweating c. Difficult to arouse d. Confusion e. Increased blood pressure f. Decreased heart rate

A,B,D,E

What information should be included in the handoff report when a patient is transferred from the OR to the postanesthesia care unit (PACU) staff? Select all that apply. a. Type and extent of surgical procedure. b. Intraoperative complications and how they were handled. c. List of usual daily medications. d. Type and amount of IV fluids and blood products given. e. Location and type of incisions, dressings, catheters, tubes, drains, or packing. f. Name, address, and phone number of next of kin.

A,B,D,E

A patient with type 1 diabetes mellitus is scheduled for surgery at 0700. Which actions must the nurse perform for this patient before he goes to the operating room? Select all that apply a. Modify the dose of insulin given based on the patient's blood glucose as ordered. b. Complete the preoperative checklist before transfer to the surgical suite. c. Teach the patient about foot care and properly fitted shoes. d. Delegate obtaining the patient's fingerstick blood glucose and vital signs to the unlicensed assistive personnel (UAP). E. Check if the patient is wearing any jewelry and call security to secure valuables if necessary. F. Place the patient on NPO status for the period ordered by the surgeon.

A,B,D,E,F

After bariatric surgery, which interventions does the nurse implement to prevent compli- cations? Select all that apply. A. Apply an abdominal binder. B. Place the patient in semi-Fowler's position. C. Keep the patient on bedrest for 24 hours. D. Monitor oxygen saturation. E. Apply sequential compression stockings. F. Observe skinfolds for redness and excoriation.

A,B,D,E,F

The nurse has given the ordered preoperative medications to the patient. What actions must the nurse take after administering these drugs? Select all that apply. a. Raise the side rails. b. Place the call light within the patient's reach. c. Ask the patient to sign the consent form. d. Instruct the patient not to get out of bed. e. Place the bed in its lowest position. f. Tell the patient that he or she may become drowsy.

A,B,D,E,F

The nurse is caring for a patient with pancre- atic cancer who had a Whipple procedure. Which interventions and assessments does the nurse implement? Select all that apply. a. Place the patient in semi-Fowler's position. b. Place the NG tube on intermittent suction. c. Monitor NG drainage, which should be bile-tinged and contain blood. d. Keep the patient NPO. e. Check blood glucose often. f. Monitor for signs of hypovolemia to prevent shock

A,B,D,E,F

Which people need immunization against hepatitis B (HBV)? Select all that apply. a. People who have unprotected sex with more than one partner b. Men who have sex with men c. Any patient scheduled for a surgical procedure d. Firefighters e. Health care providers f. Patients prescribed immunosuppressant drugs

A,B,D,E,F

Which signs/symptoms in an older adult can be an indication of "failure to thrive"? Select all that apply. a. Weakness b. Slow walking speed c. Decreased meal enjoyment d. Low physical activity e. Unintentional weight loss f. Exhaustion

A,B,D,E,F

Which statements about peritonitis are true? Select all that apply. a. Peritonitis is caused by contamination of the peritoneal cavity by bacteria or chemicals. b. Continuous ambulatory peritoneal dialysis (CAPD) can cause peritonitis. c. White blood cell counts are often decreased with peritonitis. d. Abdominal wall rigidity is a classic finding in patients with peritonitis. e. Chemical peritonitis is caused by leakage of pancreatic enzymes or gastric acids. f. Respiratory problems can be caused by increased abdominal pressure against the diaphragm.

A,B,D,E,F

Which statements about pancreatic cancer are accurate? Select all that apply. a. Venous thromboembolism (VTE) is a com- mon complication of pancreatic cancer. b. Pancreatic cancer often presents in a slow and vague manner. c. Severe pain is an early feature of this disease. d. There are no specific blood tests to diagnose pancreatic cancer. e. Chemotherapy is the treatment of choice for pancreatic cancer. f. Chronic pancreatitis predisposes a patient to pancreatic cancer.

A,B,D,F

A patient cared for in the postanesthesia care unit (PACU) had a colostomy placed for treatment of Crohn's disease. The nurse assesses that an abdominal dressing is 25% saturated with serosanguineous drainage and the incision is intact. An IV is infusing with D5/lactated Ringer's at 100 mL/hr through a 20-g peripheral IV access. Auscultation of abdomen reveals hypoactive bowel sounds in all four quadrants, abdomen soft, and no distention. Foley catheter is in place and draining yellow urine with sediment, 375 mL output in Foley bag. Which body systems have been assessed by the nurse? Select all that apply. A. Renal/urinary b. Gastrointestinal c. Respiratory d. Musculoskeletal e. Integumentary f. Cardiovascular

A,B,E

A postoperative patient in the postanesthesia care unit (PACU) has had an open reduction internal fixation of a left fractured femur. Vital signs are blood pressure 87/49 mm Hg, heart rate 100/min sinus rhythm, respirations 22/min, and temperature 98.3°F (36.8oC). The Foley catheter has a total of 110 mL of clear yellow urine in the last 4 hours. Which body systems have been assessed by the nurse? Select all that apply. a. Respiratory b. Cardiovascular c. Neurovascular d. Integumentary e. Renal/urinary f. Gastrointestinal

A,B,E

After receiving implantation of radioactive substances (brachytherapy) in the floor of the mouth, the nurse will place the patient on radiation transmission precautions that include which of the following key elements? Select all that apply. a. Place the patient in a private room with lead-lined walls. b. Visitors may stay only less than 30 minutes a day. c. Pregnant women and children can visit for only 10 minutes a day. d. Anything that goes into the patient's room may not come out. e. A tracheostomy may be needed for edema and increased secretions. f. The patient will be expected to consume foods high in iron content.

A,B,E

A patient with ulcerative colitis (UC) who has had an ileostomy is being discharged home. Which statements by the patient indicate the discharge teaching has been effective? Select all that apply. A. "I will avoid foods that cause gas." B. "I will call the health care provider if I have a fever over 101°F (38.3°C)." c. "I will change the adhesive for the appli- ance daily." D. "I know the pouch needs emptying when I feel pain in that area." E. "I will call the health care provider if I feel like my heart is beating fast." F. "I will include adequate amounts of salt and water in my diet because an ileostomy causes their loss."

A,B,E,F

The health care provider removed a patient's original surgical dressing 2 days after surgery and is discharging the patient home with daily dressing changes. Which actions will the nurse take for this patient's discharge teaching? Select all that apply. a. Ask the patient's family or significant other to observe the dressing change. b. Ask the UAP to get dressing supplies for the patient. c. Instruct that the drainage will appear serosanguineous. d. Instruct the patient to go to the emergency department (ED) for problems related to dressing changes. e. Have the case manager arrange for a home health nurse to ensure that dressing changes are done and there are no complications or infection. f. Teach the patient and family the signs and symptoms of infection.

A,B,E,F

The nurse is caring for the patient with acute appendicitis. Which interventions will the nurse perform? Select all that apply. a. Maintain the patient on NPO status. b. Administer IV fluids as prescribed. c. Apply warm compresses to the right lower abdominal quadrant. d. Maintain the patient in the supine position. e. Administer laxatives. f. If tolerated, maintain the patient in a semi-Fowler's position.

A,B,F

What are the major focus areas for interventions aimed at treating malabsorption syndromes? Select all that apply. a. Avoiding substances that aggravate malabsorption b. Use of complementary and alternative therapies c. Supplementation of nutrients d. Assessment and supplementation of coping strategies. e. Curative radiation therapy. f. Chemotherapy to prevent cancer

A,C

Which interventions must the operating room (OR) nurses provide for patient physiological integrity during the intraoperative period? Select all that apply. a. Apply padding to the OR bed to protect skin integrity. b. Communicate patient's fears about anesthesia to the nurse anesthetist. c. Monitor patient's airway, vital signs, electrocardiogram (ECG), and oxygen saturation during and after sedation. d. Assess and document skin condition before transferring patient to the postanesthesia care unit (PACU). e. Ensure that patient's wishes about advance directives are respected. f. Reposition the patient every 2 hours, especially for very long surgeries.

A,C,D

Which tasks for patient care of a male with an inguinal hernia repair should the nurse delegate to the UAP? Select all that apply. a. Record all patient intake and output. b. Perform an intermittent catheterization if the patient is unable to urinate. c. Remind the patient to consume at least 1500 mL per day. d. Assist the patient to stand while urinating. e. Assess the patient's level of pain. f. Teach the patient to keep the wound dry and clean it with antibacterial soap and water

A,C,D

A patient is scheduled for an abdominal CT scan with contrast. What preprocedural teaching should the nurse give this patient? Select all that apply. a. "You will have an IV placed for injection of the contrast." b. "You may experience the presence of borborygmus." c. "You may feel warm and flushed or experience a metallic taste after the injection." d. "The CT technician may ask you to hold your breath while a series of images are taken." e. "If you are claustrophobic, we can give you a mild sedative before the procedure." f. "The test takes 30-45 minutes."

A,C,D,E

Drug therapy for peptic ulcer disease (PUD) is implemented for which purposes? Select all that apply. a. Pain relief b. Rebuild the mucosal lining of the stomach c. Eliminate H. pylori infection d. Heal ulcerations e. Prevent recurrence f. Control bleeding

A,C,D,E

The nurse is collaborating with the dietitian to provide diet teaching for a patient with chronic pancreatitis and his family. Which are important teaching points for this teaching plan? Select all that apply. A. The patient will need increased calorie intake (4000-6000) per day to maintain weight. B. Be sure to include foods that are high in fat because they are essential for healing. C. Alcohol intake should be avoided. D. Provide a bland diet with frequent meals. E. Avoid irritating substances such as caffein- ated beverages which stimulate the GI system. F. Add rich foods to the diet to help meet the caloric requirements.

A,C,D,E

Which are advantages of minimally invasive surgery (MIS) laparoscopic cholecystectomy? Select all that apply. a. Complications are uncommon. b. The mortality is similar to traditional cholecystectomy. c. Patients recovery more rapidly. d. Postoperative pain is less severe. e. Bile duct injuries are rare. f. IV antibiotics are never needed because of decreased infection rates

A,C,D,E

Which are possible complications of chronic gastritis? Select all that apply. a. Pernicious anemia b. Thickening of the stomach lining c. Gastric cancer d. Decreased gastric acid secretion e. Peptic ulcer disease f. Local irritation from radiation therapy

A,C,D,E

Which are symptoms of early dumping syndrome? Select all that apply. a. Tachycardia b. Confusion c. Desire to lie down d. Syncope e. Occurs 30 minutes after eating f. Palpitations

A,C,D,E

The nurse is preparing discharge instructions for a patient after bariatric surgery. Which key teaching points will the nurse be sure to include? Select all that apply. A. Diet progression, importance of vitamin supplements and hydration b. Taking analgesics every 4 hours whether there is pain or not c. Restrictions on activities such as heavy lifting d. Following the health care provider's instructions for progression of activity e. Covering the wound during bath or shower f. Reporting signs of infection to the health care provider immediately

A,C,D,E,F

The nurse is providing discharge teaching for a patient after a gastrectomy. Which teaching points will the nurse include to help the patient minimize dumping syndrome? Select all that apply. a. "Eat small frequent meals." b. "Drink an 8-ounce glass of water with each meal." c. "Eliminate alcohol and caffeine from your diet." d. "Lie flat for a short time after eating." e. "Take B12 injections as prescribed by your health care provider." f. "Begin a smoking cessation program."

A,C,D,E,F

The patient with obesity asks the nurse about the benefits of exercise. Which benefits should the nurse teach about? Select all that apply. a. Increased lean muscle b. Increased body fat c. Decreased body weight d. Lower death rates at any age e. Increased cardiovascular health f. Improved psychological well-being

A,C,D,E,F

To reduce the incidence of patients with a known history or risk of malignant hyperthermia (MH), what best practices are put in place in the operating room? Select all that apply. a. List of medications available for emergency treatment of MH b. Genetic counseling after each episode of MH c. Dedicated MH cart with treatment medications d. Treatment before, during, and after surgery if the patient has a known history or risk e. Additional nursing support on call if MH develops f. Available MH hotline number

A,C,D,E,F

Which key points does the nurse include when teaching the patient with cirrhosis and his family about drug therapy before discharge? Select all that apply. a. "Do not take over-the-counter medications unless approved by your health care provider." b. "The beta-blocker called propranolol will cause your heart rate to increase." c. "The lactulose syrup should cause you to have two to three bowel movements every day." d. "Take your furosemide early in the day so that it does not keep you up at night." e. "Report any muscle weakness or lightheadedness to your health care provider right away." f. "Your health care provider may prescribe a potassium supplement to replace losses."

A,C,D,E,F

Which strategies does the nurse expect to implement in the management of dumping syndrome? Select all that apply. a. Provide more frequent smaller meals. b. Provide a high-carbohydrate diet. c. Eliminate liquids ingested with meals. d. Increase protein and fat in the diet. e. Administer acarbose to decrease carbohydrate absorption. f. Subcutaneous octreotide (Sandostatin) 2-3 times a day, 30 minutes before meals as prescribed.

A,C,D,E,F

A patient with peptic ulcer disease is receiving Maalox. Which actions does the nurse take when administering this medication? Select all that apply. a. Give the medication 2 hours after the patient's meal. b. Instruct the patient to lie on the left side after taking an antacid. c. Assess the patient for a history of renal disease before giving Maalox. d. Assess the patient for a history of heart failure before giving Maalox. e. Observe the patient for the side effect of constipation. f. Do not give other drugs within 1-2 hours of antacids.

A,C,D,F

The nurse is caring for a patient scheduled for a colonoscopy in 3 days after discharge. What does the nurse teach the patient about prepa- rations for this diagnostic test? Select all that apply. a. "Take only clear liquids the day before your colonoscopy." b. "Drink lots of red, orange, or purple (grape) beverages the day before the test." c. "You should take nothing by mouth for 4-6 hours before the test." d. "Do not take aspirin, NSAIDs, or antico- agulants for several days before the test." e. "After you drink the bowel-cleansing solu- tion, you may develop constipation in 1-2 days." f. "You will have an IV placed to receive medication to help you relax during the procedure."

A,C,D,F

The nurse is giving discharge instructions to a patient after a fundoplication procedure. The patient is instructed to avoid which activities? Select all that apply. a. Drinking carbonated beverages b. Beginning clear liquids when peristalsis has returned c. Drinking with a straw d. Eating gas-producing foods e. Eating soft foods that are easy to swallow f. Chewing gum

A,C,D,F

The nurse is teaching a patient about health promotion and maintenance to prevent gastritis. Which information does the nurse include? Select all that apply. A. "A balanced diet can help prevent gastritis." B. "To prevent gastritis, you should limit your intake of salt." C. "If you stop smoking, there is less of a chance that you will develop gastritis." D. "Yoga has been found to be effective in preventing gastritis." E. "Although regular exercise is good for you, it has not been found to influence the prevention of gastritis." F. "To prevent gastritis, alcohol should be avoided."

A,C,D,F

Which are interventions for the medical- surgical nurse to use in preventing hypoxemia for the postoperative patient? Select all that apply. a. Monitor the patient's oxygen saturation. b. Position the patient supine. c. Encourage the patient to cough and breathe deeply. D. Get the patient ambulating as soon as possible. E. Instruct the patient to rest as much as possible. F. Remind the patient to use incentive spirometry every hour while awake.

A,C,D,F

Which are potential cardiovascular complica- tions for a patient after surgery for a Whipple procedure? Select all that apply. a. Thrombophlebitis b. Pulmonary embolism c. Myocardial infarction d. Heart failure e. Renal failure f. Hemorrhage at anastomosis sites with hypovolemia

A,C,D,F

Which information does the nurse include when teaching a patient with new-onset hemorrhoids about prevention of hemorrhoid flare-up? Select all that apply. a. "Increase the fiber in your diet to prevent constipation." b. "Do not participate in any physical exercise." c. "Maintain a healthy weight." d. "Increase your amount of fluid intake." e. "Prolonged sitting or standing will not affect the development of hemorrhoids." f. "Avoid stimulant laxatives."

A,C,D,F

Which patient would be a candidate for moderate sedation? Select all that apply. a. Endoscopy b. Cesarean section delivery c. Closed fracture reduction d. Cardiac catheterization e. Abdominal surgery f. Cardioversion

A,C,D,F

Which patients are at higher risk for develop- ment of oral cavity disorders? Select all that apply. a. Homeless veteran b. Overweight adult with type 2 diabetes c. Older adult living in a long-term care facility d. Middle-aged smoker who is alcoholic e. Underweight teen with anorexia f. Middle-aged adult with history of working outdoors for over 20 years

A,C,D,F

Which statements are true regarding laparo- scopic cholecystectomy? Select all that apply. A. Laparoscopic cholecystectomy is consid- ered the "gold standard" and is performed far more often than the traditional open approach. B. Patients with chronic lung disease or heart failure who are unable to tolerate the oxygen used in the laparoscopic procedure are examples of patients who have the open surgical approach (abdominal laparotomy). C. Removing the gallbladder with the laparoscopic technique reduces the risk of wound complications. D. Patients who have their gallbladders removed by the laparoscopic technique should be taught the importance of early ambulation to promote absorption of carbon dioxide. E. Use of laparoscopic cholecystectomy puts the patient at increased risk for bile duct injuries. F. After a laparoscopic cholecystectomy, assess the patient's oxygen saturation level frequently until the effects of the anesthesia have passed.

A,C,D,F

The health care provider has prescribed qua- druple therapy for a patient with PUD due to H. pylori infection. Which drugs will the nurse expect to administer? Select all that apply. A. A proton pump inhibitor (PPI) b. Sucralfate c. Two antibiotics d. A histamine H2 antagonist e. Bismuth f. Two antacids

A,C,E

The nurse is providing care for a patient who had a minimally invasive inguinal hernia repair (MIIHR) through a laparoscope. What must the nurse be sure to include during discharge teaching? Select all that apply. a. "Avoid strenuous activity for several days before returning to work and normal activity." b. "Limit your fluid intake to 1000 to 1200 mL per day." c. "Take your prescribed stool softener to prevent constipation." d. "You will need to learn how to insert a straight catheter for the first week after surgery." e. "Observe your incisions for signs of infection or increased pain and report these immediately." f. "This procedure is fairly painless, so you should not need any pain drugs."

A,C,E

Which criteria make a patient a candidate for surgical treatment of obesity? Select all that apply. a. Repeated failure with nonsurgical interventions. B. Waist circumference greater than 40 inches. C. A BMI greater than or equal to 40. D. Waist-to-hip ratio of greater than 0.95. e. Weight more than 100% above ideal body weight. F. BMI of 35 or greater along with additional risk factors

A,C,E,F

Which gastrointestinal changes occur in older adults? (Select all that apply.) a. Increased hydrochloric acid secretion b. Decreased absorption of iron and vitamin B12 c. Decreased peristalsis may cause constipation d. Increased cholesterol synthesis e. Decreased lipase with decreased fat digestion f. Increased risk for constipation or impaction

A,C,E,F

Which patients would not be considered candidates for a liver transplant? Select all that apply. A. Patient with metastatic tumors. B. Patient with type 2 diabetes. C. Patient with severe respiratory disease. D. Patient with chronic liver disease. E. Patient with advanced cardiac disease. F. Patient who is unable to follow instructions.

A,C,E,F

Which are the two most common manifestations of gastroesophageal reflux disease (GERD)? Select all that apply. a. Dyspepsia b. Eructation c. Water brash d. Regurgitation e. Odynophagia f. Flatulence

A,D

What techniques are essential to performing a proper surgical scrub of the hands by the surgeon, assistants, and scrub nurse? Select all that apply. a. Use a broad-spectrum, surgical antimicrobial solution. b. Scrub for 2 minutes, followed by a rinse with water. c. Use an alcohol-based antimicrobial solution. d. Hold hands and arms so that water runs off, not up or down the arms. e. Scrub for 3-5 minutes, followed by a rinse with water. f. Keep hands below the elbows during the scrub and rinse.

A,D,E

Which postoperative interventions will the nurse typically teach a patient to prevent complications following surgery? Select all that apply. a. Range-of-motion exercises b. Massaging of lower extremities c. Taking pain medication only when experiencing severe pain d. Incision splinting e. Deep-breathing exercises f. Use of incentive spirometry

A,D,E,F

Which statements about gastroesophageal reflux disease (GERD) are correct? Select all that apply. A. Overweight and obese patients are at an increased risk. B. Thin and underweight patients are at an increased risk. C. It is a common disorder in the Asian and Hispanic populations. D. There is a high incidence in patients who eat mostly hot and spicy foods. E. It is a common upper gastrointestinal disorder in the United States. F. Eating large meals predisposes a patient to reflux.

A,E,F

The nurse is caring for an older adult client who experiences an exacerbation of ulcerative colitis with severe diarrhea that have lasted a week. For which complications will the nurse assess? (Select all that apply.) A) Dehydration B) Hypokalemia C) Skin breakdown D) Deep vein thrombus E) Hyperkalemia

ANS: A, B, C. The nurse will assess for complications such as dehydration, hypokalemia, and skin breakdown, all which can occur when diarrhea is profuse and fluid has been lost. Deep vein thrombosis and hyperkalemia are not complications that are associated with ulcerative colitis with severe diarrhea.

The nurse is performing medication reconciliation for a newly admitted client. The nurse recognizes that which drugs contribute to signs and symptoms of gastritis? (Select all that apply.) A) Aspirin, taken once daily to prevent cardiac concerns B) Naproxen, taken once daily for joint pain associated with arthritis C) Amoxicillin, taken over a 10-day period for an acute sinus infection D) Bacitracin ointment (over the counter), applied to minor scrapes on arms and legs E) Prednisone, tapered over a 14-day period to decrease inflammation associated with an acute sinus infection

ANS: A, B, E. Corticosteroids, erythromycin (E-Mycin, Erythromid), ASA (aspirin), and NSAIDs such as naproxen (Naprosyn) and ibuprofen (Motrin, Advil, Amersol, Novo-Profen)—as well as OTC products that contain aspirin or ibuprofen—are associated with contributing to symptoms associated with gastritis. Amoxicillin and bacitracin ointment are not.

A client completing radiation treatment has developed dysphagia and stomatitis. What teaching will the nurse provide? (Select all that apply.) A) Brush teeth twice daily with chemobrush. B) Thin liquids will make it easier to swallow. C) Limit alcohol consumption to three drinks per day. D) Rinse mouth with mild saline and water mix before and after eating. E) Refrain from using liquid dietary supplements, as these will irritate mucous membranes. F) Plan to eat soft foods like cheese, well-cooked legumes, peanut butter, and pudding.

ANS: A, D, F. A chemobrush is softer than a regular toothbrush. Teeth should be brushed gently twice daily. Thicker liquids are easier to swallow. Alcohol use should be discontinued. Rinsing the mouth can relieve xerostomia, which can affect the client's ability to eat. Liquid dietary supplements can help increase caloric intake. Soft foods should be eaten, since these pose the least risk to tissue integrity of mucous membranes.

When caring for a patient who has just had an upper GI endoscopy, the nurse assesses that the client has developed a temperature of 101.8°F (38.8°C). What is the appropriate nursing intervention? A) Promptly assess the patient for potential perforation. B) Ask the nursing assistant to bathe the client with tepid water. C) Administer acetaminophen (Tylenol) to lower the temperature. D) Delegate to an unlicensed assistive personnel (UAP) to retake the temperature.

ANS: A. A sudden spike in temperature following an endoscopic procedure may indicate perforation of the GI tract. The nurse should promptly conduct a further assessment of the client, being aware of other signs and symptoms of perforation, such as a sudden onset of acute upper abdominal pain; a rigid, board-like abdomen; and developing signs of shock.

The community clinic nurse is discussing risk factors for esophageal cancer with a group of clients. Which client behavior requires further teaching? A) Smokes 1 pack of cigarettes daily B) Walks at the shopping mall three times weekly C) Elevates pillows at night D) Eats a small snack each night before bedtime

ANS: A. Tobacco use is one of the primary risk factors for esophageal cancer (along with obesity). This client behavior requires teaching about lifestyle risks that could increase the risk for esophageal cancer. Other reported client behaviors are acceptable and do not increase risk for esophageal cancer.

Which emergency care does the nurse recognize that will be implemented for a client with malignant hyperthermia? Select all that apply. A) Removal of endotracheal tube B) Cessation (stopping) of surgery, when possible C) Insertion of Foley catheter to monitor urine output D) Transfer of client to intensive care unit when stabilized E) Assess arterial blood gases (ABGs) for respiratory alkalosis F) Use active cooling techniques, such as a cooling blanket, and ice packs around the axillae and groin

ANS: B, C, D, F. The ET tube should stay in place, and surgery should be discontinued if possible. A Foley catheter should be inserted to monitor urine output. The client should be transferred to the ICU after stabilization. ABGs will be monitored, but for metabolic acidosis (not respiratory alkalosis). Cooling techniques should be used on the axillae and groin, and may also be applied to the neck and head.

A client who recently had laparoscopic surgery to treat a ruptured appendix has developed subsequent peritonitis. The client currently has two Jackson-Pratt drains placed in the abdomen. Which finding(s) would the nurse report immediately to the surgeon? (Select all that apply.) A) Serosanguineous drainage B) Fever C) Cloudy drainage D) Painful abdominal distension E) Pain level "3" on a scale of 1 to 10

ANS: B, C, D. Fever, cloudy drainage, and painful abdominal distention can be signs of infection that should be immediately reported to the surgeon. Serosanguineous drainage is expected at this time, as is a manageable pain level of "3" on a 1 to 10 scale.

A client with obesity tells the nurse, "I would not be overweight if it weren't for my genes." What is the appropriate nursing response? (Select all that apply.) A) "Genes are responsible for obesity." B) "Tell me about your family history." C) "Let's talk about your nutrition intake." D) "How do you feel about exercise?" E) "You should get bariatric surgery."

ANS: B, C, D. Learning about the client's family history can be helpful, as there are studies that indicate that mutations in many genes may be associated with the development of obesity (although they are not solely responsible for such). Talking about the client's nutrition intake and feelings about exercise opens lines of communication to also explore other factors that may contribute to the client's nutrition status.

Which assessment data finding for a client scheduled for total knee replacement surgery is most important for the nurse to communicate to the surgeon and the anesthesia provider before the procedure? Select all that apply. A) The oxygen saturation is 97%. B) The serum potassium level is 3.0 mEq/L (3.0 mmol/L). C) The client took a total of 1300 mg of aspirin yesterday. D) The client requests to talk with a registered dietitian about weight loss. E) The client took a regularly scheduled antihypertensive drug with a sip of water 2 hours ago. F) After receiving the preoperative medications, the client tells the nurse that he lied on the assessment form and that he really is a current smoker

ANS: B, C, F. This is a low potassium value (3.0 mEq/L) which should be communicated to the surgeon and anesthesia provider prior to surgery. Taking aspirin prior to surgery can increase the risk for bleeding. This should be communicated to the surgeon and anesthesia provider prior to surgery. The client's smoking status can change important assessment information collected by the surgeon and anesthesia provider; therefore, this should be immediately communicated. The oxygen saturation level is normal, and it is acceptable that the regularly scheduled antihypertensive was taken with a sip of water 2 hours ago.

The nurse is caring for a client who has had paracentesis performed. Which nursing intervention is appropriate? (Select all that apply.) A) Keep head of bed flat. B) Measure, describe, and record drainage. C) Ambulate 30 minutes post-procedure. D) Weigh client. E) Label fluid container and send for laboratory analysis.

ANS: B, D, E. Following paracentesis, the nurse should position the patient in bed with the head of the bed elevated; monitor vital signs; measure, describe, and record drainage; maintain bed rest per protocol; weigh the client; and label the fluid container to send for laboratory analysis.

When taking a history for a client with GI problems, which daily client behavior requires further nursing assessment? (Select all that apply.) A) Eats multiple servings of vegetables B) Takes 800 mg of ibuprofen for arthritic pain C) Walks 30 minutes D) Chews tobacco E) Takes senna to assist with bowel movements F) Listens to music to promote relaxation

ANS: B, D, E. Taking routine or larger amounts of NSAIDs places the client at risk for GI distress. Chewing tobacco places the client at risk for oral cancer. Senna is an over-the-counter herbal laxative. Taking certain herbal preparations can affect appetite, absorption, and elimination. Other behaviors are healthy and do not require further nursing assessment.

The nurse is caring for a client who has been diagnosed with esophageal cancer. The client appears anxious and asks the nurse, "Does this mean I am going to die?." Which nursing responses are appropriate? (Select all that apply.) A) "No, surgery can cure you." B) "It sounds like death frightens you." C) "Let me call the hospital chaplain to talk with you." D) "You can beat this disease if you just put your mind to it." E) "Let me sit with you for awhile and we can discuss how you are feeling about this."

ANS: B, E. "B" and "E" are therapeutic responses that allow the client to express feelings and fears. "A" is nontherapeutic because it gives false hope. Although the nurse may take the action listed in "C" later in the interaction, the immediate need is to explore the client's feelings and then determine how to best meet those needs. The nurse should not assume that the client immediately wishes to see a chaplain. "D" is a nontherapeutic response that does not address the client's feelings and fears.

The community nurse is talking with four clients who have reported digestive concerns. Which client does the nurse recognize as most likely to experience gallstone production? (Select all that apply.) A) 23-year-old Caucasian vegetarian who is a dancer B) 35-year-old American Indian who works in construction C) 48-year-old Canadian who manages a fast food restaurant D) 59-year-old Asian American who is an investment banker E) 64-year-old Mexican American who resides with grandchildren

ANS: B, E. The highest frequency of gallstone production in North America lies among the American-Indian and Mexican-American populations.

"The nurse is performing an assessment on a client who has arrived in the preoperative holding area. Which client statement requires immediate nursing intervention? A) "I am a little bit anxious about my surgery." B) "When I eat shrimp, my tongue swells up and I have difficulty breathing." C) "This left knee replacement will help me to walk much more comfortably again." D) "Before I get discharged home, I want to have my eyeglasses and hearing aids returned."

ANS: B. An allergy to iodine or shellfish indicates a risk for a reaction to the agents used to clean the surgical area. With this knowledge about the client, the nurse must intervene immediately

The nurse is caring for a client who is jaundiced and reports pruritus. Which intervention will the nurse include in the plan of care? A) Monitor the client's vital signs and intake and output. B) Instruct the client to scratch with knuckles instead of nails. C) Assist the client with a hot bath and apply moisturizer. D) Encourage the client to eat a high-protein, high-cholesterol diet.

ANS: B. Pruritus occurs as a result of an accumulation of bile salts under the skin. Scratching with the knuckles instead of the nails preserves the skin's integrity and prevents tearing.

The nurse is caring for four clients. Which is at the highest risk for development of oral cancer? A) 32-year-old client with ankle fracture B) 41-year-old with human papilloma virus (HPV) infection C) 60-year old who quit smoking twenty years ago D) 83-year old that lives in a warm climate during the winter

ANS: B. Research indicates a correlation between specific strains of the human papilloma virus (HPV) and oral cancer. Oral cancer associated with HPV appears in the tonsillar area or along the base of the tongue in younger people. Because HPV-positive oral cancers account for a large number of oral cancer diagnoses, routine oral assessment is essential.

The nurse is caring for a client with a bleeding duodenal ulcer who was admitted to the hospital after vomiting bright, red blood. Which condition does the nurse anticipate when the client develops a sudden, sharp pain in the midepigastric region and a rigid, board-like abdomen? A) Pancreatitis B) Ulcer perforation C) Small bowel obstruction D) Development of additional ulcers

ANS: B. The body reacts to perforation of an ulcer by immobilizing the area as much as possible. This results in board-like abdominal rigidity, usually with extreme pain. Perforation is a medical emergency requiring immediate surgical intervention because peritonitis develops quickly after perforation. A small bowel obstruction would not cause midepigastric pain. The development of additional ulcers or pancreatitis would not cause a rigid, board-like abdomen.

The community nurse is talking with a group of individuals about colorectal cancer (CRC) risk factors. Which community participant is at the highest risk for development of CRC? A) 23-year-old vegetarian B) 30-year old with Crohn's disease C) 39-year old with no family history of cancer D) 46-year old with genetic predisposition to cancer

ANS: B. The major risk factors for the development of colorectal cancer (CRC) include being older than 50 years, genetic predisposition, personal or family history of cancer, and/or diseases that predispose the patient to cancer such as familial adenomatous polyposis (FAP), Crohn's disease, and ulcerative colitis. Only a small percentage of colorectal cancers are familial and transmitted genetically.

The nurse is caring for an older adult client who reports being "afraid to get hooked" on opioid pain medication after surgery. What is the appropriate nursing response? Select all that apply. A) "No one ever gets hooked on these drugs." B) "Don't worry, I won't give you any opioid medications." C) "Have you had concerns with drug dependence in the past?" D) "Tell me what makes you most fearful about taking opioid medication." E) "There are ways we can keep you from becoming dependent on these drugs." F) "Older adults are much less likely to rely on pain medications than younger people."

ANS: C, D. Asking the client a direct question about possible prior drug abuse allows the client to identify to the nurse whether he or she has had a problem in the past. Although a prior social history has been taken, it is not unthinkable that the client is just now ready to disclose his or her full history, particularly if he or she is embarrassed about historical drug dependence. Providing an open-ended statement about what makes the client fearful about taking opioids allows the client to express feelings, which will guide the nurse about the underlying reason for the client's concerns.

The hospice nurse is caring for a client with pancreatic cancer who has been given two to three months to live. What is the appropriate nursing response when the client's spouse states, "I know he is going to get better." A) Use therapeutic silence, and say nothing. B) "Your spouse will die in 2-3 months." C) "Let's talk about how you are feeling about your spouse's prognosis." D) "Yes, if your spouse adheres to the entire treatment plan, recovery is possible."

ANS: C. At different times within the conversation, the nurse may use therapeutic silence, yet the appropriate response at this time is to open the conversation so the spouse can freely express feelings. This can lead the nurse to clarify information, or nonjudgmentally support the spouse and client through this prognosis.

What client teaching will the nurse provide regarding postoperative leg exercises, to minimize the risk for development of deep vein thrombosis after surgery? A) Only perform each exercise one time to prevent overuse. B) Begin exercises by sitting at a 90-degree angle on the side of the bed. C) Point toes of one foot toward bottom of bed, then point toes of same leg toward their face. Repeat several times, then switch legs. D) Bend knee, and push heel of foot into the bed until the calf and thigh muscles contract. Repeat several times, then switch legs.

ANS: C. Exercises should be repeated several times for each leg. Clients should begin by lying in the bed in a 45-degree angle. Pointing toes, as described, promotes circulation. Clients should push the ball of the foot into the bed until the calf and thigh muscles contract.

The nurse is caring for four clients. Which is at the highest risk for Hepatitis B infection? A) 24-year old with abdominal pain who just returned from Central America B) 40-year old who is two days postpartum and is breastfeeding C) 65-year old who reports using street drugs ten years ago when homeless D) 81-year old who donated own blood prior to a surgical procedure

ANS: C. One mode of Hepatitis B transmission is via sharing needles, syringes, or other drug-injection equipment. The individual who used street drugs is at highest risk for development of hepatitis infection.

Which client statement requires immediate nursing intervention? A) "I used to chew tobacco but quit 5 years ago." B) "I use sunscreen to cover my face and body when I'm at the beach." C) "I do not have dental insurance, so I cannot get dental check-ups." D) "I only drink alcohol on special occasions like my birthday and anniversary."

ANS: C. The primary risk factors for oral cavity cancer and/or lesions include smoking, use of tobacco products, alcohol use, and poor dental care. The risk for basal cell carcinoma is increased by excessive sun exposure. The client without dental insurance, who cannot get dental check-ups, is more likely to have poor oral health, which increases the risk for oral cancer. This requires nursing intervention to teach the client about oral health.

The nurse closely monitors the client with acute pancreatitis for which life-threatening complication? A) Jaundice B) Type I diabetes mellitus C) Abdominal pain D) Disseminated intravascular coagulation (DIC)

ANS: D. DIC is a life-threatening condition involving hypercoagulation of the blood, with consumption of clothing factors and the development of microthrombi. This can take place when complex physiologic changes in the pancreas cause the release of necrotic tissue and enzymes into the bloodstream, resulting in altered coagulation. The client with acute pancreatitis may have jaundice and abdominal pain, and develop type 1 diabetes mellitus, as the pancreas is totally destroyed, however these are not considered life-threatening conditions.

A client with rectal bleeding who is preparing to undergo a colonoscopy tells the nurse, "I am very afraid of having polyps and cancer." What is the appropriate nursing response? A) "Let's worry about that after the procedure." B) "Polyps are never cancerous, so you do not need to worry." C) "Unfortunately all polyps are malignant, so you may already have cancer." D) "It is understandable that you are fearful. Tell me what frightens you most."

ANS: D. Polyps can be benign or malignant. Telling the client to worry about the outcome after the procedure devalues his or her feelings. The nurse will respond therapeutically and allow the client to express feelings.

Adequate nutrition is required for healing after treatment for recurrent aphthous ulcers (RAU). Which client response indicates that nursing teaching has been effective? A) "I have ordered a snack of milk and pretzels." B) "I will try to drink orange juice twice per day." C) "I ordered my sandwich on a crusty roll." D) "I would like scrambled eggs and a banana for breakfast."

ANS: D. To promote healing, clients should consume foods high in protein and vitamin C. To minimize pain, salty, spicy, hard, and acidic foods should be avoided. Scrambled eggs and bananas are soft and do not contain irritating agents.

The weight loss clinic nurse identifies the concept of nutrition for a client diagnosed with obesity. Which interventions should the nurse implement? Select all that apply. 1. Ask the client about previous diet attempts. 2. Refer the client to the dietitian. 3. Discuss maintaining a sedentary lifestyle. 4. Weigh the client. 5. Assist the client to set a realistic weight loss goal.

ANSWER: 1, 2, 4, 5. 1. Knowledge of previous weight loss attempts will assist in planning a weight loss program. 2. The dietitian will monitor the nutritional intake and help in planning a nutritionally balanced diet. 3. The client should be encouraged to maintain an active lifestyle. 4. The client's weight will be useful in determining the client's progress. 5. Clients who desire weight loss frequently want a quick fix. The nurse should assist the client to determine a consistent weight loss goal in order to achieve behavior modification to maintain the weight loss. TEST-TAKING HINT: The test taker must read each option of a "select all that apply" question as a true/false question.

The nurse is teaching a class on diverticulosis. Which interventions should the nurse discuss when teaching ways to prevent an acute exacerbation of diverticulosis? Select all that apply. 1. Eat a high-fiber diet. 2. Increase fluid intake. 3. Elevate the HOB after eating. 4. Walk 30 minutes a day. 5. Take an antacid every two (2) hours.

ANSWER: 1, 2, 4. 1. A high-fiber diet will help to prevent constipation, which is the primary reason for diverticulitis. 2. Increased fluids will help keep the stool soft and prevent constipation. 3. This will not do anything to help prevent diverticulitis. 4. Exercise will help prevent constipation. 5. No medications are prescribed to prevent an acute exacerbation of diverticulitis. Antacids are used to neutralize hydrochloric acid in the stomach. TEST-TAKING HINT: This is an alternate-type question where the test taker must select more than one option. To identify the correct answers, the test taker should think about what part of the GI system is affected. Knowing diverticulosis occurs in the sigmoid colon would help eliminate options "3" and "5" because these would be secondary to stomach disorders.

The client has undergone an abdominal perineal resection of the colon for colon cancer with a left lower quadrant colostomy. Which interventions should the nurse implement? Select all that apply. 1. Assess the stoma for color every four (4) hours and prn. 2. Encourage the client to turn, cough, and deep breathe every two (2) hours. 3. Maintain the head of the bed 30 to 40 degrees elevated at all times. 4. Auscultate for bowel sounds every four (4) hours. 5. Administer pain medications sparingly to prevent addiction

ANSWER: 1, 2, 4. 1. The colostomy stoma should be assessed to determine circulation to the stoma at least every four (4) hours. A purple or bluish purple indicates that the circulation to the stoma is impaired and is a medical emergency. 2. This is an extensive surgery requiring the client to be under general anesthesia for several hours. Turning, coughing, and deep-breathing exercises done at least every two (2) hours helps to prevent pneumonia. 3. The client is not allowed to sit on the perineal area for several days and should be maintained in a side-lying position when possible. 4. The nurse should assess for bowel activity at regularly scheduled intervals. 5. Pain medication is administered to control the client's pain; the nurse is concerned with client comfort, not addiction. Poorly controlled pain is more likely to result in drugseeking behavior than adequately treated pain. TEST-TAKING HINT: "Select all that apply" questions must be answered with each option considered a true/false freestanding question. The test taker cannot use one option to rule out another.

Which signs and symptoms should the nurse report to the health-care provider for the client recovering from an open cholecystectomy? Select all that apply. 1. Clay-colored stools. 2. Yellow-tinted sclera. 3. Amber-colored urine. 4. Wound approximated. 5. Abdominal pain.

ANSWER: 1, 2, 5. 1. Clay-colored stools are caused by recurring stricture of the common bile duct, which is a sign of post-cholecystectomy syndrome. 2. Yellow-tinted sclera and skin indicate residual effects of stricture of the common bile duct, which is a sign of post- cholecystectomy syndrome. 3. Amber-colored urine is a normal finding for a client recovering from an open cholecystectomy, so this does not warrant intervention by the nurse. 4. An approximated wound indicates the incision is intact and does not warrant intervention by the nurse. 5. Abdominal pain indicates a residual effect of a stricture of the common bile duct, inflammation, or calculi, which is a sign of post-cholecystectomy syndrome. TEST-TAKING HINT: The test taker must use knowledge of anatomy to answer this question. All answer options have something to do with the abdominal area, and the common bile duct is anatomically near the hepatic duct, which causes liver signs/symptoms.

The nurse is planning the care of a client who has had an abdominal-perineal resection for cancer of the colon. Which interventions should the nurse implement? Select all that apply. 1. Provide meticulous skin care to stoma. 2. Assess the flank incision. 3. Maintain the indwelling catheter. 4. Irrigate the (JP) drains every shift. 5. Position the client semirecumbent.

ANSWER: 1, 3, 5. 1. Colostomy stomas are openings through the abdominal wall into the colon, through which feces exit the body. Feces can be irritating to the abdominal skin, so careful and thorough skin care is needed. 2. There are midline and perineal incisions, not flank incisions. 3. Because of the perineal wound, the client will have an indwelling catheter to keep urine out of the incision. 4. Jackson Pratt drains are emptied every shift, but they are not irrigated. 5. The client should not sit upright because this causes pressure on the perineum. TEST-TAKING HINT: The test taker could eliminate option "2" because flank and abdominal-perineal are not in the same areas. This is an alternative-type question requiring the test taker to choose more than one (1) option

The nurse is interviewing a surgical client in the holding area. Which information should the nurse report to the anesthesiologist? Select all that apply. 1. The client has loose, decayed teeth. 2. The client is experiencing anxiety. 3. The client smokes two (2) packs of cigarettes a day. 4. The client has had a chest x-ray which does not show infiltrates. 5. The client reports using herbs.

ANSWER: 1, 3, 5. 1. Loose teeth or caries need to be reported to the anesthesiologist so he or she can make provisions to prevent breaking the teeth and causing the client to possibly aspirate pieces. 2. The nurse should report any client who is extremely anxious, but the nurse can address the needs of a client experiencing expected surgical anxiety. 3. Smokers are at a higher risk for complications from anesthesia. 4. No infiltrates on a chest x-ray is a normal finding and does not have to be reported. 5. Herbs—for example, St. John's wort, licorice, and ginkgo—have serious interactions with anesthesia and with bodily functions such as coagulation. TEST-TAKING HINT: This question is an alternate-type question requiring the test taker to select more than one (1) option as the correct answer. Safety is priority for a client undergoing surgery

The 85-year-old male client diagnosed with cancer of the colon asks the nurse, "Why did I get this cancer?" Which statement is the nurse's best response? 1. "Research shows a lack of fiber in the diet can cause colon cancer." 2. "It is not common to get colon cancer at your age; it is usually in young people." 3. "No one knows why anyone gets cancer, it just happens to certain people." 4. "Women usually get colon cancer more often than men but not always."

ANSWER: 1. 1. A long history of low-fiber, high-fat, and high-protein diets results in a prolonged transit time. This allows the carcinogenic agents in the waste products to have a greater exposure to the lumen of the colon. 2. The older the client, the greater the risk of developing cancer of the colon. 3. Risk factors for cancer of the colon include increasing age; family history of colon cancer or polyps; history of IBD; genital or breast cancer; and eating a high-fat, high-protein, low-fiber diet. 4. Males have a slightly higher incidence of colon cancers than do females. TEST-TAKING HINT: The test taker should realize cancers in general have an increasing incidence with age. Cancer etiologies are not an exact science, but most cancers have some risk factor, if only advancing age.

Which data should the nurse report to the healthcare provider when assessing the oral cavity of an elderly client? 1. The client's tongue is rough and beefy red. 2. The client's tonsils are +1 on a grading scale. 3. The client's mucosa is pink and moist. 4. The client's uvula rises with the mouth open.

ANSWER: 1. 1. A rough, beefy-red tongue may indicate the client has pernicious anemia and should be evaluated by the health-care provider. 2. A score of +1 on the tonsil grading scale shows the tonsils are extending to the haryngopalatine arch, which is normal. 3. Mucosa should be pink and moist; therefore, the nurse would not need to notify the healthcare provider. 4. Symmetrical movement of the uvula is normal and should not be reported to the health-care provider.

Which problem should the nurse identify as priority for client who is one (1) day postoperative? 1. Potential for hemorrhaging. 2. Potential for injury. 3. Potential for fluid volume excess. 4. Potential for infection.

ANSWER: 1. 1. All clients who undergo surgery are at risk for hemorrhaging, which is the priority problem. 2. The client is at risk for injury, but the priority problem the first day postoperative is hemorrhaging. 3. A potential fluid imbalance would be for less fluid as a result of blood loss and decreased oral intake; it would not be for fluid volume excess. 4. Infection would be a potential problem but not priority over hemorrhaging on the first postoperative day. TEST-TAKING HINT: Remember to apply the ABCs of care: airway, breathing, and circulation. The test taker must apply testing rules when answering questions which require identifying priority problems.

Which nursing intervention is priority for the client experiencing acute pain? 1. Assess the client's verbal and nonverbal behavior. 2. Wait for the client to request pain medication. 3. Administer the pain medication on a scheduled basis. 4. Teach the client to use only imagery every hour for the pain.

ANSWER: 1. 1. Assessing verbal and nonverbal cues is the priority intervention because pain is subjective. 2. Some clients are hesitant to ask for medication or believe it is a sign of weakness to ask. 3. There are times when pain medications are given on a routine basis, but it is not the best answer because assessment takes priority. 4. Alternative therapies, such as imagery, are used in combination with medications, but they never replace medications. TEST-TAKING HINT: Options such as option "4" which have absolute words such as "only" usually can be eliminated as a correct answer. The test taker should remember to apply the nursing process, and the first step is assessment

The nurse clears the PCA pump and discovers the client has used only a small amount of medication during the shift. Which intervention should the nurse implement? 1. Determine why the client is not using the PCA pump. 2. Document the amount and take no action. 3. Chart the client is not having pain. 4. Contact the HCP and request oral medication.

ANSWER: 1. 1. Assessing why the client is not using the medication is a priority and then, based on the client's response, a plan of care can be determined. 2. The fact a client is not using pain medication warrants the nurse determining the cause so appropriate action can be taken. 3. This may or may not be why the client is not using the PCA pump. The nurse must first determine why the client is not using pain medication. 4. This may or may not be indicated, but until the nurse determines why the client is not taking the medication, this action should not be implemented. TEST-TAKING HINT: Assessment is priority when caring for a client. It is the first step of the nursing process, and if the test taker is unsure of the correct answer, it is the best choice to select.

Which physical examination should the nurse implement first when assessing the client diagnosed with peptic ulcer disease? 1. Auscultate the client's bowel sounds in all four quadrants. 2. Palpate the abdominal area for tenderness. 3. Percuss the abdominal borders to identify organs. 4. Assess the tender area progressing to nontender.

ANSWER: 1. 1. Auscultation should be used prior to palpation or percussion when assessing the abdomen. Manipulation of the abdomen can alter bowel sounds and give false information. 2. Palpation gives appropriate information the nurse needs to collect, but if done prior to auscultation, the sounds will be altered. 3. Percussion of the abdomen does not give specific information about peptic ulcer disease. 4. Tender areas should be assessed last to prevent guarding and altering the assessment. This includes palpation, which should be done after auscultation. TEST-TAKING HINT: The word "first" requires the test taker to rank in order the interventions needing to be performed. The test taker should visualize caring for the client. This will assist the test taker in making the correct choice.

The client is diagnosed with irritable bowel syndrome (IBS). Which intervention should the nurse teach the client to reduce symptoms? 1. Instruct the client to avoid drinking fluids with meals. 2. Explain the need to decrease intake of flatusforming foods. 3. Teach the client how to perform gentle perianal care. 4. Encourage the client to attend a support group meeting.

ANSWER: 1. 1. Avoidance of fluids during meals will help prevent abdominal distention, which causes symptoms of IBS. Do not confuse inflammatory bowel disease (IBD) and irritable bowel syndrome (IBS). 2. Avoidance of flatus (gas)-forming foods helps with the symptoms of IBD, not IBS. 3. Clients with IBS do have altered bowel habits such as diarrhea and constipation, but perianal care will not prevent IBS. 4. IBS does have a psychological component, but a client recently diagnosed should be taught other interventions before a psychologist is recommended

The client has end-stage liver failure secondary to alcoholic cirrhosis. Which complication indicates the client is at risk for developing hepatic encephalopathy? 1. Gastrointestinal bleeding. 2. Hypoalbuminemia. 3. Splenomegaly. 4. Hyperaldosteronism

ANSWER: 1. 1. Blood in the intestinal tract is digested as a protein, which increases serum ammonia levels and increases the risk of developing hepatic encephalopathy. 2. Decreased albumin causes the client to develop ascites. 3. An enlarged spleen increases the rate at which red blood cells (RBCs), WBCs, and platelets are destroyed, causing the client to develop anemia, leukopenia, and thrombocytopenia, but not hepatic encephalopathy. 4. An increase in aldosterone causes sodium and water retention, resulting in the development of ascites and generalized edema. TEST-TAKING HINT: Some questions require the test taker to have specific knowledge to be able to identify the correct answer. This is one of those questions

The client receiving antibiotic therapy complains of white, cheesy plaques in the mouth. Which intervention should the nurse implement? 1. Notify the health-care provider to obtain an antifungal medication. 2. Explain the patches will go away naturally in about two (2) weeks. 3. Instruct to rinse the mouth with diluted hydrogen peroxide and water daily. 4. Allow the client to verbalize feelings about having the plaques.

ANSWER: 1. 1. Candidiasis, or thrush, presents as white, cheesy plaques, which bleed when rubbed and is a side effect of antibiotic therapy. Candidiasis is treated with antifungal solution, swished around the mouth, held for at least one (1) minute, and then swallowed. Candidiasis can be prevented if Lactobacillus acidophilus is administered concurrently with antibiotic therapy. 2. White painless patches disappearing in approximately two (2) weeks are leukoplakia, caused by tobacco use, which may be cancerous and should be evaluated by an HCP. 3. A solution of hydrogen peroxide is not recommended to treat candidiasis. 4. The nurse needs to treat the client's mouth, not use therapeutic communication

The client has an eviscerated abdominal wound. Which intervention should the nurse implement? 1. Apply sterile normal saline dressing. 2. Use sterile gloves to replace protruding parts. 3. Place the client in reverse Trendelenburg position. 4. Administer intravenous antibiotic immediately (STAT).

ANSWER: 1. 1. Evisceration is a life-threatening condition in which the abdominal contents protrude through the ruptured incision. The nurse must protect the bowel from the environment by placing a sterile normal saline gauze on it, which prevents the intestines from drying out and necrosing. 2. The nurse should not attempt to replace the protruding bowel. 3. This position places the client with the head of the bed elevated, which will make the situation worse. 4. Antibiotics will not protect the protruding bowels, which must be priority. Antibiotics will be administered at a later time to prevent infection, but this is not urgent. TEST-TAKING HINT: The test taker must understand the word "evisceration" to answer this question

Which nursing problem is priority for the 76-year-old client diagnosed with gastroenteritis from staphylococcal food poisoning? 1. Fluid volume deficit. 2. Nausea. 3. Risk for aspiration. 4. Impaired urinary elimination.

ANSWER: 1. 1. Fluid volume deficit secondary to diarrhea is the priority because of the potential for metabolic acidosis and hypokalemia, which are both life threatening, especially in the elderly. 2. Nausea may occur, but it is not priority. However, excessive vomiting could lead to potential complications. 3. Risk for aspiration could result from vomiting; however, vomiting does not usually occur in food poisoning, but it may be secondary to botulism. 4. Impaired urinary elimination is not a priority. The client has diarrhea, not urine output problems. TEST-TAKING HINT: Always notice the client's age because it is usually a significant clue as to the correct answer. Prioritizing questions may have more than one (1) potential appropriate nursing problem but only one (1) has priority. Remember Maslow's hierarchy of needs.

The client with acute diverticulitis has a nasogastric tube draining green liquid bile. Which intervention should the nurse implement? 1. Document the findings as normal. 2. Assess the client's bowel sounds. 3. Determine the client's last bowel movement. 4. Insert the N/G tube at least two (2) more inches.

ANSWER: 1. 1. Green bile contains hydrochloric acid and should be draining from the N/G tube; therefore, the nurse should take no action and document the findings. 2. There is no reason for the nurse to assess the client's bowel sounds because the drainage is normal. 3. The client's last bowel movement would not affect the N/G drainage. 4. Bile draining from the N/G tube indicates the tube is in the stomach and there is no need to advance the tube further. TEST-TAKING HINT: The test taker must know what drainage is normal for tubes inserted into the body. Any type of blood or coffeeground drainage would be abnormal and require intervention by the nurse.

The client diagnosed with liver failure is experiencing pruritus secondary to severe jaundice. Which action by the unlicensed assistive personnel (UAP) warrants intervention by the nurse? 1. The UAP is assisting the client to take a hot, soapy shower. 2. The UAP applies an emollient to the client's legs and back. 3. The UAP puts mittens on both hands of the client. 4. The UAP pats the client's skin dry with a clean towel.

ANSWER: 1. 1. Hot water increases pruritus, and soap will cause dry skin, which increases pruritus; therefore, the nurse should discuss this with the UAP. 2. Applying emollient lotion will help prevent dry skin, which will help decrease pruritus; therefore, this would not require any intervention by the nurse. 3. Mittens will help prevent the client from scratching the skin and causing skin breakdown. This would not require intervention by the nurse. 4. The skin should be patted dry, not rubbed, because rubbing the skin will cause increased irritation. This action does not require intervention by the nurse. TEST-TAKING HINT: A concept accepted for most clients during a.m. care is not to use hot water because it causes dilation of vessels, which may cause orthostatic hypotension. This is not the rationale for not using hot water with a client who has pruritus, but sometimes the test taker can apply broad concepts when answering questions.

The circulating nurse notes a discrepancy in the needle count. What intervention should the nurse implement first? 1. Inform the other members of the surgical team about the problem. 2. Assume the original count was wrong and change the record. 3. Call the radiology department to perform a portable x-ray. 4. Complete an occurrence report and notify the risk manager

ANSWER: 1. 1. If the needle count does not correlate, the surgical technologist and the other surgical team members should be informed. After repeating the count, a search for the missing needle should be conducted. 2. Assuming the original count was wrong is illegal and dangerous for the client. 3. If the needle is not located, an x-ray will be done, but this is not the first intervention. 4. If the missing needle is not located, an occurrence report should be completed and sent to the risk manager, but this it is not the first intervention

The client diagnosed with a hiatal hernia is scheduled for a laparoscopic Nissen fundoplication. Which statement indicates the nurse's teaching is effective? 1. "I will have four (4) to five (5) small incisions." 2. "I will be in the hospital for at least one (1) week." 3. "I will not have any pain because this is laparoscopic surgery." 4. "I will be returning to work the day after my surgery."

ANSWER: 1. 1. In a laparoscopic Nissen fundoplication, there are four (4) to five (5) incisions approximately one (1) inch apart allowing for the passage of equipment to visualize the abdominal organs and perform the operation. 2. Many clients come through the day surgery department and go home the same day. Some clients may remain in the hospital for one (1) or two (2) days but not for a week. 3. All surgeries will result in pain for the client. 4. The client should not return to work the next day; the client should wait at least one (1) week before returning to work

The nurse is caring for clients on a medical unit. Which client information should be brought to the attention of the HCP immediately? 1. A serum sodium of 128 mEq/L in a client diagnosed with obstipation. 2. The client diagnosed with fecal impaction who had two (2) hard formed stools. 3. A serum potassium level of 3.8 mEq/L in a client diagnosed with diarrhea. 4. The client with diarrhea who had two (2) semiliquid stools totaling 300 mL.

ANSWER: 1. 1. Normal serum sodium levels are 135 to 152 mEq/L, so the client's 128 mEq/L value requires intervention. 2. The client diagnosed with a fecal impaction is beginning to move the stool; this indicates an improvement. 3. Normal potassium levels are 3.5 to 5.5 mEq/L. A level of 3.8 mEq/L is within normal limits and does not require intervention. 4. This client has been having diarrhea and now is having semiliquid stools, so this client is getting better. TEST-TAKING HINT: The test taker must determine if the client is experiencing a potentially life-threatening complication, such as potential for seizures. Answer options "2," "3," and "4" are expected for the disease process and are normal or show improvement.

The nurse working in a skilled nursing facility is collaborating with the dietitian concerning the meals of an immobile client. Which foods are most appropriate for this client? 1. Oatmeal and wheat toast. 2. Cream of wheat and biscuits. 3. Cottage cheese and canned peaches. 4. Tuna on a croissant and applesauce.

ANSWER: 1. 1. Oatmeal and wheat toast are highfiber foods and are recommended for clients who are immobile to help prevent constipation. 2. Cream of wheat and biscuits are low-fiber foods. 3. Cottage cheese and canned peaches are low-fiber foods. 4. Tuna is a good source of protein for the client, but croissants have a high fat content and are a factor in weight gain if consistently eaten. Applesauce is low in fiber

The clinic nurse is caring for a client who is 67 inches tall and weighs 100 kg. The client complains of occasional pyrosis, which resolves with standing or with taking antacids. Which treatment should the nurse expect the HCP to order? 1. Place the client on a weight loss program. 2. Instruct the client to eat three (3) balanced meals. 3. Tell the client to take an antiemetic before each meal. 4. Discuss the importance of decreasing alcohol intake.

ANSWER: 1. 1. Obesity increases the risk of pyrosis (heartburn); therefore, losing weight could help decrease the incidence. 2. Eating small, frequent meals along with decreased intake of spicy foods have been linked to the prevention of heartburn (pyrosis). 3. Antiemetics decrease nausea, which does not occur with heartburn. Antacids neutralize the acid of the stomach and are used to treat heartburn. 4. Drinking alcoholic beverages increases heartburn and should be avoided, not decreased.

The nurse is caring for an adult client diagnosed with GERD. Which condition is the most common comorbid disease associated with GERD? 1. Adult-onset asthma. 2. Pancreatitis. 3. Peptic ulcer disease. 4. Increased gastric emptying

ANSWER: 1. 1. Of adult-onset asthma cases, 80% to 90% are caused by gastroesophageal reflux disease (GERD). 2. Pancreatitis is not related to GERD. 3. Peptic ulcer disease is related to H. pylori bacterial infections and can lead to increased levels of gastric acid, but it is not related to reflux. 4. GERD is not related to increased gastric emptying. Increased gastric emptying would be a benefit to a client with decreased functioning of the lower esophageal sphincter. TEST-TAKING HINT: Some questions are knowledge based. There are no test-taking strategies for knowledge-based questions.

The nurse is caring for a client in acute pain as a result of surgery. Which intervention should the nurse implement? 1. Administer pain medication as soon as the time frame allows. 2. Use nonpharmacological methods to replace medications. 3. Use cryotherapy after heat therapy because it works faster. 4. Instruct family members to administer medication with the PCA.

ANSWER: 1. 1. Pain medications should be administered at the frequency ordered by the HCP, not just when the client requests them, especially for acute pain. 2. Nonpharmacological methods should never replace medications, but they should be used in combination to help keep the client comfortable. 3. Cryotherapy (cold) is used immediately postoperative or postinjury. Heat applications are applied at a later time. 4. Only clients should activate the PCA to prevent overdosing. TEST-TAKING HINT: Option "4" should be eliminated because a basic concept is the client should be the person in control of the pain, not a family member; pain is subjective.

The nurse is performing an admission assessment on a client diagnosed with GERD. Which signs and symptoms would indicate GERD? 1. Pyrosis, water brash, and flatulence. 2. Weight loss, dysarthria, and diarrhea. 3. Decreased abdominal fat, proteinuria, and constipation. 4. Midepigastric pain, positive H. pylori test, and melena.

ANSWER: 1. 1. Pyrosis is heartburn, water brash is the feeling of saliva secretion as a result of reflux, and flatulence is gas—all symptoms of GERD. 2. Gastroesophageal reflux disease does not cause weight loss. 3. There is no change in abdominal fat, no proteinuria (the result of a filtration problem in the kidney), and no alteration in bowel elimination for the client diagnosed with GERD. 4. Midepigastric pain, a positive H. pylori test, and melena are associated with gastric ulcer disease. TEST-TAKING HINT: Frequently, incorrect answer options will contain the symptoms of a disease of the same organ system.

The nurse writes the problem "imbalanced nutrition: less than body requirements" for the client diagnosed with hepatitis. Which intervention should the nurse include in the plan of care? 1. Provide a high-calorie intake diet. 2. Discuss total parenteral nutrition (TPN). 3. Instruct the client to decrease salt intake. 4. Encourage the client to increase water intake

ANSWER: 1. 1. Sufficient energy is required for healing. Adequate carbohydrate intake can spare protein. The client should eat approximately 16 carbohydrate kilocalories for each kilogram of ideal body weight daily. 2. TPN is not routinely prescribed for the client with hepatitis; the client must lose a large of amount of weight and be unable to eat anything for TPN to be ordered. 3. Salt intake does not affect the healing of the liver. 4. Water intake does not affect healing of the liver, and the client should not drink so much water as to decrease caloric food intake. TEST-TAKING HINT: The test taker should key in on "less than body requirements" in the stem and select the answer option addressing increasing calories, which eliminates options "3" and "4."

The client is diagnosed with salmonellosis secondary to eating some slightly cooked hamburger meat. Which clinical manifestations should the nurse expect the client to report? 1. Abdominal cramping, nausea, and vomiting. 2. Neuromuscular paralysis and dysphagia. 3. Gross amounts of explosive bloody diarrhea. 4. Frequent "rice water stool" with no fecal odor

ANSWER: 1. 1. Symptoms develop 8 to 48 hours after ingesting the Salmonella bacteria and include diarrhea, abdominal cramping, nausea, and vomiting, along with lowgrade fever, chills, and weakness. 2. Neuromuscular paralysis and dysphagia occur with botulism, a severe lifethreatening form of food poisoning caused by Clostridium botulinum. 3. Gross explosive bloody diarrhea is a clinical manifestation of hemorrhagic colitis caused by Escherichia coli. 4. Gray-cloudy diarrhea with no fecal odor, blood, or pus is caused by cholera, which is endemic in parts of Asia, the Middle East, and Africa. TEST-TAKING HINT: Often when two (2) options have the same clinical manifestation, such as diarrhea and stool, this should make the test taker realize either one of these two (2) options is correct, so the other two (2) options can be eliminated, or both are incorrect

The client diagnosed with IBD is prescribed total parenteral nutrition (TPN). Which intervention should the nurse implement? 1. Check the client's glucose level. 2. Administer an oral hypoglycemic. 3. Assess the peripheral intravenous site. 4. Monitor the client's oral food intake.

ANSWER: 1. 1. TPN is high in dextrose, which is glucose; therefore, the client's blood glucose level must be monitored closely. 2. The client may be on sliding-scale regular insulin coverage for the high glucose level. 3. The TPN must be administered via a subclavian line because of the high glucose level. 4. The client is NPO to put the bowel at rest, which is the rationale for administering the TPN. TEST-TAKING HINT: The test taker may want to select option "3" because it has the word "assess," but the test taker should remember to note the adjective "peripheral," which makes this option incorrect. Remember, the words "check" and "monitor" are words meaning "assess."

The client who had an abdominal surgery has a Jackson Pratt (JP) drainage tube. Which assessment data warrant immediate intervention by the nurse? 1. The bulb is round and has 40 mL of fluid. 2. The drainage tube is taped to the dressing. 3. The JP insertion site is pink and has no drainage. 4. The JP bulb has suction and is sunken in

ANSWER: 1. 1. The JP bulb should be depressed, which indicates suction is being applied. A round bulb indicates the bulb is full and needs to be emptied and suction reapplied. 2. The tube should be taped to the dressing to prevent accidentally pulling the drain out of the insertion site. 3. The insertion site should be pink and without any signs of infection, which include drainage, warmth, and redness. 4. The JP bulb should be sunken in or depressed, indicating suction is being applied. TEST-TAKING HINT: The stem is asking which data need intervention by the nurse. Option "2" can be ruled out because all tubes and drains should be secured. A pink insertion site with no drainage is expected, which would cause the test taker to eliminate option "3" as a possible correct answer

The PACU nurse is receiving the client from the OR. Which intervention should the nurse implement first? 1. Assess the client's breath sounds. 2. Apply oxygen via nasal cannula. 3. Take the client's blood pressure. 4. Monitor the pulse oximeter reading

ANSWER: 1. 1. The airway should be assessed first. When caring for a client, the nurse should follow the ABCs: airway, breathing, and circulation. 2. After assessing the client's airway and breathing, the nurse can apply oxygen via a nasal cannula if it is necessary. 3. The blood pressure is taken automatically by the monitor, but this is not priority over airway. 4. The pulse oximeter is applied to the client's finger to obtain the peripheral oxygenation status, but the nurse should assess the client's breathing first. TEST-TAKING HINT: When the stem of the question asks the test taker to implement a nursing intervention first, the test taker should think of assessment and then apply Maslow's hierarchy of needs.

The nurse is conducting an interview with a 75-year-old client admitted with acute pain. Which question would have priority when assisting with pain management? 1. "Have you ever had difficulty getting your pain controlled?" 2. "What types of surgery have you had in the last 10 years?" 3. "Have you ever been addicted to narcotics?" 4. "Do you have a list of your prescription medications?"

ANSWER: 1. 1. The answer to this request would indicate if the client has had a negative experience which may influence the client's pain management. 2. Previous surgeries would be pertinent information but not for pain management. 3. Before asking this question, the nurse should have specific information to suspect drug use. 4. Discussing the client's prescription medications is necessary, but asking for a list of medications will not address the client's pain management. TEST-TAKING HINT: Assessment, the first step of the nursing process, of pain perception is indicated when caring for a client with acute pain.

Which outcome should the nurse identify for the client diagnosed with aphthous stomatitis? 1. The client will be able to cope with perceived stress. 2. The client will consume a balanced diet. 3. The client will deny any difficulty swallowing. 4. The client will take antacids as prescribed.

ANSWER: 1. 1. The cause of canker sores (aphthous stomatitis) is unknown. The small ulcerations of the soft oral tissue are linked to stress, trauma, allergies, viral infections, and metabolic disorders. Therefore, being able to cope with stress is a desired outcome. 2. The client with recurrent erythematous macule cankers will not have malnutrition; therefore, a balanced diet is not applicable to this client. 3. The client with cankers should not have difficulty swallowing. 4. Antacids are not a treatment for canker sores.

Which activities are the circulating nurse's responsibilities in the operating room? 1. Monitor the position of the client, prepare the surgical site, and ensure the client's safety. 2. Give preoperative medication in the holding area and monitor the client's response to anesthesia. 3. Prepare sutures; set up the sterile field; and count all needles, sponges, and instruments. 4. Prepare the medications to be administered by the anesthesiologist and change the tubing for the anesthesia machine.

ANSWER: 1. 1. The circulating nurse has many responsibilities in the OR, including coordinating the activities in the OR; keeping the OR clean; ensuring the safety of the client; and maintaining the humidity, lighting, and safety of the equipment. 2. This is the role of the nurse anesthetist or anesthesiologist. 3. This is the role of the scrub nurse or technologist. 4. If there is an anesthesia technologist, this would be his or her role, or the nurse anesthetist and the anesthesiologist would assume the role. TEST-TAKING HINT: Options "2" and "4" discuss anesthesia and an anesthesiologist, which may lead the test taker to eliminate these as possible correct answers. Some questions are knowledge-based questions which require the test taker to know the information; this is an example of this type of question.

The female client diagnosed with anorexia nervosa is admitted to the hospital. The client is 67 inches tall and weighs 40 kg. Which client problem has the highest priority? 1. Altered nutrition. 2. Low self-esteem. 3. Disturbed body image. 4. Altered sexuality.

ANSWER: 1. 1. The client is 67 inches tall (5'7'') and weighs 88 pounds (40 kg 3 2.2 5 88). This client is severely underweight and nutrition is the priority. 2. Clients with anorexia have a chronic low self-esteem problem, but this is a psychosocial problem and actual physical problems are priority. 3. Disturbed body image is a psychosocial problem manifested in a physical one. The physical problem is priority; this would be an appropriate long-term goal. 4. This client thinks her body is not appealing and this could be a problem, but it is a psychosocial issue and not priority. TEST-TAKING HINT: The test taker must decide which problem is priority when all the problems could apply to the client. Unless the client is considering suicide and has a plan to carry it out, physical problems are priority

The nurse caring for a client diagnosed with GERD writes the client problem of "behavior modification." Which intervention should be included for this problem? 1. Teach the client to sleep with a foam wedge under the head. 2. Encourage the client to decrease the amount of smoking. 3. Instruct the client to take over-the-counter medication for relief of pain. 4. Discuss the need to attend Alcoholics Anonymous to quit drinking.

ANSWER: 1. 1. The client should elevate the head of the bed on blocks or use a foam wedge to use gravity to help keep the gastric acid in the stomach and prevent reflux into the esophagus. Behavior modification is changing one's behavior. 2. The client should be encouraged to quit smoking altogether. Referral to support groups for smoking cessation should be made. 3. The nurse should be careful when recommending OTC medications. This is not the most appropriate intervention for a client with GERD. 4. The client should be instructed to discontinue using alcohol, but the stem does not indicate the client is an alcoholic. TEST-TAKING HINT: Clients are encouraged to quit, not decrease, smoking. Current research indicates smoking is damaging to many body systems, including the gastrointestinal system. The test taker should not assume anything not in the stem of a question.

The nurse requests the client to sign a surgical informed consent form for an emergency appendectomy. Which statement by the client indicates further teaching is needed? 1. "I will be glad when this is over so I can go home today." 2. "I will not be able to eat or drink anything prior to my surgery." 3. "I can practice relaxing by listening to my favorite music." 4. "I will need to get up and walk as soon as possible."

ANSWER: 1. 1. The client will be in the hospital for a few days. This is not a day-surgery procedure. The client needs more teaching. 2. Clients are NPO (nothing by mouth) prior to surgery to prevent aspiration during and after anesthesia. The client understands the teaching. 3. Listening to music and other relaxing techniques can be used to alleviate anxiety and pain. This statement indicates the client understands the teaching. 4. Clients are encouraged to get out of bed as soon as possible and progress until a return to daily activity is achieved. The client understands the teaching. TEST-TAKING HINT: This question is asking the test taker to identify the answer option which is incorrect. Three (3) options will be appropriate statements which indicate the client understands the teaching.

The nurse is reviewing the pathology report of a client post-cervical neck node dissection. The health-care provider has explained the results of the biopsy to the client. Which should the nurse implement? 1. Allow the client the opportunity to discuss feelings about the results. 2. Assess the client's neck dissection dressing for bleeding. 3. Monitor the client's white blood cell count for elevation. 4. Call the pathology department to verify the report is correct

ANSWER: 1. 1. The client will have been told that he/she has cancer. The nurse should allow the client to verbalize feelings. 2. The nurse should routinely monitor any dressing, but assessing the dressing does not address the pathology report and its implications for the client. 3. The report indicates a diagnosis of cancer, not infection. 4. The pathology report is performed on the actual cells removed from the tissues. The report does not need to be verified. TEST-TAKING HINT: The test taker could reason out the correct answer from some key words in the report. Reed-Sternberg cells are only associated with Hodgkin's lymphoma, but if the test taker did not know this then "Stage 3" is usually used to designate the severity of the cancer being diagnosed.

Which intervention should the nurse implement specifically for the client in end-stage liver failure who is experiencing hepatic encephalopathy? 1. Assess the client's neurological status. 2. Prepare to administer a loop diuretic. 3. Check the client's stool for blood. 4. Assess for an abdominal fluid wave.

ANSWER: 1. 1. The increased serum ammonia level associated with liver failure causes the hepatic encephalopathy, which, in turn, leads to neurological deficit. 2. Administering a loop diuretic is an appropriate intervention for ascites and portal hypertension. 3. Checking the stool for bleeding is an appropriate intervention for esophageal varices and decreased vitamin K. 4. Assessing the abdominal fluid wave is an appropriate intervention for ascites and portal hypertension.

The client who is morbidly obese has undergone gastric bypass surgery. Which immediate postoperative intervention has the greatest priority? 1. Monitor respiratory status. 2. Weigh the client daily. 3. Teach a healthy diet. 4. Assist in behavior modification.

ANSWER: 1. 1. The morbidly obese client will have a large abdomen, preventing the lungs from expanding, which predisposes the client to respiratory complications. 2. The client may be weighed daily, but this is not priority. 3. The client should be taught proper nutrition for weight loss, but this is not the priority in the immediate postoperative period. 4. This is very important for the long term, but respiratory status is priority. TEST-TAKING HINT: Regardless of the procedure or the size of the client, respiratory status is priority in the immediate postoperative period. The test taker should apply Maslow's hierarchy of needs.

The 68-year-old client scheduled for intestinal surgery does not have clear fecal contents after three (3) tap water enemas. Which intervention should the nurse implement first? 1. Notify the surgeon of the client's status. 2. Continue giving enemas until clear. 3. Increase the client's IV fluid rate. 4. Obtain STAT serum electrolytes

ANSWER: 1. 1. The nurse should contact the surgeon because the client is at risk for fluid and electrolyte imbalance after three (3) enemas. Clients who are NPO, elderly clients, and pediatric clients are more likely to have these imbalances. 2. Administering more enemas will put the client at further risk for fluid volume deficit and electrolyte imbalance. 3. The IV may need to be increased, but the nurse would need to fully assess the fluid status of the client prior to initiating this intervention. Intravenous fluids would not directly affect the peristalsis of the gastrointestinal tract so that the bowels would move more rapidly. 4. The electrolyte status may need to be assessed, but the nurse would need an order for this intervention. TEST-TAKING HINT: Very few questions will require the nurse to notify the health-care provider, but there will be some; the test taker must know when a potential complication may occur. The nurse cannot prescribe or order laboratory tests without a health-care provider's order

The client is complaining of left shoulder pain. Which intervention should the nurse implement first? 1. Assess the neurovascular status of the left hand. 2. Check the medication administration record (MAR). 3. Ask if the client wants pain medication. 4. Administer the client's pain medication.

ANSWER: 1. 1. The nurse should first assess the client for potential complications to determine if this is expected pain or pain requiring notifying the health-care provider. 2. The nurse must check the MAR to determine when the last pain medication was administered, but it is not the first intervention. 3. The nurse must rule out complications which require medical intervention prior to medicating the client. 4. The nurse should not administer any pain medication prior to ruling out complications and checking the MAR. TEST-TAKING HINT: The test taker should apply the nursing process when answering the question and select an option that addresses assessment

The client complains to the nurse of unhappiness with the health-care provider. Which intervention should the nurse implement next? 1. Call the HCP and suggest he or she talk with the client. 2. Determine what about the HCP is bothering the client. 3. Notify the nursing supervisor to arrange a new HCP to take over. 4. Explain the client cannot request another HCP until after discharge.

ANSWER: 1. 1. The nurse should first assess the situation prior to informing the HCP of the client's concerns and then allow the HCP and client to discuss the situation. 2. The nurse should determine what is concerning the client. It could be a misunderstanding or a real situation where the client's care is unsafe or inadequate. 3. If a new HCP is to be arranged, it is the HCP's responsibility to arrange for another HCP to assume responsibility for the care of the client. 4. The choice of HCP is ultimately the client's. If the HCP cannot arrange for another HCP, the client may be discharged and obtain a new health-care provider. TEST-TAKING HINT: The nurse should assess the situation; the first step in the nursing process is assessment.

The nurse caring for a client one (1) day postoperative sigmoid resection notes a moderate amount of dark reddish brown drainage on the midline abdominal incision. Which intervention should the nurse implement first? 1. Mark the drainage on the dressing with the time and date. 2. Change the dressing immediately using sterile technique. 3. Notify the health-care provider immediately. 4. Reinforce the dressing with a sterile gauze pad

ANSWER: 1. 1. The nurse should mark the drainage on the dressing to determine if active bleeding is occurring, because dark reddish-brown drainage indicates old blood. This allows the nurse to assess what is actually happening. 2. Surgical dressings are initially changed by the surgeon; the nurse should not remove the dressing until the surgeon orders the dressing change to be done by the nurse. 3. The nurse should assess the situation before notifying the HCP. 4. The nurse may need to reinforce the dressing if the dressing becomes saturated, but this would be after a thorough assessment is completed. TEST-TAKING HINT: The question is asking the test taker to determine which intervention must be implemented first, and assessment is the first step of the nursing process. Options "2," "3," and "4" would not be implemented prior to assessing. Marking the dressing allows the nurse to assess the dressing and determine if active bleeding is occurring.

The nurse is caring for the client who is one (1) day post-upper gastrointestinal (UGI) series. Which assessment data warrant intervention? 1. No bowel movement. 2. Oxygen saturation 96%. 3. Vital signs within normal baseline. 4. Intact gag reflex

ANSWER: 1. 1. The nurse should monitor the client for the first bowel movement to document elimination of barium, which should be eliminated within two (2) days. If the client does not have a bowel movement, a laxative may be needed to help the client to eliminate the barium before it becomes too hard to pass. 2. An oxygen saturation of 96% is acceptable and does not require intervention. 3. Vital signs should be monitored to recognize and treat complications before the client is in danger. Baseline is a desired outcome. 4. The client's throat is not anesthetized for this procedure, so the gag reflex is not pertinent information in this procedure.

The nurse is teaching the client diagnosed with colon cancer who is scheduled for a colostomy. Which behavior indicates the nurse is utilizing adult learning principles? 1. The nurse repeats the information as indicated by the client's questions. 2. The nurse teaches in one session all the information the client needs. 3. The nurse uses a video so the client can hear the medical terms. 4. The nurse waits until the client asks questions about the surgery.

ANSWER: 1. 1. The nurse should realize the client is anxious about the diagnosis of cancer and the impending surgery. Therefore, the nurse should be prepared to repeat information as necessary. The learning principle the nurse needs to consider is "anxiety decreases learning." 2. Small manageable sessions increase learning, especially when the client is anxious. 3. Videos are not the best teaching tool for adults. Short videos are useful for children. 4. The nurse should assess the client's readiness and willingness to learn and not wait until the client asks questions about the surgery.

The nurse is facilitating a support group for clients diagnosed with Crohn's disease. Which information is most important for the nurse to discuss with the clients? 1. Discuss coping skills to assist with adaptation to lifestyle modifications. 2. Teach about drug administration, dosages, and scheduled times. 3. Teach dietary changes necessary to control symptoms. 4. Explain the care of the ileostomy and necessary equipment.

ANSWER: 1. 1. The objectives for support groups are to help members cope with chronic diseases and help manage symptom control. 2. Drug administration, dosage, and scheduled times should be discussed in the hospital prior to discharge or in the health-care provider's office; therefore, this is not a priority at the support group meeting. 3. Dietary changes should be taught at the time the disease is diagnosed, but this is not a priority at the support group meeting. 4. An ileostomy may be the surgical option for clients who do not respond to medical treatment, but other nonsurgical treatments would be topics of discussions during support group meetings.

The nurse is preparing the postoperative nursing care plan for the client recovering from a hemorrhoidectomy. Which intervention should the nurse implement? 1. Establish rapport with the client to decrease embarrassment of assessing site. 2. Encourage the client to lie in the lithotomy position twice a day. 3. Milk the tube inserted during surgery to allow the passage of flatus. 4. Digitally dilate the rectal sphincter to express old blood.

ANSWER: 1. 1. The site of the surgery can cause embarrassment when the nurse assesses the site; therefore, the nurse should establish a positive relationship. 2. The lithotomy position is with the client's legs in stirrups for procedures such as Pap smears and some surgeries such as transurethral resection of the prostate, not for the client who is postoperative hemorrhoidectomy. 3. A tube is not placed in the client's rectum after this surgery. 4. The rectal sphincter does not need to be digitally dilated.

Which action by the client indicates to the nurse preoperative teaching has been effective? 1. The client demonstrates how to use the incentive spirometer device. 2. The client demonstrates the use of the patientcontrolled analgesia pump. 3. The client can name two (2) anesthesia agents used during surgery. 4. The client ambulates down the hall to the nurse's station each hour.

ANSWER: 1. 1. The teaching is effective if the client is able to demonstrate the use of the spirometer prior to surgery. 2. The patient-controlled analgesia pump would not be available prior to surgery because the pumps are charged to the client on a daily basis and the client would not be able to demonstrate how to use it. 3. Determining allergies to anesthesia medications is important prior to surgery, but the nurse would not teach the specific medication names. 4. This would demonstrate increased mobility and would be encouraged after surgery, but it would not determine if teaching was effective.

The charge nurse is monitoring client laboratory values. Which value is expected in the client with cholecystitis who has chronic inflammation? 1. An elevated white blood cell (WBC) count. 2. A decreased lactate dehydrogenase (LDH). 3. An elevated alkaline phosphatase. 4. A decreased direct bilirubin level.

ANSWER: 1. 1. The white blood cell count should be elevated in clients with chronic inflammation. 2. A decreased lactate dehydrogenase (LDH) indicates liver abnormalities. 3. An elevated alkaline phosphatase indicates liver abnormalities. 4. A decreased bilirubin indicates an obstructive process. TEST-TAKING HINT: If the test taker does not know what the values mean, the test taker should look to the disease process. The -itis means inflammation, and an educated guess would be elevated WBCs in an inflammatory process.

The client diagnosed with acute diverticulitis is complaining of severe abdominal pain. On assessment, the nurse finds a hard, rigid abdomen and T 102°F. Which intervention should the nurse implement? 1. Notify the health-care provider. 2. Prepare to administer a Fleet's enema. 3. Administer an antipyretic suppository. 4. Continue to monitor the client closely.

ANSWER: 1. 1. These are signs of peritonitis, which is life threatening. The health-care provider should be notified immediately. 2. A Fleet's enema will not help a lifethreatening complication of diverticulitis. 3. A medication administered to help decrease the client's temperature will not help a life-threatening complication. 4. These are signs/symptoms indicating a possible life-threatening situation and require immediate intervention. TEST-TAKING HINT: In most instances, the test taker should not select the option stating to notify the HCP immediately, but in some situations, it is the correct answer. The test taker should look at all the other options and determine if the option is information the HCP requires or if it is an independent intervention which will help the client.

The nurse received a male client from the postanesthesia care unit. Which assessment data would warrant immediate intervention? 1. The client's vital signs are T 97˚F, P 108, R 24, and BP 80/40. 2. The client is sleepy but opens the eyes to his name. 3. The client is complaining of pain at a "5" on a 1-to-10 pain scale. 4. The client has 20 mL of urine in the urinary drainage bag.

ANSWER: 1. 1. These are symptoms of hypovolemic shock and require immediate intervention. 2. This is a common response to anesthesia. Clients are sleepy until the anesthesia wears off. 3. Pain management is required, but this does not indicate a life-threatening complication. 4. Urine outputs should be monitored in the postoperative period, but indwelling catheter bags are emptied in the PACU prior to transferring the client to the floor, so 20 mL would not warrant immediate intervention.

The nurse is caring for a postoperative client with a nasogastric tube to low intermittent suction. Which intervention should the nurse implement first based on the blood gas results? pH 7.48. O2 sat 96%. PaCO2 46. HCO3 20. 1. Assess the output in the suction canister. 2. Apply oxygen by nasal cannula. 3. Have the client take slow, deep breaths. 4. Place the client on stool specimen collection.

ANSWER: 1. 1. This blood gas indicates metabolic alkalosis with partial compensation. The pH level is high, indicating alkalosis. The HCO3 level is low, indicating an alkalosis problem. The Paco2 is outside the normal range, indicating lungs are trying to retain acid to help correct the pH but the pH is not in normal range (partial compensation). The N/G tube could be removing too much acid from the stomach. The nurse should assess the NGT output. 2. This is a metabolic problem, not respiratory. 3. This is a metabolic problem, not respiratory. 4. The client has an NGT removing acidic contents from the stomach; stool specimen or observation is not indicated. TEST-TAKING HINT: The nurse must be able to interpret common laboratory results. Part of the assessment of a symptom requires determining what therapies can impact the result.

Which diagnostic data should be reported to the health-care provider (HCP) immediately? 1. The ABG result of pH 7.11, Paco2 45, HCO3 20, and Pao2 98 for a client diagnosed with type 1 diabetes. 2. Sodium 137 mEq/L, potassium 4 mEq/L, glucose 120 mg/dL for a client diagnosed with gastroenteritis. 3. Hemoglobin 9.4 g/dL and hematocrit 29% for a client who received a blood transfusion of the previous shift. 4. A pulse oximetry reading of 93% for a client diagnosed with chronic obstructive pulmonary disease (COPD).

ANSWER: 1. 1. This blood gas is metabolic acidosis, a potentially lethal situation. The nurse should notify the HCP immediately. 2. These results for sodium and potassium are WNL. The glucose is at the edge of the normal. 3. The results of the hemoglobin and hematocrit (H&H) are not in the range to require more blood at this time. The nurse can give the results to the HCP on rounds. 4. This pulse oximetry reading is WNL. TEST-TAKING HINT: The nurse must be able to interpret common laboratory results. Part of the assessment of a symptom requires knowing the normal ranges.

The female client presents to the clinic for an examination because she has not had a menstrual cycle for several months and wonders if she could be pregnant. The client is 5′10″ tall and weighs 45 kg. Which assessment data should the nurse obtain first? 1. Ask the client to recall what she ate for the last 24 hours. 2. Determine what type of birth control the client has been using. 3. Reweigh the client to confirm the data. 4. Take the client's pulse and blood pressure.

ANSWER: 1. 1. This client is 5'10" tall and weighs 99 pounds (45 kg 3 2.2 5 99). Menses will cease if the client is severely emaciated. A 24-hour dietary recall is a step toward assessing the client's eating patterns. 2. The type of birth control could be asked, but the client is asking about missing menstrual periods. Birth control does not interfere with having a period; if anything, some forms of birth control will make the cycles more regular. 3. The nurse can look at the client and see a very thin young woman, which should confirm more assessment is needed, not reweighing. 4. The pulse and blood pressure will not provide the nurse any information as to why the client's menstrual cycles have ceased. TEST-TAKING HINT: The stem of the question provides information about the client's height and weight, and the test taker must determine if this is important information. Information in the stem must be eliminated as not pertinent to the question or closely regarded to let the test taker know what the question is asking

The circulating nurse is planning the care for an intraoperative client. Which statement is the expected outcome? 1. The client has no injuries from the OR equipment. 2. The client has no postoperative infection. 3. The client has stable vital signs during surgery. 4. The client recovers from anesthesia

ANSWER: 1. 1. This expected outcome addresses the safety of the client while in the OR. 2. This would be an expected outcome in the postoperative period. 3. The anesthesiologist or nurse anesthetist would monitor the client's vital signs during surgery. 4. This would be an expected outcome for the anesthesiologist or nurse anesthetist. TEST-TAKING HINT: The adjectives "intraoperative" and "circulating" are the key words to answering this question. Safety is priority in the operating room.

The nurse is administering a proton pump inhibitor to a client diagnosed with peptic ulcer disease. Which statement supports the rationale for administering this medication? 1. It prevents the final transport of hydrogen ions into the gastric lumen. 2. It blocks receptors controlling hydrochloric acid secretion by the parietal cells. 3. It protects the ulcer from the destructive action of the digestive enzyme pepsin. 4. It neutralizes the hydrochloric acid secreted by the stomach.

ANSWER: 1. 1. This statement is the rationale for proton pump inhibitors. 2. This statement explains the rationale for histamine receptor antagonists. 3. This statement describes how mucosal protective agents work in the body. 4. This statement is the rationale for antacids.

The client one (1) day postoperative develops an elevated temperature. Which intervention would have priority for the client? 1. Encourage the client to deep breathe and cough every hour. 2. Encourage the client to drink 200 mL of water every shift. 3. Monitor the client's wound for drainage every eight (8) hours. 4. Assess the urine output for color and clarity every four (4) hours.

ANSWER: 1. 1. When a postoperative client develops a fever within the first 24 hours, the cause is usually in the respiratory system. The client should increase deep breathing and coughing to assist the client to expand the lungs and decrease pulmonary complications. 2. Drinking fluid can bring down temperature, but 200 mL would not be a sufficient amount to accomplish this. Unless contraindicated, the client should drink from one (1) to two (2) L/day. 3. Wound infections may cause the fever later in the recovery but will usually not elevate within the first 24 hours after surgery. 4. A urinary tract infection may occur later but would probably not be the cause of elevated temperature within 24 hours after surgery

Which problem is most appropriate for the nurse to identify for the client with diarrhea? 1. Alteration in skin integrity. 2. Chronic pain perception. 3. Fluid volume excess. 4. Ineffective coping

ANSWER: 1. 1. When clients have multiple liquid stools, the rectal area can become irritated. The integrity of the skin can be impaired. 2. Pain experienced by this client would be acute, rather than chronic. 3. Fluid volume deficit is appropriate, rather than fluid volume excess. 4. Ineffective coping is a psychosocial problem and is not appropriate for a client with diarrhea.

The client at the eating disorder clinic weighs 35 kg and is 5 ft 7 inches tall. Which would the nurse document as the Body Mass Index (BMI)? ________________

ANSWER: 12.06 BMI. This client is extremely underweight. To figure the BMI the test taker must first multiply the height in inches times the height in inches. This client is 67 inches tall. 67 × 67 = 4489 The next step is to divide the weight in pounds by the sum of the height times the height: 35 × 2.2 = 77 pounds 77 ÷ 4489 = 0.01715 Then multiply this number times the conversion of 703: 0. 01715 × 703 = 12.06 TEST-TAKING HINT: The nurse must be able to work common math problems to determine the client needs

The client has a nasogastric tube. The healthcare provider orders IV fluid replacement based on the previous hour's output plus the baseline IV fluid ordered of 125 mL/hr. From 0800 to 0900 the client's N/G tube drained 45 mL. At 0900, what rate should the nurse set for the IV pump? _______

ANSWER: 170 mL/hr. The N/G tube drainage of 45 mL must be added to the 125 mL/hr IV rate, which equals 170 (125 + 45 = 170). The nurse should infuse 170 mL in the next hour. TEST-TAKING HINT: The stem states the previous hour's N/G tube output plus the baseline IV rate. The test taker must observe the key words in the stem. Don't forget to use the drop-down calculator when taking the NCLEX-RN.

Which interventions should the nurse discuss regarding prevention of an acute exacerbation of diverticulosis? Select all that apply. 1. Eat a low-fiber diet. 2. Drink 2,500 mL of water daily. 3. Avoid eating foods with seeds. 4. Walk 30 minutes a day. 5. Take an antacid every two (2) hours.

ANSWER: 2, 3, 4. 1. A high-fiber diet will prevent constipation, the primary reason for diverticulosis/diverticulitis. A low-fiber (residue) diet is prescribed for acute diverticulitis. 2. Increased fluids help to keep the stool soft and prevent constipation. 3. It is controversial if seeds cause an exacerbation of diverticulosis, but this is an appropriate intervention to teach until proven otherwise. 4. Exercise helps to prevent constipation, which can cause an exacerbation of diverticulitis. 5. There are no medications used to help prevent an acute exacerbation of diverticulosis/ diverticulitis. Antacids are used to neutralize hydrochloric acid in the stomach.

The nurse is receiving a client from the postanesthesia care unit (PACU). Which interventions should the nurse implement? Select all that apply. 1. Ambulate the client to the bathroom to void. 2. Take the client's vital signs to compare with PACU data. 3. Monitor all lines into and out of the client's body. 4. Assess the client's surgical site. 5. Push the client's PCA button to treat for pain during movement

ANSWER: 2, 3, 4. 1. The client should not be ambulated until the nurse has a chance to assess for the client's ability to ambulate safely. 2. The nurse should assess the vital signs from PACU with the current vital signs to be sure that the client is stable. 3. The nurse should assess the intravenous lines, indwelling catheters, and tubes upon receiving the client. 4. The nurse must assess the surgical site for bleeding to know if the client is actually stable or not. 5. Only the client should push the PCA pump's button; otherwise the client may receive an overdose of medication. TEST-TAKING HINT: The client is "returning" from PACU. This client may still be groggy from anesthesia and should not be ambulating until the nurse has assessed the client and is aware the client is awake enough to ambulate safely.

The nurse is caring for a male client scheduled for abdominal surgery. Which interventions should the nurse include in the plan of care? Select all that apply. 1. Perform passive range-of-motion exercises. 2. Discuss how to cough and deep breathe effectively. 3. Tell the client he can have a meal in the PACU. 4. Teach ways to manage postoperative pain. 5. Discuss events which occur in the postanesthesia care unit.

ANSWER: 2, 4, 5. 1. Passive means the nurse performs the rangeof-motion exercises. The client in the PACU should do active range-of-motion exercises. 2. Coughing effectively aids in the removal of pooled secretions, which can cause pneumonia. Deep-breathing exercises keep the alveoli inflated and prevent atelectasis. 3. The client having abdominal surgery will be NPO until bowel sounds return, which will not occur in the PACU; therefore, the client is not given a meal. 4. The client's postoperative pain should be kept within a tolerable range. 5. These interventions help decrease the client's anxiety. TEST-TAKING HINT: This is an alternate-type question, which requires the test taker to select more than one (1) option as the correct answer. The nurse's priority after surgery is to prevent postoperative complications.

Which nursing interventions should be included in the care plan for the 84-year-old client diagnosed with acute gastroenteritis? Select all that apply. 1. Assess the skin turgor on the back of the client's hands. 2. Monitor the client for orthostatic hypotension. 3. Record the frequency and characteristics of sputum. 4. Use Standard Precautions when caring for the client. 5. Institute safety precautions when ambulating the client.

ANSWER: 2, 4, 5. 1. The nurse should assess skin turgor over the sternum in the elderly client because loss of subcutaneous fat associated with aging makes skin turgor assessment on the arms less reliable. 2. Orthostatic hypotension indicates fluid volume deficit, which can occur in an elderly client who is having many episodes of diarrhea. 3. The nurse should record frequency and characteristics of stool, not sputum, in the client diagnosed with gastroenteritis. 4. Standard Precautions, including wearing gloves and hand washing, help prevent the spread of the infection to others. 5. The elderly client is at risk for orthostatic hypotension; therefore, safety precautions should be instituted to ensure the client doesn't fall as a result of a decrease in blood pressure. TEST-TAKING HINT: This is an alternate-type question requiring the test taker to choose all interventions that apply. The test taker should look at each option and consider if this is an intervention for an "elderly" client. The elderly are a special population usually requiring specific interventions addressing the aging process no matter what the disease process.

The 26-year-old male client in the PACU has a heart rate of 110 and a rising temperature and complains of muscle stiffness. Which interventions should the nurse implement? Select all that apply. 1. Give a back rub to the client to relieve stiffness. 2. Apply ice packs to the axillary and groin areas. 3. Prepare an ice slush for the client to drink. 4. Prepare to administer dantrolene, a smoothmuscle relaxant. 5. Reposition the client on a warming blanket.

ANSWER: 2, 4. 1. A back rub is a therapeutic intervention, but it is not appropriate for a life-threatening complication of surgery. 2. Ice packs should be applied to the axillary and groin areas for a client experiencing malignant hyperthermia. 3. The client would be NPO to prepare for intubation, but an ice slush would be used to irrigate the bladder and stomach per nasogastric tube. 4. Dantrolene is the drug of choice for treatment. 5. Cooling blankets, not a warming blanket, are used to decrease the fast-rising temperature. TEST-TAKING HINT: This is an alternate-type question, which requires the test taker to select more than one (1) option as the correct answer. Malignant hyperthermia is a medical emergency requiring immediate treatment.

The client who is obese presents to the clinic before beginning a weight loss program. Which interventions should the nurse teach? Select all that apply. 1. Walk for 30 minutes three (3) times a day. 2. Determine situations that initiate eating behavior. 3. Weigh at the same time every day. 4. Limit sodium in the diet. 5. Refer to a weight support group

ANSWER: 2, 5. 1. Exercise recommendations for weight loss are to exercise for 30 minutes at least three (3) times per week, not three (3) times a day. 2. The client should be aware of situations triggering the consumption of food when the client is not hungry, such as anger, boredom, and stress. Food-seeking behaviors are not associated only with hunger in the client who is obese. 3. The client should weigh himself or herself about once a week. If weight loss is not observed, the client becomes discouraged and feels powerless to control the weight. This can lead to diet failure. 4. Sodium is limited in clients with hypertension, not obesity. 5. Weight loss support groups such as Weight Watchers or Take Off Pounds Sensibly (TOPS) are helpful to keep the client participating in a weight loss program. TEST-TAKING HINT: This is an alternate-type question. The test taker must judge each answer option for itself; one (1) option does not eliminate another. The NCLEX-RN gives credit for the entire question. The test taker must identify all the right answers or the answer will be counted as incorrect.

The nurse is caring for clients on a surgical unit. Which client should the nurse assess first? 1. The client who had an inguinal hernia repair and has not voided in four (4) hours. 2. The client who was admitted with abdominal pain who suddenly has no pain. 3. The client four (4) hours postoperative abdominal surgery with no bowel sounds. 4. The client who is one (1) day postappendectomy and is being discharged

ANSWER: 2. 1. A client who has not voided within four (4) hours after any surgery is not priority. This is an acceptable occurrence, but if the client hasn't voided for eight (8) hours, then the nurse should assess further. 2. A sudden cessation of pain may indicate a ruptured appendix, which could lead to peritonitis, a life-threatening complication; therefore, the nurse should assess this client first. 3. Bowel sounds should return within 24 hours after abdominal surgery. Absent bowel sounds at four (4) hours postoperative is not of great concern to the nurse. 4. The client being discharged is stable and not a priority for the nurse. TEST-TAKING HINT: The stem is asking which client the nurse should see first. Therefore, the test taker should look for life-threatening or serious complications or abnormal assessment data for the disease process

Which specific data should the nurse obtain from the client who is suspected of having peptic ulcer disease? 1. History of side effects experienced from all medications. 2. Use of nonsteroidal anti-inflammatory drugs (NSAIDs). 3. Any known allergies to drugs and environmental factors. 4. Medical histories of at least three (3) generations

ANSWER: 2. 1. A history of problems the client has experienced with medications is taken during the admission interview. This information does not specifically address peptic ulcer disease. 2. Use of NSAIDs places the client at risk for peptic ulcer disease and hemorrhage. NSAIDs suppress the production of prostaglandin in the stomach, which is a protective mechanism to prevent damage from hydrochloric acid. 3. Allergies are included for safety, but this is not specific for peptic ulcer disease. 4. Information needs to be obtained about past generations so the nurse can analyze any potential health problems, but this is not specific for peptic ulcer disease. TEST-TAKING HINT: The words "specific data" indicate there will be appropriate data in one (1) or more of the answer options but only one (1) is specific to peptic ulcer disease.

The client diagnosed with ulcerative colitis has an ileostomy. Which statement indicates the client needs more teaching concerning the ileostomy? 1. "My stoma should be pink and moist." 2. "I will irrigate my ileostomy every morning." 3. "If I get a red, bumpy, itchy rash I will call my HCP." 4. "I will change my pouch if it starts leaking."

ANSWER: 2. 1. A pink and moist stoma indicates viable tissue and adequate circulation. A purple stoma indicates necrosis. 2. An ileostomy will drain liquid all the time and should not routinely be irrigated. A sigmoid colostomy may need daily irrigation to evacuate feces. 3. A red, bumpy, itchy rash indicates infection with the yeast Candida albicans, which should be treated with medication. 4. The ileostomy drainage has enzymes and bile salts, which are irritating and harsh to the skin; therefore, the pouch should be changed if any leakage occurs. TEST-TAKING HINT: This is an "except" question, and the test taker must identify which option is not a correct action for the nurse to implement. Sometimes flipping the question—"Which interventions indicate the client understands the teaching?"—can assist in identifying the correct answer.

Which type of precaution should the nurse implement to protect from being exposed to any of the hepatitis viruses? 1. Airborne Precautions. 2. Standard Precautions. 3. Droplet Precautions. 4. Exposure Precautions.

ANSWER: 2. 1. Airborne Precautions are required for transmission occurring by dissemination of either airborne droplet nuclei or dust particles containing the infectious agent. 2. Standard Precautions apply to blood, all body fluids, secretions, and excretions, except sweat, regardless of whether they contain visible blood. 3. Droplet transmission involves contact of the conjunctivae of the eyes or mucous membranes of the nose or mouth with largeparticle droplets generated during coughing, sneezing, talking, or suctioning. 4. Exposure Precautions is not a designated isolation category. TEST-TAKING HINT: The test taker must know Standard Precautions are used by all healthcare workers who have direct contact with clients or with their body fluids or have indirect contact with objects used by clients who are infected, such as would be involved in emptying trash, changing linens, or cleaning the room.

Which assessment data indicate to the nurse the client recovering from an open cholecystectomy may require pain medication? 1. The client's pulse is 65 beats per minute. 2. The client has shallow respirations. 3. The client's bowel sounds are 20 per minute. 4. The client uses a pillow to splint when coughing

ANSWER: 2. 1. An increased pulse is expected in the client who is in acute pain. 2. An open cholecystectomy requires a large incision under the diaphragm. Deep breathing places pressure on the diaphragm and the incision, causing pain. Shallow respirations indicate inadequate pain control, and the nurse should intervene. 3. Twenty bowel sounds a minute is normal data and does not require further action. 4. Splinting the abdomen allows the client to increase the strength of the cough by increasing comfort and does not indicate a need for pain medication. TEST-TAKING HINT: The stem asks which data would warrant pain medication. Therefore, the test taker should select an answer not expected or not normal for clients who are postoperative abdominal surgery

The nurse is caring for a client who uses cathartics frequently. Which statement made by the client indicates an understanding of the discharge teaching? 1. "In the future I will eat a banana every time I take the medication." 2. "I don't have to have a bowel movement every day." 3. "I should limit the fluids I drink with my meals." 4. "If I feel sluggish, I will eat a lot of cheese and dairy products."

ANSWER: 2. 1. Bananas are encouraged for clients with potassium loss from diuretics; a banana is not needed for harsh laxative (cathartic) use. Harsh laxatives should be discouraged because they cause laxative dependence and a narrowing of the colon with long-term use. 2. It is not necessary to have a bowel movement every day to have normal bowel functioning. 3. Limiting fluids will increase the problem; the client should be encouraged to increase the fluids in the diet. 4. If the client is feeling "sluggish" from not being able to have a bowel movement, these foods increase constipation because they are low in residue (fiber). TEST-TAKING HINT: The test taker must understand words such as "cathartic." Limiting fluids is used for clients in renal failure or congestive heart failure, but increasing fluids is recommended for most other conditions.

The dietitian and the nurse in a long-term care facility are planning the menu for the day. Which foods should be recommended for the immobile clients for whom swallowing is not an issue? 1. Cheeseburger and milk shake. 2. Canned peaches and a sandwich on wholewheat bread. 3. Mashed potatoes and mechanically ground red meat. 4. Biscuits and gravy with bacon

ANSWER: 2. 1. Cheeseburgers and milk shakes are lowresidue foods and can make constipation worse. 2. Canned peaches are soft and can be chewed and swallowed easily while providing some fiber; whole-wheat bread is higher in fiber than white bread. These foods will be helpful for clients whose gastric motility is slowed as a result of lack of exercise or immobility. 3. Mashed potatoes and mechanically ground meat do not provide high fiber. 4. Biscuits, gravy, and bacon are refined flour foods or processed meat (fat). These will not help clients to prevent constipation. TEST-TAKING HINT: The test taker must realize the consequences of immobility include constipation

The client is being admitted to the outpatient psychiatric clinic diagnosed with bulimia. Which question should the nurse ask to identify behaviors suggesting bulimia? 1. "When was the last time you exercised?" 2. "What over-the-counter medications do you take?" 3. "How long have you had a positive self-image?" 4. "Do you eat a lot of high-fiber foods for bowel movements?"

ANSWER: 2. 1. Clients diagnosed with anorexia exercise excessively; clients diagnosed with bulimia do not. 2. Clients diagnosed with bulimia frequently take cathartic laxatives to prevent absorption of calories from the food consumed. 3. Clients diagnosed with bulimia and anorexia have low self-esteem. The client feels ugly or unlovable if he or she is overweight (by his or her perception). 4. High-fiber foods do help the body to produce larger stools, but this client would use a cathartic laxative. TEST-TAKING HINT: The test taker must distinguish between bulimia and anorexia to answer this question. Clients with anorexia are usually underweight, whereas clients with bulimia may be of a normal or slightly larger size.

The nurse is caring for a client diagnosed with bulimia nervosa. Which nursing intervention should the nurse implement after the client's evening meal? 1. Praise the client for eating all the food on the tray. 2. Stay with the client for 45 minutes to an hour. 3. Allow the client to work out on the treadmill. 4. Place the client on bedrest until morning.

ANSWER: 2. 1. Clients diagnosed with bulimia will eat the entire meal and more food if available. This is not unusual behavior for a client diagnosed with bulimia. 2. By having someone stay with the client for 45 minutes to one (1) hour after a meal, the client will be prevented from inducing vomiting and ridding the body of the meal before it can be metabolized. 3. Clients diagnosed with anorexia nervosa tend to overexercise to prevent weight gain and to lose imagined excess weight. 4. Bedrest is not needed for this client. TEST-TAKING HINT: The test taker must be able to differentiate between bulimia and anorexia. It can be difficult to keep these processes separate, especially because some clients have both anorexia and bulimia.

The male client tells the nurse he has been experiencing "heartburn" at night that awakens him. Which assessment question should the nurse ask? 1. "How much weight have you gained recently?" 2. "What have you done to alleviate the heartburn?" 3. "Do you consume many milk and dairy products?" 4. "Have you been around anyone with a stomach virus?"

ANSWER: 2. 1. Clients with heartburn are frequently diagnosed as having GERD. GERD can occasionally cause weight loss but not weight gain. 2. Most clients with GERD have been self-medicating with over-the-counter medications prior to seeking advice from a health-care provider. It is important to know what the client has been using to treat the problem. 3. Milk and dairy products contain lactose, which are important if considering lactose intolerance but are not important for "heartburn." 4. Heartburn is not a symptom of a viral illness. TEST-TAKING HINT: Clients will use common terms such as "heartburn" to describe symptoms. The nurse must be able to interpret or clarify the meaning of terms used with the client. Part of the assessment of a symptom requires determining what aggravates and alleviates the symptom.

The nurse is caring for the client scheduled for an abdominal perineal resection for Stage IV colon cancer. Which client problem should the nurse include in the intraoperative care plan? 1. Fluid volume deficit. 2. Impaired tissue perfusion. 3. Infection of surgical site. 4. Risk for immunosuppression.

ANSWER: 2. 1. Fluid deficit is a potential problem, not an actual problem. The client's fluid balance should be managed by intravenous fluids. 2. The perfusion of the surgical site is compromised as a result of the surgical incision, especially when a graft is used. 3. Infection is a potential problem but not at the time of surgery. 4. After surgery, not during surgery, the client may require chemotherapy, which can cause immunosuppression.

The client diagnosed with gastroenteritis is being discharged from the emergency department. Which intervention should the nurse include in the discharge teaching? 1. If diarrhea persists for more than 96 hours, contact the health-care provider. 2. Instruct the client to wash hands thoroughly before handling any type of food. 3. Explain the importance of decreasing steroids gradually as instructed. 4. Discuss how to collect all stool samples for the next 24 hours

ANSWER: 2. 1. If the diarrhea persists more than 48 hours, the client should notify the HCP. Diarrhea for more than 96 hours could lead to metabolic acidosis, hypokalemia, and possible death. 2. Washing hands should be done by the client at all times, but especially when the client has gastroenteritis. The bacteria in feces may be transferred to other people via food if hands are not washed properly. 3. Steroids are not used in the treatment of gastroenteritis; antidiarrheal medication is usually prescribed. 4. The client may be asked to provide a stool specimen for culture, ova, parasites, and fecal leukocytes, but the client is not asked for a 24-hour stool collection. TEST-TAKING HINT: If the test taker did not know the answer to this question, hand washing should be selected because it is the number-one intervention for preventing any type of contamination or nosocomial infection

The 22-year-old female who is obese is discussing weight loss programs with the nurse. Which information should the nurse teach? 1. Jog for two (2) to three (3) hours every day. 2. Lifestyle behaviors must be modified. 3. Eat one (1) large meal every day in the evening. 4. Eat 1,000 calories a day and don't take vitamins.

ANSWER: 2. 1. Jogging is not an appropriate exercise for a client who is obese: there is too much stress on the heart and joints. 2. If lifestyle behaviors such as patterns of eating and daily exercise are not modified, the client who loses weight will regain the weight and usually more. 3. The client should eat frequent small meals during the day to keep from being hungry. Breakfast should not be skipped. 4. Diets containing fewer than 1,200 calories per day need to be supplemented with a multivitamin to provide the body with the nutrients needed to stay healthy. TEST-TAKING HINT: The test taker could eliminate answer option "4" because healthcare professionals should not discourage health promotion activities.

The client with a history of peptic ulcer disease is admitted into the intensive care department with frank gastric bleeding. Which priority intervention should the nurse implement? 1. Maintain a strict record of intake and output. 2. Insert a nasogastric (N/G) tube and begin saline lavage. 3. Assist the client with keeping a detailed calorie count. 4. Provide a quiet environment to promote rest.

ANSWER: 2. 1. Maintaining a strict record of intake and output is important to evaluate the progression of the client's condition, but it is not the most important intervention. 2. Inserting a nasogastric tube and lavaging the stomach with saline is the most important intervention because this directly stops the bleeding. 3. A calorie count is important information assisting in the prevention and treatment of a nutritional deficit, but this intervention does not address the client's immediate and lifethreatening problem. 4. Promoting a quiet environment aids in the reduction of stress, which can cause further bleeding, but this will not stop the bleeding. TEST-TAKING HINT: The test taker is required to rank the importance of interventions in the question. Using Maslow's hierarchy of needs to rank physiological needs first, the test taker should realize inserting a nasogastric tube and beginning lavage is solving a circulation or fluid deficit problem.

The postanesthesia care nurse is caring for a client who had abdominal surgery and is complaining of nausea. Which intervention should the nurse implement first? 1. Medicate the client with a narcotic analgesic (IVP). 2. Assess the nasogastric tube for patency. 3. Check the temperature for elevation. 4. Hyperextend the neck to prevent stridor.

ANSWER: 2. 1. Medicating the client with an analgesic could increase the client's nausea unless the nausea is caused by pain. The nurse should assess the etiology to determine the interventions. 2. A client who had abdominal surgery usually has a nasogastric (N/G) tube in place. If the N/G tube is not patent, this will cause nausea. Irrigating the N/G tube may relieve nausea. 3. Checking the temperature will not treat the nausea. 4. Hyperextending the neck will assist the client to breathe but will not treat nausea. TEST-TAKING HINT: Assessment is the first step in the nursing process. Checking the N/G tube for patency and taking the temperature are the only assessment interventions. Temperature does not correlate with nausea. Medication may be administered but it would be an antiemetic, not a narcotic analgesic.

Which intervention is appropriate for the nurse to delegate to the unlicensed assistive personnel (UAP) when caring for the female client experiencing acute pain? 1. Take the pain medication to the room. 2. Apply an ice pack to the site of pain. 3. Check on the client 30 minutes after she takes the pain medication. 4. Observe the client's ability to use the PCA.

ANSWER: 2. 1. Medication administration cannot be delegated to a UAP. 2. This task does not require teaching, evaluating, or nursing judgment and therefore can be delegated. 3. Assessment cannot be delegated to a UAP. 4. Evaluation of teaching cannot be delegated to a UAP. TEST-TAKING HINT: The terms "observe" and "check" in options "3" and "4" are different from the term "evaluate," but reading the options, the tasks are clearly addressing the evaluation step of the nursing process. Evaluation cannot be delegated to the UAP.

The nursing manager is making assignments for the OR. Which case should the manager assign to the inexperienced nurse? 1. The client having open-heart surgery. 2. The client having a biopsy of the breast. 3. The client having laser eye surgery. 4. The client having a laparoscopic knee repair

ANSWER: 2. 1. Open-heart surgery is complex, and the care of the client should be assigned to an experienced nurse with special training. 2. The case of a client having a biopsy of the breast would be a good case for an inexperienced nurse because it is simple. 3. Laser eye surgery requires the nurse in the OR to have additional training to operate the equipment. 4. Additional training to be in the OR would be required for this case because special care to prevent infection is needed in orthopedic cases. TEST-TAKING HINT: The test taker should select the option which requires the least amount of additional training because the nurse is new to the operating room. Technology required for specific surgeries requires additional training.

The nurse is preparing to administer the initial dose of an aminoglycoside antibiotic to the client diagnosed with acute diverticulitis. Which intervention should the nurse implement? 1. Obtain a serum trough level. 2. Ask about drug allergies. 3. Monitor the peak level. 4. Assess the vital signs

ANSWER: 2. 1. Peak and trough levels are drawn after the client has received at least three (3) to four (4) doses of medication, not after the initial dose because the client has just been admitted. 2. The nurse should always ask about allergies to medication when administering medications, but especially when administering antibiotics, which are notorious for allergic reactions. 3. The peak and trough levels are not drawn prior to the first dose; they are ordered after multiple doses. 4. The nurse should question when to administer the medication, but there is no vital sign preventing the nurse from administering an antibiotic. TEST-TAKING HINT: The test taker must read the stem closely to realize the client is receiving the initial dose, causing the test taker to eliminate options "1" and "3" as possible correct answers. Both options "2" and "4" are assessment data, but the test taker should ask which one will directly affect the administration of the medication.

The occupational health nurse is preparing a presentation to a group of factory workers about preventing colon cancer. Which information should be included in the presentation? 1. Wear a high-filtration mask when around chemicals. 2. Eat several servings of cruciferous vegetables daily. 3. Take a multiple vitamin every day. 4. Do not engage in high-risk sexual behaviors.

ANSWER: 2. 1. Some cancers have a higher risk of development when the client is occupationally exposed to chemicals, but cancer of the colon is not one of them. 2. Cruciferous vegetables, such as broccoli, cauliflower, and cabbage, are high in fiber. One of the risks for cancer of the colon is a high-fat, low-fiber, and high-protein diet. The longer the transit time (the time from ingestion of the food to the elimination of the waste products), the greater the chance of developing cancer of the colon. 3. A multiple vitamin may improve immune system function, but it does not prevent colon cancer. 4. High-risk sexual behavior places the client at risk for sexually transmitted diseases. A history of multiple sexual partners and initial sexual experience at an early age does increase the risk for the development of cancer of the cervix in females. TEST-TAKING HINT: The colon processes waste products from eating foods, and option "2" is the only option to mention food. Therefore, option "2" would be the best option to select if the test taker did not know the correct answer.

Which assessment data indicate to the nurse the client's gastric ulcer has perforated? 1. Complaints of sudden, sharp, substernal pain. 2. Rigid, boardlike abdomen with rebound tenderness. 3. Frequent, clay-colored, liquid stool. 4. Complaints of vague abdominal pain in the right upper quadrant

ANSWER: 2. 1. Sudden sharp pain felt in the substernal area indicates angina or myocardial infarction. 2. A rigid, boardlike abdomen with rebound tenderness is the classic sign/symptom of peritonitis, which is a complication of a perforated gastric ulcer. 3. Clay-colored stools indicate liver disorders, such as hepatitis. 4. Clients with gallbladder disease report vague to sharp abdominal pain in the right upper quadrant. TEST-TAKING HINT: The only two (2) answer options that refer to the abdomen are options "2" and "4." Therefore, the test taker should select one (1) of these two (2) because a gastric ulcer involves the stomach.

The client diagnosed with inflammatory bowel disease has a serum potassium level of 3.4 mEq/L. Which action should the nurse implement first? 1. Notify the health-care provider (HCP). 2. Assess the client for muscle weakness. 3. Request telemetry for the client. 4. Prepare to administer potassium IV.

ANSWER: 2. 1. The HCP should be notified so potassium supplements can be ordered, but this is not the first intervention. 2. Muscle weakness may be a sign of hypokalemia; hypokalemia can lead to cardiac dysrhythmias and can be life threatening. Assessment is priority for a potassium level just below normal level, which is 3.5 to 5.5 mEq/L. 3. Hypokalemia can lead to cardiac dysrhythmias; therefore, requesting telemetry is appropriate, but it is not the first intervention. 4. The client will need potassium to correct the hypokalemia, but it is not the first intervention. TEST-TAKING HINT: When the question asks which action should be implemented first, remember assessment is the first step in the nursing process. If the answer option addressing assessment is appropriate for the situation in the question, then the test taker should select it as the correct answer.

The 79-year-old client diagnosed with acute gastroenteritis is admitted to the medical unit. Which task would be most appropriate for the nurse to delegate to the unlicensed assistive personnel (UAP)? 1. Evaluate the client's intake and output. 2. Take the client's vital signs. 3. Change the client's intravenous solution. 4. Assess the client's perianal area.

ANSWER: 2. 1. The UAP can calculate the client's intake and output, but the nurse must evaluate the data to determine if it is normal for the elderly client diagnosed with acute gastroenteritis. 2. The UAP can take the vital signs for a client who is stable; the nurse must interpret and evaluate the vital signs. 3. The UAP cannot administer medications, and IV solutions are considered to be medications 4. The nurse cannot delegate assessment. The client may have an excoriated perianal area secondary to diarrhea; therefore, the nurse should assess the client. TEST-TAKING HINT: The nurse should not delegate any nursing task requiring judgment or assessment and cannot delegate the administration of medications. Words such as "evaluate" mean the same thing as "assess"; therefore, options "1," "3," and "4" can be eliminated.

The client is in the lithotomy position during surgery. Which nursing intervention should be implemented to decrease a complication from the positioning? 1. Increase the intravenous fluids. 2. Lower one leg at a time. 3. Raise the foot of the stretcher. 4. Administer epinephrine, a vasopressor.

ANSWER: 2. 1. The anesthesiologist, not the nurse in the operating room, manages the intravenous fluids. 2. The lithotomy position has both legs elevated and placed in stirrups. The legs should be lowered one leg at a time to prevent hypotension from the shift of the blood. 3. Raising the foot of the bed would be a treatment of hypotension, but not hypotension resulting from the lithotomy position. 4. Epinephrine, a vasopressor, is used during codes to shunt blood from the periphery to the central circulation.

The client is one (1) day postoperative major abdominal surgery. Which client problem is priority? 1. Impaired skin integrity. 2. Fluid and electrolyte imbalance. 3. Altered bowel elimination. 4. Altered body image.

ANSWER: 2. 1. The client has a surgical incision, which impairs the skin integrity, but it is not the priority because it is sutured under sterile conditions. 2. After abdominal surgery, the body distributes fluids to the affected area as part of the healing process. These fluids are shifted from the intravascular compartment to the interstitial space, which causes potential fluid and electrolyte imbalance. 3. Bowel elimination is a problem, but after general anesthesia wears off, the bowel sounds will return, and this is not a lifethreatening problem. 4. Psychosocial problems are not priority over actual physiological problems. TEST-TAKING HINT: When identifying priority problems, the test taker can eliminate any psychosocial problem as a potential correct answer if there are applicable physiological problems.

The female client came to the clinic complaining of abdominal cramping and at least 10 episodes of diarrhea every day for the last two (2) days. The client just returned from a trip to Mexico. Which intervention should the nurse implement? 1. Instruct the client to take a cathartic laxative daily. 2. Encourage the client to drink lots of Gatorade. 3. Discuss the need to increase protein in the diet. 4. Explain the client should weigh herself daily

ANSWER: 2. 1. The client would be taking antidiarrheal medication, not medications to stimulate bowel movements. 2. The client probably has traveler's diarrhea, and oral rehydration is the preferred choice for replacing fluids lost as a result of diarrhea. An oral glucose electrolyte solution, such as Gatorade, All-Sport, or Pedialyte, is recommended. 3. The client should be encouraged to stay on liquids and eat bland foods of all three (3) food groups—carbohydrates, proteins, and fats. 4. There is no need for the client to weigh herself daily. Symptoms usually resolve within two (2) to three (3) days without complications. TEST-TAKING HINT: Be sure to note the adjectives and adverbs in the stem and the answer options, such as "cathartic" laxative and weigh "daily." These words are very often important in ruling out answers and identifying the correct answer.

The client diagnosed with end-stage liver failure is admitted with esophageal bleeding. The HCP inserts and inflates a triple-lumen nasogastric tube (Sengstaken-Blakemore). Which nursing intervention should the nurse implement for this treatment? 1. Assess the gag reflex every shift. 2. Stay with the client at all times. 3. Administer the laxative lactulose (Chronulac). 4. Monitor the client's ammonia level

ANSWER: 2. 1. The client's throat is not anesthetized during the insertion of a nasogastric tube, so the gag reflex does not need to be assessed. 2. While the balloons are inflated, the client must not be left unattended in case they become dislodged and occlude the airway. This is a safety issue. 3. This laxative is administered to decrease the ammonia level, but the question does not say the client's ammonia level is elevated. 4. Esophageal bleeding does not cause the ammonia level to be elevated. TEST-TAKING HINT: In most cases, the test taker should not select an option containing the word "all," but in some instances, it may be the correct answer. Although the ammonia level is elevated in liver failure, the test taker must be clear as to what the question is asking. "Inflate" is the key to answering the question correctly

The female nurse sticks herself with a contaminated needle. Which action should the nurse implement first? 1. Notify the infection control nurse. 2. Cleanse the area with soap and water. 3. Request postexposure prophylaxis. 4. Check the hepatitis status of the client.

ANSWER: 2. 1. The nurse must notify the infection control nurse as soon as possible so treatment can start if needed, but this is not the first intervention. 2. The nurse should first clean the needle stick with soap and water and attempt stick bleed to help remove any virus injected into the skin. 3. Postexposure prophylaxis may be needed, but this is not the first action. 4. The infection control/employee health nurse will check the status of the client whom the needle was used on before the nurse stuck herself. TEST-TAKING HINT: The question requires the test taker to identify the first intervention. The test taker should think about which intervention will directly help the nurse—and that is to clean the area.

Which intervention should the nurse implement when administering a potassium supplement? 1. Determine the client's allergies. 2. Ask the client about leg cramps. 3. Monitor the client's blood pressure. 4. Monitor the client's complete blood count.

ANSWER: 2. 1. The nurse should inquire about drug allergies before administering all medications, not just potassium. 2. Leg cramps occur when serum potassium levels are too low or too high. If the client has leg cramps, this could indicate an imbalance, which could lead to cardiac dysrhythmias. 3. The blood pressure does not evaluate for dysrhythmias, a possible result of abnormal potassium levels. 4. The complete blood count does not include the potassium level; a chemistry panel is needed

The nurse is completing the preoperative checklist for the client who is scheduled for a laparoscopic cholecystectomy. The preoperative complete blood count (CBC) results are as follows: WBC 12.3, RBC 6.8, Hgb 14.4, Hct 41.8, Platelets 168. Which action should the nurse implement first? 1. Check off that the CBC report is on the chart. 2. Notify the surgeon of the WBC. 3. Assess the client for dyspnea. 4. Teach the client to turn, cough, and deep breathe.

ANSWER: 2. 1. The nurse should recognize the WBC count is high, possibly indicating a current infection. The HCP should be notified immediately, and surgery may be cancelled because a laparoscopic cholecystectomy is an elective procedure. 2. The nurse should notify the surgeon. Surgery may be cancelled because a laparoscopic cholecystectomy is an elective procedure. 3. The client may have any of a number of infections. The high WBC could indicate pneumonia but it could indicate another type of infection. 4. The client may have any of a number of infections. The high WBC could indicate pneumonia but it could indicate another type of infection. TEST-TAKING HINT: The test taker should find the abnormal data in the chart before deciding what action to take. If all the data presented are within normal limits, then option "1" would be the answer.

The nurse is caring for the client with Clostridium difficile. Which intervention should the nurse implement to prevent health-care associated infection (HAI) spread to other clients? 1. Wash hands with Betadine for two (2) minutes after giving care. 2. Wear nonsterile gloves when handling GI excretions. 3. Clean the perianal area with soap and water after each stool. 4. Flush the commode twice when disposing of stool.

ANSWER: 2. 1. The nurse should use soap and water for 15 to 30 seconds before and after caring for the client. Betadine is surgical scrub. 2. Clean gloves should be worn when providing care to prevent the transfer of the bacteria found in the stool. This will prevent the spread of the bacteria to other clients in the health-care facility (nosocomial). But this is not a substitute for good hand hygiene. 3. The nurse should clean the perianal area or instruct the client to clean the area, but this will not prevent the spread of the bacteria to other clients. 4. Flushing the commode twice is not necessary when disposing of stool and will not prevent a nosocomial infection.

Which statement would be an expected outcome for the postoperative client who had general anesthesia? 1. The client will be able to sit in the chair for 30 minutes. 2. The client will have a pulse oximetry reading of 97% on room air. 3. The client will have a urine output of 30 mL per hour. 4. The client will be able to distinguish sharp from dull sensations.

ANSWER: 2. 1. The postoperative client is expected to be out of bed as soon as possible, but this goal is not specific to having general anesthesia. 2. The anesthesia machine takes over the function of the lungs during surgery, so the expected outcome should directly reflect the client's respiratory status; the alveoli can collapse, causing atelectasis. 3. Urine output should be 30 mL/hr, but the expected outcome is not specific to general anesthesia. 4. Sensation would be an outcome assessed after use of a spinal anesthesia or block, but it is not specific to general anesthesia. TEST-TAKING HINT: If the test taker has no idea what the answer is, the test taker should apply Maslow's hierarchy of needs and select the option addressing the airway. This will not always result in the correct answer, but the rule can be followed if the test taker has no idea of the correct answer.

The nurse is caring for the client one (1) day postoperative sigmoid colostomy. Which independent nursing intervention should the nurse implement? 1. Change the infusion rate of the intravenous fluid. 2. Encourage the client to ventilate feelings about body image. 3. Administer opioid narcotic medications for pain management. 4. Assist the client out of bed to sit in the chair twice daily.

ANSWER: 2. 1. The rate of the intravenous fluid is a collaborative nursing intervention because it requires an order from the health-care provider. 2. Encouraging the client to verbalize feelings about body changes assists the client to accept these changes. This is an independent intervention not requiring a health-care provider's order. 3. Medication administration is a collaborative intervention because it requires an order by the health-care provider. 4. Activity level immediately postoperative requires an order by the health-care provider

The client diagnosed with liver problems asks the nurse, "Why are my stools clay-colored?" On which scientific rationale should the nurse base the response? 1. There is an increase in serum ammonia level. 2. The liver is unable to excrete bilirubin. 3. The liver is unable to metabolize fatty foods. 4. A damaged liver cannot detoxify vitamins.

ANSWER: 2. 1. The serum ammonia level is increased in liver failure, but it is not the cause of clay-colored stools. 2. Bilirubin, the by-product of red blood cell destruction, is metabolized in the liver and excreted via the feces, which causes the feces to be brown in color. If the liver is damaged, the bilirubin is excreted via the urine and skin. 3. The liver excretes bile into the gallbladder and the body uses the bile to digest fat, but it does not affect the feces. 4. Vitamin deficiency, resulting from the liver's inability to detoxify vitamins, may cause steatorrhea, but it does not cause clay-colored stool. TEST-TAKING HINT: The test taker should have a grasp of physiology to help answer this question. Clay-colored stool indicates no color in the feces. Because color in the feces is caused by bilirubin, lack of color would be the result of the liver's inability to excrete bilirubin.

The nurse in the holding area of the surgery department is interviewing a client who requests to keep his religious medal on during surgery. Which intervention should the nurse implement? 1. Notify the surgeon about the client's request to wear the medal. 2. Tape the medal to the client and allow the client to wear the medal. 3. Request the family member take the medal prior to surgery. 4. Explain taking the medal to surgery is against the policy

ANSWER: 2. 1. The surgeon does not need to be notified of the client's request; this can be addressed by the nursing staff. 2. The medal should be taped and the client should be allowed to wear the medal because meeting spiritual needs is essential to this client's care. 3. The client should be allowed to bring the medal to surgery if the medal is taped to the client. 4. Hospital policies should be established for the well-being of clients, and spiritual needs should be addressed. TEST-TAKING HINT: Because options "3" and "4" do not allow the client to wear the medal to surgery, these can be eliminated as possible answers because they are both saying the same thing.

The parents of a female toddler bring the child to the pediatrician's office with nausea, vomiting, and diarrhea. Which intervention should the nurse implement first? 1. Ask the parent about the child's diet. 2. Assess the child's tissue turgor. 3. Give the child a sucker if she is "good." 4. Notify the HCP the child is waiting to be seen.

ANSWER: 2. 1. The usual diet is not a concern at this time, when the child goes home the nurse will need to explain what the parents can offer the child to prevent dehydration. 2. Assessing the skin turgor will give the nurse information about the hydration status of the toddler. The nurse should perform an assessment based on the presenting symptoms. 3. Suckers are sugar based and rarely given to children for "good behavior" anymore. 4. The nurse should perform his/her own assessment prior to notifying the HCP to see the child. TEST-TAKING HINT: The nurse must assess the client. This is a professional responsibility regardless of other health-care professionals and their responsibilities.

The charge nurse has just received the shift report. Which client should the nurse see first? 1. The client diagnosed with Crohn's disease who had two (2) semiformed stools on the previous shift. 2. The elderly client admitted from another facility who is complaining of constipation. 3. The client diagnosed with AIDS who had a 200-mL diarrhea stool and has elastic skin tissue turgor. 4. The client diagnosed with hemorrhoids who had some spotting of bright red blood on the toilet tissue.

ANSWER: 2. 1. This client is improving; semiformed stools are better than diarrhea. 2. This client has just arrived, so the nurse does not know if the complaint is valid and needs intervention unless assessed. The elderly have difficulty with constipation as a result of decreased gastric motility, medications, poor diet, and immobility. 3. The client has diarrhea, but only 200 mL, and has elastic tissue turgor, indicating the client is not dehydrated. 4. This is not normal, but it is expected for a client with hemorrhoids. TEST-TAKING HINT: The test taker should notice descriptive words such as "elderly," which should alert the test taker to the age range having an implication in answering the question. Answer options "3" and "4" are expected for the disease processes.

The client with hepatitis asks the nurse, "I went to an herbalist, who recommended I take milk thistle. What do you think about the herb?" Which statement is the nurse's best response? 1. "You are concerned about taking an herb." 2. "The herb has been used to treat liver disease." 3. "I would not take anything that is not prescribed." 4. "Why would you want to take any herbs?"

ANSWER: 2. 1. This is a therapeutic response, and the nurse should provide factual information. 2. Milk thistle has an active ingredient, silymarin, which has been used to treat liver disease for more than 2,000 years. It is a powerful oxidant and promotes liver cell growth. 3. The nurse should not discourage complementary therapies. 4. This is a judgmental statement, and the nurse should encourage the client to ask questions. TEST-TAKING HINT: The test taker may not have any idea what milk thistle is but should apply test-taking strategies, including not selecting options with "why" unless interviewing the client. Only use therapeutic responses when unable to provide factual information. At times, the test taker may not like any answer option but should always apply the rules to help determine the correct answer.

The nurse identifies the nursing diagnosis "risk for injury related to positioning" for the client in the operating room. Which nursing intervention should the nurse implement? 1. Avoid using the cautery unit which does not have a biomedical tag on it. 2. Carefully pad the client's elbows before covering the client with a blanket. 3. Apply a warming pad on the OR table before placing the client on the table. 4. Check the chart for any prescription or overthe-counter medication use.

ANSWER: 2. 1. This would prevent an electrical injury, but the interventions must address positioning, which is the etiology of the nursing diagnosis. 2. Padding the elbows decreases pressure so nerve damage and pressure ulcers are prevented. This addresses the etiology of the nursing diagnosis. 3. This would help to decrease hypothermia, but it does not address the etiology of the nursing diagnosis. 4. Checking the chart for medication use would help prevent interactions between anesthesia and routine medications, but it does not address the etiology of the nursing diagnosis. TEST-TAKING HINT: The test taker must be knowledgeable of nursing diagnosis and the nursing process. The assessment data support the response "risk for injury" and the interventions address the etiology "positioning."

The client is admitted to the medical floor with acute diverticulitis. Which collaborative intervention should the nurse anticipate the health-care provider ordering? 1. Administer total parenteral nutrition. 2. Maintain NPO and nasogastric tube. 3. Maintain on a high-fiber diet and increase fluids. 4. Obtain consent for abdominal surgery

ANSWER: 2. 1. Total parenteral nutrition is not an expected order for this client. 2. The bowel must be put at rest. Therefore, the nurse should anticipate orders for maintaining the client NPO and a nasogastric tube. 3. These orders would be instituted when the client is getting better and the bowel is not inflamed. 4. Surgery is not the first consideration when the client is admitted into the hospital. TEST-TAKING HINT: "Collaborative" means the nurse must care for the client with another discipline, and the health-care provider would have to order all of the distracters. The test taker should remember food and fluid probably should be stopped in the client with lower gastrointestinal problems.

The nurse identifies the client problem "excess fluid volume" for the client in liver failure. Which short-term goal would be most appropriate for this problem? 1. The client will not gain more than two (2) kg a day. 2. The client will have no increase in abdominal girth. 3. The client's vital signs will remain within normal limits. 4. The client will receive a low-sodium diet.

ANSWER: 2. 1. Two (2) kg is more than four (4) pounds, which indicates severe fluid retention and is not an appropriate goal. 2. Excess fluid volume could be secondary to portal hypertension. Therefore, no increase in abdominal girth would be an appropriate short-term goal, indicating no excess of fluid volume. 3. Vital signs are appropriate to monitor, but they do not yield specific information about fluid volume status. 4. Having the client receive a low-sodium diet does not ensure the client will comply with the diet. The short-term goal must evaluate if the fluid volume is within normal limits. TEST-TAKING HINT: Remember, goals evaluate the interventions; therefore, option "4" could be eliminated as the correct answer because it is an intervention, not a goal. Short-term weight fluctuations tend to reflect fluid balance, and any weight gain in 24 hours indicates retention of fluid, which is not an appropriate goal.

The client is admitted with end-stage liver failure and is prescribed the laxative lactulose (Chronulac). Which statement indicates the client needs more teaching concerning this medication? 1. "I should have two to three soft stools a day." 2. "I must check my ammonia level daily." 3. "If I have diarrhea, I will call my doctor." 4. "I should check my stool for any blood."

ANSWER: 2. 1. Two to soft three stools a day indicates the medication is effective. 2. There is no instrument used at home to test daily ammonia levels. The ammonia level is a serum level requiring venipuncture and laboratory diagnostic equipment. 3. Diarrhea indicates an overdosage of the medication, possibly requiring the dosage to be decreased. The HCP needs to make this change in dosage, so the client understands the teaching. 4. The client should check the stool for brightred blood as well as dark, tarry stool. TEST-TAKING HINT: This is an "except" question. The test taker must realize three (3) options indicate an understanding of the teaching. If the test taker does not know the answer, notice that all the options except "2" have something to do with stool, and laxative affects the stool

Which expected outcome should the nurse include for a client diagnosed with peptic ulcer disease? 1. The client's pain is controlled with the use of NSAIDs. 2. The client maintains lifestyle modifications. 3. The client has no signs and symptoms of hemoptysis. 4. The client takes antacids with each meal.

ANSWER: 2. 1. Use of NSAIDs increases and causes problems associated with peptic ulcer disease. 2. Maintaining lifestyle changes such as following an appropriate diet and reducing stress indicate the client is complying with the medical regimen. Compliance is the goal of treatment to prevent complications. 3. Hemoptysis is coughing up blood, which is not a sign or symptom of peptic ulcer disease. This would not be an expected outcome. 4. Antacids should be taken one (1) to three (3) hours after meals, not with each meal. TEST-TAKING HINT: Expected outcomes are positive completion of goals; maintaining lifestyle modifications would be an appropriate goal for any client with any chronic illness

The nurse is caring for a client diagnosed with hemorrhaging duodenal ulcer. Which collaborative interventions should the nurse implement? Select all that apply. 1. Perform a complete pain assessment. 2. Assess the client's vital signs frequently. 3. Administer a proton pump inhibitor intravenously. 4. Obtain permission and administer blood products. 5. Monitor the intake of a soft, bland diet

ANSWER: 3, 4. 1. A pain assessment is an independent intervention the nurse should implement frequently. 2. Evaluating vital signs is an independent intervention the nurse should implement. If the client is able, BPs should be taken lying, sitting, and standing to assess for orthostatic hypotension. 3. This is a collaborative intervention the nurse should implement. It requires an order from the HCP. 4. Administering blood products is collaborative, requiring an order from the HCP. 5. The diet requires an order by the health-care provider, but a diet will not be ordered because the client is NPO. TEST-TAKING HINT: Descriptive words such as "collaborative" or "independent" can be the deciding factor when determining if an answer option is correct or incorrect. These are key words the test taker should identify

The nurse is caring for clients in an outpatient clinic. Which information should the nurse teach regarding the American Cancer Society's recommendations for the early detection of colon cancer? 1. Beginning at age 60, a digital rectal examination should be done yearly. 2. After reaching middle age, a yearly fecal occult blood test should be done. 3. Have a colonoscopy at age 50 and then once every five (5) to 10 years. 4. A flexible sigmoidoscopy should be done yearly after age 40.

ANSWER: 3. 1. A digital rectal examination is done to detect prostate cancer and should be started at age 40 years. 2. "Middle age" is a relative term; specific ages are used for recommendation. 3. The American Cancer Society recommends a colonoscopy at age 50 and every five (5) to 10 years thereafter, and a flexible sigmoidoscopy and a barium enema every five (5) years. 4. A flexible sigmoidoscopy should be done at five (5)-year intervals between the colonoscopy. TEST-TAKING HINT: A digital examination is an examination performed by the examiner's finger and does not examine the entire colon.

Which disease is the client diagnosed with GERD at greater risk for developing? 1. Hiatal hernia. 2. Gastroenteritis. 3. Esophageal cancer. 4. Gastric cancer.

ANSWER: 3. 1. A hiatal hernia places the client at risk for GERD; GERD does not predispose the client for developing a hiatal hernia. 2. Gastroenteritis is an inflammation of the stomach and intestine, usually caused by a virus. 3. Barrett's esophagus results from longterm erosion of the esophagus as a result of reflux of stomach contents secondary to GERD. This is a precursor to esophageal cancer. 4. The problems associated with GERD result from the reflux of acidic stomach contents into the esophagus, which is not a precursor to gastric cancer. TEST-TAKING HINT: The test taker may associate hiatal hernia with GERD. One can be a result of the other, and this can confuse the test taker. If the test taker did not have any idea of the correct answer, option "3" has the word "esophageal" in it, as does the stem of the question, and, therefore, the test taker should select this as the correct answer.

The nurse is working in an outpatient clinic. Which client is most likely to have a diagnosis of diverticulosis? 1. A 60-year-old male with a sedentary lifestyle. 2. A 72-year-old female with multiple childbirths. 3. A 63-year-old female with hemorrhoids. 4. A 40-year-old male with a family history of diverticulosis.

ANSWER: 3. 1. A sedentary lifestyle may lead to obesity and contribute to hypertension or heart disease but usually not to diverticulosis. 2. Multiple childbirths are not a risk factor for developing diverticulosis. 3. Hemorrhoids would indicate the client has chronic constipation, which is a strong risk factor for diverticulosis. Constipation increases the intraluminal pressure in the sigmoid colon, leading to weakness in the intestinal lining, which, in turn, causes outpouchings, or diverticula. 4. A family history is not a risk factor. Having daily bowel movements and preventing constipation will decrease the chance of developing diverticulosis. TEST-TAKING HINT: The test taker must know constipation is the leading risk factor for diverticulosis, and if the test taker knows hemorrhoids are caused by constipation, it would lead the test taker to select option "3" as the correct answer

The unlicensed assistive personnel (UAP) reports the vital signs for a first-day postoperative client as T 100.8˚F, P 80, R 24, and BP 148/80. Which intervention would be most appropriate for the nurse to implement? 1. Administer the antibiotic earlier than scheduled. 2. Change the dressing over the wound. 3. Have the client turn, cough, and deep breathe every two (2) hours. 4. Encourage the client to ambulate in the hall.

ANSWER: 3. 1. Antibiotics need to be administered at the scheduled time. 2. These data would not support the need to change the dressing, and surgeons usually want to change the surgical dressing for the first time. 3. Having the client turn, cough, and deep breathe is the best intervention for the nurse to implement because, if a client has a fever within the first day, it is usually caused by a respiratory problem. 4. The client is first-day postoperative, and ambulating in the hall would not be appropriate. TEST-TAKING HINT: With clients who have undergone surgery, the priority problems are respiration and hemorrhaging. The test taker should select an option which addresses one of these two (2) areas.

The postoperative client is transferred from the PACU to the surgical floor. Which action should the nurse implement first? 1. Apply antiembolism hose to the client. 2. Attach the drain to 20 cm suction. 3. Assess the client's vital signs. 4. Listen to the report from the anesthesiologist.

ANSWER: 3. 1. Applying antiembolism hose may be appropriate, but it is not the first intervention. 2. Attaching a drain would be appropriate but not before assessing the client. 3. Assessing the client's status after transfer from the PACU should be the nurse's first intervention. 4. Receiving reports is not the nurse's first intervention. TEST-TAKING HINT: The test taker should apply the nursing process when answering questions which require identifying the first intervention. Assessment is the first step of the nursing process.

The client is one (1) hour post-endoscopic retrograde cholangiopancreatogram (ERCP). Which intervention should the nurse include in the plan of care? 1. Instruct the client to cough forcefully. 2. Encourage early ambulation. 3. Assess for return of a gag reflex. 4. Administer held medications.

ANSWER: 3. 1. Asking the client to cough forcefully may irritate the client's throat. 2. Early ambulation does not enhance safety because the client will be sedated. 3. The ERCP requires an anesthetic spray be used prior to insertion of the endoscope. If medications, food, or fluid are given orally prior to the return of the gag reflex, the client may aspirate. 4. Medications are not administered until the gag reflex has returned. TEST-TAKING HINT: The test taker must notice adjectives such as "endoscopic," which means the procedure includes going down the mouth; option "3" is the only option that has anything to do with the mouth. Selecting a distracter addressing assessment would be appropriate because assessment is the first step of the nursing process.

The nurse has received the a.m. shift report. Which client should the nurse assess first? 1. The 44-year-old client diagnosed with peptic ulcer disease who is complaining of acute epigastric pain. 2. The 74-year-old client diagnosed with acute gastroenteritis who has had four (4) diarrhea stools during the night. 3. The 65-year-old client diagnosed with IBD who has tented skin turgor and dry mucous membranes. 4. The 15-year-old client diagnosed with food poisoning who has vomited several times during the night shift

ANSWER: 3. 1. Epigastric pain is expected in a client diagnosed with peptic ulcer disease. 2. Four (4) diarrheal stools are not unusual in a client diagnosed with gastroenteritis. 3. Tented skin turgor and dry mucous membranes indicate dehydration, which warrants the nurse assessing this client first. 4. Vomiting is expected in a client diagnosed with food poisoning. TEST-TAKING HINT: When managing clients, the nurse must be able to prioritize care. Therefore, the test taker must be able to determine which client's complaints, signs, or symptoms are not expected of the disease process. The test taker should always look at the client's age because it may help determine the best answer

The nurse is discussing the therapeutic diet for the client diagnosed with diverticulosis. Which meal indicates the client understands the discharge teaching? 1. Fried fish, mashed potatoes, and iced tea. 2. Ham sandwich, applesauce, and whole milk. 3. Chicken salad on whole-wheat bread and water. 4. Lettuce, tomato, and cucumber salad and coffee.

ANSWER: 3. 1. Fried foods increase cholesterol. Mashed potatoes do not have the peel, which is needed for increased fiber. 2. Applesauce does not have the peel, which is needed for increased fiber, and the option does not identify which type of bread; whole milk is high in fat. 3. Chicken salad, which has vegetables such as celery, grapes, and apples, and whole-wheat bread are high in fiber, which is the therapeutic diet prescribed for clients with diverticulosis. An adequate intake of water helps prevent constipation. 4. Tomatoes and cucumbers have seeds, and many health-care providers recommend clients with diverticulosis avoid seeds because of the possibility of the seeds entering the diverticulum and becoming trapped, leading to peritonitis. TEST-TAKING HINT: The test taker must know a high-fiber diet is prescribed for diverticulosis and at least five (5) to six (6) foods are encouraged or discouraged for the different types of diets. High-fiber foods are foods with peels (potato, apple) and whole-wheat products.

The client diagnosed with ulcerative colitis is prescribed a low-residue diet. Which meal selection indicates the client understands the diet teaching? 1. Grilled hamburger on a wheat bun and fried potatoes. 2. A chicken salad sandwich and lettuce and tomato salad. 3. Roast pork, white rice, and plain custard. 4. Fried fish, whole grain pasta, and fruit salad.

ANSWER: 3. 1. Fried potatoes, along with pastries and pies, should be avoided. 2. Raw vegetables should be avoided because this is roughage. 3. A low-residue diet is a low-fiber diet. Products made of refined flour or finely milled grains, along with roasted, baked, or broiled meats, are recommended. 4. Fried foods should be avoided, and whole grain is high in fiber. Nuts and fruits with peels should be avoided. TEST-TAKING HINT: The test taker must know about therapeutic diets prescribed by healthcare providers. Remember, low residue is the same as low fiber.

The nurse is preparing a client diagnosed with GERD for surgery. Which information warrants notifying the HCP? 1. The client's Bernstein esophageal test was positive. 2. The client's abdominal x-ray shows a hiatal hernia. 3. The client's WBC count is 14,000/mm3 . 4. The client's hemoglobin is 13.8 g/dL.

ANSWER: 3. 1. In a Bernstein test, acid is instilled into the distal esophagus, causing immediate heartburn for a client diagnosed with GERD. This would not warrant notifying the HCP. 2. Hiatal hernias are frequently the cause of GERD; therefore, this finding would not warrant notifying the HCP. 3. The client's WBC count is elevated, indicating a possible infection, which warrants notifying the HCP. 4. This is a normal hemoglobin result and would not warrant notifying the HCP. TEST-TAKING HINT: When the test taker is deciding when to notify a health-care provider, the answer should be data not normal for the disease process or signaling a potential or life-threatening complication.

Which assessment question is priority for the nurse to ask the client diagnosed with end-stage liver failure secondary to alcoholic cirrhosis? 1. "How many years have you been drinking alcohol?" 2. "Have you completed an advance directive?" 3. "When did you have your last alcoholic drink?" 4. "What foods did you eat at your last meal?"

ANSWER: 3. 1. It really doesn't matter how long the client has been drinking alcohol. The diagnosis of alcoholic cirrhosis indicates the client has probably been drinking for many years. 2. An advance directive is important for the client who is terminally ill, but it is not the priority question. 3. The nurse must know when the client had the last alcoholic drink to be able to determine when and if the client will experience delirium tremens, the physical withdrawal from alcohol. 4. This is not a typical question asked by the nurse unless the client is malnourished, which is not information provided in the stem. TEST-TAKING HINT: Because the word "alcohol" is in the stem of the question, if the test taker had no idea what the correct answer is, the test taker should select an option with the word "alcohol" in it and look closely at options "1" and "3."

The circulating nurse observes the surgical scrub technician remove a sponge from the edge of the sterile field with a clamp and place the sponge and clamp in a designated area. Which action should the nurse implement? 1. Place the sponge back where it was. 2. Tell the technician not to waste supplies. 3. Do nothing because this is the correct procedure. 4. Take the sponge out of the room immediately

ANSWER: 3. 1. Items which are on the edge of the sterile field are considered contaminated and should be removed from the field. 2. The technician is not wasting supplies; the technician is following principles of asepsis. 3. The technician followed the correct procedure. Sponges are counted to maintain client safety, so all sponges must be kept together to repeat the count before the incision site is sutured. The sponge must be removed, not used, and placed in a designated area to be counted later. 4. Taking the contaminated sponge out of the room would cause a discrepancy in the sponge count. TEST-TAKING HINT: When answering this question, the test taker must consider safety, which is always of the utmost importance during surgery. Sponge count is a basic concept in operating room nursing theory

Which statement indicates to the emergency department nurse the client diagnosed with acute gastroenteritis understands the discharge teaching? 1. "I will probably have some leg cramps while I have gastroenteritis." 2. "I should decrease my fluid intake until the diarrhea subsides." 3. "I should reintroduce solid foods very slowly back into my diet." 4. "I should only drink bottled water until the abdominal cramping stops."

ANSWER: 3. 1. Leg cramps could indicate hypokalemia, which is a potential complication of excessive diarrhea and should be reported to the health-care provider. 2. The client should increase the fluid intake because oral rehydration is the primary treatment for gastroenteritis to replace lost fluid as a result of diarrhea and to prevent dehydration. 3. Reintroducing solid foods slowly, in small amounts, will allow the bowel to rest and the mucosa to return to normal functioning after acute gastroenteritis. 4. Bottled water should be consumed when contaminated water is suspected, and an oral glucose-electrolyte solution, such as Gatorade or Pedialyte, should be recommended. TEST-TAKING HINT: Both options "2" and "4" refer to fluids, which should make the test taker either eliminate both of these or select from one (1) of these two (2) as the right answer.

Which assessment data indicate the postoperative client who had spinal anesthesia is suffering a complication of the anesthesia? 1. Loss of sensation at the lumbar (L5) dermatome. 2. Absence of the client's posterior tibial pulse. 3. The client has a respiratory rate of eight (8). 4. The blood pressure is within 20% of the client's baseline.

ANSWER: 3. 1. Loss of sensation in the L5 dermatome is expected from spinal anesthesia. 2. Absence of a posterior tibial pulse is indicative of a block in the blood supply, but it is not a complication of spinal anesthesia. 3. If the effects of the spinal anesthesia move up rather than down the spinal cord, respirations can be depressed and even blocked. 4. This is an expected outcome and does not indicate a complication. TEST-TAKING HINT: The test taker must know normal rates for vital signs, and a respiratory rate of eight (8) would be significantly low for any client and indicate a possible complication.

The occupational health nurse observes the chief financial officer eat large lunch meals. The client disappears into the restroom after a meal for about 20 minutes. Which observation by the nurse would indicate the client has bulimia? 1. The client jogs two (2) miles a day. 2. The client has not gained weight. 3. The client's teeth are a green color. 4. The client has smooth knuckles

ANSWER: 3. 1. Many clients jog one (1) to two (2) miles per day as part of their exercise program. This does not indicate bulimia. 2. Not gaining weight may be an end result of bulimia, but it does not identify bulimia. 3. Bulimia is characterized by bingeing and purging by inducing vomiting after a meal. Stomach contents are acidic and the acid wears away the enamel on the teeth, leaving the teeth a green color. 4. The client has calluses on the knuckles from pushing them into the throat to induce vomiting. TEST-TAKING HINT: The question requires the nurse to be knowledgeable of the signs/ symptoms of bulimia. Vomiting should lead the test taker to green teeth secondary to hydrochloric acid.

The nurse assesses a large amount of red drainage on the dressing of a client who is six (6) hours postoperative open cholecystectomy. Which intervention should the nurse implement? 1. Measure the abdominal girth. 2. Palpate the lower abdomen for a mass. 3. Turn client onto side to assess for further drainage. 4. Remove the dressing to determine the source.

ANSWER: 3. 1. Measuring the abdominal girth helps further assess internal bleeding, not external bleeding. 2. Palpating the lower abdomen assesses the bladder, not bleeding. 3. Turning the client to the side to assess the amount of drainage and possible bleeding is important prior to contacting the surgeon. 4. The first dressing change is usually done by the surgeon; the nurse can reinforce the dressing. TEST-TAKING HINT: The adjectives "large" and "red" indicate the client is bleeding, and assessment is always priority when the client is having a possible complication of a surgery. Remember, assessment is the first step in the nursing process.

The charge nurse is making assignments. Staffing includes a registered nurse with five (5) years of medical-surgical experience, a newly graduated registered nurse, and two (2) unlicensed assistive personnel (UAPs). Which client should be assigned to the most experienced nurse? 1. The 39-year-old client diagnosed with lower esophageal dysfunction who is complaining of pyrosis. 2. The 54-year-old client diagnosed with Barrett's esophagus who is scheduled to have an endoscopy this morning. 3. The 46-year-old client diagnosed with gastroesophageal reflux disease who has wheezes in all five (5) lobes. 4. The 68-year-old client who is three (3) days postoperative for hiatal hernia and needs to be ambulated four (4) times today

ANSWER: 3. 1. Pyrosis is heartburn and is expected in a client diagnosed with GERD. The new graduate can care for this client. 2. Barrett's esophagus is a complication of GERD; new graduates can prepare a client for a diagnostic procedure. 3. This client is exhibiting symptoms of asthma, a complication of GERD. This client should be assigned to the most experienced nurse. 4. This client can be cared for by the new graduate, and ambulating can be delegated to the unlicensed assistive personnel (UAP). TEST-TAKING HINT: The most experienced nurse should be assigned to the client whose assessment and care require more experience and knowledge about the disease process, potential complications, and medications. The term "most experienced" in the stem is the key to answering this question.

The client with type 2 diabetes is prescribed prednisone, a steroid, for an acute exacerbation of inflammatory bowel disease (IBD). Which intervention should the nurse discuss with the client? 1. Take this medication on an empty stomach. 2. Notify the HCP if experiencing a moon face. 3. Take the steroid medication as prescribed. 4. Notify the HCP if the blood glucose is over 160.

ANSWER: 3. 1. Steroids can cause erosion of the stomach and should be taken with food. 2. A moon face is an expected side effect of steroids. 3. This medication must be tapered off to prevent adrenal insufficiency; therefore, the client must take this medication as prescribed. 4. Steroids may increase the client's blood glucose, but diabetic medication regimens are usually not altered for the short period of time the client with an acute exacerbation is prescribed steroids. TEST-TAKING HINT: The test taker should know few medications must be taken on an empty stomach, which would cause option "1" to be eliminated. All medications should be taken as prescribed—don't think the answer is too easy

The nurse must obtain surgical consent forms for the scheduled surgery. Which client would not be able to consent legally to surgery? 1. The 65-year-old client who cannot read or write. 2. The 30-year-old client who does not understand English. 3. The 16-year-old client who has a fractured ankle. 4. The 80-year-old client who is not oriented to the day

ANSWER: 3. 1. The 65-year-old client who cannot read can mark an "X" on the form and is legally able to sign a surgical permit as long as the client understands the benefits, alternatives, and all potential complications of the surgery. 2. The client who does not speak English can and should have information given and questions answered in the client's native language. 3. A 16-year-old client is not legally able to give permission for surgery unless the adolescent has been given an emancipated status by a judge. This information was not given in the stem. 4. A client is able to give permission unless determined incompetent. Not knowing the day of the week is not significant. TEST-TAKING HINT: Age in a stem or option gives the test taker a clue as to the correct answer. The nurse must be aware of legal issues when caring for the client.

The client diagnosed with bulimia has a BMI of 20. The nurse understands which scientific rationale explains this finding? (Underweight: under 19; Ideal weight: 19-24.9; Overweight: 25-30; Obese: 30.1 and over). 1. The BMI is low because the client does not eat and exercises frequently. 2. The BMI is within normal range because the client's therapy is effective. 3. The BMI is WNL because the client vomits or uses laxatives to prevent weight gain. 4. The BMI is high and the client needs to try new methods of weight control.

ANSWER: 3. 1. The BMI is not low; these symptoms are associated with anorexia. 2. The BMI is WNL but does not indicate therapy is effective. 3. The patient diagnosed with bulimia binge eats and then induces vomiting or uses laxatives to prevent weight gain. 4. The BMI is WNL. TEST-TAKING HINT: The test taker will be required to interpret graphs on the NCLEX-RN and associate the graph information with disease processes.

Which diagnostic test should the nurse monitor for the client diagnosed with severe anorexia nervosa? 1. Liver function tests. 2. Kidney function tests. 3. Cardiac function tests. 4. Bone density scan

ANSWER: 3. 1. The client diagnosed with anorexia will have muscle tissue wasting; liver function tests will not monitor for this. 2. Kidney function tests will not monitor nutrition or muscle wasting. 3. The heart is a muscle; in severe anorexia (more than 60% under ideal body weight), muscle tissue is catabolized to provide energy to the body. The client is at risk for death from cardiac complications. 4. The client's entire body will be involved in the process as a result of malnutrition, but bone density tests are not done. TEST-TAKING HINT: The test taker needs to be aware of the complications associated with specific disease processes.

The client is being prepared for discharge after a laparoscopic cholecystectomy. Which intervention should the nurse implement? 1. Discuss the need to change the abdominal dressing daily. 2. Tell the client to check the T-tube output every eight (8) hours. 3. Include the significant other in the discharge teaching. 4. Instruct the client to stay off clear liquids for two (2) days.

ANSWER: 3. 1. The client has three (3) to four (4) incisions with Band-Aids in the upper quadrant, not an abdominal dressing. 2. The client will not have a T-tube with a laparoscopic cholecystectomy. 3. A laparoscopic cholecystectomy is done in day surgery. The nurse must make sure the significant others taking care of the client are knowledgeable of postoperative care. 4. The client will be on a regular diet after being discharged from the day surgery clinic.

The nurse is planning the care of a client diagnosed with lower esophageal sphincter dysfunction. Which dietary modifications should be included in the plan of care? 1. Allow any of the client's favorite foods as long as the amount is limited. 2. Have the client perform eructation exercises several times a day. 3. Eat four (4) to six (6) small meals a day and limit fluids during mealtimes. 4. Encourage the client to consume a glass of red wine with one (1) meal a day.

ANSWER: 3. 1. The client is instructed to avoid spicy and acidic foods and any food producing symptoms. 2. Eructation means belching, which is a symptom of GERD. 3. Clients should eat small, frequent meals and limit fluids with the meals to prevent reflux into the esophagus from a distended stomach. 4. Clients are encouraged to forgo all alcoholic beverages because alcohol relaxes the lower esophageal sphincter and increases the risk of reflux. TEST-TAKING HINT: The word "any" in option "1" should give the test taker a clue that, unless there are absolutely no dietary restrictions, this is an incorrect answer. Option "2" requires knowledge of medical terminology

Which situation demonstrates the circulating nurse acting as the client's advocate? 1. Plays the client's favorite audio book during surgery. 2. Keeps the family informed of the findings of the surgery. 3. Keeps the operating room door closed at all times. 4. Calls the client by the first name when the client is recovering.

ANSWER: 3. 1. The client is not awake during surgery, so playing a favorite audio book would not be an example of client advocacy. 2. This would be a nice action to take, but it is not an example of client advocacy. 3. This would keep the client's dignity by maintaining privacy. With this action, the nurse is speaking for the client while the client cannot speak as a result of anesthesia; this is an example of client advocacy. 4. Clients should be referred to by their last name, rather than first, unless the client requests the staff to use his or her first name. This is not an example of client advocacy. TEST-TAKING HINT: The definition of a client advocate is a person designated to speak up for the client's rights when the client cannot

The client is diagnosed with peritonitis. Which assessment data indicate to the nurse the client's condition is improving? 1. The client is using more pain medication on a daily basis. 2. The client's nasogastric tube is draining coffee-ground material. 3. The client has a decrease in temperature and a soft abdomen. 4. The client has had two (2) soft-formed bowel movements.

ANSWER: 3. 1. The client needing more pain medication indicates the client's condition is getting worse. 2. Coffee-ground material indicates old blood from the gastrointestinal system. 3. Because the signs of peritonitis are elevated temperature and rigid abdomen, a reversal of these signs indicates the client is getting better. 4. Two soft-formed bowel movements are normal, but this does not have anything to do with peritonitis. TEST-TAKING HINT: The -itis of peritonitis means inflammation, which is associated with an elevated temperature. A decrease in temperature would be a sign the client is improving

Which instruction should be discussed with the client diagnosed with gastroesophageal reflux disease (GERD)? 1. Eat a low-carbohydrate, low-sodium diet. 2. Lie down for 30 minutes after eating. 3. Do not eat spicy foods or acidic foods. 4. Drink two (2) glasses of water before bedtime.

ANSWER: 3. 1. The client should eat a low-fat, high-fiber diet. 2. The client should not lie down for at least two (2) hours after each meal to prevent gastric reflux. 3. The client should avoid irritants, such as spicy foods or acidic foods, as well as alcohol, caffeine, and tobacco, because they increase gastric secretions. 4. The client should avoid food or drink two (2) hours before bedtime or lying down after eating.

The client has had a liver biopsy. Which postprocedure intervention should the nurse implement? 1. Instruct the client to void immediately. 2. Keep the client NPO for eight (8) hours. 3. Place the client on the right side. 4. Monitor blood urea nitrogen (BUN) and creatinine level.

ANSWER: 3. 1. The client should empty the bladder immediately prior to the liver biopsy, not after the procedure. 2. Foods and fluids are usually withheld two (2) hours after the biopsy, after which the client can resume the usual diet. 3. Direct pressure is applied to the site, and then the client is placed on the right side to maintain site pressure. 4. Blood urea nitrogen (BUN) and creatinine levels are monitored for kidney function, not liver function, and the renal system is not affected with the liver biopsy. TEST-TAKING HINT: The adjective "postprocedure" should help the test taker rule out option "1." Knowing the anatomical position of the liver should help the test taker select option "3" as the correct answer. The test taker must know laboratory data for each organ, which helps rule out option "4" as a possible correct answer

The client is admitted to the medical unit with a diagnosis of acute diverticulitis. Which healthcare provider's order should the nurse question? 1. Insert a nasogastric tube. 2. Start an IV with D5W at 125 mL/hr. 3. Put the client on a clear liquid diet. 4. Place the client on bedrest with bathroom privileges.

ANSWER: 3. 1. The client will have a nasogastric tube because the client will be NPO, which will decompress the bowel and remove hydrochloric acid. 2. Preventing dehydration is a priority with the client who is NPO. 3. The nurse should question a clear liquid diet because the bowel must be put on total rest, which means NPO. 4. The client is in severe pain and should be on bedrest, which will help rest the bowel. TEST-TAKING HINT: This is an "except" question. Therefore, the test taker must identify which answer option is incorrect for the stem. Sometimes flipping the question helps in selecting the correct answer. In this question, the test taker could ask, "Which HCP orders would be expected for a client diagnosed with diverticulitis?" The unexpected option would be the correct answer.

The client presents with a complete blockage of the large intestine from a tumor. Which healthcare provider's order would the nurse question? 1. Obtain consent for a colonoscopy and biopsy. 2. Start an IV of 0.9% saline at 125 mL/hr. 3. Administer 3 liters of GoLYTELY. 4. Give tap water enemas until it is clear

ANSWER: 3. 1. The client will need to have diagnostic tests, so this is an appropriate intervention. 2. The client who has an intestinal blockage will need to be hydrated. 3. This client has an intestinal blockage from a solid tumor blocking the colon. Although the client needs to be cleaned out for the colonoscopy, GoLYTELY could cause severe cramping without a reasonable benefit to the client and could cause a medical emergency. 4. Tap water enemas until clear would be instilling water from below the tumor to try to rid the colon of any feces. The client can expel this water. TEST-TAKING HINT: The stem states a "complete blockage," which indicates the client needs surgery. Therefore, options "1" and "2" are appropriate for surgery. The stem asks the test taker which order would be questioned, so this is an "except" question.

The client is diagnosed with ulcerative colitis. Which sign/symptom warrants immediate intervention by the nurse? 1. The client has 20 bloody stools a day. 2. The client's oral temperature is 99.8°F. 3. The client's abdomen is hard and rigid. 4. The client complains of urinating when coughing

ANSWER: 3. 1. The colon is ulcerated and unable to absorb water; 10 to 20 bloody diarrhea stools is the most common symptom of ulcerative colitis and does not warrant immediate intervention. 2. This is not an elevated temperature and does not warrant immediate intervention by the nurse. 3. A hard, rigid abdomen indicates peritonitis, a complication of ulcerative colitis, and warrants immediate intervention. 4. Stress incontinence is not a symptom of colitis and does not warrant immediate intervention.

The emergency department nurse is working in a community hospital. During the past two (2) hours, 15 clients have been admitted with Salmonella food poisoning. Which information should the nurse discuss with the clients? 1. Explain the incubation period is 48 to 72 hours. 2. Explain the source of this poisoning is contaminated water. 3. Explain the sources of contamination are eggs and chicken. 4. Explain the bacterial contaminant came from canned foods.

ANSWER: 3. 1. The incubation period for Salmonella food poisoning is 8 to 48 hours. 2. Salmonellae bacteria are not transmitted to humans via water. 3. Eggs, poultry, and pet turtles are sources of the Salmonellae bacteria, which cause food poisoning. 4. Clostridium botulinum is transmitted via improperly canned food.

Which task is most appropriate for the nurse to delegate to the unlicensed assistive personnel (UAP)? 1. Draw the serum liver function test. 2. Evaluate the client's intake and output. 3. Perform the bedside glucometer check. 4. Help the ward clerk transcribe orders

ANSWER: 3. 1. The laboratory technician draws serum blood studies, not the UAP. 2. The UAP can obtain the intake and output, but the nurse must evaluate the data to determine if the results are normal for the client's disease process or condition. 3. The UAP can perform a bedside glucometer check, but the nurse must evaluate the result and determine any action needed. 4. The ward clerk has specific training that allows the transcribing of health-care provider orders. TEST-TAKING HINT: The test taker must be knowledgeable of delegation rules; the nurse cannot delegate assessing, teaching, medication administration, evaluating, and any task for an unstable client.

The 84-year-old client comes to the clinic complaining of right lower abdominal pain. Which question is most appropriate for the nurse to ask the client? 1. "When was your last bowel movement?" 2. "Did you have a high-fat meal last night?" 3. "Can you describe the type of pain?" 4. "Have you been experiencing any gas?"

ANSWER: 3. 1. The last bowel movement does not help identify the cause of the client's right lower abdominal pain. This might be appropriate for a client with left lower abdominal pain. 2. Information about a high-fat meal would be asked if the nurse suspected the client had a gallbladder problem. 3. An elderly client may experience a ruptured appendix with minimal pain; therefore, the nurse should assess the characteristics of the pain. 4. The passage of flatus (gas) does not help determine the cause of right lower abdominal pain. TEST-TAKING HINT: The test taker should go back to basics and assess the client.

The client is diagnosed with gastroenteritis. Which laboratory data warrant immediate intervention by the nurse? 1. A serum sodium level of 137 mEq/L. 2. Arterial blood gases of pH 7.37, Pao2 95, Paco2 43, HCO3 24. 3. A serum potassium level of 3.3 mEq/L. 4. A stool sample positive for fecal leukocytes

ANSWER: 3. 1. The normal serum sodium level is 135 to 145 mEq/L; therefore, an intervention by the nurse is not needed. 2. These are normal arterial blood gas results; therefore, the nurse would not need to intervene. 3. In gastroenteritis, diarrhea often results in metabolic acidosis and loss of potassium. The normal serum potassium level is 3.5 to 5.5 mEq/L; therefore, a level of 3.3 mEq/L would require immediate intervention. Hypokalemia can lead to life-threatening cardiac dysrhythmias. 4. A stool specimen showing fecal leukocytes supports the diagnosis of gastroenteritis and does not warrant immediate intervention by the nurse. TEST-TAKING HINT: The test taker should read the stem and be certain he or she understands what the question is asking—in this case, which data require "immediate intervention"? Therefore, the test taker is identifying an answer not normal for the disease process.

The nurse is assessing the client recovering from abdominal surgery who has a patientcontrolled analgesia (PCA) pump. The client has shallow respirations and refuses to deep breathe. Which intervention should the nurse implement? 1. Insist the client take deep breaths. 2. Notify the surgeon to request a chest x-ray. 3. Determine the last time the client used the PCA pump. 4. Administer oxygen at 2 L/min via nasal cannula

ANSWER: 3. 1. The nurse cannot force the client to do anything; this would be considered assault. 2. There are no data to support the need for a chest x-ray. 3. Shallow respirations and refusal to deep breathe could be the result of abdominal pain. The nurse should assess the client for pain and determine the last time the PCA pump was used. 4. Based on the information given, the client does not need oxygen. TEST-TAKING HINT: If the test taker is unsure of the answer, identifying key words in the stem—"abdominal surgery" and "PCA"— should guide the test taker to select an option related to one (1) of these key words. "Determine" can be substituted for the word "assess," which is the first step of the nursing process

The client who had abdominal surgery tells the nurse, "I felt something give way in my stomach." Which intervention should the nurse implement first? 1. Notify the surgeon immediately. 2. Instruct the client to splint the incision. 3. Assess the abdominal wound incision. 4. Administer pain medication intravenously

ANSWER: 3. 1. The nurse may notify the surgeon if warranted, but it is not the first intervention. 2. The nurse should instruct the client to splint the incision when coughing, then take further action. 3. Assessing the surgical incision is the first intervention because this may indicate the client has wound dehiscence. 4. The nurse should never administer pain medication without assessing for potential complications. TEST-TAKING HINT: The stem is asking which intervention is first. This means all four (4) answer options could be possible actions but only one (1) is first. The test taker should use the nursing process and select the option addressing assessment because it is the first step in the nursing process.

The postoperative client complains of hearing a "popping sound" and feeling "something opening" when ambulating in the room. Which intervention should the nurse implement first? 1. Notify the surgeon the client has had an evisceration. 2. Contact the surgery department to prepare for emergency surgery. 3. Assess the operative site and cover the site with a moistened dressing. 4. Explain this is a common feeling and tell the client to continue with activity.

ANSWER: 3. 1. The nurse should assess the client before notifying the surgeon the client felt or heard a "pop" and "something opening." 2. The surgery department may or may not need to be notified. The incision should be assessed. 3. The nurse should assess the surgical site and, if the site has eviscerated, cover the opening with a sterile dressing moistened with sterile 0.9% saline. This will prevent the tissues from becoming dry and infected. 4. The nurse should not dismiss any complaint from a client without further assessment.

The client diagnosed with Crohn's disease is crying and tells the nurse, "I can't take it anymore. I never know when I will get sick and end up here in the hospital." Which statement is the nurse's best response? 1. "I understand how frustrating this must be for you." 2. "You must keep thinking about the good things in your life." 3. "I can see you are very upset. I'll sit down and we can talk." 4. "Are you thinking about doing anything like committing suicide?"

ANSWER: 3. 1. The nurse should never tell a client he or she understands what the client is going through. 2. Telling the client to think about the good things is not addressing the client's feelings. 3. The client is crying and is expressing feelings of powerlessness; therefore, the nurse should allow the client to talk. 4. The client is crying and states "I can't take it anymore," but this is not a suicidal comment or situation. TEST-TAKING HINT: There are rules applied to therapeutic responses. Do not say "understand" and do not ask "why." The test taker should select an option where some type of feeling is being reflected in the statement

The nurse, a licensed practical nurse (LPN), and an unlicensed assistive personnel (UAP) are caring for clients on a medical floor. Which nursing task would be most appropriate to assign to the LPN? 1. Assist the UAP to learn to perform blood glucose checks. 2. Monitor the potassium levels of a client with diarrhea. 3. Administer a bulk laxative to a client diagnosed with constipation. 4. Assess the abdomen of a client who has had complaints of pain.

ANSWER: 3. 1. The nurse will be responsible for signing off on the UAP as to being competent to perform the blood glucose. The nurse should do this to determine the competency of the UAP. 2. The laboratory values may require the nurse to interpret and act on the results. The nurse cannot delegate tasks requiring professional judgment. 3. The LPN can administer medications such as a laxative. 4. The nurse cannot delegate assessment. TEST-TAKING HINT: Nurses cannot delegate any activity requiring professional judgment, assessment, teaching, or evaluation.

The client diagnosed with end-stage renal failure and ascites is scheduled for a paracentesis. Which client teaching should the nurse discuss with the client? 1. Explain the procedure will be done in the operating room. 2. Instruct the client a Foley catheter will have to be inserted. 3. Tell the client vital signs will be taken frequently after the procedure. 4. Provide instructions on holding the breath when the HCP inserts the catheter.

ANSWER: 3. 1. The procedure is done in the client's room, with the client seated either on the side of the bed or in a chair. 2. The client should empty the bladder prior to the procedure to avoid bladder puncture, but there is no need for an indwelling catheter to be inserted. 3. The client is at risk for hypovolemia; therefore, vital signs will be assessed frequently to monitor for signs of hemorrhaging. 4. The client does not have to hold the breath when the catheter is inserted into the peritoneum; this is done when obtaining a liver biopsy. TEST-TAKING HINT: If the test taker had no idea what the answer is, knowing vital signs are assessed after all procedures should make the test taker select this option.

The nurse has been assigned to care for a client diagnosed with peptic ulcer disease. Which assessment data require further intervention? 1. Bowel sounds auscultated 15 times in one (1) minute. 2. Belching after eating a heavy and fatty meal late at night. 3. A decrease in systolic blood pressure (BP) of 20 mm Hg from lying to sitting. 4. A decreased frequency of distress located in the epigastric region.

ANSWER: 3. 1. The range for normoactive bowel sounds is from five (5) to 35 times per minute. This would require no intervention. 2. Belching after a heavy, fatty meal is a symptom of gallbladder disease. Eating late at night may cause symptoms of esophageal disorders. 3. A decrease of 20 mm Hg in blood pressure after changing position from lying, to sitting, to standing is orthostatic hypotension. This could indicate the client is bleeding. 4. A decrease in the quality and quantity of discomfort shows an improvement in the client's condition. This would not require further intervention. TEST-TAKING HINT: When the question asks about further intervention, the test taker should examine the answer options for an unexpected outcome requiring further assessment.

The surgical client's vital signs are T 98˚F, P 106, R 24, and BP 88/40. The client is awake and oriented times three (3) and the skin is pale and damp. Which intervention should the nurse implement first? 1. Call the surgeon and report the vital signs. 2. Start an IV of D5RL with 20 mEq KCl at 125 mL/hr. 3. Elevate the feet and lower the head. 4. Monitor the vital signs every 15 minutes.

ANSWER: 3. 1. The surgeon should be notified, but this is not the first action; the client must be cared for. 2. The postoperative client had lactated Ringer's infused during surgery. The rate should be increased during hemorrhage—which the vital signs indicate is occurring—but potassium should not be added. 3. By lowering the head of the bed and raising the feet, the blood is shunted to the brain until volume-expanding fluids can be administered, which is the first intervention for a client who is hemorrhaging. 4. When signs and symptoms of shock are observed, the nurse will monitor the vital signs more frequently than every 15 minutes. TEST-TAKING HINT: These are the signs of hypovolemic shock. The test taker should select the intervention which will directly affect the client's problem and can be implemented the fastest to ensure the client's safety.

Which nursing task would be most appropriate to delegate to the unlicensed assistive personnel (UAP) on a postoperative unit? 1. Change the dressing over the surgical site. 2. Teach the client how to perform incentive spirometry. 3. Empty and record the amount of drainage in the JP drain. 4. Auscultate the bowel sounds in all four (4) quadrants

ANSWER: 3. 1. The surgeon usually removes the first surgical dressing. The nurse performs surgical dressing changes so asepsis is maintained and the incision can be assessed. 2. Nurses cannot delegate teaching and assessment. 3. Emptying the drainage devices and recording the amounts on the bedside intake and output forms can be delegated. 4. Listening to bowel sounds is assessing and cannot be delegated

The client returned to the medical surgical unit at 1800 following a two (2)-hour surgery and one (1) hour in the postanesthesia recovery unit (PACU). The nurse is reviewing the client's intake and output at midnight. Oral intake: 250 mL. IV intake: 740 mL. Urine output: 120 mL. Other output: 120 mL. Which intervention should the nurse implement based on the recorded data? 1. Immediately place an indwelling catheter in the client. 2. Assess the client's skin turgor in the abdominal area. 3. Recheck the client's urinary output in two (2) hours. 4. Encourage the client to drink 500 mL of clear liquids.

ANSWER: 3. 1. This client has been without food or water for several hours prior to the surgery, so the client needs fluid intake to urinate. Indwelling catheters carry with them the risk for a urinary tract infection. The nurse should continue to monitor the client's output before taking this action. 2. Skin turgor should not be assessed for indicating a fluid volume deficit yet. The client received fluids during surgery and would have had an indwelling catheter to prevent incontinence during surgery. 3. The nurse should wait and monitor the client before taking another action. The client has an IV running at 150 mL per hour. The client should be given a chance to produce urine before rushing to take another action. 4. The client should be allowed to rest. It has been a long day of waiting for the surgery to take place and then having the surgery. Pushing liquids at midnight is not advised, considering the need for rest. TEST-TAKING HINT: Timing is important to consider when answering this question— midnight, return from surgery, and PACU at 1800. Basic knowledge of surgical procedure is the client is NPO for at least eight (8) hours before general anesthesia.

Which problem is appropriate for the nurse to identify for a client in the intraoperative phase of surgery? 1. Alteration in comfort. 2. Disuse syndrome. 3. Risk for injury. 4. Altered gas exchange

ANSWER: 3. 1. This client problem would be appropriate for a postoperative client or, in some circumstances, a preoperative client, but not for a client in surgery. 2. This is a problem of long-term immobility and would not apply during surgery. 3. This problem would be appropriate for the intraoperative phase. The circulating nurse would strap and carefully pad areas to prevent damage to tissues and nerves. 4. The client is receiving oxygen or breathing by the ventilator. The client should not have an alteration in gas exchange

Which situation is an example of the nurse fulfilling the role of client advocate? 1. The nurse brings the client pain medication when it is due. 2. The nurse collaborates with other disciplines during the care conference. 3. The nurse contacts the health-care provider when pain relief is not obtained. 4. The nurse teaches the client to ask for medication before the pain gets to a "5."

ANSWER: 3. 1. This exemplifies the role of provider of care, and it does not address client advocacy. 2. This action is addressing the role of collaborator. 3. When the nurse contacts the HCP about unrelieved pain, the nurse is speaking when the client cannot, which is the definition of a client advocate. 4. This action is providing care to the client and does not address client advocacy. TEST-TAKING HINT: One (1) of the most important roles of the nurse is to be a client advocate. The nurse must always identify problems and follow through to their resolution.

Which problem would be most appropriate for the nurse to identify for the client experiencing acute pain? 1. Ineffective coping. 2. Potential for injury. 3. Alteration in comfort. 4. Altered sensory input.

ANSWER: 3. 1. This is a psychosocial problem, which is not appropriate for an acute physiological problem. 2. A potential problem is not priority for a client in acute pain. 3. Alteration in comfort is addressing the client's acute pain. 4. Altered sensory input does not address the client's acute physical pain. TEST-TAKING HINT: The test taker should be familiar with NANDA's list of client problems and nursing diagnoses, which includes alteration in comfort for pain. Potential problems do not have priority over actual problems.

The client is diagnosed with end-stage liver failure. The client asks the nurse, "Why is my doctor decreasing the doses of my medications?" Which statement is the nurse's best response? 1. "You are worried because your doctor has decreased the dosage." 2. "You really should ask your doctor. I am sure there is a good reason." 3. "You may have an overdose of the medications because your liver is damaged." 4. "The half-life of the medications is altered because the liver is damaged."

ANSWER: 3. 1. This is a therapeutic response and is used to encourage the client to verbalize feelings but does not provide factual information. 2. This is passing the buck; the nurse should be able to answer this question. 3. This is the main reason the HCP decreases the client's medication dose and is an explanation appropriate for the client. 4. This is the medical explanation as to why the medication dose is decreased, but it should not be used to explain to a layperson. TEST-TAKING HINT: The test taker should provide factual information when the client asks "why." Therefore, options "1" and "2" could be eliminated as possible correct answers. Both options "3" and "4" explain the rationale for decreasing the medication dose, but the nurse should answer in terms the client can understand. Would a layperson know what "half-life" means?

The nurse is completing a preoperative assessment on a male client who states, "I am allergic to codeine." Which intervention should the nurse implement first? 1. Apply an allergy bracelet on the client's wrist. 2. Label the client's allergies on the front of the chart. 3. Ask the client what happens when he takes the codeine. 4. Document the allergy on the medication administration record.

ANSWER: 3. 1. This is an important step for the nurse to implement, but it is not the first intervention. 2. This must be done, but it is not the first intervention. 3. The nurse should first assess the events which occurred when the client took this medication because many clients think a side effect, such as nausea, is an allergic reaction. 4. This information must be put on the medication administration record (MAR), but it is not the first intervention. TEST-TAKING HINT: The stem is asking the test taker to identify the first intervention. Therefore, all four (4) options could be interventions which should be implemented, but assessment is the first part of the nursing process, so option "3" is the correct answer.

The nurse is caring for the immediate postoperative client who had a laparoscopic cholecystectomy. Which task could the nurse delegate to the unlicensed assistive personnel (UAP)? 1. Check the abdominal dressings for bleeding. 2. Increase the IV fluid if the blood pressure is low. 3. Ambulate the client to the bathroom. 4. Auscultate the breath sounds in all lobes.

ANSWER: 3. 1. This is assessment and cannot be delegated. 2. This intervention would require nursing judgment, and increasing IV fluid is medication administration; neither task can be delegated. 3. A day surgery client can be ambulated to the bathroom, so this task can be delegated to the UAP. 4. This would require assessment and cannot be delegated. TEST-TAKING HINT: The nurse cannot delegate teaching, assessing, medication administration, and evaluating or any task for an unstable client to a UAP

The three (3)-day postoperative client is complaining of unrelieved pain at the incision site one (1) hour after the administration of narcotic pain medication. Which action should the nurse implement first? 1. Check the MAR for another medication to administer. 2. Teach the client to use guided imagery to relieve the pain. 3. Assess the client for complications. 4. Elevate the head of the client's bed.

ANSWER: 3. 1. This may be an appropriate intervention, but unrelieved pain three (3) days after the surgery may indicate a problem. The nurse should assess the client first. 2. A client in pain is not ready to learn. If narcotic medication is not successful, something else may be occurring. 3. The first step of the nursing process is to assess. Pain unrelieved three (3) days postoperative needs to be investigated. 4. Repositioning the client may or may not help, but the nurse should assess the client. TEST-TAKING HINT: The first step of the nursing process is ASSESS. The test taker must have some systematic method of problem solving. The test taker must also remember "if in stress—do not assess." In other words, the test taker has been given enough information to implement an intervention immediately.

The nurse is assessing a client in the day surgery unit who states, "I am really afraid of having this surgery. I'm afraid of what they will find." Which statement would be the most therapeutic response by the nurse? 1. "Don't worry about your surgery. It is safe." 2. "Tell me why you're worried about your surgery." 3. "Tell me about your fears of having this surgery." 4. "I understand how you feel. Surgery is frightening."

ANSWER: 3. 1. This statement is giving false reassurance. 2. "Why" is never therapeutic. The client does not owe the nurse an explanation. 3. This statement focuses on the emotion which that the client identified and is therapeutic. 4. This statement belittles the client's fear, and no person understands how another person feels. TEST-TAKING HINT: There are rules the test taker should implement when answering these types of questions. The test taker should not select an option which asks the client "why," such as option "2," or an option which states, "I understand," such as option "4."

Which client outcome would the nurse identify for the preoperative client? 1. The client's abnormal laboratory data will be reported to the anesthesiologist. 2. The client will not have any postoperative complications for the first 24 hours. 3. The client will demonstrate the use of a pillow to splint while deep breathing. 4. The client will complete an advance directive before having the surgery

ANSWER: 3. 1. This would be an outcome for the health-care team, not the client. 2. This would be an appropriate postoperative, not preoperative, outcome. 3. This would be the expected outcome for the client during the preoperative phase. After the teaching has been completed, the client should be able to demonstrate how to splint with the pillow while deep breathing and coughing. 4. This would not be an expected outcome for the preoperative client. All clients should be encouraged to complete an advance directive, but it is not required by law.

Which task should the nurse delegate to the unlicensed assistive personnel (UAP) to improve the desire to eat in a 14-year-old client diagnosed with anorexia? 1. Administer an antiemetic 30 minutes before the meal. 2. Provide mouth care with lemon-glycerin swabs prior to the meal. 3. Create a social atmosphere by interacting with the client. 4. Encourage the client's parents to sit with the client during meals.

ANSWER: 3. 1. Unlicensed assistive personnel (UAP) cannot administer medications, and this is not appropriate for a client with anorexia. 2. Mouth care should be provided before and after meals but not with alcohol-based mouth wash and lemon-glycerin swabs, which can decrease the appetite. 3. The UAP assisting the client with meals should increase interaction to improve the client's appetite and make it an enjoyable occasion. 4. Often the parents are the cause of the client's stress and anxiety, which may have led to the client's anorexia; therefore, the parents should not be asked to stay with the client.

The client is diagnosed with an acute exacerbation of IBD. Which priority intervention should the nurse implement? 1. Weigh the client daily and document in the client's chart. 2. Teach coping strategies such as dietary modifications. 3. Record the frequency, amount, and color of stools. 4. Monitor the client's oral fluid intake every shift.

ANSWER: 3. 1. Weighing the client daily will help identify if the client is experiencing malnutrition, but it is not the priority intervention during an acute exacerbation. 2. Coping strategies help develop healthy ways to deal with this chronic disease, which has remissions and exacerbations, but it is not the priority intervention. 3. The severity of the diarrhea helps determine the need for fluid replacement. The liquid stool should be measured as part of the total output. 4. The client will be NPO when there is an acute exacerbation of IBD to allow the bowel to rest. TEST-TAKING HINT: The test taker can apply Maslow's hierarchy of needs and select the option addressing a physiological need.

The nurse is teaching the American Diabetes Association diet to a client diagnosed with diabetes mellitus type 2. Which should the nurse teach the client? 1. Instruct the client to weigh all food before cooking it. 2. Teach the client to eat only carbohydrates if the blood glucose is low. 3. Demonstrate how to determine the amount of carbohydrates being eaten. 4. Explain that proteins should be 75% of the recommended diet.

ANSWER: 3. 1. Weighing the food is no longer recommended but if the client weighs the food then it should be weighed after cooking because cooking changes the weight of food. 2. The client should eat some carbohydrates but also protein foods to maintain the blood glucose levels. 3. The client can estimate the amount of carbohydrates being consumed by using the fist as a guideline. The fist is always with the client and is an unobtrusive method of determining the amount of food and calories being ingested. 4. Seventy-five percent of the diet being protein places a great burden on the kidneys and can result in acidosis. This is unhealthy. TEST-TAKING HINT: The nurse must be able to provide basic teaching for diets.

The male client had abdominal surgery and the nurse suspects the client has peritonitis. Which assessment data support the diagnosis of peritonitis? 1. Absent bowel sounds and potassium level of 3.9 mEq/L. 2. Abdominal cramping and hemoglobin of 14 g/dL. 3. Profuse diarrhea and stool specimen shows Campylobacter. 4. Hard, rigid abdomen and white blood cell count 22,000/mm3

ANSWER: 4. 1. Absent bowel sounds indicate a paralytic ileus, not peritonitis, and the potassium level is within normal limits (3.5 to 5.5 mEq/L). 2. Abdominal cramping would not make the nurse suspect peritonitis, and the hemoglobin is normal (13 to 17 g/dL). 3. Campylobacter is a cause of profuse diarrhea, but it does not support a diagnosis of peritonitis. 4. A hard, rigid abdomen indicates an inflamed peritoneum (abdominal wall cavity) resulting from an infection, which results in an elevated WBC level. TEST-TAKING HINT: The -itis of peritonitis means inflammation, and if the test taker has no idea what the answer is, an elevated WBC count should provide a key to selecting option "4" as the correct answer

Which data indicate to the nurse the client who is one (1) day postoperative right total hip replacement is progressing as expected? 1. Urine output was 160 mL in the past eight (8) hours. 2. Paralysis and paresthesia of the right leg. 3. T 99.0˚F, P 98, R 20, and BP 100/60. 4. Lungs are clear bilaterally in all lobes

ANSWER: 4. 1. Adequate urine output should be 30 mL/hr or at least 240 mL in an eight (8)-hour period. 2. Paralysis (inability to move) and paresthesia (numbness and tingling) indicate neurovascular compromise to the right leg, which indicates a complication and is not an expected outcome. 3. The client's temperature and pulse are slightly elevated and the BP is low, which does not indicate effective nursing care. 4. Lung sounds which are clear bilaterally in all lobes indicate the client has adequate gas exchange, which prevents postoperative complications and indicates effective nursing care. TEST-TAKING HINT: If the test taker does not know the answer, then applying the testing rule that airway is priority would cause the test taker to select option "4" as the correct answer.

The client is four (4) hours postoperative open cholecystectomy. Which data warrant immediate intervention by the nurse? 1. Absent bowel sounds in all four (4) quadrants. 2. The T-tube has 60 mL of green drainage. 3. Urine output of 100 mL in the past three (3) hours. 4. Refusal to turn, deep breathe, and cough

ANSWER: 4. 1. After abdominal surgery, it is not uncommon for bowel sounds to be absent. 2. This is a normal amount and color of drainage. 3. The minimum urine output is 30 mL/hr. 4. Refusing to turn, deep breathe, and cough places the client at risk for pneumonia. This client needs immediate intervention to prevent complications. TEST-TAKING HINT: The test taker should recognize normal data such as the normal urine output and normal data for postoperative clients. The test taker should apply basic concepts when answering questions. Normal or expected outcomes do not require action.

Which problem is highest priority for the nurse to identify in the client who had an open cholecystectomy surgery? 1. Alteration in nutrition. 2. Alteration in skin integrity. 3. Alteration in urinary pattern. 4. Alteration in comfort.

ANSWER: 4. 1. Alteration in nutrition may be an appropriate client problem, but it is not priority. 2. Alteration in skin integrity may be an appropriate client problem but is not priority. 3. Alteration in urinary elimination may be an appropriate client problem but is not priority. 4. Acute pain management is the highest priority client problem after surgery because pain may indicate a life-threatening problem. TEST-TAKING HINT: When a question asks for the highest priority problem, the test taker should look for a life-threatening complication. Pain may be expected, but it may indicate a complication.

The client diagnosed with IBD is prescribed sulfasalazine (Asulfidine), a sulfonamide antibiotic. Which statement best describes the rationale for administering this medication? 1. It is administered rectally to help decrease colon inflammation. 2. This medication slows gastrointestinal (GI) motility and reduces diarrhea. 3. This medication kills the bacteria causing the exacerbation. 4. It acts topically on the colon mucosa to decrease inflammation.

ANSWER: 4. 1. Asulfidine cannot be administered rectally. Corticosteroids may be administered by enema for the local effect of decreasing inflammation while minimizing the systemic effects. 2. Antidiarrheal agents slow the gastrointestinal motility and reduce diarrhea. 3. IBD is not caused by bacteria. 4. Asulfidine is poorly absorbed from the gastrointestinal tract and acts topically on the colonic mucosa to inhibit the inflammatory process. TEST-TAKING HINT: If the test taker doesn't know the answer, then the test taker could eliminate options "2" and "3" because they do not contain the word "inflammation"; IBD is inflammatory bowel disease.

The nurse has administered an antibiotic, a proton pump inhibitor, and Pepto-Bismol for peptic ulcer disease secondary to H. pylori. Which data would indicate to the nurse the medications are effective? 1. A decrease in alcohol intake. 2. Maintaining a bland diet. 3. A return to previous activities. 4. A decrease in gastric distress.

ANSWER: 4. 1. Decreasing the alcohol intake indicates the client is making some lifestyle changes. 2. The client with peptic ulcer disease (PUD) is prescribed a regular diet, but the type of diet does not determine if the medication is effective. 3. The return to previous activities indicates the client has not adapted to the lifestyle changes and has returned to the previous behaviors which precipitated the peptic ulcer disease. 4. Antibiotics, proton pump inhibitors, and Pepto-Bismol are administered to decrease the irritation of the ulcerative area and cure the ulcer. A decrease in gastric distress indicates the medication is effective. TEST-TAKING HINT: To determine the effectiveness of a medication, the test taker must know the scientific rationale for administering the medication. Peptic ulcer disease causes gastric distress. If gastric distress is relieved, then the medication is effective.

The school nurse is discussing methods to prevent an outbreak of hepatitis A with a group of high school teachers. Which action is the most important to teach the high school teachers? 1. Do not allow students to eat or drink after each other. 2. Drink bottled water as much as possible. 3. Encourage protected sexual activity. 4. Sing the happy birthday song while washing hands.

ANSWER: 4. 1. Eating after each other should be discouraged, but it is not the most important intervention. 2. Only bottled water should be consumed in third world countries, but this precaution is not necessary in American high schools. 3. Hepatitis B and C, not hepatitis A, are transmitted by sexual activity. 4. Hepatitis A is transmitted via the fecal- oral route. Good hand washing helps to prevent its spread. Singing the happy birthday song takes approximately 30 seconds, which is how long an individual should wash his or her hands. TEST-TAKING HINT: The test taker must realize good hand washing is the most important action in preventing transmission of hepatitis A virus. Often, the test taker will not select the answer option that seems too easy—but remember, do not overlook the obvious

The nurse is preparing a client for surgery who will be receiving general anesthesia. Which medication should the nurse question administering? 1. Metoprolol PO. 2. Cefazolin sodium IVPB. 3. EMLA cream topical. 4. Dabigatran etexilate.

ANSWER: 4. 1. Even though this is an oral medication and the client is NPO, beta blocker medications are not held for surgery. They are administered with a sip of water. Beta blockers can cause rebound cardiac dysrhythmias if not administered as routinely taken. Beta blocker medications should be tapered off to avoid cardiac issues. The nurse would not question this medication. 2. An antibiotic medication would be expected to be administered prophylactically to prevent infections postoperatively. This is a cephalosporin, not penicillin. The nurse would not question this medication. 3. Some facilities apply EMLA cream prior to initiating the IV to decrease pain during the procedure. The nurse would not question this medication. 4. Dabigatran is an anticoagulant; the nurse would question this medication. The medication should have stopped five (5) to seven (7) days preoperatively. The nurse would question this medication and the surgery may have to be postponed because the client is receiving an anticoagulant. TEST-TAKING HINT: Because options "1" and "4" are oral medications and clients who are to receive general anesthesia are NPO, the test taker would choose from these options. The nurse must know which medications have specific requirements, such as steroids and beta blockers are tapered off.

The client received naloxone (Narcan), an opioid antagonist, in the postanesthesia care unit. Which nursing intervention should the nurse include in the care plan? 1. Measure the client's intake and output hourly. 2. Administer sleep medications at night. 3. Encourage the client to verbalize feelings. 4. Monitor respirations every 15 to 30 minutes.

ANSWER: 4. 1. Narcan does not alter the urinary elimination; therefore, this is not an appropriate intervention for this client. 2. Anesthesia may alter sleep patterns, but this nursing intervention does not take into account the need for Narcan to be administered to the client. 3. This nursing intervention does not address the use of Narcan. 4. Narcan is given to reverse respiratory depression from opioid analgesic medications and has a short half-life. The client may experience a rebound respiratory depression in 15 to 20 minutes, so this nursing intervention of monitoring respirations every 15 to 30 minutes is appropriate.

The nurse is assessing a client complaining of abdominal pain. Which data support the diagnosis of a bowel obstruction? 1. Steady, aching pain in one specific area. 2. Sharp back pain radiating to the flank. 3. Sharp pain increases with deep breaths. 4. Intermittent colicky pain near the umbilicus.

ANSWER: 4. 1. Steady, aching pain is associated with a peritoneal inflammation, which may be secondary to a ruptured spleen or perforated ulcer or other abdominal organ. 2. Sharp pain in the back and flank indicates kidney involvement. 3. Sharp pain increasing with deep breaths indicates muscular involvement. 4. Intermittent and colicky pain located near the umbilicus is indicative of a small bowel obstruction; lumbar pain is indicative of colon involvement.

The client is scheduled for a colostomy secondary to colon cancer, and the surgeon tells the client the stool will be a formed consistency. Where would the nurse teach the client the stoma will be located? 1. RLQ. 2. RUQ. 3. LUQ. 4. RLQ.

ANSWER: 4. 1. Stools are liquid in the ascending colon. 2. Stools are mushy in the right transverse colon. 3. The left transverse colon has semi-mushy stool. 4. The sigmoid colon is located in the left lower quadrant, and the client expels solid feces.

The nurse and an unlicensed assistive personnel (UAP) are caring for clients on a surgery unit. Which task would be most appropriate to delegate to the UAP? 1. Explain to the client how to cough and deep breathe. 2. Discuss preoperative plans with the client and family. 3. Determine the ability of the caregivers to provide postoperative care. 4. Assist the client to take a povidone-iodine (Betadine) shower.

ANSWER: 4. 1. Teaching cannot be delegated. 2. Discussing the preoperative plans is part of the planning process and cannot be delegated. 3. Evaluation cannot be delegated to the UAP. 4. The UAP can assist a stable client to take a shower whether or not it is with Betadine.

The charge nurse is making shift assignments. Which postoperative client should be assigned to the most experienced nurse? 1. The 4-year-old client who had a tonsillectomy and is able to swallow fluids. 2. The 74-year-old client with a repair of the left hip who is unable to ambulate. 3. The 24-year-old client who had an uncomplicated appendectomy the previous day. 4. The 80-year-old client with small bowel obstruction and congestive heart failure.

ANSWER: 4. 1. The client appears stable; pediatric clients can become unstable quickly, but the most experienced nurse would not need to care for this client. 2. A client with a fractured hip will be ambulated by the physical therapist and this client is stable, so the most experienced nurse does not need to care for this client. 3. A young client who had an appendectomy would require routine postoperative care. 4. An older client with a chronic disease would be a complicated case, requiring the care of a more experienced nurse. TEST-TAKING HINT: When questions ask for assignments for the most experienced nurse, the test taker should realize clients whose condition can change quickly, such as elderly clients who have a complicated condition, should be assigned to the most experienced nurse.

The female client is more than 10% over ideal body weight. Which nursing intervention should the nurse implement first? 1. Ask the client why she is eating too much. 2. Refer the client to a gymnasium for exercise. 3. Have the client set a realistic weight loss goal. 4. Determine the client's eating patterns.

ANSWER: 4. 1. The client does not owe the nurse an explanation. 2. If the HCP determines it is safe for the client to exercise, a gymnasium might be recommended, but walking is the best exercise, and this can be done in the neighborhood or at an enclosed shopping mall. 3. The client should set realistic weight loss goals. A realistic weight loss goal is one (1) to one and one-half (11/2) pounds per week, but this should be done after assessing the client. 4. Determining the client's eating patterns and what triggers the client to eat—stress or boredom, for example—and where and when the client consumes most of the calories—snacking in front of the TV at night, for example—is needed to assist the client to change eating behaviors. TEST-TAKING HINT: This question is an example of using the nursing process to arrive at the correct answer. Assessing the client has priority

The client with a diagnosis of rule-out colon cancer is two (2) hours post-sigmoidoscopy procedure. Which assessment data warrant immediate intervention by the nurse? 1. The client has hyperactive bowel sounds. 2. The client is eating a hamburger the family brought. 3. The client is sleepy and wants to sleep. 4. The client's BP is 96/60 and apical pulse is 108.

ANSWER: 4. 1. The client has been NPO and had laxatives; therefore, hyperactive bowel sounds do not warrant immediate intervention. 2. The client is able to eat after the procedure, so this does not warrant immediate intervention. 3. The client received sedation during the procedure and may have been up during the night having bowel movements, resulting in the client being exhausted and sleepy. 4. These are signs/symptoms of hypovolemic shock requiring immediate intervention by the nurse.

The client developed a paralytic ileus after abdominal surgery. Which intervention should the nurse include in the plan of care? 1. Administer a laxative of choice. 2. Encourage the client to increase oral fluids. 3. Encourage the client to take deep breaths. 4. Maintain a patent nasogastric tube.

ANSWER: 4. 1. The client is NPO; therefore, no medication would be administered. 2. The client is NPO so no food or fluids are allowed. 3. Deep breathing will help prevent pulmonary complications but does not address the client's paralytic ileus. 4. A paralytic ileus is the absence of peristalsis; therefore, the bowel will be unable to process any oral intake. A nasogastric tube is inserted to decompress the bowel until surgical intervention or until bowel sounds return spontaneously. TEST-TAKING HINT: If the test taker realizes the stem of the question says part of the gastrointestinal system, the ileus, is paralyzed, the test taker should know allowing the client to take anything by mouth would be an inappropriate action, so options "1" and "2" could be eliminated. Deep breathing addresses the respiratory system, not the gastrointestinal system, so option "3" could also be eliminated.

The nurse is preparing a client diagnosed with GERD for discharge following an esophagogastroduodenoscopy (EGD). Which statement indicates the client understands the discharge instructions? 1. "I should not eat for at least one (1) day following this procedure." 2. "I can lie down whenever I want after a meal. It won't make a difference." 3. "The stomach contents won't bother my esophagus but will make me nauseous." 4. "I should avoid orange juice and eating tomatoes until my esophagus heals."

ANSWER: 4. 1. The client is allowed to eat as soon as the gag reflex has returned. 2. An esophagogastroduodenoscopy is a diagnostic procedure, not a cure. Therefore, the client still has GERD and should be instructed to stay in an upright position for two (2) to three (3) hours after eating. 3. Stomach contents are acidic and will erode the esophageal lining. 4. Orange juice and tomatoes are acidic, and the client diagnosed with GERD should avoid acidic foods until the esophagus has had a chance to heal. TEST-TAKING HINT: This question assumes the test taker has knowledge of diagnostic procedures for specific disease processes.

The nurse is caring for a client diagnosed with GERD. Which nursing interventions should be implemented? 1. Place the client prone in bed and administer nonsteroidal anti-inflammatory medications. 2. Have the client remain upright at all times and walk for 30 minutes three (3) times a week. 3. Instruct the client to maintain a right lateral side-lying position and take antacids before meals. 4. Elevate the head of the bed (HOB) 30 degrees and discuss lifestyle modifications with the client

ANSWER: 4. 1. The client is encouraged to lie with the head of the bed elevated, but this is difficult to achieve when on the stomach. NSAIDs inhibit prostaglandin synthesis in the stomach, which places the client at risk for developing gastric ulcers. The client is already experiencing gastric acid difficulty. 2. The client will need to lie down at some time, and walking will not help with GERD. 3. If lying on the side, the left side-lying position, not the right side, will allow less chance of reflux into the esophagus. Antacids are taken one (1) and three (3) hours after a meal. 4. The head of the bed should be elevated to allow gravity to help in preventing reflux. Lifestyle modifications of losing weight, making dietary modifications, attempting smoking cessation, discontinuing the use of alcohol, and not stooping or bending at the waist all help to decrease reflux. TEST-TAKING HINT: Option "2" has an "all," which should alert the test taker to eliminate this option. If the test taker has no idea of the answer, lifestyle modifications are an educated guess for most chronic problems

The client is placed on percutaneous endoscopic gastrostomy (PEG) tube feedings. Which occurrence warrants immediate intervention by the nurse? 1. The client tolerates the feedings being infused at 50 mL/hr. 2. The client pulls the nasogastric feeding tube out. 3. The client complains of being thirsty. 4. The client has green, watery stool

ANSWER: 4. 1. The client is tolerating the feeding change, so there is no need for an immediate action. 2. The client has a PEG tube inserted into the stomach through the abdominal wall. The client does not have a nasogastric feeding tube. 3. Complaints of being thirsty should be addressed; the client may require some ice chips in the mouth or oral care, but this is not priority over assessing the client's ability to swallow. 4. This client needs to be cleaned immediately, the abdomen must be assessed, and a determination must be made regarding the type of feeding and the additives and medications being administered and skin damage occurring. This occurrence is priority. TEST-TAKING HINT: The test taker must identify assessment data indicating a complication secondary to the disease process when the stem asks which occurrence warrants immediate intervention.

Which statement by the client diagnosed with hepatitis warrants immediate intervention by the clinic nurse? 1. "I will not drink any type of beer or mixed drink." 2. "I will get adequate rest so I don't get exhausted." 3. "I had a big hearty breakfast this morning." 4. "I took some cough syrup for this nasty head cold."

ANSWER: 4. 1. The client should avoid alcohol to prevent further liver damage and promote healing. 2. Rest is needed for healing of the liver and to promote optimum immune function. 3. Clients with hepatitis need increased caloric intake, so this is a good statement. 4. The client needs to understand some types of cough syrup have alcohol and all alcohol must be avoided to prevent further injury to the liver; therefore, this statement requires intervention. TEST-TAKING HINT: If the test taker did not know the answer, the test taker could apply the rule of avoiding any over-the-counter (OTC) medications unless approved by a health-care provider

Which data should the nurse expect to assess in the client diagnosed with acute gastroenteritis? 1. Decreased gurgling sounds on auscultation of the abdominal wall. 2. A hard, firm, edematous abdomen on palpation. 3. Frequent, small melena-type liquid bowel movements. 4. Bowel assessment reveals loud, rushing bowel sounds.

ANSWER: 4. 1. The client would have increased gurgling sounds, revealing hyperactive bowel movements. 2. A hard, firm, edematous abdomen is not expected in a client with gastroenteritis; this would indicate a possible complication and require further assessment. 3. The client has increased liquid bowel movements (diarrhea) but should not have blood in the stool, which is the definition of melena. 4. Borborygmi, or loud, rushing bowel sounds, indicates increased peristalsis, which occurs in clients with diarrhea and is the primary clinical manifestation in a client diagnosed with acute gastroenteritis. TEST-TAKING HINT: The test taker should realize that, in an acute condition, the assessment data would be abnormal, which may help select the correct answer for some questions.

The client is two (2) hours post colonoscopy. Which assessment data warrant intermediate intervention by the nurse? 1. The client has a soft, nontender abdomen. 2. The client has a loose, watery stool. 3. The client has hyperactive bowel sounds. 4. The client's pulse is 104 and BP is 98/60.

ANSWER: 4. 1. The client's abdomen should be soft and nontender; therefore, this finding would not require immediate intervention. 2. The client had to clean the bowel prior to the colonoscopy; therefore, watery stool is expected. 3. The client was NPO and received bowel preparation prior to the colonoscopy; therefore, hyperactive bowel sounds might occur and do not warrant immediate intervention. 4. Bowel perforation is a potential complication of a colonoscopy. Therefore, signs of hypotension—decreased BP and increased pulse—warrant immediate intervention from the nurse. TEST-TAKING HINT: This is an "except" question. The test taker is being asked to select which data are abnormal for a procedure. The test taker should remember any invasive procedure could possibly lead to hemorrhaging, and signs of shock should always be considered a possible correct answer

The nurse is preparing a client for surgery. Which intervention should the nurse implement first? 1. Check the permit for the spouse's signature. 2. Take and document intake and output. 3. Administer the "on call" sedative. 4. Complete the preoperative checklist.

ANSWER: 4. 1. The client's signature, not the spouse's, should be on the surgical permit. 2. This would be important information if abnormal, but it is not the first intervention. 3. "On call" sedatives should be administered after the surgical checklist is completed. 4. Completing the preoperative checklist has the highest priority to ensure all details are completed without omissions. TEST-TAKING HINT: A client should never be sedated until the permit has been verified and all legal issues are settled. The test taker should not read into a question by inserting facts not in the stem. For example, the test taker may think option "1" could be a correct answer if the client is confused, but the stem does not include this information

The nurse writes a problem "low self-esteem" for a 16-year-old client diagnosed with anorexia. Which client goal should be included in the plan of care? 1. The client will spend one (1) hour a day with the parents. 2. The client eats 50% of the meals provided. 3. Dietary will provide high-protein milk shakes (tid). 4. The client will verbalize one positive attribute.

ANSWER: 4. 1. The goal is written in terms of client behavior; this option is a nursing intervention, not a client goal. 2. Eating 50% of meals provided does not address low self-esteem. 3. High-protein shakes are a dietary intervention. 4. The problem of "low self-esteem" requires the client to verbalize psychosocial feelings. Identifying one positive attribute is an appropriate goal. TEST-TAKING HINT: The test taker could eliminate distracter "3" as a health-care discipline intervention. Psychosocial problems require goals addressing a nonphysiological need.

Which task would be most appropriate for the nurse to delegate to the unlicensed assistive personnel (UAP)? 1. Complete the preoperative checklist. 2. Assess the client's preoperative vital signs. 3. Teach the client about coughing and deep breathing. 4. Assist the client to remove clothing and jewelry.

ANSWER: 4. 1. The nurse should complete this form because it requires analysis, which cannot be delegated to the UAP. 2. Nurses cannot delegate assessment. 3. The nurse cannot delegate teaching to a UAP. 4. The UAP can remove clothing and jewelry. TEST-TAKING HINT: The nurse should consider the knowledge and training of the person receiving the assignments. The nurse should never delegate assessment, teaching, administering medications, evaluation, or care of an unstable client to a UAP.

The nurse is preparing the client for a fiberoptic colonoscopy for colon polyps. Which task can be delegated to the unlicensed assistive personnel (UAP)? 1. Administer the polyethylene glycol electrolyte lavage solution. 2. Explain to the client why this morning's breakfast is withheld. 3. Start an intravenous site with 0.9% normal saline fluid. 4. Administer a cleansing enema until the return is clear.

ANSWER: 4. 1. The polyethylene glycol electrolyte lavage solution is a medication and cannot be delegated. 2. Teaching is the responsibility of the nurse and cannot be delegated. 3. Starting an intravenous site and managing the delivery of the fluid is the responsibility of the nurse and cannot be delegated. 4. The administration of enemas can be delegated to the unlicensed assistive personnel (UAP)

The nurse is assessing the integumentary system of the client diagnosed with anorexia nervosa. Which finding supports the diagnosis? 1. Preoccupation with calories. 2. Thick body hair. 3. Sore tongue. 4. Dry, brittle hair

ANSWER: 4. 1. The preoccupation with food, calories, and preparing meals are psychosocial behaviors suggesting the client has an eating disorder. 2. Clients who have anorexia nervosa have thin, fine body hair. 3. Iron-deficiency anemia causes clients to experience a sore tongue. 4. Thin, brittle hair occurs in clients with anorexia.

Which assessment data supports the client's diagnosis of gastric ulcer to the nurse? 1. Presence of blood in the client's stool for the past month. 2. Reports of a burning sensation moving like a wave. 3. Sharp pain in the upper abdomen after eating a heavy meal. 4. Complaints of epigastric pain 30 to 60 minutes after ingesting food.

ANSWER: 4. 1. The presence of blood does not specifically indicate diagnosis of an ulcer. The client could have hemorrhoids or cancer resulting in the presence of blood. 2. A wavelike burning sensation is a symptom of gastroesophageal reflux. 3. Sharp pain in the upper abdomen after eating a heavy meal is a symptom of gallbladder disease. 4. In a client diagnosed with a gastric ulcer, pain usually occurs 30 to 60 minutes after eating but not at night. In contrast, a client with a duodenal ulcer has pain during the night often relieved by eating food. Pain occurs one (1) to three (3) hours after meals. TEST-TAKING HINT: This question asks the test taker to identify assessment data specific to the disease process. Many diseases have similar symptoms, but the timing of symptoms or their location may help rule out some diseases and provide the health-care provider with a key to diagnose a specific disease—in this case, peptic ulcer disease. Nurses are usually the major source for information to the health-care team.

The client who has had an abdominal perineal resection is being discharged. Which discharge information should the nurse teach? 1. The stoma should be a white, blue, or purple color. 2. Limit ambulation to prevent the pouch from coming off. 3. Take pain medication when the pain level is at an "8." 4. Empty the pouch when it is one-third to one-half full.

ANSWER: 4. 1. The stoma should be light to a medium pink, the color of the intestines. A blue or purple color indicates a lack of circulation to the stoma and is a medical emergency 2. The stoma should be pouched securely for the client to be able to participate in normal daily activities. The client should be encouraged to ambulate to aid in recovery. 3. Pain medication should be taken before the pain level reaches a "5." Delaying taking medication will delay the onset of pain relief and the client will not receive full benefit from the medication. 4. The pouch should be emptied when it is one-third to one-half full to prevent the contents from becoming too heavy for the seal to hold and to prevent leakage from occurring. TEST-TAKING HINT: Normal mucosa is pink, not white, and clients are always encouraged to ambulate after surgery to prevent the complications related to immobility. Remember basic concepts when answering questions, especially about postoperative nursing care.

Which violation of surgical asepsis would require immediate intervention by the circulating nurse? 1. Surgical supplies were cleaned and sterilized prior to the case. 2. The circulating nurse is wearing a longsleeved sterile gown. 3. Masks covering the mouth and nose are being worn by the surgical team. 4. The scrub nurse setting up the sterile field is wearing artificial nails.

ANSWER: 4. 1. These are appropriate activities in a surgery department; therefore, no intervention is required. 2. This is required to maintain surgical asepsis. 3. This follows the principles of surgical asepsis. 4. According to the Centers for Disease Control and Prevention (CDC), the Association of Operating Room Nurses (AORN), and the Association for Practitioners in Infection Control, artificial nails harbor microorganisms, which increase the risk for infection. TEST-TAKING HINT: The adjective "artificial" in option "4" and the word "violation" in the stem should cause the test taker to select option "4" as the correct answer if the test taker had no idea which distracter to select.

The client diagnosed with diverticulitis is complaining of severe pain in the left lower quadrant and has an oral temperature of 100.6°F. Which intervention should the nurse implement first? 1. Notify the health-care provider. 2. Document the findings in the chart. 3. Administer an oral antipyretic. 4. Assess the client's abdomen.

ANSWER: 4. 1. These are classic signs/symptoms of diverticulitis; therefore, the HCP does not need to be notified. 2. These are normal findings for a client diagnosed with diverticulitis, but on admission the nurse should assess the client and document the findings in the client's chart. 3. The nurse should not administer any food or medications. 4. The nurse should assess the client to determine if the abdomen is soft and nontender. A rigid tender abdomen may indicate peritonitis. TEST-TAKING HINT: The test taker must remember to apply the nursing process when answering test questions. Assessment is the first step in the nursing process. Although the signs/symptoms are normal and could be documented, the nurse should always assess.

The occupational health nurse has had five (5) clients come to the clinic complaining of abdominal cramping, nausea, and vomiting. Which information should the nurse teach the employees to decrease the spread of this condition? 1. Teach the employees to cough into the sleeve. 2. Teach the housekeepers to use an antibacterial soap. 3. Teach the coworkers to get a hepatitis vaccine. 4. Teach the employees to wash their hands frequently.

ANSWER: 4. 1. This is a gastrointestinal complaint, not a respiratory virus. 2. Antibacterial soap will not affect a virus. A virus is not a bacterium. 3. A hepatitis vaccine prevents hepatitis but this is a gastrointestinal viral illness. 4. Hand washing will prevent the spread of the virus and decrease the risk for the employees. TEST-TAKING HINT: The test taker should remember basic infection control standards.

The nurse is caring for a client scheduled for total hip replacement. Which behavior indicates the need for further preoperative teaching? 1. The client uses the diaphragm and abdominal muscles to inhale through the nose and exhale through the mouth. 2. The client demonstrates dorsiflexion of the feet, flexing of the toes, and moves the feet in a circular motion. 3. The client uses the incentive spirometer and inhales slowly and deeply so the piston rises to the preset volume. 4. The client gets out of bed by lifting straight upright from the waist and then swings both legs along the side of the bed.

ANSWER: 4. 1. This is the correct way to perform deepbreathing exercises; therefore, no further teaching is needed. 2. This is the correct way to perform rangeof-motion exercises; therefore, no further teaching is needed. 3. This is the way to use a volume incentive spirometer; therefore, no further teaching is needed. 4. The correct way to get out of bed postoperatively is to roll onto the side, grasp the side rail to maneuver to the side, and then push up with one hand while swinging the legs over the side. The client needs further teaching. TEST-TAKING HINT: This is an "except" question. Therefore, the test taker must select an option which shows the client does not understand the teaching. Sometimes flipping the question and asking which behavior indicates the client understands the teaching will help in answering this type of question.

Which intervention should the nurse include when discussing ways to help prevent potential episodes of gastroenteritis from Clostridium botulism? 1. Make sure all hamburger meat is well cooked. 2. Ensure all dairy products are refrigerated. 3. Discuss why campers should drink only bottled water. 4. Discard damaged canned goods

ANSWER: 4. 1. Well-cooked meat will help prevent gastroenteritis secondary to staphylococcal food poisoning. 2. Refrigerating dairy products will help prevent gastroenteritis secondary to eating foods kept at room temperature, causing staphylococcal food poisoning. 3. Drinking bottled water will help prevent gastroenteritis secondary to Escherichia coli found in contaminated water. 4. Any discolored food, food from a damaged can or jar, or food from a can or jar not having a tight seal should be destroyed without tasting or touching it. TEST-TAKING HINT: The test taker should be careful with words such as "all," "only," and "never"; few absolutes exist in the healthcare field.

The circulating nurse and the scrub technician find a discrepancy in the sponge count. Which action should the circulating nurse take first? 1. Notify the client's surgeon. 2. Complete an occurrence report. 3. Contact the surgical manager. 4. Recount all sponges

ANSWER: 4. 1. When discrepancies occur in the count, it is usually a simple mistake discovered with a recount. The surgeon will be notified if the count is wrong after a recount. 2. If an error is found to have been made, an occurrence report will be completed, but it is not the first intervention. 3. This would be done if a correct count is not maintained, but it is not the first intervention. 4. A recount of sponges may lead to the discovery of the cause of the presumed error. Usually it is just a miscount or a result of a sponge being placed in a location other than the sterile field, such as the floor or a lower shelf. TEST-TAKING HINT: When the test taker has no idea of the correct answer, the test taker should apply the nursing process and choose the option which addresses assessment because it is the first step in the nursing process.

The nurse is preparing to administer 250 mL of intravenous antibiotic to the client. The medication must infuse in one (1) hour. An intravenous pump is not available and the nurse must administer the medication via gravity with IV tubing at 10 gtts/min. At what rate should the nurse infuse the medication? _________

ANSWER: 42 gtts/min. The nurse must use the formula: amount to be infused × drops per minute minutes for infusion 250 mL × 10 gtts 60 minutes or, 2,500 ÷ 60 minutes = 41.66 gtts/min, which should be rounded up to 42 gtts/min. TEST-TAKING HINT: The test taker must know how to calculate dosage and calculation questions. Remember to use the drop-down calculator if needed; the test taker can ask for an erase slate during state board examinations.

The nurse is preparing to hang a new bag of total parenteral nutrition for a client with an abdominal perineal resection. The bag has 1,500 mL of 50% dextrose, 10 mL of trace elements, 20 mL of multivitamins, 20 mL of potassium chloride, and 500 mL of lipids. The bag is to infuse over the next 24 hours. At what rate should the nurse set the pump? _________

ANSWER: 85 mL/hr. First determine the total amount to be infused over 24 hours: 1500 + 500 + 20 + 20 = 2,040 mL over 24 hours Then, determine the rate per hour: 2,040 ÷ 24 = 85 mL/hr TEST-TAKING HINT: Check and recheck calculations. Division should be carried out to the second or third decimal place before rounding

The client diagnosed with appendicitis has undergone an appendectomy. At two (2) hours postoperative, the nurse takes the vital signs and notes T 102.6˚F, P 132, R 26, and BP 92/46. Which interventions should the nurse implement? List in order of priority. 1. Increase the IV rate. 2. Notify the health-care provider. 3. Elevate the foot of the bed. 4. Check the abdominal dressing. 5. Determine if the IV antibiotics have been administered.

ANSWER: In order of priority: 1, 3, 4, 5, 2. 1. The nurse should increase the IV rate to maintain the circulatory system function until further orders can be obtained. 3. The foot of the bed should be elevated to help treat shock, the symptoms of which include elevated pulse and decreased blood pressure. Those signs and an elevated temperature indicate an infection may be present and the client could be developing septicemia. 4. The dressing should be assessed to determine if bleeding is occurring. 5. The nurse should administer any IV antibiotics ordered after addressing hypovolemia. The nurse will need this information when reporting to the health-care provider. 2. The health-care provider should be notified when the nurse has the needed information.

The client is admitted to the emergency department complaining of acute epigastric pain and reports vomiting a large amount of brightred blood at home. Which interventions should the nurse implement? List in order of priority. 1. Assess the client's vital signs. 2. Insert a nasogastric tube. 3. Begin iced saline lavage. 4. Start an IV with an 18-gauge needle. 5. Type and crossmatch for a blood transfusion.

ANSWER: In order of priority: 1, 4, 5, 2, 3. 1. The nurse should assess the vital signs to determine if the client is in hypovolemic shock. The stem of the question does not provide information indicating the client is hypovolemic. The client's perception of a large amount of blood may differ from the nurse's assessment. 4. The nurse should start the IV line to replace fluid volume. 5. While the nurse is starting the IV, a blood sample for typing and crossmatching should be obtained and sent to the laboratory. 2. An N/G tube should be inserted so direct iced saline can be instilled to cause constriction, which will decrease the bleeding. 3. The iced saline lavage will help decrease bleeding.

_____: The surgical removal of the sigmoid colon, rectum, and anus through combined abdominal and perineal incisions. This resection is performed when rectal tumors are present.

Abdominoperineal (AP) resection

_____: Inflammation of the gallbladder occurring in the absence of gallstones; typically associated with biliary stasis caused by any condition that affects the regular filling or emptying of the gallbladder.

Acalculous cholecystitis

_____: Inflammation of a salivary gland; can be caused by infectious agents, irradiation, or immunologic disorders.

Acute sialadenitis

_____: Surgical reattachment. Also a general term meaning "a connection."

Anastomosis

_____: An induced state of partial or total loss of sensation with or without loss of consciousness.

Anesthesia

_____: A localized induration and fluctuance that is caused by inflammation of the soft tissue near the rectum or anus and is most often the result of obstruction of the ducts of glands in the anorectal region by feces, foreign bodies, or trauma.

Anorectal abscess

The nurse is instructing a client on measures to maintain effective oral health. Which measures does the nurse include in the client's teaching plan? (Select all that apply). A) Regular dental checkups. B) Use of mouthwashes containing alcohol. C) Ensuring that dentures are slightly loose-fitting. D) Managing stress as much as possibleE) Eating a balanced diet

Answer: A, D, E. Regular dental checkups are important so potential problems can be prevented or attended to promptly. Stress suppresses the immune system, which can increase the client's risk for infections such as Candida albicans. Eating a balanced diet can reduce the risk for dental caries and infections such as C. albicans or stomatitis.Mouthwashes that contain alcohol may cause inflammation and should be avoided. Dentures must be in good repair and need to fit properly

A nurse is providing dietary teaching for a client who has chronic pancreatitis. Which of the following food selections by the client indicates an understanding of the teaching? A) 1 cup sliced banana. B) 1 oz cheddar cheese. C) 8 oz whole milk. D) One slice of beef bologna.

Answer: A. A) Foods that are high in fat can cause diarrhea for clients who have pancreatitis. One cup of sliced banana, which contains 0.49 g of fat, is a low-fat food option. Clients who have pancreatitis should consume a high-protein and low-fat diet with an adequate amount of carbohydrates and calories. B) The nurse should inform the client that high-fat foods, such as cheddar cheese, should be avoided because foods that are high in fat can cause diarrhea. An ounce of cheddar cheese contains 9.33 g of total fat. C) The nurse should inform the client that high-fat foods, such as whole milk, should be avoided because foods that are high in fat can cause diarrhea. A cup of whole milk contains 7.93 g of fat. The nurse should recommend fat-free milk. D) The nurse should inform the client that high-fat foods, such as bologna, should be avoided because foods that are high in fat can cause diarrhea. One slice of beef bologna contains 7.84 g of fat.

A nurse is assessing a client immediately following a paracentesis for the treatment of ascities. Which of the following findings indicates the procedure was effective? A) Decreased shortness of breath. B) Increased heart rate C) Equal pre- and postprocedure weights D) Presence of a fluid wave

Answer: A. A) Increased abdominal fluid can limit the expansion of the diaphragm and prevent the client from taking a deep breath. Once excess peritoneal fluid is removed, the diaphragm will expand more freely. The nurse should identify this finding as an indicator the procedure was effective. B) The nurse should identify an increased heart rate as an indication of hypovolemia, which is a potential complication of a paracentesis. This finding does not indicate the procedure was effective. C) The nurse should expect the client's postprocedure weight to be less than the preprocedure weight due to the withdrawal of fluid from the peritoneal cavity. This finding does not indicate the procedure was effective. D) The nurse should identify the presence of a fluid wave as an indication of ascites. This finding does not indicate the procedure was effective.

A nurse is caring for a client who is 2 days postoperative following a cholecystectomy. The client has been vomiting for the past 24 hr and reports a pain level of 8 on a scale from 0 to 10. The nurse notes a hard, distended abdomen and absent bowel sounds. After conferring with the provider, which of the following actions should the nurse take first? A) Insert an NG tube. B) Draw the client's blood for electrolytes. C) Initiate intake and output. D) Administer pain medication.

Answer: A. A) The greatest risk to the client is fluid and electrolyte imbalance as a result of accumulated fluid and gas in the gastrointestinal tract. The first action the nurse should take is to insert an NG tube to begin decompression of the bowel. B) Drawing the client's blood for electrolytes is an important action because electrolyte imbalance is a possible cause of paralytic ileus; however, there is another action the nurse should take first. C) Initiating intake and output is an important action because it provides information about the client's fluid balance; however, there is another action the nurse should take first. D) Administering pain medication is an important action to promote comfort; however, there is another action the nurse should take first.

A nurse is providing discharge teaching for a client who has peptic ulcer disease and a new prescription for famotidine. Which of the following statements by the client indicates an understanding of the teaching? A) "I should take this medication at bedtime." B) "I will monitor my blood glucose level regularly while taking this medication." C) "I will drink iced tea with my meals and snacks." D) "I should expect this medication to discolor my stools."

Answer: A. A) The nurse should instruct the client to take the medication at bedtime to inhibit the action of histamine at the H2-receptor site in the stomach. B) There is no indication that famotidine therapy has an adverse effect on blood glucose levels. Therefore, it is unnecessary for the client to monitor daily glucose levels. C) The nurse should instruct the client to avoid excessive intake of caffeinated beverages, such as coffee and tea. D) Famotidine can cause constipation, anorexia, and cramps. Bismuth, another medication prescribed for peptic ulcer disease, can turn the client's stools black.

A nurse is caring for a client who is preoperative and is asking multiple questions about the risks of the procedure. Which of the following actions should the nurse take? A) Ask the surgeon to speak to the client for clarification. B) Reassure the client that the procedure is necessary for recovery. C) Explain the risks and benefits of the surgery to the client. D) Notify the circulating nurse that the client has questions about the procedure.

Answer: A. A) The nurse should notify the surgeon that the client has questions about the procedure. It is the responsibility of the surgeon to explain the risks and benefits of the surgery. B) This response dismisses the client's concerns and is an example of false reassurance. C) It is not the nurse's responsibility to explain the risks and benefits of the surgery. The nurse should verify that the client signs the consent form prior to transfer to surgery. D) The nurse should verify that the client signs the consent form prior to transfer to surgery. The circulating nurse is not responsible for explaining the risks and benefits of the procedure to the client. This action is the responsibility of the surgeon.

The nurse is caring for a patient who is to be discharged after a bowel resection and the creation of a colostomy. Which patient statement demonstrates that additional instruction from the nurse is needed? A) "I can drive my car in about 2 weeks." B) "I need to avoid drinking carbonated sodas." C) "It may take 6 weeks to see the effects of some foods on my bowel patterns." D) "Stool softeners will help me avoid straining."

Answer: A. Additional instruction is needed from the nurse when the patient who is about to be discharged after a bowel resection and colostomy says, "I can drive my car in about 2 weeks." The patient who has had a bowel resection and colostomy would avoid driving for 4 to 6 weeks.The patient needs to avoid drinking sodas and other carbonated drinks because of the gas they produce. He or she may not be able to see the effects of certain foods on bowel patterns for several weeks. The patient must avoid straining at stool.

The nurse working during the day shift on the medical unit has just received report. Which patient does the nurse plan to assess first? A) Young adult with epigastric pain, hiccups, and abdominal distention after having a total gastrectomy B) Adult who had a subtotal gastrectomy and is experiencing dizziness and diaphoresis after each meal C) Middle-aged patient with gastric cancer who needs to receive omeprazole (Prilosec) before breakfast D) Older adult with advanced gastric cancer who is scheduled to receive combination chemotherapy

Answer: A. After receiving shift report, the nurse would first assess the post-op total gastrectomy young adult with epigastric pain, hiccups and abdominal pain. This patient is experiencing symptoms of acute gastric dilation, which can disrupt the suture line. The surgeon must be notified immediately because the nasogastric tube may need irrigation or repositioning.The patient who had a subtotal gastrectomy is not in a life-threatening situation and does not require immediate assessment. The patient with gastric cancer and the older adult with advanced gastric cancer are in stable condition and do not require immediate assessment.

An older female patient is diagnosed with gastric cancer. Which statement made by the patient's family demonstrates a correct understanding of the disorder? A) "This may be related to her recurring ulcer disease." B) "This cancer is probably curable with surgery." C) "Gastric cancer has a strong genetic component." D) "Thank goodness she won't have to undergo surgery."

Answer: A. Correct understanding of a patient's diagnosis of gastric cancer is indicated when they family states that the diagnosis could be related to the patient's ulcer disease. Infection with Helicobacter pylori is the largest risk factor for gastric cancer because it carries the cytotoxin-associated antigen A (CagA) gene. Patients with chronic ulcers are probably infected with this organism.Surgery is often not curative because most gastric cancers do not present with symptoms until late in the disease and have a high fatality rate. There is no strong genetic predisposition to gastric cancer. Surgery is part of the treatment.

While working in the outpatient procedure unit, the RN is assigned to these clients. Which client does the nurse assess first? A) A 51-year-old who recently had an endoscopic retrograde cholangiopancreatography (ERCP) B) A 54-year-old who is ready for discharge following a colonoscopy C) A 58-year-old who has just arrived for basal gastric secretion and gastric acid stimulation testing D) A 60-year-old with questions about an endoscopic ultrasound examination

Answer: A. ERCP requires conscious sedation, so the client needs immediate assessment of respiratory and cardiovascular status. The endoscopic procedure and nursing care for a client having an ERCP are similar to those for the EGD procedure, except that the endoscope is advanced farther into the duodenum and into the biliary tractA 54-year old client being discharged after a colonoscopy, a 58-year old client who is going to have a gastric acid test, and a 60-year old client with questions about an endoscopic ultrasound examination are not at risk for depressed respiratory status.

A patient diagnosed with ulcerative colitis (UC) is to be discharged on loperamide (Imodium) for symptomatic management of diarrhea. What does the nurse include in the teaching about this medication? A) "Be aware of the signs/symptoms of toxic megacolon that we discussed." B) "If diarrhea increases, you must let your primary health care provider know." C) "You must avoid pregnancy." D) "You will need to decrease your dose of sulfasalazine (Azulfidine)."

Answer: A. In teaching a UC patient discharged on loperamide, the nurse tells the patient to be aware of signs/symptoms of toxic megacolon that were discussed. Antidiarrheal drugs may precipitate colonic dilation and toxic megacolon. Toxic megacolon is characterized by an enlarged colon with fever, leukocytosis, and tachycardia.Loperamide will decrease diarrhea rather than increase it. Constipation is sometimes a problem. No contraindication for pregnancy is noted. Sulfasalazine therapy typically continues on a long-term basis.

A patient is scheduled for discharge after surgery for inflammatory bowel disease. The patient's spouse will be assisting home health services with the patient's care. What is most important for the home health nurse to assess in the patient and the spouse with regard to the patient's home care? A) Ability of the patient and spouse to perform incision care and dressing changes B) Effective coping mechanisms for the patient and spouse after the surgical experience C) Knowledge about the patient's requested pain medications D) Understanding of the importance of keeping scheduled follow-up appointments

Answer: A. It is most important for the home health nurse to assess the patient's and spouse's ability to carry out incision care and dressing changes. This assessment is essential to avoid further development of the infectious process, as well as infection of the surgical incision itself.Assessing coping mechanisms and knowledge of the patient's pain medication are important but are not the priority. Understanding the importance of scheduled follow-up appointments is important but is also not the priority.

A client who had been hospitalized with pancreatitis is being discharged with home health services. The client is severely weakened after this illness. Which nursing intervention is the highest priority in conserving the client's strength? A) Limiting the client's activities to one floor of the home B) Instructing the client to take an as-needed (PRN) sleeping medication at night C) Arranging for the client to have a nutritional consult to assess the client's diet D) Asking the health care provider for a request for PRN nasal oxygen

Answer: A. Limiting the client's activities to one floor of the home is the highest priority nursing intervention. This will prevent tiring the client unnecessarily with stair climbing.A PRN sleeping medication will not increase the client's strength level or conserve strength.Arranging for a nutritional consult or placing the client on PRN nasal oxygen will not necessarily result in an increase in the client's strength level or conserve strength. No information suggests that the client has any history of breathing difficulties.

When caring for a client with oral cancer who has developed stomatitis as a complication of radiation and chemotherapy, which action does the nurse delegate to the unlicensed assistive personnel (UAP)? A) Provide oral care using disposable foam swabs. B) Inspect the oral mucosa for evidence of oral candidiasis. C) Instruct the client on how to use nystatin (Mycostatin) oral rinses. D) Teach the client how to make appropriate dietary choices.

Answer: A. Providing oral care for a client with oral lesions is an appropriate assignment for a UAP.Assessments, client teaching, and assisting clients with oral problems in making appropriate dietary choices are the responsibilities of licensed nursing staff.

A client has an acute case of opioid depression and receives a dose of naloxone (Narcan). Which statement is true about this client? A) Supplemental pain reduction is needed. B) One dose is needed. C) This is an acute emergency. D) The client will be hostile.

Answer: A. Supplemental pain reduction is needed. The client has breakthrough pain after the opioid antagonist is given, so other interventions to promote comfort are needed.Several doses of naloxone may be needed because the drug has a short half-life. Opioid depression is a manageable situation, not an acute emergency. The client with opioid depression usually is not fully conscious.

The nurse is assessing an alert client who had abdominal surgery yesterday. What method provides the most accurate data about resumption of peristalsis in the client? A) Asking the client whether he or she has passed flatus (gas) within the previous 12 to 24 hours B) Auscultating bowel sounds in all abdominal quadrants C) Counting the number of bowel sounds in each abdominal quadrant over one minute. D) Observing the abdomen for symmetry and distention

Answer: A. The best and most reliable method for assessing the return of peristalsis following abdominal surgery is the client's report of passing flatus within the past 8 hours or stool within the past 12 hours.Although auscultation and counting the number of sounds was once a method of assessing for bowel activity, it is no longer considered the most effective method. Observing the abdomen is one method of examining a client's abdomen, but it is not a reliable way to assess for resumption of activity after surgery.

Who is the most likely person to administer blood products in an operating suite? A) Circulating nurse. B) Holding area nurse. C) Scrub nurse. D) Specialty nurse.

Answer: A. The circulating nurse is the most likely person to administer blood products to a client in the operating suite. Circulating nurses or "circulators" are registered nurses who coordinate, oversee, and are involved in the client's nursing care in the operating room.Holding area nurses manage the client's care before surgery; blood would not yet be needed at this point. Scrub nurses set up the sterile field, drape the client, and hand sterile supplies, sterile equipment, and instruments to the surgeon and the assistant. Specialty nurses may be in charge of a particular type of surgical specialty. They are responsible for nursing care specific to clients who need that type of surgery, such as assessing, maintaining, and recommending equipment, instruments, and supplies.

A patient with colorectal cancer had colostomy surgery performed yesterday. The patient is very anxious about caring for the colostomy and states that the primary health care provider's instructions "seem overwhelming." What does the nurse do first for this patient? A) Encourage the patient to look at and touch the colostomy stoma B) Instruct the patient about complete care of the colostomy C) Schedule a visit from a patient who has a colostomy and is successfully caring for it D) Suggest that the patient involve family members in the care of the colostomy

Answer: A. The first action the nurse does for the postoperative colostomy patient who is very anxious about caring for the colostomy is to encourage the patient to look at and touch the colostomy stoma. The initial intervention is to get the patient comfortable looking at and touching the stoma before providing instructions on its care.Instructing the patient about colostomy care will be much more effective after the patient's anxiety level has stabilized. Talking with someone who has gone through a similar experience may be helpful to the patient only after his or her anxiety level has stabilized. The patient has begun to express feelings regarding the colostomy and its care. It is too soon to involve others. The patient must get comfortable with this body image change before attempting to involve family members in colostomy care.

A patient has been discharged home after surgery for gastric cancer, and a case manager will follow up with the patient. To ensure a smooth transition from the hospital to the home setting, which information provided by the hospital nurse to the case manager is given the highest priority? A) Schedule of the patient's follow-up examinations and diagnostic testing B) Information on family members' progress in learning how to perform dressing changes C) Copy of the diet plan prepared for the patient by the hospital dietitian D) Detailed account of what occurred during the patient's surgical procedure

Answer: A. The highest priority information the hospital nurse would give to the home case manager is a schedule of the patient's follow-up exams and diagnostic testing. Because recurrence of gastric cancer is common, it is important for the patient to have follow-up examinations and x-rays so that a recurrence can be detected quickly.It may take family members a long time to become proficient at tasks such as dressing changes. Although the case manager must be aware of the diet, family members will likely be preparing the patient's daily diet, and they would be provided with this information. It is not necessary for the case manager to have details of the patient's surgical procedure unless a significant event occurred during the procedure.

The RN who usually works on the pediatric unit is floated to the GI medical-surgical unit. Which client is most appropriate for the charge nurse to assign to the float nurse? A) A 20-year-old with anorexia nervosa receiving total parenteral nutrition through a central venous line. B) A 35-year-old who had a laparoscopic gastroplasty yesterday and is now taking sips of clear liquids. C) A 60-year-old with gastric cancer receiving elemental feedings through a jejunostomy tube. D) A 65-year-old with morbid obesity who requires a preoperative bariatric surgery assessment.

Answer: A. The most appropriate client for the charge nurse to assign to the float nurse is the 20 year-old anorexia client. A pediatric nurse would be most familiar with the pathophysiology and collaborative treatment of this client.The client with a laparoscopic gastroplasty, the client with gastric cancer with a jejunostomy tube, and the client with morbid obesity requiring a preoperative bariatric surgery assessment all require care by a nurse with more familiarity with adult nutritional disorders and bariatric surgery.

The nurse admits an immunocompromised client who has contracted herpes simplex stomatitis. The nurse anticipates that the health care provider will request which medication? A) Acyclovir (Zovirax) B) Diphenhydramine (Benadryl) C) Nystatin (Mycostatin) D) Tetracycline syrup (Sumycin syrup)

Answer: A. The nurse anticipates that the health care provider will request acyclovir for the client. Acyclovir (Zovirax) is an antiviral agent that is prescribed for immunocompromised clients who contract herpes simplex stomatitis.Diphenhydramine is an antihistamine that is not indicated for treating this condition. Nystatin is indicated for treatment of fungal infection. Tetracycline syrup is indicated for treatment of recurrent aphthous ulcers (canker sores).

The nurse is caring for a patient diagnosed with esophageal cancer who is experiencing diarrhea after conventional esophageal surgery. The nurse anticipates that the primary health care provider will request which medication to manage diarrhea? A) Loperamide (Imodium) B) Mesalamine (Pentasa) C) Minocycline (Minocin) D) Pantoprazole (Protonix)

Answer: A. The nurse anticipates that the primary health care provider will order Loperamide to manage the diarrhea. Diarrhea is thought to be the result of vagotomy syndrome, which develops as a result of interruption of vagal fibers to the abdominal viscera during surgery. It can occur 20 minutes to 2 hours after eating and can be symptomatically managed with loperamide.Mesalamine is used to treat patients with mild to moderate ulcerative colitis. Minocycline is an antibiotic used for treatment of infection. Pantoprazole is used to treat gastroesophageal reflux disease.

A nurse is teaching a patient with Crohn's disease about managing the disease with the drug adalimumab (Humira). Which instruction does the nurse emphasize to the patient? A) "Avoid large crowds and anyone who is sick." B) "Do not take the medication if you are allergic to foods with fatty acids." C) "Expect difficulty with wound healing while you are taking this drug." D) "Monitor your blood pressure and report any significant decrease in it."

Answer: A. The nurse emphasizes that the patient taking adalimumab for Crohn's disease needs to avoid being around large crowds to prevent developing an infection. Adalimumab (Humira), a biologic response modifier (BRM), also known as a monoclonal antibody drug, has been approved for use in Crohn's disease when other drugs have been ineffective. BRMs are approved for refractory (not responsive to other therapies) cases. These drugs cause immunosuppression and should be used with caution. Patients must be taught to report any signs of a beginning infection, including a cold, and to also avoid others who are sick.The patient would not take the medication if he or she is allergic to certain proteins. Although immune suppression may occur to some degree, the patient would not experience difficulty with wound healing while taking adalimumab. Also, the patient would not experience a decrease in blood pressure from taking this drug.

A patient with a history of osteoarthritis has a 10-inch (25.5 cm) incision following a colon resection. The incision has become infected, and the wound requires extensive irrigation and packing. What aspect of the patient's care does the nurse make certain to discuss with the primary health care provider before the patient's discharge? A) Having a home health consultation for wound care B) Requesting an antianxiety medication C) Requesting pain medication for the patient's osteoarthritis D) Placing the patient in a skilled nursing facility for rehabilitation

Answer: A. The nurse makes sure to discuss an order for a home health consultation for wound care with the primary health care provider. Home health services are most appropriate for this patient because wound care will be extensive and the patient's mobility may be limited.No indication suggests that the patient is experiencing anxiety regarding postoperative care. Pain medication may be needed for the patient's osteoarthritis, but this is not the highest priority. A skilled nursing facility is not necessary if the patient can remain in his or her home with sufficient support services.

Which practice does the nurse include when teaching a client about proper oral care? A) Perform self-examination of the mouth every week, and report any unusual findings. B) Brush the teeth daily and floss as needed. C) Use drugs that reduce the flow of saliva unless lesions are present. D) Regularly rinse mouth with alcohol-based agent.

Answer: A. The nurse needs to teach the client that proper oral care involves self-examination of the mouth every week and reporting any unusual findings to the Health Care Provider.Clients need to brush teeth and floss every day. Clients would be instructed to avoid, if possible, drugs that can cause inflammation of the mouth or that can reduce the flow of saliva, and to avoid contact with agents that may cause inflammation of the mouth (such as alcohol based mouthwashes).

When assessing a patient for GERD, the nurse expects to find dyspepsia (heartburn), excessive salivation, flatulence which is common after eating, and regurgitation (backward flow of food and fluid into the throat).Blood-tinged sputum and excessive salivation are not symptoms of GERD. A) Place food at the back of the mouth as you eat. B) Do not be overly concerned with tongue or lip movements. C) Before swallowing, tilt the head back to straighten the esophagus. D) Do not attempt to reach food particles that are on the lips or around the mouth.

Answer: A. The nurse should instruct the patient to place food at the back of the mouth when eating. This will help the patient avoid aspiration of food. Food aspiration can cause airway obstruction, pneumonia, or both, especially in older adults.Both tongue movements and sealing of the lips should be monitored in this patient. The patient's head should be tilted forward in the chin-tuck position and not back. The patient needs to be able to reach food particles on her or his lips and around the mouth with the tongue.

A client is scheduled for a colonoscopy. What does the nurse tell the client to do before the procedure is performed? A) "Begin a clear liquid diet at least 24 hours before the test." B) "Do not eat or drink anything for 12 hours before the test." C) "Give yourself tap water enemas until the fluid returns are clear." D) "Be sure to take all currently prescribed medications prior to the procedure."

Answer: A. The nurse tells the client to be on a clear liquid diet for at least 24 hours to cleanse the bowel before a colonoscopy.The client must be NPO (except for water) 4 to 6 hours before a colonoscopy, not 12 hours. Also, the client needs to avoid aspirin, anticoagulants, and antiplatelet drugs for several days before the procedure. Diabetic clients need to check with their health care provider about drug therapy requirements on the day of the test because they are NPO. The client would not give him/herself a tap water enema. Clients must not take all currently prescribed medications without first checking with their doctor.

A patient has been diagnosed with mild gastroesophageal reflux disease (GERD) and asks the nurse about nonpharmacologic treatments to prevent symptoms. What does the nurse tell this patient? A) "Avoid caffeine-containing foods and beverages." B) "Eat three meals each day and avoid snacking between meals." C) "Peppermint lozenges help to reduce stomach upset." D) "Sleep on your left side with a pillow between your knees."

Answer: A. The nurse tells the patient to avoid caffeine-containing foods and beverages. The nurse also teaches the patient to limit or eliminate foods that decrease lower esophageal sphincter (LES) pressure and that irritate inflamed tissue, causing heartburn. These foods include peppermint, chocolate, alcohol, fatty foods (especially fried), caffeine, and carbonated beverages.The nurse also needs to remind the patient to eat four to six small meals each day rather than three large ones and avoid snacking between meals. Large meals increase the volume of and pressure in the stomach and delay gastric emptying. Peppermint decreases LES pressure and increases the risk of symptoms. Patients need to be taught to elevate the head by 6 to 12 inches (30 cm) for sleep to prevent nighttime reflux.

A male patient with a long history of ulcerative colitis experienced massive bleeding and had emergency surgery for creation of an ileostomy. He is very concerned that sexual intercourse with his wife will be impossible because of his new ileostomy pouch. How does the nurse respond? A) "A change in position may be what is needed for you to have intercourse with your wife." B) "Have you considered going to see a marriage counselor with your wife?" C) "What has your wife said about your pouch system?" D) "You must get clearance from your primary health care provider before you attempt to have intercourse."

Answer: A. The nurse tells the patient who had an emergency ileostomy that a simple change in positioning during intercourse may alleviate the patient's apprehension and facilitate sexual relations with his wife.Suggesting marriage counseling may address the patient's concerns, but it focuses on the wrong issue. The patient has not stated that he has relationship problems. Asking the patient what his wife has said about the pouch may address some of the patient's concerns, but it similarly focuses on the wrong issue. Telling the patient that he needs to get clearance from his primary health care provider is an evasive response that does not address the patient's primary concern.

A patient with ulcerative colitis (UC) has stage 1 of a restorative proctocolectomy with ileo-anal anastomosis (RPC-IPAA) procedure performed. The patient asks the nurse, "How long do people with this procedure usually have a temporary ileostomy?" How does the nurse respond? A) "It is usually ready to be closed in about 1 to 2 months." B) "You need to talk to your primary health care provider about how long you will have this temporary ileostomy." C) "The period of time is indefinite—I am sorry that I cannot say." D) "You will probably have it for 6 months or longer, until things heal."

Answer: A. The nurse tells the patient with a temporary ileostomy that it is usually ready to be closed in about 1 to 2 months. The RPC-IPAA has become the most effective alternate method for ulcerative colitis (UC) patients who have surgery to remove diseased portions of intestines. Stage 1 creates a temporary ileostomy to be used while an internally created pouch is healing. Stage 2 closes the ileostomy, and the patient begins to use the pouch for storage of stool. The time between the surgeries is generally 1 to 2 months.Telling the patient that he or she will have to discuss it with the primary health care provider evades the question. The nurse can give generalities to the patient based on past practice and available data. The time that the patient has the ileostomy is not "indefinite." The intent of this procedure is to eliminate the need to have a permanent ileostomy. The pouch would heal in 1 to 2 months, not 6 months. This estimate is not based on the expected outcome.

After receiving change-of-shift report on these clients, which client does the nurse plan to assess first? A) Young adult client with acute pancreatitis who is dyspneic and has a respiratory rate of 34 to 38 breaths/min B) Adult client admitted with cholecystitis who is experiencing severe right upper quadrant abdominal pain C) Middle-aged client who has an elevated temperature after undergoing endoscopic retrograde cholangiopancreatography D) Older adult client who is receiving total parenteral nutrition after a Whipple procedure and has a glucose level of 235 mg/dL (13.1 mmol/L)

Answer: A. The nurse would first assess the young adult client with acute pancreatitis who is dyspneic and has a respiratory rate of 34 to 38 breaths/min. Airway and breathing are the two most important criteria the nurse will use to determine which client to assess first. The dyspneic client is at greatest risk for rapid deterioration and requires immediate assessment and intervention. Acute respiratory distress syndrome is a possible complication of acute pancreatitis.The client with cholecystitis and the client with an elevated temperature will require further assessment and intervention, but these are not medical emergencies requiring the nurse's immediate attention. The older adult client's glucose level will require intervention but, again, is not a medical emergency.

The RN has just received reports about all of these clients on the inpatient surgical unit. Which client does the nurse care for first? A) A 43-year-old who had a bowel resection 7 days ago and has new serosanguineous drainage on the dressing. B) A 46-year-old who had a thoracotomy 5 days ago and needs discharge teaching before going home. C) A 48-year-old who had bladder surgery earlier in the day and is reporting pain when coughing. D) A 49-year-old who underwent repair of a dislocated shoulder this morning and has a temperature of 100.4°F (38°C).

Answer: A. The nurse would first care for the 7-day postoperative client who has new serosanguineous drainage. New drainage on the seventh postoperative day is unusual and suggests a complication that would require further assessment and possible immediate action.The client awaiting discharge teaching is not a priority. A temperature of 100.4°F (38°C) and pain upon coughing following bladder surgery are normal on the first postsurgical day.

When caring for a client with hepatic encephalopathy, in which situation does the nurse question the use of neomycin (Mycifradin)? A) Kidney failure. B) Refractory ascites. C) Fetor hepaticus. D) Paracentesis scheduled for today.

Answer: A. The nurse would question the use of neomycin for a client with kidney failure. Aminoglycoside drugs, which include neomycin, are nephrotoxic and ototoxic, and must not be taken by clients with hepatic encephalopathy.Cirrhosis and hepatic failure cause both ascites and encephalopathy; no contraindication for neomycin is known. Fetor hepaticus causes an ammonia smell to the breath when serum ammonia levels are elevated; neomycin is used to decrease serum ammonia levels. The client may be NPO for a few hours before paracentesis, but may take neomycin when the procedure is complete, or with less than 30 mL of water, depending on hospital policy.

The nurse has placed a nasogastric (NG) tube in a patient with upper gastrointestinal (GI) bleeding to administer gastric lavage. The patient asks the nurse about the purpose of the NG tube for the procedure. What is the nurse's best response? A) "A fluid solution goes down the tube to help clean out your stomach." B) "The medication goes down the tube to help clean out your stomach." C) "The primary health care provider requested the tube to be placed just in case it was needed." D) "We'll start feeding you through it once your stomach is cleaned out.""

Answer: A. The nurse's best response to the patient with upper GI bleeding about the purpose of a NG tube for gastric lavage is that fluid is put down the tube to clean out the stomach. Gastric lavage involves the instillation of a room-temperature solution of water or saline in volumes of 200 to 300 mL through an NG tube to clear out stomach contents and blood clots.Gastric lavage does not involve the instillation of medication. An NG tube is not typically placed in a patient without a particular purpose in mind. Gastric lavage does not involve enteral feeding.

A patient is scheduled to be discharged home after a gastrectomy and will need to perform daily dressing changes on the surgical wound. What is the nurse's highest priority intervention? A) Providing both oral and written instructions to the patient and his spouse on changing the dressing and on symptoms of infection that must be reported to the provider B) Asking the primary health care provider for a referral for home health services to assist with dressing changes C) Asking the spouse if any other family members are in the medical profession and could help change the dressing D) Offer literature on dressing changes and schedule follow-up phone calls with the patient and spouse to talk them through dressing changes when at home.

Answer: A. The nurse's highest priority intervention for a post-op gastrectomy patient about to be discharged with daily dressing changes is to provide the patient and spouse with both oral and written instructions on what to report to the provider, and how to perform the dressing changes. This will reinforce important points needed about what information to report to the provider as well as properly caring for the wound.Obtaining a referral and recruiting other family members prevents the patient and spouse from taking responsibility for the patient's care. Follow-up phone calls and written literature won't provide assurance that wound care is being done properly or that teaching was effective.

The nurse reviews a routine discharge teaching plan concerning postoperative care with a client. Which statement by the client indicates that teaching about wound care was effective? A) "I may need to restrict my activities for several months." B) "I should remove the dressing if the wound is draining." C) "Some bleeding from the incision is normal for several weeks." D) "The wound will completely heal in about 2 months."

Answer: A. To protect the integrity of the wound, activities may need to be restricted.The wound is usually open to air for healing, but draining wounds need to be covered. Bleeding and serosanguineous drainage is not normal after 5 days. The length of time it takes for a wound to heal varies, and can take up to 2 years to heal.

A patient with an exacerbation of ulcerative colitis has been prescribed Vivonex PLUS. The patient asks the nurse how this is helpful for improving signs/symptoms. How does the nurse reply? A) "It is absorbed quickly and allows the affected part of the GI tract to rest and heal." B) "It provides key nutrients and extra calories to promote healing." C) "It is bland and reduces the secretion of gastric acids." D) "It does not contain caffeine or other GI tract stimulants."

Answer: A. Vivonex PLUS is an enteral elemental formula with components that are quickly absorbed in the small bowel that reduces bowel stimulation allowing the affected part of the GI tract to rest and heal. It helps to improve signs/symptoms of ulcerative colitis. For less severe exacerbations, a semielemental product of Vivonex PLUS may induce remission. These products are absorbed in the jejunum and therefore permit the distal small intestine and colon to rest.Nutritional supplements such as Ensure or Sustacal are added to provide nutrients and more calories. GI stimulants such as caffeinated beverages and alcohol need to be avoided, but this is not the reason for using Vivonex PLUS.

A client who has undergone surgery and completed radiation therapy to treat oral cancer reports persistent dry mouth. What will the nurse teach this client about managing this symptom? A) Use saliva substitutes, especially when eating dry foods. B) This condition is common but is temporary. C) Use lozenges and hard candies to prevent dry mouth. D) This indicates a complication of therapy.

Answer: A. Xerostomia is a common effect of oral irradiation and may be permanent. Clients should be advised to use saliva substitutes.The condition is common, but often permanent. Lozenges and hard candies are not as effective as saliva substitutes. Dry mouth is a side effect of therapy, not a symptom of complications. Frequent sips of water is the preferred method of treating xerostomia during radiation therapy.

The nurse is instructing a group of overweight clients on the complications of obesity that develop when weight is not controlled through diet and exercise. Which lifestyle changes does the nurse emphasize? (Select all that apply). A) "Begin a weight-training program for building muscle mass." B) "Consume a diet that is moderate in salt and sugar and low in fats and cholesterol." C) "Eat a variety of foods, especially grain products, vegetables, and fruits." D) "Engage in moderate physical activity for at least 30 minutes each day." E) "Foods eaten away from home tend to be higher in fat, cholesterol, and salt and lower in calcium than foods prepared at home." F) "Liquid dietary supplements can be substituted safely for solid food while attempting to lose weight."

Answer: B, C, D, E. Lifestyle changes the nurse emphasizes include consuming a diet that is moderate in salt and sugar and low in fats and cholesterol, and moderate physical activity for at least 30 minutes each day. These are smart strategies for a person who wants to lose weight. Eating a variety of foods, especially grain products, vegetables, and fruits, helps people achieve weight loss. These are foods that "burn" more calories as they are metabolized. Many foods eaten away from home tend to be higher in fat, cholesterol, and salt and lower in calcium than foods prepared at home. When dining out, people can make smart choices, but they have to be educated and careful.A weight-training program for building muscle mass does not need to be included in a weight loss program. Muscle weighs more and tends to increase weight in people who weight-train. Liquid dietary supplements cannot safely be substituted for solid food while attempting to lose weight.

Which factors place a client at risk for gastrointestinal (GI) problems? (Select all that apply). A) Eating a high-fiber diet B) Smoking a half-pack of cigarettes per day C) Socioeconomic status D) Some herbal preparations E) Use of nonsteroidal anti-inflammatory drugs (NSAIDs)

Answer: B, C, D, E. Smoking or any tobacco use places a client in a higher-risk category for GI problems. Socioeconomic status can also influence the risk for GI problems; clients may not be able to afford to seek care or treatment and may put off seeking help. Some herbal preparations contribute to GI problems, such as Ayurvedic herbs, which can affect appetite, absorption, and elimination. NSAIDs can predispose clients to peptic ulcer disease or GI bleeding.High-fiber diets are generally believed to be healthy for most clients.

A nurs is assessing a client who has appendicitis. Which of the following findings should the nurse expect? (Select all that apply). A) WBC 6,000/mm3. B) Right lower quadrant pain C) Nausea and vomiting D) Oral temperature 38.4° C (101.1° F) E) Bloody diarrhea

Answer: B, C, D. A) A WBC of 10,000 to 18,000/mm3 is an expected finding in a client who has appendicitis. B) Right lower quadrant pain is an expected finding in a client who has appendicitis. C) Nausea and vomiting are expected findings in a client who has appendicitis. D) A low-grade temperature is an expected finding in a client who has appendicitis. E) Bloody diarrhea is an expected finding in a client who has colorectal cancer.

A nurse is caring for a client who has ulcerative colitis. The client has had several exacerbations over the past 3 years. Which of the following instructions should the nurse include in the plan of care to minimize the risk of further exacerbations? (Select all that apply). A) Drink two 240 mL (8 oz) glasses of milk per day. B) Restrict intake of carbonated beverages. C) Use progressive relaxation techniques. D) Arrange activities to allow for daily rest periods. E) Increase dietary fiber intake.

Answer: B, C, D. A) Dairy products, such as milk, are poorly tolerated by clients who have ulcerative colitis and should be avoided. B) The client should avoid gastrointestinal stimulants, such as carbonated beverages, nuts, peppers, and smoking. C) Progressive relaxation techniques, a form of biofeedback, are recommended to help the client minimize stress, which can precipitate an exacerbation. D) Daily rest periods decrease stress and reduce intestinal motility. E) The client should restrict intake of dietary fiber, which can cause diarrhea and cramping.

An obese client has been taking orlistat (Xenical) 60 mg orally three times a day for 4 weeks, but has only lost 10 pounds (4.5 kg). The health care provider doubles the dosage and recommends behavioral changes. What behavioral changes does the nurse include in the teaching plan? (Select all that apply) A) Cognitive restructuring to learn negative coping statements. B) Keeping a daily food diary. C) Identifying emotional and situational factors that stimulate eating. D) Increasing exercise. E) Seeking behaviors in others that one can model.

Answer: B, C, D. Self-monitoring techniques the nurse includes in the teaching plan are keeping a record of foods eaten (food diary), identifying emotional and situational factors that stimulate eating, and exercise patterns. Stimulus control involves controlling the external cues that promote overeating.Cognitive restructuring involves modifying negative beliefs by learning positive, not negative, coping self-statements. Healthy eating behaviors must be learned or modified by the client as an individual and not through copying or modeling others' behaviors.

A patient is diagnosed with irritable bowel syndrome (IBS). What factors does the nurse suspect as possibly contributing to the patient's condition? (Select all that apply). A) Antihistamines. B) Caffeinated drinks. C) Stress. D) Sleeping pills. E) Combination of genetic, immunological, and hormonal factors

Answer: B, C, E. The factors that the nurse suspects may contribute to IBS include: caffeinated drinks, stress, and combinations of genetic, immunological and hormonal factors. The etiology of IBS remains unclear. Research suggests that a combination of environmental, immunologic, genetic, hormonal, and stress factors play a role in the development and course of the disorder. Examples of environmental factors include foods and fluids like caffeinated or carbonated beverages and dairy products. Infectious agents have also been identified. Several studies have found that patients with IBS often have small-bowel bacterial overgrowth, which causes bloating and abdominal distention.Antihistamines and sleeping pills are not suspected of causing IBS.

The nurse is caring for a patient with esophageal cancer who has received photodynamic therapy using porfimer sodium (Photofrin). What instructions does the nurse include in teaching the patient about porfimer sodium? (Select all that apply). A) Avoid sunlight for 2 weeks. B) Cover or shield all exposed body areas from sunlight. C) Follow a clear liquid diet for 3 to 5 days after the procedure. D) Monitor for hypertension. E) Tissue particles may be found in the sputum.

Answer: B, C, E. The nurse teaches the patient that porfimer sodium causes photosensitivity, and sunglasses and protective clothing covering all exposed body areas are essential. Also, a clear liquid diet would be followed for 3 to 5 days after the procedure and then advanced to full liquids as tolerated. In addition, the patient would be warned that tissue particles may be released from the tumor site and may be present in the sputum.Sunlight needs to be avoided for 1 to 3 months, and not for 2 weeks. Side effects are rare and may include nausea, fever, and constipation. Hypertension is not a side effect of porfimer sodium.

Which client does the charge nurse on the adult medical unit assign to an RN who has floated from the outpatient gastrointestinal (GI) clinic? A) A 38-year-old who needs discharge instructions after having an endoscopic retrograde cholangiopancreatography (ERCP) B) A 40-year-old who needs laxatives administered and effectiveness monitored before a colonoscopy C) A 43-year-old recently admitted with nausea, abdominal pain, and abdominal distention D) A 50-year-old with epigastric pain who needs conscious sedation during a scheduled endoscopy procedure

Answer: B. A 40-year-old who needs laxatives administered and effectiveness monitored before a colonoscopy is the least complicated client. This client would be assigned to the float nurse who would have the experience and training to adequately care for this client. A clinic nurse typically cares for clients with chronic conditions.Discharge instructions following an ERCP, assessment of an admitted acutely ill client, and monitoring a client who is receiving conscious sedation would be accomplished best by nurses with experience in caring for adults with acute GI problems.

The nurse expects that which client will be discharged to the home environment first? A) Older obese adult who has had a laparoscopic cholecystectomy B) Middle-aged thin adult who has had a laparoscopic cholecystectomy C) Middle-aged thin adult with a heart murmur who has had a traditional cholecystectomy D) Older obese adult with chronic obstructive pulmonary disease (COPD) who has had a traditional cholecystectomy

Answer: B. A middle-aged client with a thin frame, who had a laparoscopic cholecystectomy, will be discharged first.Although the older obese client also had a laparoscopic cholecystectomy, the client's obesity and age probably will require a longer stay. A traditional cholecystectomy will always require a longer recovery time. The older obese client with a history of COPD will likely have a more lengthy recovery because of associated breathing problems.

A nurse is reviewing the laboratory values of a client who has colorectal cancer. Which of the following findings should the nurse expect? A) Negative fecal occult blood test. B) Hemoglobin 9.1 g/dL C) Hematocrit 43% D) Decreased serum carcinoembryonic antigen (CEA) level

Answer: B. A) A positive fetal occult blood test is an expected finding for a client who has colorectal cancer because colorectal cancer causes bleeding in the gastrointestinal tract. B) A hemoglobin level of 9.1 g/dL is below the expected reference range. Decreased hemoglobin is an expected finding in a client who has colorectal cancer due to occult intestinal bleeding. C) A hematocrit level of 43% is within the expected reference range. The nurse should expect a decreased hematocrit level in a client who has colorectal cancer due to occult intestinal bleeding. D) An elevated CEA level is an expected finding in a client who has colorectal cancer.

A nurse in the PACU is assessing a client who is postoperative. Which of the following findings hsould the nurse report to the provider? A) Small amount of sanguinous drainage on dressing. B) Presence of inspiratory stridor. C) Pain level of 4 on 0 to 10 scale. D) Blood pressure 10% lower than baseline.

Answer: B. A) A small amount of sanguinous drainage is an expected finding following surgery. The nurse should note the amount of sanguinous drainage on the dressing and notify the provider if there is a large amount of drainage. B) The nurse should report inspiratory stridor to the provider because it is a manifestation of tracheal edema and requires intervention. C) Pain is an expected finding following surgery. The nurse should report pain that is not controlled by the administration of analgesic medication. D) The nurse should report a change in blood pressure of 25% above or below baseline to the provider.

A nurse is assessing a client who has acute hepatitis B. Which of the following findings should the nurse expect? A) Abdominal distenion. B) Joint pain. C) Periumbilical discoloration. D) Obstipation.

Answer: B. A) Abdominal distention is an expected finding in a client who has a small bowel obstruction. B) Joint pain is an expected finding in a client who has acute hepatitis B. C) Periumbilical discoloration is an expected finding in a client who has intraperitoneal bleeding. D) Obstipation, or failure to pass stools, is an expected finding in a client who has a complete bowel obstruction.

A nurse is providing teaching for a client who is scheduled to undergo moderate sedation for a bronchoscopy. The nurse should verify that the client understands the procedure when the cient states which of the following? A) "I will need to complete a bowel prep the day before the procedure." B) "I can expect to feel sleepy for several hours after the procedure." C) "I can eat as soon as the procedure is over." D) "I will drink plenty of fluids the morning of the procedure."

Answer: B. A) Bowel preparation is not necessary prior to a bronchoscopy. B) Following moderate sedation, the client should expect to feel drowsy for several hours and should avoid all activities requiring concentration. C) The client should not resume eating until his gag reflex returns to prevent the risk of aspiration. D) The client should be NPO for 4 to 8 hr prior to a bronchoscopy to reduce the risk of aspiration.

A nurse is assessing a client's recovery from spinal anesthesia. Which of the following sensations should the nurse expect to return to the first? A) Pain. B) Touch. C) Warmth. D) Cold.

Answer: B. A) Following spinal anesthesia, the second sensation the nurse should expect the client to feel is the sense of pain. B) Following spinal anesthesia, the first sensation the nurse should expect the client to feel is the sense of touch. C) Following spinal anesthesia, the third sensation the nurse should expect the client to feel is the sense of warmth. D) Following spinal anesthesia, the fourth sensation the nurse should expect the client to feel is the sense of cold.

A nurse is assessing a client who has peritonitis. Which of the following findings should the nurse expect? A) Periumbilical cyanosis. B) Board-like abdomen. C) Bloody diarrhea. D) Increased bowel sounds.

Answer: B. A) Periumbilical cyanosis is an expected finding in a client who has pancreatitis. B) A board-like, distended abdomen, accompanied by extreme pain and tenderness, is an expected finding in a client who has peritonitis. C) Bloody diarrhea is an expected finding in a client who has colorectal cancer. D) Diminished bowel sounds is an expected finding in a client who has peritonitis.

A client is transferredfrom the surgical suite to the PACU following oral surgery. While monitoring the client's vital signs, the nurse finds that the client's tongue has become swollen and is obstructing the airway. Which of the following actions should the nurse take first? A) Assist with endotracheal intubation. B) Use the head-tilt, chin-lift method to open the airway. C) Increase the client's flow of oxygen. D) Contact the anesthesiologist.

Answer: B. A) The nurse might need to assist with intubation; however, this is not the first action the nurse should take. B) The first action the nurse should take when using the airway, breathing, circulation approach to client care is to establish a patent airway by tilting the client's head back and pushing the lower jaw forward. C) The nurse might need to increase the client's flow of oxygen to maintain the oxygen saturation at 90% or higher; however, this is not the first action the nurse should take. D) The nurse should contact the anesthesiologist to provide emergency treatment; however, this is not the first action the nurse should take.

A nurse is providing preoperative teaching for a client who is about to have a below-the-knee amputation. Which of the following should the nurse include? A) "You should avoid lying on your abdomen after surgery." B) "Your surgeon might prescribe an antibiotic before surgery." C) "To promote wound healing, it is important for you to reduce your intake of carbohydrates once you return home." D) "It is important for you to sit in a chair at the bedside for several hours every day to reduce the risk of pneumonia."

Answer: B. A) The nurse should instruct the client that he will be assisted into the prone position every 3 to 4 hr after surgery to prevent a hip flexion contracture. B) A client who has a surgical amputation of an extremity is at risk for infection. Therefore, the provider often prescribes a broad-spectrum, prophylactic antibiotic to reduce the risk of infection. C) A client who is postoperative should increase the intake of carbohydrates and protein. Calories from carbohydrates are used for energy and ensure that adequate proteins are available for wound healing. D) The client should avoid sitting for long periods of time to reduce the risk of a hip flexion contracture.

A nurse is providing dietary teaching for a client who is postoperative following a gastrectomy. Which of the following foods should the nurse encourage the client to include in her diet to prevent dumping syndrome? A) Grape juice. B) Eggs. C) Ice cream. D) Honey.

Answer: B. A) The nurse should instruct the client to avoid drinking sweetened fruit juice due to the client's increased risk for manifestations of dumping syndrome. B) The nurse should instruct the client to increase dietary intake of protein-containing foods, such as eggs, to decrease the risk for manifestations of dumping syndrome. The client should eat some form of protein at each meal. C) The nurse should instruct the client to avoid eating ice cream, a food that is high in sugar, due to the client's increased risk for manifestations of dumping syndrome. D) The nurse should instruct the client to avoid consuming honey and other simple sugars due to the client's increased risk for manifestations of dumping syndrome.

A nurse is creating a plan of care for a client who is preoperative for a total hip arthroplasty, practices judaism, and adheres to a kosher diet. Which of the following interventions is the nurse's priority? A) Listen and allow the client to express feelings about the surgery. B) Determine if the client's faith conflicts with the treatment plan. C) Ensure the client's meal plan serves only kosher food following surgery. D) Teach the client how to perform various relaxation exercises.

Answer: B. A) The nurse should listen and allow the client to express feelings about the surgery to provide support and comfort to the client prior to the surgery. However, another intervention is the nurse's priority at this time. B) The nurse's priority intervention when using the nursing process is assessment. By determining if the client's faith, religious practices, or views conflict with the current treatment plan or surgical procedure, the nurse can take the necessary steps to inform the provider and prevent an issue during or after the surgical procedure. C) The nurse should ensure the client's meal plan serves only kosher food following surgery because it is important to be knowledgeable and respectful of the client's dietary restrictions. However, another intervention is the nurse's priority at this time. D) The nurse should teach the client how to perform various relation exercises, as well as meditation, and guided imagery because these methods are associated with improved physiological and psychological outcomes. However, another intervention is the nurse's priority at this time.

A client had an open transverse colectomy 5 days ago. The nurse enters the client's room and recognizes that the wound has eviscerated. After covering the wound with a sterile, salikne-soaked dressing, which of the following actions should the nurse take? A) Place the client in a reverse Trendelenburg position. B) Obtain vital signs to assess for shock. C) Go to the nurses' station to seek assistance. D) Reinsert the organs into the abdominal cavity.

Answer: B. A) The nurse should place the client in a supine position with hips and knees bent and the head of the bed elevated 15° to 20°. B) The nurse should obtain vital signs to assess the client's current status. C) The nurse should stay with a client who has experienced a wound evisceration. The nurse should press the call light to seek assistance. D) The nurse should make no attempt to reinsert the eviscerated contents.

A nurse is reviewing the medical record of a client who is to undergo general anesthesia for surgery. The nurse should report which of the following findings to the provider? A) BUN 12 mg/dL B) Serum potassium 2.8 mEq/L C) INR 1.5 D) Serum sodium 140 mEq/L

Answer: B. A) The nurse should recognize that the client's BUN level is within the expected reference range of 10 to 20 mg/dL. B) The nurse should recognize that the client's potassium level is below the expected reference range of 3.5 to 5 mEq/L, which places the client at risk for cardiac dysrhythmias. Therefore, this finding should be reported to the provider immediately. C) The nurse should recognize that the client's INR is within the expected reference range of 0.7 to 1.8. D) The nurse should recognize that the client's sodium level is within the expected reference range of 136 to 145 mEq/L.

A nurse is providing teaching for a client who is in the immediate postoperative period and has a PCA pump. Which of the following statements should the nurse include in the teaching? A) "You will receive a dose of medication every time you push the button." B) "Do not allow your family to push the PCA button if you are sleeping." C) "Do not push the PCA button until your pain reaches a severe level." D) "You cannot receive too much medication by pushing the button."

Answer: B. A) The nurse should teach the client that the amount of medication delivered, as well as the time span between each dose, is set by the PCA device. However, the device maintains a therapeutic level of pain control and can be adjusted if the provider determines a need. The PCA has a lock-out period to decrease the risk of opioid overdose. B) The nurse should instruct the client that she should be awake when receiving a dose of the medication and that she is the only authorized user of the PCA pump. Therefore, allowing a family member to push the button is unauthorized and a safety risk for the client. C) The nurse should instruct the client that lower doses of opioids more frequently provide effective pain relief if she uses the PCA pump before the pain gets severe. It requires less medication to stay ahead of the pain rather than waiting until the pain reaches a level that requires extra medication to decrease the client's pain to a manageable level. D) Clients have different tolerance levels for opioid analgesics. Therefore, the nurse should instruct the client to report nausea, dizziness, and other adverse effects indicating excessive sedation. Opioid analgesia places the client at risk for respiratory depression. Therefore, the nurse should monitor the client closely for indications of sedation and respiratory depression.

A surgical nurse enters a surgical suite to ensure surgical asepsis is maintained. Which of the following findings requires intervention by the nurse? A) The circulating nurse opens dressing packages before applying sterile gloves. B) The scrub technologist is wearing a watch under his scrubs. C) The holding area nurse is performing client education. D) The surgeon has her hands folded 5 cm (2 in) above her waist.

Answer: B. A) The outside wrapping of dressing packages is nonsterile. Touching the outside of the wrapper maintains the sterility of the contents inside the wrapper. It is acceptable for the nurse to open packages before applying sterile gloves. Therefore, this action does not require intervention by the nurse. B) Finger and wrist jewelry are likely contaminated with microorganisms and bacteria. Therefore, the scrub technologist should remove jewelry before handling sterile objects. C) The role of the holding area nurse includes verifying that the preoperative checklist is complete and performing client education. Therefore, this action does not require intervention by the nurse. D) The surgeon should hold her hands above the waist to maintain surgical asepsis. Any items held below the waist are deemed unsterile or contaminated. Therefore, this action does not require intervention by the nurse.

A patient with gastroesophageal reflux disease has undergone a laparoscopic Nissen fundoplication (LNF). What will the nurse include in postoperative home care instructions? A) "Consume carbonated beverages if you experience stomach upset." B) "Remain on a soft diet for about a week and avoid raw vegetables that are difficult to swallow." C) "You may resume running and weight lifting if you wish." D) "You may stop taking your antireflux medications after 1 week."

Answer: B. After LNF, patients need to be taught to remain on a soft diet for 1 week and to avoid raw vegetables that are difficult to swallow.Carbonated beverages should be avoided. Patients may walk but need to avoid heavy lifting. Antireflux medications need to be taken for 1 month after the procedure.

The RN receives a change-of-shift report about four patients. Which patient does the nurse assess first? A) A 20-year-old with ulcerative colitis (UC) who had six liquid stools during the previous shift B) A 25-year-old who has just been admitted with possible appendicitis and has a temperature of 102°F (37.9°C) C) A 56-year-old who had a colon resection earlier in the day and whose colostomy bag does not have any stool in it D) A 60-year-old admitted with acute gastroenteritis who is reporting severe cramping and nausea

Answer: B. After a change-of-shift report the RN first assess a 25-year-old who has just been admitted with possible appendicitis and has a temperature of 102°F (37.9°C). This patient with possible appendicitis may have developed a perforation and may be at risk for peritonitis. Rapid assessment and possible surgical intervention are needed.The patient with UC who had six liquid stools, the patient whose colostomy bag does not have any stool in it, and the patient who was admitted with acute gastroenteritis all need assessment and intervention by an RN, but they are not at immediate risk for life-threatening complications.

The RN on the medical-surgical unit receives a shift report about four patients. Which patient does the nurse assess first? A) A 34-year-old who has returned to the unit after a colon resection with a new colostomy stoma, which is pink and moist. B) A 36-year-old admitted after a motor vehicle collision (MVC) with areas of ecchymosis on the abdomen in a "lap-belt" pattern C) A 40-year-old with a reducible inguinal hernia asking questions about surgery. D) A 51-year-old with familial adenomatous polyposis (FAP) who is scheduled for a colonoscopy

Answer: B. After the shift-report, the RN first assesses the 36-year-old admitted after a MVC with areas of ecchymosis on the area in a "lap-belt" pattern. Ecchymosis in the abdominal area may indicate intraperitoneal or intraabdominal bleeding. This patient requires rapid assessment and interventions.The patient who is post colon resection, the patient with preoperative questions, and the patient with FAP do not have an urgent need for further assessment or intervention.

A client has a primary problem of inadequate nutrition caused by the effects of chemotherapy. The client is receiving continuous enteral feedings through a nasogastric (NG) tube. What does the RN ask the LPN/LVN to do for this client? A) Assess nutritional parameters on the client every 3 days. B) Check the residual volume of the NG tube every 4 hours. C) Monitor the client for signs and symptoms of pneumonia. D) Teach the client about the purpose of enteral feedings.

Answer: B. Checking the residual volume of the client's NG tube every 4 hours is within the scope of knowledge and practice for the LPN/LVN.Assessing nutritional parameters on the client, monitoring the client for signs and symptoms of pneumonia, and teaching the client about the purpose of enteral feedings are complex and require broad knowledge about the physiology associated with malnutrition and possible complications of tube feedings. These activities need to be performed by an RN.

A client had a routine sigmoidoscopy with a tissue biopsy. What postprocedure complication would the nurse report to the health care provider? A) Gas and flatulence. B) Excessive bleeding. C) Nausea and vomiting. D) Severe rectal pain.

Answer: B. Excessive or heavy bleeding is a possible complication following a sigmoidoscopy. It must be reported immediately to the health care provider.Nausea, vomiting, and severe rectal pain are not common complications of sigmoidoscopy. Gas and flatulence are expected assessment findings post-sigmoidoscopy

A client is admitted to the hospital with severe right upper quadrant (RUQ) abdominal pain. Which assessment technique does the nurse use for this client? A) Assesses the abdomen in the following sequence: inspection, palpation, percussion, auscultation B) Examines the RUQ of the abdomen last following all other assessment techniques. C) Have the client lie in a supine position with legs straight and arms at the sides D) Gently palpates any bulging mass and documents findings.

Answer: B. If the client reports pain in the RUQ, the nurse would examine this area last in the examination sequence. This sequence prevents the client from tensing abdominal muscles because of the pain, which would make the examination difficult.The sequence for examining the abdomen is inspection, auscultation, percussion, and then palpation. This sequence prevents the increase in intestinal activity and bowel sounds caused by palpation and percussion. The client would be positioned supine with the knees bent, while keeping the arms at the sides to prevent tensing of the abdominal muscles. If a bulging, pulsating mass is present during assessment of the abdomen, do not touch the area because the client may have an abdominal aortic aneurysm, a life-threatening problem. Notify the health care provider of this finding immediately!

Which task would be best for the charge nurse to assign to the LPN/LVN working in the surgery admitting area? A) Provide preoperative teaching to a client who needs insertion of a tunneled central venous catheter. B)Insert a retention catheter in a client who requires a flap graft of a sacral pressure ulcer. C) Obtain the medical history from a client who is scheduled for a total hip replacement. D) Assess the client who is being admitted for an elective laparoscopic cholecystectomy.

Answer: B. Insertion of a catheter is the best task within the scope of skills approved for the LPN/LVN.Preoperative teaching and physical assessment of a preoperative client are under the scope of the RN. History information would be completed by the RN on the unit.

A Certified Wound, Ostomy, and Continence Nurse (CWOCN) is teaching a patient about caring for a new ileostomy. What information is most important to include? A) "After surgery, output from your ileostomy may be a loose, dark-green liquid with some blood present." B) "Call your primary health care provider if your stoma has a bluish or pale look." C) "Notify the primary health care provider if output from your stoma has a sweetish odor." D) "Remember that you must wear a pouch system at all times."

Answer: B. It is most important for the Certified Wound, Ostomy, and Continence nurse to tell the patient with a new ileostomy to call the primary health care provider if the stoma has a bluish or pale look. If the stoma has a bluish, pale, or dark look, its blood supply may be compromised and the primary health care provider must be notified immediately.It is true that output from the stoma after surgery may be a loose, greenish-colored liquid that may contain some blood, but this information is not the highest priority for instruction. It is normal for output from the stoma to have very little odor or a sweetish smell. Although it is true that the patient will be required to wear a pouch system at all times, this is not the highest priority for instruction.

When providing community education, the nurse emphasizes that which group needs to receive immunization for hepatitis B? A) Clients who work with shellfish B) Men who engage in sex with men C) Clients traveling to a third-world country D) Clients with elevations of aspartate aminotransferase and alanine aminotransferase

Answer: B. Men who prefer sex with men are at increased risk for hepatitis B, which is spread by the exchange of blood and body fluids during sexual activity.Consuming raw or undercooked shellfish may cause hepatitis A, not hepatitis B. Travel to third-world countries exposes the traveler to contaminated water and risk for hepatitis A. Hepatitis B is not of concern, unless the client is exposed to blood and body fluids during travel. Clients who have liver disease should receive the vaccine, but men who have sex with men are at higher risk for contracting hepatitis B.

A patient has vague symptoms that indicate an acute inflammatory bowel disorder. Which signs/symptoms are most indicative of Crohn's disease (CD)? A) Abdominal pain relieved by bending the knees, constipation B) Chronic diarrhea, abdominal colicky pain, and fever C) Epigastric cramping & persistent rectal bleeding D) Hypotension with vomiting and headache

Answer: B. Signs/symptoms that are most indicative of Crohn's disease (CD) are: chronic diarrhea, abdominal colicky pain, and fever. These signs/symptoms are more specific to CD than any of the other acute inflammatory bowel disorders.Abdominal pain that is relieved by bending the knees is indicative of peritonitis or pancreatitis. Epigastric cramping is a sign/symptom more indicative of appendicitis. Hypotension with vomiting is not characteristic of CD.

A client has just been diagnosed with pancreatic cancer. The client's upset spouse tells the nurse that they have recently moved to the area, have no close relatives, and are not yet affiliated with a church. What is the nurse's best response? A) "Maybe you should find a support group to join." B) "Would you like me to contact the hospital chaplain for you?" C) "Do you want me to try to find a therapist for you?" D) "Do you have any friends whom you want me to call?"

Answer: B. Suggesting to contact the hospital chaplain is the best and most appropriate response for the nurse to take when talking with the cancer client's spouse.Suggesting that the client find a support group does not assist the client and the family with the problem. It is inappropriate for the nurse to suggest that the client and the family need a therapist. The spouse has already told the nurse that they have recently moved to the area, so it is unlikely that they have already made close friends.

The nurse is educating a group of older adults about screening for colorectal cancer. Which statement by a group member indicates the need for further clarification about these guidelines? A) "A barium enema every 5 years is a screening option." B) "I will need to have a routine colonoscopy every 5 years." C) "My routine flexible sigmoidoscopy every 5 years is OK." D) "The 'virtual' colonoscopy every 5 years is acceptable."

Answer: B. The 2015 guidelines indicate that routine screening with colonoscopy is performed every 10 years, not every 5 years.Other options are performed at 5-year intervals. A barium enema every 5 years is a screening option. A flexible sigmoidoscopy and a "virtual" colonoscopy every 5 years are also acceptable for screening. A "virtual" colonoscopy or CT colonography is a noninvasive imaging procedure that takes multidimensional views of the entire colon.

A client has undergone a partial glossectomy for cancer. What community resource does the nurse refer the client to when dressing supplies will be needed at home? A) Oral Cancer Foundation B) American Cancer Society (ACS) C) Client Advocate Foundation D) American Medical Supply Foundation

Answer: B. The ACS supplies dressings and transportation to and from follow-up visits or medical treatments for clients with cancer. A partial glossectomy is removal of part of the tongue (see Chapter 29).The Oral Cancer Foundation is an organization for local support groups and resources. The Client Advocate Foundation provides education, legal counseling, and referrals to clients with cancer and survivors concerning managed care, insurance, financial issues, job discrimination, and debt crisis matters. The American Medical Supply Foundation does not exist.

A male patient in a long-term care facility is 2 days postoperative after an open repair of an indirect inguinal hernia. Which nursing action does the RN delegate to unlicensed assistive personnel (UAP)? A) Assessing the patient's incision for signs of infection B) Assisting the patient to stand to void C) Instructing the patient in how to deep-breathe D) Monitoring the patient's pain level

Answer: B. The RN delegates the UAP to assist the 2 day postoperative open repair of an indirect inguinal hernia patient to stand and void. Assisting the patient with activities is part of the UAP role.Assessment of the patient's incision and monitoring the patient's pain level requires broader education and scope of practice than a UAP and would be performed by licensed nursing personnel. Patient teaching—even about something as fundamental as taking "deep breaths"—likewise requires broader education and scope of practice and would be done by licensed nursing personnel.

The nurse practitioner is performing an abdominal assessment on a newly admitted client. In which order should the nurse proceed with assessment technique? A) Auscultation, percussion, palpation, inspection B) Inspection, auscultation, percussion, palpation C) Palpation, percussion, inspection, auscultation D) Percussion, auscultation, palpation, inspection

Answer: B. The assessment technique proceeds as inspection, auscultation, percussion, palpation. This sequence is different from that used for other body systems. It is used so that palpation and percussion do not increase intestinal activity and bowel sounds. Nurse generalists may perform inspection, auscultation, and light palpation; percussion and deep palpation may be done by advanced practice nurses.Inspection must be the first assessment technique. Options beginning with auscultation, palpation, or percussion are incorrect.

Which activity by the nurse will best relieve symptoms associated with ascites? A) Administering oxygen B) Elevating the head of the bed C) Monitoring serum albumin levels D) Administering intravenous fluids.

Answer: B. The best action by the nurse caring for a client with ascites is to elevate the head of the bed. The enlarged abdomen of ascites limits respiratory excursion. Fowler's position will increase excursion and reduce shortness of breath.The client may need oxygen, but first the nurse would raise the head of the bed to improve respiratory excursion and oxygenation. Monitoring serum albumin levels will detect anticipated decreased levels associated with cirrhosis and hepatic failure but does not relieve the symptoms of ascites. Administering IV fluids will contribute to fluid volume excess and fluid shifts into the peritoneal cavity, worsening ascites.

An RN and an LPN/LVN are working together in caring for a client who needs all of these interventions after orthopedic surgery. Which action(s) would be best for the RN to accomplish? A) Reinforce the need to cough and deep-breathe every 2 to 4 hours. B) Develop the discharge teaching plan in conjunction with the client. C) Administer narcotic pain medications before assisting the client with ambulation. D) Listen for bowel sounds and monitor the abdomen for distention and pain.

Answer: B. The best and most appropriate action for the nurse to take is to develop the discharge teaching plan with the client. Education and preparation for discharge are within the scope of practice of the RN, but not within that of the LPN/LVN.Reinforcing the need to cough and deep-breathe and monitoring the client are within the scope of the LPN/LVN nurse. LPN/LVNs can also administer pain medications.

Which nursing action is best for the charge nurse to delegate to an experienced LPN/LVN? A) Retape the nasogastric tube for a patient who has had a subtotal gastrectomy and vagotomy. B) Reinforce the teaching previously done by the RN about avoiding alcohol and caffeine for a patient with chronic gastritis. C) Document instructions for a patient with chronic gastritis about how to use "triple therapy." D) Assess the gag reflex for a patient who has arrived from the post anesthesia care unit after a laparoscopic gastrectomy.

Answer: B. The best nursing action to delegate to the experienced LPN/LVN is to reinforce patient teaching previously done by the RN to a patient with chronic gastritis about avoiding alcohol and caffeine. Reinforcement of teaching done by the RN is within the scope of practice for an LPN/LVN.Retaping the nasogastric tube for a patient who has had a subtotal gastrectomy and vagotomy is a complex task that would be done by the RN. Documenting instructions about how to use triple therapy are nursing functions that would be done by the RN. Assessment of a patient's gag reflex is also an RN nursing function.

A patient has been diagnosed with terminal esophageal cancer. The patient is interested in obtaining support from hospice, but expresses concern that pain management will not be adequate. What is the nurse's best response? A) "Haven't you received adequate pain management in the hospital?" B) "Would you like me to get a nurse from hospice to come talk with you?" C) "Do you want me to call the hospital chaplain to explain hospice to you?" D) "Talk to your primary health care provider about hospice services."

Answer: B. The best way to alleviate the patient's concerns would be to have a hospice nurse talk with the patient and answer any questions.Suggesting that the patient has had adequate pain management sounds defensive. Referring the patient to the chaplain or the primary health care provider is evasive and attempts to shift responsibility away from the nurse.

Which patient does the charge nurse assign to an experienced LPN/LVN? A) A 28-year-old who requires teaching about how to catheterize a Kock ileostomy B) A 30-year-old who must receive neomycin sulfate (Mycifradin) before a colectomy C) A 34-year-old with ulcerative colitis (UC) who has a white blood cell count of 23,000/mm3 (23 × 109/L) D) A 38-year-old with gastroenteritis who is receiving IV fluids at 250 mL/hr

Answer: B. The charge nurse assigns to an experienced LPN/LVN a 30-year-old who needs to receive neomycin sulfate before a colectomy. The LPN/LVN would be familiar with the purpose, adverse effects, and patient teaching required for neomycin.Teaching about how to catheterize a Kock ileostomy, assessing the patient with UC with a high white blood cell count, and monitoring the patient with gastroenteritis receiving IV fluids present complex problems that require assessment or intervention by an RN.

The nurse suspects that which client is at highest risk for developing gallstones? A) Obese male with chronic obstructive pulmonary disease B) Obese female receiving hormone replacement therapy C) Thin male with a history of coronary artery bypass grafting D) Thin female who has recently given birth

Answer: B. The client at highest risk is the obese female receiving hormone replacement therapy. Both obesity and hormone replacement therapy have been found to increase a woman's risk for developing gallstones. Other risk factors for developing gallstones are type 2 diabetes, dyslipidemia, and insulin resistance.Men are at lower risk than women for developing gallstones. Although pregnancy increases the risk for a woman to develop gallstones, a woman's thin frame lessens that risk.

An older adult with severe rheumatoid arthritis in the upper extremities is malnourished. What does the nurse suspect as the cause of this client's malnutrition? A) A decrease in the client's appetite B) Decreasing ability to manipulate eating utensils C) Inadequate income to purchase sufficient food D) Metabolic requirements that are increased owing to immobility

Answer: B. The client's severe rheumatoid arthritis in the hands and arms would produce a decrease in the client's ability to manipulate utensils.No evidence suggests that the client is experiencing a decrease in appetite or is financially unable to purchase adequate food. No evidence suggests that the client is immobile because of osteoarthritis in the extremities. Metabolic requirements would decrease, not increase, with less mobility

A patient has developed gastroenteritis while traveling outside the country. What is the likely cause of the patient's symptoms? A) Bacteria on the patient's hands B) Ingestion of parasites in the water C) Insufficient vaccinations D) Overcooked food.

Answer: B. The likely cause of gastroenteritis when a patient travels outside the country is ingestion of water that is infested with parasites.Bacteria on the patient's hands will not produce gastroenteritis unless food or water is contaminated with the bacteria. Insufficient vaccinations may cause other disease processes, but not gastroenteritis. Undercooked, not overcooked, food may produce gastroenteritis.

The nurse is teaching a group of adults in the community about the 2015-2020 Dietary Guidelines for Americans. What does the nurse emphasize as a dietary strategy suggested in these guidelines? A) Half of each meal should consist of dairy, fruits, and proteins. B) Adults should focus on variety and nutrient density and not calories. C) Older adults should consider lacto-ovarian diets for improved health. D) Adults should include a multivitamin with iron and vitamin B12 in their diet.

Answer: B. The nurse emphasizes the need to focus on "shifts" to include a variety of nutrient-dense foods rather than less nutritious foods. The focus involves the client making active choices. This strategy is included in the 2015-2020 Dietary Guidelines for Americans. Examples of other guidelines are listed in Table 60-1.The most recent guidelines in 2015-2020 do not recommend that half of the diet include proteins and dairy. Using the My Plate recommendations, half of the diet should be fruits and vegetables. Lacto-ovarian diets are not emphasized. A multivitamin with iron and B12 is not recommended if the diet is adequate.

A patient admitted with severe gastroenteritis has been started on an IV, but the patient continues having excessive diarrhea. Which medication does the nurse expect the primary health care provider to prescribe? A) Balsalazide (Colazal) B) Loperamide (Imodium) C) Mesalamine (Asacol) D) Milk of Magnesia (MOM)

Answer: B. The nurse expects the primary health care provider to prescribe loperamide for a patient with severe gastroenteritis who still has excessive diarrhea. If the primary health care provider determines that antiperistaltic agents are necessary, an initial dose of loperamide (Imodium) 4 mg can be administered orally, followed by 2 mg after each loose stool, up to 16 mg daily.Balsalazide is not the best choice for control of diarrhea in this scenario. Mesalamine is used for patients with ulcerative colitis for long-term therapy. MOM is a laxative.

A client has undergone an 8-hour surgical procedure under general anesthesia. In assessing the client for complications related to positioning, the nurse is most concerned with which finding? A) Decreased sensation in the lower extremities. B) Diminished peripheral pulses in the lower extremities. C) Pale, cool extremities. D) Reddened areas over bony prominences.

Answer: B. The nurse is most concerned with diminished peripheral pulses in the lower extremities. This could indicate diminished blood flow.Decreased sensation; pale, cool extremities; and reddened areas over bony prominences can be normal occurrences in clients who have undergone a long surgical procedure.

The admission assessment for a patient with acute gastric bleeding indicates blood pressure 82/40 mm Hg, pulse 124 beats/min, and respiratory rate 26 breaths/min. Which admission request does the nurse implement first? A) Type and crossmatch for 4 units of packed red blood cells. B) Infuse 0.9% normal saline solution at 200 mL/hr. C) Give pantoprazole (Protonix) 40 mg IV now and then daily. D) Insert a nasogastric tube and connect to low intermittent suction.

Answer: B. The nurse must first infuse 0.9% normal saline solution at 200 mL/hr for the patient with acute gastric bleeding and hypotension associated with volume loss. The most rapidly available volume expanders are crystalloids to treat hypovolemia.A type and crossmatch, administration of pantoprazole, and insertion of a nasogastric tube must all be done, but the nurse's immediate concern is correcting the patient's hypovolemia.

The nurse is assessing a client who comes to the emergency department with acute abdominal pain. The nurse notes a bulging, pulsating mass when inspecting the abdomen. Which action by the nurse is correct? A) Auscultate the abdomen to determine the presence of bowel sounds. B) Notify the provider about this finding immediately. C) Palpate the client's abdomen to determine the outlines of the mass. D) Question the client about recent stool habits.

Answer: B. The nurse needs to immediately notify the health care provider because a bulging, pulsating mass may indicate an abdominal aortic aneurysm requiring emergency actions.Palpating the abdomen or even touching the abdomen with a stethoscope may cause this to rupture, which would be a life-threatening emergency. Because this is a potential life-threatening situation, questioning the client about stool habits is not appropriate.

A nurse is teaching a patient about dietary methods to help manage exacerbations (flare-ups) of diverticulitis. What does the nurse advice the patient? A) "Be sure to maintain an exclusively low-fiber diet to prevent pain on defecation." B) "Consume a low-fiber diet while your diverticulitis is active. When inflammation resolves, consume a high-fiber diet." C) "Maintain a high-fiber diet to prevent the development of hemorrhoids that frequently accompany this condition." D) "Make sure you consume a high-fiber diet while diverticulitis is active. When inflammation resolves, consume a low-fiber diet."

Answer: B. The nurse teaches the patient that the most effective way to manage diverticulitis is to consume a low-fiber diet while inflammation is present, followed by a high-fiber diet once the inflammation has subsided.Neither an exclusively low-fiber diet nor an exclusively high-fiber diet will effectively manage diverticulitis. A high-fiber diet while diverticulitis is active will only worsen the disease and its symptoms.

A patient who developed viral gastroenteritis with vomiting and diarrhea is scheduled to be seen in the clinic the following day. What will the nurse teach the patient to do in the meantime? A) "Avoid all solid foods to allow complete bowel rest." B) "Consume extra fluids to replace fluid losses." C) "Take an over-the-counter antidiarrheal medication." D) "Contact your primary health care provider for an antibiotic medication."

Answer: B. The nurse tells the patient to drink extra fluids to replace fluid lost through vomiting and diarrhea.It is not necessary to stop all solid food intake. Antidiarrheal medications are used if diarrhea is severe. Antibiotics are used if the infection is bacterial.

A patient suspected of having irritable bowel syndrome (IBS) is scheduled for a hydrogen breath test. What does the nurse tell the patient about this test? A) "During the test, you will drink small amounts of an antacid as directed by the technician." B) "If you have IBS, hydrogen levels may be increased in your breath samples and can be an indication that you have IBS." C) "The test will take between 30 and 45 minutes to complete." D) "You must have nothing to drink (except water) for 24 hours before the test."

Answer: B. The nurse tells the patient with IBS who has a hydrogen breath test prescribed that "hydrogen levels may be increased in your breath samples and can indicate that you have IBS." Excess hydrogen levels in patients with IBS are due to bacterial overgrowth in the small intestine that accompanies the disease. The hydrogen travels to the lungs to be excreted.The patient will ingest small amounts of sugar during the test, not an antacid. The test takes longer than 45 minutes to complete. The patient has breath samples taken every 15 minutes for 1 to 2 hours. The patient needs to be NPO (except for water) for 12 hours before the test.

As the nurse obtains informed consent, the client asks, "Now what exactly are they going to do to me?" What is the nurse's response? A) Contact the anesthesiologist. B) Contact the surgeon. C) Explain the procedure. D) Have the client sign the form

Answer: B. The nurse will contact the surgeon to convey the client's question. The nurse is not responsible for explaining or providing detailed information about the surgical procedure. Rather, the nurse's role is to clarify facts that have been presented by the health care provider and dispel myths that the client or family may have heard about the surgical experience.The anesthesiologist is responsible for the anesthesia, not the surgical details. Although the nurse is only witnessing the signature, it is the nurse's role to ensure that the facts are clarified before the consent form is signed.

The nurse is caring for clients in the outclient clinic. Which of these phone calls would the nurse return first? A) Client with hepatitis A reporting severe and ongoing itching B) Client with severe ascites who has a temperature of 101.4°F (38°C) C) Client with cirrhosis who has had a 3-pound (1.4 kg) weight gain over 2 days D) Client with esophageal varices and mild right upper quadrant pain

Answer: B. The nurse will first call the client with severe ascites and a temperature of 101.4 (38°C).This client may have spontaneous bacterial peritonitis.Itching is anticipated with jaundice, so this client may be called last. Weight gain with cirrhosis is not uncommon owing to low albumin levels. Cirrhosis may cause mild right upper quadrant pain. This client would be called after the client with severe ascites.

The RN has just received the change-of-shift report for the medical unit. Which client should the RN see first? A) Client with ascites who had a paracentesis 2 hours ago and is reporting a headache B) Client with portal-systemic encephalopathy (PSE) who has become increasingly difficult to arouse C) Client with hepatic cirrhosis and jaundice who has hemoglobin of 10.9 g/dL (109 mmol/l) and thrombocytopenia D) Client with hepatitis A who has elevated alanine aminotransferase (ALT) and aspartate aminotransferase (AST)

Answer: B. The nurse would first see the client with PSE who is now difficult to arouse. A change in the level of consciousness (LOC) of the client with PSE is the greatest concern. Actions to improve the client's LOC must be rapidly implemented.Although uncomfortable, a headache in the client with ascites is not likely related to liver disease and does not pose an immediate threat or complication. A hemoglobin of 10.9 g/dL (109 mmol/L) and thrombocytopenia are expected findings in a client with cirrhosis and do not pose an immediate threat. Elevated ALT and AST levels are expected for the client with hepatitis A and do not indicate a risk for severe complications.

A patient has a long-term history of Crohn's disease and has recently developed acute gastritis. The patient asks the nurse whether Crohn's disease was a direct cause of the gastritis. What is the nurse's best response? A) "Yes, Crohn's disease is known to be a direct cause of the development of chronic gastritis." B) "We know that there can be an association between Crohn's disease and chronic gastritis, but Crohn's does not directly cause acute gastritis to develop." C) "What has your doctor told you about how your gastritis developed?" D) "Yes, a familial tendency to inherit Crohn's disease and gastritis has been reported. Have your other family members been tested for Crohn's disease?"

Answer: B. The nurse's best response is that Crohn's disease may be an underlying disease process when chronic gastritis develops, but not when acute gastritis occurs. It is not known to be a direct cause of the disease.Although Crohn's disease tends to run in families, gastritis is a symptom of other disease processes. It is not a disease process in and of itself. Asking the patient what the doctor has said is an evasive response on the part of the nurse and does not help answer the patient's question.

A patient with peptic ulcer disease (PUD) asks the nurse whether licorice and slippery elm might be useful in managing the disease. What is the nurse's best response? A) "No, they probably won't be useful. You should use only prescription medications in your treatment plan." B) "These herbs could be helpful. However, you should talk with your primary health care provider before adding them to your treatment regimen." C) "Yes, these are known to be effective in managing this disease but make sure you research the herbs thoroughly before taking them." D) "No, herbs are not useful for managing this disease. You can use any type of over-the-counter drugs though. They have been shown to be safe."

Answer: B. The nurse's best response is that although licorice and slippery elm may be helpful in managing PUD, the patient must consult his or her primary health care provider before making a change in the treatment regimen.Alternative therapies may or may not be helpful in managing PUD. The patient must not use over-the-counter medications without first discussing it with his or her primary health care provider.

A patient who has been diagnosed recently with esophageal cancer states, "I'm not comfortable going to my father's birthday lunch at our family-owned restaurant because I'm afraid I'll choke in public." What is the nurse's best response? A) "I understand your concerns, but you can't give up your normal activities. You should go anyway and try not to worry about it." B) "Tell me more about the lunch, what will be served and who is going with you." C) "Why not take one of your antianxiety pills before going? That will keep you from worrying about everything so much." D) "You need to talk to your doctor about your concerns. The doctor may recommend that you join a support group for cancer survivors."

Answer: B. The nurse's best response is to ask the patient for more information to help determine the specific fear and discuss possible alternatives so choking and/or fear of choking can be minimized or avoided in public.Telling the patient not to worry about it or to call the provider is evasive and unhelpful; it is used to placate the patient and does not address the patient's concerns. The patient should use problem-solving and coping skills before resorting to the use of medication.

A patient in the outpatient clinic tells the nurse about experiencing heartburn and nighttime coughing episodes. Which action does the nurse take first? A) Teach the patient about antacid effects and side effects. B) Ask the patient about oral intake, current medications and description of episodes. C) Suggest that the patient sleep with the head elevated 6 inches (15 cm). D) Tell the patient to avoid drinking alcohol late in the evening.

Answer: B. The nurse's first action would be further assessment of the patient's risk factors for gastroesophageal reflux disease (GERD). Before suggesting interventions or beginning patient teaching, the nurse must elicit more information about the patient's symptoms.The nurse needs additional data before telling the patient about antacid effects, sleeping with the head elevated, or not drinking alcohol late in the evening.

The nurse assesses a client's wound 24 hours postoperatively. Which finding causes the nurse the greatest concern and should be reported to the surgeon? A) Crusting along the incision line. B) Redness and swelling around the incision. C) Sanguineous drainage at the suture site. D) Serosanguineous drainage on the dressing.

Answer: B. The nurse's greatest concern is redness and swelling around the incision. This needs to be reported to the surgeon because these signs could indicate an infection.Crusting along the incision line, sanguineous drainage, and serosanguineous drainage are normal.

A client is scheduled to undergo a liver transplantation. Which nursing intervention is most likely to prevent the complications of bile leakage and abscess formation? A) Preventing hypotension. B) Keeping the T-tube in a dependent position C) Administering antibiotic vaccinations. D) Administering immune-suppressant drugs

Answer: B. The nursing intervention most likely to prevent the complications of bile leakage and abscess formation is keeping the T-tube in a dependent position and secured to the client. This action will likely prevent bile leakage, abscess formation, and hepatic thrombosis.Preventing hypotension will help to prevent the complication of acute kidney injury. Administering antibiotic vaccinations will help to prevent infection. Administering immune-suppressant drugs will help to prevent graft rejection.

A patient with irritable bowel syndrome (IBS) is constipated. The nurse instructs the patient about a management plan. Which patient statement shows an accurate understanding of the nurse's teaching? A) "A cup (236 mL) of caffeinated coffee with cream & sugar at dinner is OK for me." B) "I need to go for a walk every evening." C) "Maintaining a low-fiber diet will manage my constipation." D) "Limiting the amount of fluid that I drink with meals is very important."

Answer: B. The patient statement, "I need to go for a walk every evening," shows that the patient accurately understands the nurse's management plan to treat IBS. Walking every day is an excellent exercise for promoting intestinal motility. Increased ambulation is part of the management plan for IBS, along with increased fluids and fiber and avoiding caffeinated beverages.Caffeinated beverages can cause bloating or diarrhea and need to be avoided in patients with IBS. Fiber is encouraged in patients with IBS because it produces a bulky soft stool and aids in establishing regular bowel habits. At least 8 to 10 cups (2 to 2.5 liters) of fluid need to be consumed daily to promote normal bowel function.

The nurse is teaching a patient who has undergone a hemorrhoidectomy about a follow-up plan of care. Which patient statement demonstrates a correct understanding of the nurse's instructions? A) "I would take Ex-Lax after the surgery to 'keep things moving'." B) "I will need to eat a diet high in fiber." C) "Limiting my fluids will help me with constipation." D) "To help with the pain, I'll apply ice to the surgical area."

Answer: B. The statement that shows that the hemorrhoidectomy patient correctly understands the nurse's instruction is, "I will need to eat a diet high in fiber." A diet high in fiber serves as a natural stool softener and will prevent irritation to hemorrhoids caused by painful bowel movements.Ex-lax is a stimulant laxative. Stimulant laxatives are discouraged because they are habit-forming. Increased amounts of fluids are needed to prevent constipation. Moist heat (sitz baths) will be more effective with postoperative discomfort than cold applications. Cold therapy is sometimes recommended and useful before surgery for inflamed hemorrhoids.

When providing dietary teaching to a client with hepatitis, what practice does the nurse recommend? A) Having a larger meal early in the morning B) Consuming increased carbohydrates and moderate protein C) Restricting fluids to 1500 mL/day D) Limiting alcoholic beverages to once weekly

Answer: B. To repair the liver, the nurse recommends that the client adopt a high-carbohydrate and moderate-protein diet. Fats may cause dyspepsia.The client with hepatitis feels full easily and needs to have four to six small meals daily. Fluids are restricted with ascites caused by cirrhosis. Not all clients with hepatitis progress to cirrhosis. Complete abstention from alcohol is necessary until the liver enzymes return to normal.

A patient with a recent surgically created ileostomy refuses to look at the stoma and asks the nurse to perform all required stoma care. What does the nurse do next? A) Asks the patient whether family members could be trained in stoma care B) Has another patient with a stoma who performs self-care talk with the patient C) Requests that the primary health care provider request antidepressants and a psychiatric consult D) Suggests that the primary health care provider request a home health consultation so stoma care can be performed by a home health nurse

Answer: B. When a patient with a recently created ileostomy refuses to look at the stoma and wants the nurse to perform all required stoma care, the nurse has another patient with a stoma who performs self-care talk with the patient.If at all possible, the patient would perform stoma care so that he or she can be as independent as possible. Although the patient may need medication for depression, the priority is to encourage the patient to look at, touch, and begin caring for the stoma. A home health nurse can be a support but cannot provide all of the care that the patient will need.

A patient is admitted with severe viral gastroenteritis caused by norovirus. The patient asks the nurse, "How did I get this disease?" Which answer by the nurse is correct? A) "You may have contracted it from an infected infant." B) "You may have consumed contaminated food or water." C) "You may have come into contact with an infected animal." D) "You may have had contact with the blood of an infected person."

Answer: B. When a patient with severe viral gastroenteritis caused by norovirus asks, "How did I get this disease?", the nurse answers, "You may have consumed contaminated food or water." Norovirus is the leading foodborne disease that causes gastroenteritis. It is transmitted via the fecal-oral route from person to person and from contaminated food and water. Vomiting causes the virus to become airborne.Campylobacter, not novovirus, can be transmitted by contact with infected infants or animals. Escherichia coli, not novovirus, may be spread via animals and contaminated food, water, or fomites. HIV, not novovirus, may be spread via the blood. Campylobacter and E. coli both cause bacterial gastroenteritis, while norovirus causes viral gastroenteritis.

The nurse is reviewing admitting requests for a patient admitted to the intensive care unit with perforation of a duodenal ulcer. Which request does the nurse implement first? A) Apply antiembolism stockings. B) Place a nasogastric (NG) tube, and connect to suction. C) Insert an indwelling catheter, and check output hourly. D) Give famotidine (Pepcid) 20 mg IV every 12 hours.

Answer: B. When caring for an ICU patient with a perforated duodenal ulcer, the nurse or primary care provider must first insert a nasogastric (NG) tube and connect it to suction. To decrease spillage of duodenal contents into the peritoneum, NG suction would be rapidly initiated. This will minimize the risk for peritonitis.Antiembolism stockings will need to be applied, monitoring urined output is important, and famotidine (Pepcid) will need to be administered, but these are done after the NG tube is inserted and connected to suction.

A nurse is developing a plan of care for a client who has cirrhosis and ascites. Which of the following interventions should the nurse include in the plan? A) Check mental status once daily. B) Measure the client's abdominal girth daily. C) Provide a daily intake of 4 g of sodium for the client. D) Assess the client's breath sounds every 12 hr.

Answer: B. A) A client who has cirrhosis is at risk for hepatic encephalopathy. The nurse should assess the client's mental status every 4 to 8 hr. B) The nurse should measure the client's abdominal girth and weigh the client daily to monitor the amount of fluid accumulation in the abdomen and the effectiveness of treatment measures. C) A client who has cirrhosis can have edema and ascites. The client who has cirrhosis is usually prescribed a 1 to 2 g sodium-restricted diet to prevent ascites. D) A client who has cirrhosis is at risk for dyspnea due to ascites. The nurse should monitor the client's breath sounds every 4 to 8 hr.

A nurse is caring for a client who has a surgical wound with a Penrose drain in place. Which of the following interventions should the nurse plan to perform? A) Apply negative pressure when emptying the drain. B) Establish a clamping schedule prior to removal. C) Use the sterile technique when performing dressing changes. D) Cut a slit in a 4-inch square gauze pad to place around the drain.

Answer: C. A) A Penrose drain is an open system and drains by gravity. B) Clamping a Penrose drain can lead to infection. C) The nurse should change the Penrose drain dressing using the surgical aseptic technique. D) A drain sponge should be used around a Penrose drain. A gauze pad should never be cut and used around a drain due to the risk of dressing fibers becoming embedded in the wound.

A nurse is reviewing the laboratory results of a client who has hepatic cirrhosis. Which of the following laboratory findings should the nurse report to the provider? A) Direct bilirubin 0.5 mg/dL. B) INR 1.0. C) Ammonia 180 mcg/dL. D) Albumin 4.0 g/dL

Answer: C. A) A direct bilirubin level of 0.5 mg/dL is within the expected reference range of 0.3 to 1.0 mg/dL. B) An INR of 1.0 is within the expected reference range of 0.8 to 1.1. C) An ammonia level of 180 mcg/dL is above the expected reference range of 10 to 80 mcg/dL. The nurse should report an increased ammonia level because it can indicate portal-systemic encephalopathy. D) An albumin level of 4.0 g/dL is within the expected reference range of 3.5 to 5 g/dL.

A nurse is caring for a client who is 12 hr postoperative from a gastrectomy and has an NG tube set to continuous low suction. Which of the following findings requires intervention by the nurse? A) Urine output of 150 mL over the last 4 hr B) Yellow drainage in the NG tube C) Gastric distention D) Absent bowel sounds

Answer: C. A) A urine output of 150 mL is within the expected reference range. The nurse should report a urine output of less than 30 mL/hr to the provider. B) Yellow drainage indicates gastric secretions and is an expected finding after a gastrectomy. C) Gastric distention is an indication that the NG tube is not patent. The nurse should check the tubing for kinks, blockages, and loose connections. The nurse should also reposition the client to facilitate drainage. The nurse should avoid removing or irrigating the tube unless directed to do so by the surgeon. D) Absent bowel sounds are an expected finding for the first 24 hr after abdominal surgery.

A nurse is reviewing the medication administration record for a client who is scheduled for surgery the next day. The nurse should identify that which of the following medications places the client at risk for complications during surgery and should be reported to the provider? A) Atorvastatin. B) Alendronate. C) Clopidogrel. D) Ranitidine.

Answer: C. A) Atorvastatin is an antilipoprotein used to reduce cholesterol levels, which decreases the client's risk of cardiovascular events. Therefore, the nurse should identify that atorvastatin does not increase the client's risk for surgical complications. B) Alendronate is a bisphosphonate that decreases the activity of osteoclasts and, therefore, inhibits bone resorption in clients who have osteoporosis. The nurse should identify that alendronate does not increase the client's risk for surgical complications. C) The nurse should identify that clopidogrel is an oral antiplatelet medication used to prevent coronary artery stenosis and other vascular incidents. Therefore, the mediation should be discontinued 5 days prior to surgery because it acts similarly to aspirin and can cause the client to experience increased bleeding during and after surgery. D) Ranitidine is a histamine receptor antagonist that suppresses gastric acid secretion in order to improve the client's gastroesophageal reflux disease. Therefore, the nurse should identify that ranitidine does not increase the client's risk for surgical complications.

A nurse is admitting a client who has acute pancreatitis. Which of the following actions should the nurse take first? A) Administer ceftazidime to the client.. B) Insert a nasogastric tube for the client. C) Identify the client's current level of pain. D) Instruct the client to remain NPO.

Answer: C. A) Clients who have acute pancreatitis are at risk for infection. The nurse should administer prescribed antibiotics. However, there is another action the nurse should perform first. B) Clients who have acute pancreatitis are at risk for paralytic ileus and might require gastric decompression. The nurse should insert a nasogastric tube, if needed. However, there is another action the nurse should perform first. C) The first action the nurse should take when using the nursing process is to assess the client. Clients who have acute pancreatitis often have severe abdominal pain. By assessing the client's level of pain, the nurse can identify the need for and implement interventions to alleviate the client's pain. D) Clients who have acute pancreatitis are often placed on NPO status to decrease stress on the pancreas. The nurse should explain this intervention to the client. However, there is another action the nurse should perform first.

A nurse is providing discharge teaching for a client following an ileostomy. The nurse should instruct theclient to report which of the following findings to the provider? A) Intolerance to high-fiber foods. B) Sensation of burning during bowel elimination. C) Dark purple stoma. D) Liquid ileostomy output.

Answer: C. A) Clients who have an ileostomy often experience intolerance to high-fiber foods. The client might need to eliminate these foods from her diet. However, the client does not need to report this intolerance to the provider. B) Clients who have an ileostomy are expected to experience a burning sensation during bowel elimination due to decreased absorption of gastric acid in the ileum. The nurse should provide the client with instructions about skin care, such as washing the area around the ostomy with warm soap and water after each bowel movement, drying the area gently, and applying a thin coat of ointment to the area. C) The nurse should instruct the client to contact the provider if the stoma is a dark purple color, which is an indication of bowel ischemia. D) Clients who have an ileostomy are expected to have loose liquid output. The nurse should provide the client with information about how to avoid dehydration due to excessive liquid output, including recommending the consumption of oral replacement solutions.

A nurse is providing dietary teaching for a client wh ohas a new diagnosis o f celiac disease. Which of the following statementsby the client indicates an understanding of the teaching? A) "I will need to avoid taking vitamin supplements while on this diet." B) "I can return to my regular diet when I am free of symptoms." C) "I will eat beans to ensure I get enough fiber in my diet." D) "I need to avoid drinking liquids with my meals while on this diet."

Answer: C. A) Clients who have celiac disease are at risk for malabsorption of vitamins and minerals; therefore, the client should continue taking vitamin and mineral supplements. B) Celiac disease is an autoimmune disorder that causes changes to the intestinal mucosa, resulting in an intolerance to gluten, which is found in wheat, barley, and rye. The client should continue to avoid eating foods that contain gluten. C) Clients who have celiac disease must maintain a gluten-free diet which eliminates fiber-rich whole wheat products. Clients should eat beans, nuts, fruits, and vegetables to ensure an adequate intake of fiber. D) Clients who have dumping syndrome should avoid drinking liquids with their meals to slow the movement of food through the intestinal tract. Clients who have celiac disease do not need to refrain from drinking liquids with meals.

A nurse is reviewing the medical record of a client who is scheduled for an elective surgery. Which of the following medications should the nurse expect the provider to discontinue prior to surgery to minimize the risk of complications? A) Digoxin. B) Cefazolin. C) Warfarin. D) Ondansetron.

Answer: C. A) Digoxin is an antiarrhythmic that does not increase the client's risk for surgical complications. B) Cefazolin is an antibiotic that can be used prophylactically to reduce the risk of infection for some types of surgical procedures. C) The nurse should anticipate that the provider will instruct the client to discontinue warfarin, an anticoagulant, because it increases the risk of bleeding during and following surgery. D) Ondansetron is a serotonin receptor antagonist that is used to treat postoperative nausea and vomiting, which could place the client at risk for aspiration of gastric secretions.

A circulating nurse is monitoring the temperature in a surgical suite. The nurse should identify that cool temperatures reduce a client's risk for which of the following potential complications of surgery? A) Malignant hyperthermia. B) Blood clots. C) Infection. D) Hypoxia.

Answer: C. A) Malignant hyperthermia is a genetic condition that can cause a reaction to the surgical anesthetic. Therefore, a cool room temperature does not reduce the risk for malignant hyperthermia. B) Decreased mobility during a surgical procedure can increase the risk for a blood clot. Therefore, a cool room temperature does not reduce the risk for blood clots. C) The nurse should identify that a cool room temperature with humidity between 30% and 60%, along with a proper air exchange and filtering system, reduces the risk of infection for clients during surgery. D) Hypoxia is caused by the surgical anesthetic and inadequate ventilation. Therefore, a cool room temperature does not reduce the risk for hypoxia.

A nurse is providing discharge teaching for a client who has chronic hepatitis C. Which of the following statements by the client indicates an understanding of the teaching? A) "I will need treatment for 3 months." B) "I will decrease my intake of calories." C) "I will avoid medications that contain acetaminophen." D) "I will avoid alcohol until I'm no longer contagious."

Answer: C. A) Medications used to treat hepatitis include a combination of peginterferon, boceprevir, and ribavirin. A common course of treatment is peginterferon and ribavirin for 4 weeks followed by peginterferon, ribavirin, and boceprevir for 6 to 8 weeks. B) A client who has hepatitis C should eat small, frequent meals that are high in carbohydrates and calories. C) A client who has hepatitis C should avoid medications that contain acetaminophen, which can cause additional liver damage. D) A client who has hepatitis C should avoid alcohol consumption at all times due to the client's increased risk for cirrhosis.

A nurse is reviewing the prescriptions for a client wh ohas campylobacter enteritis. Which of the following prescriptions should the nruse clarify with the provider? A) Potassium. B) 0.45% sodium chloride IV C) Magnesium hydroxide D) Ciprofloxacin

Answer: C. A) Nausea, vomiting, and diarrhea are manifestations of enteritis, which places the client at risk for hypokalemia. The nurse should recognize that a prescription for potassium is appropriate. B) Nausea, vomiting, and diarrhea are manifestations of enteritis. The nurse should recognize that a prescription for an IV administration of 0.45% sodium chloride, a hypotonic solution, is appropriate to reverse the effects of dehydration. C) Nausea, vomiting, and diarrhea are manifestations of enteritis. The nurse should clarify a prescription for magnesium hydroxide, also known as milk of magnesia, with the provider. This medication increases gastrointestinal motility, which can increase the client's risk for an electrolyte imbalance and contribute to dehydration. D) The nurse should recognize that a prescription for ciprofloxacin, an antibiotic, is appropriate because Campylobacter enteritis is a bacterial form of gastroenteritis.

A nurse is receiving evening shift report on four clients who returned from the PACU that morning. The nurse should assess which of the following clients first? A) A client who is postoperative following a total knee arthroplasty and is reporting a knee pain level of 7 on a scale from 0 to 10. B) A client who is postoperative following a tonsillectomy and has had one episode of coffee-ground emesis. C) A client who is postoperative following a thoracotomy and has a chest tube with 150 mL of bright-red blood in the collection chamber from the past 1 hr. D) A client who is postoperative following a small bowel resection and has a temporary colostomy along with absent bowel sounds in all four quadrants.

Answer: C. A) The nurse should assess the client who is postoperative from a total knee arthroplasty and has postoperative pain of 7 on a scale from 0 to 10; however, this is an expected finding and there is another client the nurse should assess first. B) The nurse should assess the client who is postoperative from a tonsillectomy and has coffee-ground emesis; however, postoperative coffee-ground emesis is an expected finding and there is another client the nurse should assess first. C) When using the airway, breathing, circulation approach to client care, the nurse should first assess the client who has 150 mL of blood in the collection chamber because this finding is above the expected reference range and can be an indication of hemorrhage. D) The nurse should assess the client who is postoperative from a small bowel resection and has absent bowel sounds; however, absent bowel sounds are an expected finding and there is another client the nurse should assess first.

A nurse is assessing a client who is preoperative. The nurse should identify that which of the following factors reported by the client increases the risk for a postoperative wound infection? A) Frequent use of echinacea. B) History of osteoporosis C) Long-term use of corticosteroids D) Diet high in vitamin C

Answer: C. A) The nurse should identify that echinacea is a dietary supplement used to stimulate immune function. Therefore, it does not increase the client's risk for infection. B) The nurse should identify that a history of osteoporosis increases the client's risk for bone fracture, but does not increase the client's risk for infection. C) The nurse should identify that the use of corticosteroids inhibits leukocyte response, which increases the client's risk for infection. D) The nurse should identify that a diet high in vitamin C promotes wound healing. Therefore, it does not increase the client's risk for infection.

A nurse is providing discharge teaching for a client who has a new prescription for medications to treat peptic ulcer disease. The nurse should identify that which of the following medications inhibits gastric acid secretion? A) Aluminum hydroxide. B) Calcium carbonate. C) Famotidine. D) Sucralfate.

Answer: C. A) The nurse should inform the client that aluminum hydroxide is an antacid that neutralizes gastric acid but does not inhibit its secretion. B) The nurse should inform the client that calcium carbonate is an antacid that neutralizes gastric acid but does not inhibit its secretion. C) The nurse should inform the client that famotidine is an H2-receptor antagonist that is prescribed for the treatment of peptic ulcer disease to inhibit the secretion of gastric acid. D) The nurse should inform the client that sucralfate is a mucosal barrier fortifier that forms a protective coating over the ulcer but does not inhibit gastric acid secretion.

A nurse is providing discharge teaching for a client who has GERD. Which of the followng statements by the client indicates an understanding of the teaching? A) "I will avoid drinking liquids for 30 minutes after taking a chewable antacid tablet." B) "I will eat a snack before going to bed." C) "I will decrease the amount of carbonated beverages I drink." D) "I will lie down for at least 30 minutes after eating each meal."

Answer: C. A) The nurse should instruct the client to drink a glass of water immediately after taking an antacid tablet. B) The nurse should instruct the client to eat four to six small meals per day and avoid snacking before bed. C) The nurse should instruct the client to limit or eliminate fatty foods, coffee, cola, tea, carbonated beverages, and chocolate from his diet because they irritate the lining of the stomach. D) The nurse should instruct the client to sit upright for 1 to 2 hr after meals.

A nurse is planning care for a client who is postoperative and has a closed-wound drainage system in place. Which of the following interventions should the nurse plan to include? A) Clamp the drain while the client is ambulating. B) Check the patency of the drain every 12 hr. C) Cleanse the drain plug with alcohol after emptying. D) Secure the drain to the client's bed sheet.

Answer: C. A) The purpose of a closed-wound drainage system is to provide continuous suction. Therefore, the nurse should not clamp the drain while the client is ambulating. B) The nurse should check the amount, color, and type of drainage at least every 8 hr. C) After emptying the drain, the nurse should use one hand to compress the top and bottom of the device together and the other to cleanse the plug before replacing it. D) The nurse should secure the drain to the client's gown to prevent dislodgement.

A surgical client has signed do-not-resuscitate (DNR) orders before going to the operating room (OR). A complication requiring resuscitation occurs during surgery. What is the nurse's proper action? A) Call the legal department. B) Call the client's primary health care provider. C) Honor the DNR order. D) Resuscitaet per OR procedure.

Answer: C. According to the Association of Perioperative Registered Nurses, suspending a DNR order during surgery violates a client's right to self-determination.Calling the legal department or the client's health care provider is not an appropriate response. Resuscitating this client after a DNR has been signed is inappropriate.

A patient admitted with severe diarrhea is experiencing skin breakdown from frequent stools. What is an important comfort measure for this patient? A) Applying hydrocortisone cream B) Cleaning the area with soap and hot water C) Using sitz baths three times daily D) Wearing absorbent cotton underwear

Answer: C. An important comfort measure for a patient admitted with severe diarrhea experiencing skin breakdown is using sitz baths three times daily.Barrier creams, not hydrocortisone creams, may be used. The skin would be cleaned gently with soap and warm, not hot, water. Absorbent cotton underwear helps keep the skin dry but is not a comfort measure.

The nurse is instructing a client about the use of antiembolism stockings. Which statement by the client indicates the need for further teaching? A) "I will take off my stockings one to three times a day for 30 minutes." B) "My stockings are too loose." C) "It's better if they are too tight rather than too loose." D) "These stockings help promote blood flow."

Answer: C. Antiembolism stockings should fit properly to achieve the desired result. Stockings that are too tight will impede blood flow.Frequent removal of the stockings is appropriate to allow for hygiene and a break from their wear. Stockings that are too loose are ineffective. Antiembolism stockings may be used during and after surgery to promote venous return.

At 8:00 a.m., the registered nurse is admitting a client scheduled for sinus surgery to the outpatient surgery department. Which information given by the client is of most immediate concern to the nurse? A) An allergy to iodine and shellfish. B) Being nauseated after a previous surgery. C) Having a small glass of juice at 7:00 am. D) Expressing anxiety about the srugery.

Answer: C. Clients need to be NPO for a sufficient length of time before surgery to prevent aspiration of fluid into the lungs. Intake of food or fluids may delay the start time of the surgery, so the nurse must notify the surgeon and anesthesiologist for possible rescheduling.The nurse would confirm that all allergies are charted, and that the client has the correct allergy band identification. Many clients experience nausea after surgery; the nurse would document this in the client's information as well. The nurse would talk with the client and explore the anxiety; this is a normal feeling before surgery.

The nurse suspects that a client may have acute pancreatitis as evidenced by which group of laboratory results? A) Deceased calcium, elevated amylase, decreased magnesium B) Elevated bilirubin, elevated alkaline phosphatase C) Elevated lipase, elevated white blood cell (WBC) count, elevated glucose D) Decreased blood urea nitrogen (BUN), elevated calcium, elevated magnesium

Answer: C. Elevated lipase, along with increased WBC and increased glucose, suggests acute pancreatitis. Also, increased are serum amylase, serum trypsin, and serum elastase.Many pancreatic and nonpancreatic disorders can cause increased serum amylase levels. Bilirubin and alkaline phosphatase levels will be increased only if pancreatitis is accompanied by biliary dysfunction. Usually, calcium and magnesium will be increased and BUN increased, not decreased, in acute pancreatitis.

The outpatient clinic nurse is caring for a recovering client who had a colonoscopy. The client asks for a drink. How does the nurse respond to this request? A) "After I hear bowel sounds, you can have a drink." B) "Twenty minutes after the procedure was completed, you may have some liquids." C) "When you are able to pass flatus (gas), you can have a drink." D) "You can have fluids when you get home and are settled."

Answer: C. Fluids are permitted after the client's peristalsis has returned, which is validated by the client's passing flatus (p. 34).Ability to pass flatus (gas) is more reliable than auscultation of bowel sounds when assessing a client's status to drink after a colonoscopy. There is no set time period after the procedure that is considered safe for the client to have something to drink. The client will not be discharged home without the nurse determining that peristalsis has returned. The client must report that he or she is passing flatus to go home; therefore, the client should be given a drink before being sent home.

The nurse is assessing a client who has recurrent attacks of pancreatitis and is concerned about possible alcohol abuse as an underlying cause of these attacks. To elicit this information, what will the nurse do initially? A) Ask the client about binge drinking. B) Question the client whether drinking increases on weekends. C) Provide privacy and use the CAGE questionnaire (Cut down, Annoyed by criticism, Guilt about drinking, and Eye-opener drinks) D) Ask the client's spouse to describe the client's drinking

Answer: C. Initially, the nurse needs to provide privacy and establish a trusting relationship to help obtain information from the client about alcohol use. The CAGE questionnaire is useful as well.Topics such as binge drinking or tending to drink more on holidays or weekends may put the client on the defensive rather than provide the desired information. It has not yet been determined whether the client engages in binge drinking. Asking the client client's spouse will decrease nurse-client trust.

The nurse is caring for a postoperative client who had an extensive oral and neck surgery. The client is now describing throbbing pain in the head. The nurse anticipates that the health care provider will request which medication for this client? A) Diphenhydramine (Benadryl) B) Midazolam (Versed) intravenously C) Morphine sulfate intravenously D) Oxycodone plus acetaminophen (Percocet, Tylox)

Answer: C. Intravenous morphine sulfate is indicated for severe pain and is given initially. Clients undergoing surgery for oral cancer describe their pain as throbbing or pounding.Diphenhydramine is an anti-inflammatory agent and is not indicated for treatment of pain. Midazolam is used for conscious sedation and is not indicated for pain. Oxycodone/acetaminophen is given for systematic relief of moderate pain. This client may also have trouble swallowing.

Which substance, produced in the stomach, facilitates the absorption of vitamin B12? A) Glucagon B) Hydrochloric acid. C) Intrinsic factor. D) Pepsinogen

Answer: C. Intrinsic cells are produced by the parietal cells in the stomach. This substance facilitates the absorption of vitamin B12. Absence of intrinsic factor causes pernicious anemia.Glucagon, which is produced by the alpha cells in the pancreas, is essential for the regulation of metabolism. Parietal cells secrete hydrochloric acid, but this does not facilitate the absorption of vitamin B12. Pepsinogen is secreted by the chief cells; pepsinogen is a precursor to pepsin, a digestive enzyme.

A patient with a bowel obstruction is ordered a Salem sump nasogastric tube (NGT). After the nurse inserts the tube, which nursing intervention is the highest priority for this patient? A) Attaching the tube to low intermittent suction B) Auscultating for bowel sounds and peristalsis while the suction runs C) Connecting the tube to low continuous suction D) Flushing the tube with 30 mL of normal saline every 24 hours

Answer: C. Most patients with an obstruction have an NGT unless the obstruction is mild. A Salem sump tube is inserted through the nose and placed into the stomach. It is attached to low continuous suction unless otherwise requested by the primary health care provider. This tube has a vent (pigtail) that prevents the stomach mucosa from being pulled away during suctioning. This tube does not require intermittent suctionLevin tubes (no pigtail) do not have a vent and therefore should only be connected to low intermittent suction. They are used much less often than the Salem sump tubes. Bowel sounds would not be auscultated with suction on and running. After appropriate placement is established, the contents are aspirated and the tube is irrigated with 30 mL of normal saline every 4 hours or as requested by the primary health care provider.

Which intervention does the nurse implement for an older adult client to minimize skin breakdown related to surgical positioning? A) Apply elastic stockings to lower extremities. B) Monitr for excessive blood loss. C) Pad bony prominences. D) Secure joints on a board in anatomic positions.

Answer: C. Padding bony prominences best minimizes skin breakdown.Elastic stockings assist in increased venous return. Monitoring for blood loss and securing joints do not protect the skin.

After gastric surgery, a client arrives in the post-anesthesia care unit. Which nursing action is most appropriate for the RN to delegate to an experienced nursing assistant? A) Monitor respiratory rate and airway patency. B) Irrigate the nasogastric tube with saline. C) Position the client on the left side. D) Assess the client's pain level.

Answer: C. Positioning the client on the left side would most likely be delegated to an experienced, unlicensed care provider.Airway patency requires the care of a nurse in case of emergency management requirements. Irrigating the nasogastric tube with saline is a nursing skill and care by a nurse would be required. Pain assessment is also within the scope of a nurse.

Which problem for a client with cirrhosis takes priority? A) Insufficient knowledge related to the prognosis of the disease process B) Discomfort related to the progression of the disease process C) Potential for injury related to hemorrhage D) Inadequate nutrition related to an inability to tolerate usual dietary intake

Answer: C. Potential for injury related to hemorrhage is the priority client problem because this complication could be life threatening.Insufficient knowledge of the prognosis of the disease process, discomfort, and inadequate nutrition are not priorities because these issues are not immediately life threatening.

The nurse is providing instructions to a client who has a history of stomatitis. Which instructions does the nurse include in the client's teaching plan? A) Encourage the client to eat acidic foods to decrease bacteria. B) Mouth care should be performed twice daily. C) Rinse the mouth with warm saline or sodium bicarbonate. D) Use a medium-bristled toothbrush for oral care.

Answer: C. Rinsing the mouth with warm saline or sodium bicarbonate or a combination of the two decreases inflammation and pain.Acidic foods increase inflammation and should be avoided. Mouth care should be done after each meal and as often as needed. If stomatitis is not controlled, mouth care may have to be done every 2 hours or more frequently. A soft toothbrush not medium-bristled one would be used for oral care.

Which food does the nurse instruct a client undergoing chemotherapy for oral cancer with secondary stomatitis to avoid? A) Broiled fish. B) Ice cream. C) Salted pretzels. D) Scrambled eggs.

Answer: C. Salty foods like pretzels can further irritate ulcers in the client's mouth, causing pain.Cool or cold foods and foods high in protein, such as fish, eggs, and ice cream, may be included in the diet of the client with stomatitis.

The nurse is performing a dressing change on a client who underwent abdominal surgery 6 days prior. The nurse notes a moderate amount of serosanguineous drainage on the old dressing. What will the nurse do? A) Apply extra gauze to the new dressing. B) Contact the surgeon to discuss the need for antibiotics. C) Notify the surgeon about possible wound dehiscence. D) Perform the dressing change according to unit protocol.

Answer: C. Serosanguineous discharge persisting past the 5th postoperative day may indicate wound dehiscence and would be reported to the surgeon.The nurse would not just reinforce the dressing, but would notify the surgeon. Serosanguineous discharge does not indicate infection. Persistent serosanguineous discharge is an abnormal finding and to be reported.

A client has recently developed acute sialadenitis. Which intervention does the nurse include in this client's care? A) Applying cold compresses B) Avoiding the use of fruit or citrus-flavored candy C) Massaging the salivary gland D) Restrict fluids

Answer: C. Sialadenitis (sialoadenitis) is inflammation of a salivary gland. The salivary gland is massaged to stimulate the flow of saliva. This is done by milking the edematous gland with the fingertips toward the ductal opening.To promote the flow of saliva, warm, not cold, compresses are applied to the affected salivary gland. Sialagogues such as lemon slices and fruit- or citrus-flavored candy are used to stimulate the flow of saliva. The client is kept well hydrated to promote salivary flow.

A client has just undergone a surgical procedure with general anesthesia. Which finding indicates that the client needs further assessment in the post-anesthesia care unit? A) Pain at the surgical site. B) Requirement for verbal stimuli to awaken. C) Snoring sounds when inhaling. D) Sore throat on swallowing.

Answer: C. Snoring sounds when inhaling may indicate respiratory depression.Postsurgical pain at the surgical site is normal. Requiring verbal stimuli to awaken and a sore throat on swallowing are normal post-sedation.

Five RNs from other units have been assigned to the post-anesthesia care unit for the day. A 16-year-old client with diabetes has also just arrived from the operating room (OR) after having laparoscopic abdominal surgery. The charge nurse assigns the RN with which kind of experience to care for this new client? A) RN who usually works on the inpatient pediatric unit. B) RN who provides education to diabetic clients in a clinic. C) RN who has 5 years of experience in the delivery room. D) RN who ordinarily works as a scrub nurse in the OR

Answer: C. The RN with delivery room experience would have experience with abdominal surgery and with postoperative care of clients with diabetes, and would be aware of possible postoperative complications for this client.The RN who usually works on the pediatric unit would not be aware of potential complications and routine assessments for this client. The RN who provides education to diabetic clients in a clinic would be able to provide required care for the client's diabetes but not the postoperative aspect of care. The RN who works as a scrub nurse would not have the knowledge and understanding of routine postoperative care that is needed for this client.

A female client is concerned that her inability to conceive a child is connected to her morbid obesity. How does the nurse respond? A) "Do you feel that your obesity is keeping you from getting pregnant?" B) "Have you considered adoption as an option?" C) "Tell me about any changes in your menstrual cycle each month." D) "What has your health care provider told you about your problems in getting pregnant?"

Answer: C. The best response by the nurse is to ask the client who is concerned about her inability to conceive, is to ask her about changes in her menstrual cycle each month. Obesity has been known to produce changes in the menstrual cycle, thus causing difficulties in getting pregnant. Asking the client about her menstrual cycle directly addresses the client's concern and is designed to elicit helpful assessment information.Asking the client if she feels her obesity is keeping her from getting pregnant only asks the client to restate the obvious. It is also a closed question that requires only a "yes-or-no" response. Telling the client that adoption is an option is an intrusive response by the nurse and may alienate the client. It also does not address the client's concern about obesity. Asking what her health care provider told her is an evasive response from the nurse and does not address the client's concerns.

The nurse is performing a health assessment on an obese client who states, "I have tried many diets in an effort to lose weight, but have been unsuccessful." How does the nurse assess whether the client's response to stress is related to the client's obesity? A) "Do you have a history of mental problems, especially depression?" B) "Do you usually use alcohol or drugs when you feel stressed?" C) "Tell me what you do to relieve stress in your daily life." D) "What is it about your obesity that causes you to feel uncomfortable?"

Answer: C. The best way to assess a client's response to obesity and stress is to say, "Tell me what you do to relieve stress in your daily life." This open-ended type of question is best because it cannot be answered with a "yes" or "no."Asking the client about mental health problems will cause the client to feel uncomfortable with the assessment; problems in handling stress do not mean mental health or depression problems. More effective methods can be used to determine the client's alcohol and drug habits. Having the client tell you what makes him or her uncomfortable about obesity will only cause the client to restate the obvious. It does not determine the effect that stress has on the client.

An 87-year-old resident from an extended care facility has not been eating for several days and is admitted to the hospital with a diagnosis of malnutrition. She has an enteral feeding tube placed in her left nostril. Her medications include digoxin (Lanoxin), ranitidine (Zantac), and potassium chloride elixir (Kay Ciel). The nurse checks the gastric pH of the feeding tube and obtains a value of 6.0, which may indicate that the feeding tube is in the client's lungs. Is there another possible explanation for the nurse to consider? A) No; the feeding tube must be removed. B) No; the potassium effect will prevent the pH from reaching 6.0. C) Yes; the client is taking Zantac. D) Yes; the pH paper has expired and is giving a false reading.

Answer: C. The client may be taking the drug Zantac. The pH may be as high as 6.0 if the client takes certain medications, such as histamine2 blockers (e.g., ranitidine [Zantac], famotidine [Pepcid]). This finding, given the circumstances, does not mean that the tube is displaced and in the client's lungs.The NG tube does not have to be removed at this time. The potassium effect would not cause the pH to become more alkaline. Expired pH paper will provide no data that are reliable, so it would not be possible to have a reading of "6.0."

The nurse is teaching a middle-aged adult client with a body mass index (BMI) of 27.5 and a height of 5'2" (157.5 cm) about what the BMI number means, and about malnutrition. Which client statement indicates a need for further instruction A) "If I could get my BMI below 25, my risk for malnutrition would decrease." B) "I realize that this means that I have some increased health risks." C) "My goal should be to get my BMI below 18.5." D) "This means that I have an increased amount of total fat stored in my body."

Answer: C. The client statement showing a need for further instruction is, "My goal should be to get my BMI below 18.5." The least risk for malnutrition is associated with scores between 18.5 and 25.Older adults need to have a BMI between 23 and 27. The client with a BMI greater than 24.9 does have increased health risks that a client with a lower number would not have. The client's BMI of 27.5 does mean that an increased amount of fat is stored in the body in relation to the client's height.

The nurse completes the preoperative checklist on a client scheduled for general surgery. Which factor contributes the greatest risk for the planned procedure? A) Age 59 years. B) General anesthesia complications experienced by the client's brother. C) Diet-controlled diabetes mellitus. D) Ten pounds (4.5 kg) over the client's ideal body weight.

Answer: C. The client's greatest risk factor is diabetes mellitus. Diabetes contributes an increased risk for surgery or postsurgical complications.Older adults are at greater risk for surgical procedures, but this client is not classified as an older adult. Family medical history and problems with anesthetics may indicate possible reactions to anesthesia, but this is not the best answer. Obesity increases the risk for poor wound healing, but being 10 pounds (4.5 kg) overweight does not categorize this client as obese.

A young adult man says that he cannot stay on a diet because of trouble finding one that will incorporate his food preferences. How does the nurse most effectively plan nutritional care for this client? A) Calculates his body mass index (BMI) B) Records a 24-hour diary of his physical activities C) Obtains a 24-hour recall (diary) of his food intake D) Measures his accurate height and weight measurements

Answer: C. The most effective way to plan nutritional care for a client is to obtain a 24-hour recall of food intake. This will determine the client's food preferences and eating patterns so that they can be incorporated into the diet.Although calculating a BMI and measuring height and weight are important parts of a nutritional assessment, they do not address the issue of the client's food preferences. Keeping an activity diary will also not reveal any information related to the client's food preferences.

Which assessment finding in a postoperative client after general anesthesia requires immediate intervention? A) Heart rate of 58 beats/min. B) Pale, cool extremities. C) Respiratory rate of 6 breaths/min. D) Suppressed gag reflex.

Answer: C. The most immediate postoperative assessment is respiratory assessment, and a rate less than 10 breaths/min is too low.A heart rate of 58 beats/min, pale and cool extremities, and a suppressed gag reflex are all normal postoperative findings.

A patient with colorectal cancer is scheduled for colostomy surgery. Which comment from the nurse is most therapeutic for this patient? A) "Are you afraid of what your spouse will think of the colostomy?" B) "Don't worry. You will get used to the colostomy eventually." C) "Tell me what worries you the most about this procedure." D) "Why are you so afraid of having this procedure done?"

Answer: C. The most therapeutic comment by the nurse to a patient scheduled for colostomy surgery is "Tell me what worries you the most about this procedure." Asking the patient about what worries him or her is the only question that allows the patient to express fears and anxieties about the diagnosis and treatment.Asking the patient if he or she is afraid is a closed question (i.e., it requires only a "yes" or "no" response). It closes the dialogue and is not therapeutic. Telling the patient not to worry offers reassurance and is a "pat" statement, making it nontherapeutic. "Why" questions place patients on the defense and are not therapeutic because they close the conversation.

The nurse is teaching a class of older adults in the community about engaging in "regular" exercise. What does the nurse advise them? A) "One to two hours of cardiovascular exercise every day is a good idea." B) "Joining a fitness program or gym will help greatly with your exercise." C) "Walking 30 to 40 minutes provides the same benefit as long periods of exercise." D) "You will benefit most if you get into a group that shares your exercise goals."

Answer: C. The nurse advises the class of older adults to walk 30 to 40 minutes five days per week. Although some people think that regular exercise has to include joining a fitness program or exercising for long periods of time, simple forms of exercise like walking can provide the same type of benefit. Older adults can engage in this type of exercise which does not cost anything (unlike joining a program) and provides health benefits such as strengthening joints and improving cardiovascular health.One to two hours of cardiovascular exercise every day is not required to achieve benefits of exercise. Joining a gym is not necessary. In addition, many older adults have a fixed income and cannot afford memberships. A 30-minute walk can be accomplished with a group (such as "mall walking") or alone.

The nurse anesthetist notices that a surgical client has an unexpected rise in the end-tidal carbon dioxide level, with a decrease in oxygen saturation and sinus tachycardia. What is the nurse anesthetist's initial action? A) Administer cardiopulmonary resuscitation. B) Continue as normal. C) Immediately stop all inhalation anesthetic agents and succinylcholine. D) Inform the surgeon.

Answer: C. The nurse anesthetist's initial action is to stop all inhalation anesthetic agents and succinylcholine. This client is exhibiting early symptoms of malignant hyperthermia (MH). The most sensitive indication of MH is an unexpected rise in the end-tidal carbon dioxide level, along with a decrease in oxygen saturation. Another early indication is sinus tachycardia. Survival depends on early diagnosis and the actions of the entire surgical team. Time is crucial when MH is diagnosed, and MH requires immediate intervention.This client does not require resuscitation. Continuing as normal is inappropriate. Informing the surgeon is not the priority.

When assessing a client for hepatic cancer, the nurse anticipates finding an elevation in which laboratory test result? A) Hemoglobin and hematocrit. B) Leukocytes. C) Alpha-fetoprotein. D) Serum albumin

Answer: C. The nurse anticipates finding an elevation in the laboratory test for alpha-fetoprotein. Fetal hemoglobin (alpha-fetoprotein) is abnormal in adults, and is a tumor marker indicative of cancers.Although anemia may be present, elevated hemoglobin and hematocrit are not diagnostic of hepatic cancer. White blood cells (leukocytes) are not used to specifically diagnose cancers. Serum albumin levels may be low in liver cancer and in malnutrition.

The nurse is reviewing the medication history for a patient diagnosed with gastroesophageal reflux disease (GERD) who has been prescribed esomeprazole (Nexium) once daily. The patient reports that this proton pump inhibitor medication doesn't completely control the symptoms. The nurse contacts the primary health care provider to discuss which intervention? A) Adding a second proton pump inhibitor medication B) Increasing the dose of esomeprazole C) Changing to a twice-daily dosing regimen D) Switching to omeprazole (Prilosec)

Answer: C. The nurse contacts the primary health care provider about changing the Proton pump inhibitor to twice daily. These medications are usually effective when given once daily but can be given twice daily if symptoms are not well controlled.Adding a second medication, increasing the dose, or switching to another proton pump inhibitor is not recommended.

A patient with ulcerative colitis (UC) is prescribed sulfasalazine (Azulfidine) and corticosteroid therapy. As the disease improves, what change does the nurse expect in the patient's medication regimen? A) Corticosteroid therapy will be stopped. B) Sulfasalazine (Azulfidine) will be stopped. C) Corticosteroid therapy will be tapered. D) Sulfasalazine (Azulfidine) will be tapered.

Answer: C. The nurse expects that corticosteroid therapy will be tapered as the UC improves in the patient who was taking both sulfasalazine and corticosteroids. Once clinical improvement has been established, corticosteroids are tapered over a 2- to 3-month period.Stopping corticosteroid therapy abruptly is unsafe—steroids must be gradually decreased in patients. Usually the amount that they have been taking dictates how quickly or slowly they can be stopped. Sulfasalazine therapy will be taken on a long-term basis. It may be increased or decreased, depending on the patient's symptoms, but will likely never be stopped. These decisions are made over a long period of therapy.

An obese client is prescribed orlistat (Xenical). The client asks the nurse how the drug works. How does the nurse respond? A) "It decreases the amount of norepinephrine in your brain. This action will increase your feeling of being satisfied on less food." B) "It increases the amount of serotonin in your brain. This action will greatly increase your metabolic rate, and you will burn calories quicker." C) "It inhibits enzymes and changes the way your body digests fats. Because fats are only partially digested and absorbed, calorie intake is decreased." D) "It will alter the chemistry of your brain. Consequently, you will feel full before you overeat."

Answer: C. The nurse explains to the client that Orlistat inhibits lipase and leads to partial hydrolysis of triglycerides. Because fats are only partially digested and absorbed, calorie intake is decreased.Orlistat does not decrease the amount of norepinephrine in the brain, increase the amount of serotonin in the brain, or alter the chemistry of the brain.

Which of these assigned patients does the nurse assess first after receiving the change-of-shift report? A) Young adult admitted the previous day with abdominal pain who is scheduled for a computed tomography (CT) scan in 30 minutes B) Adult with gastroesophageal reflux disease (GERD) who is describing epigastric pain at a level of 6 (0-to-10 pain scale) C) Middle-aged adult with an esophagectomy done 2 days earlier who has bright-red drainage from the nasogastric (NG) tube D) Older adult admitted with an ileus who has absent bowel sounds and a prescription for metoclopramide (Reglan) on an as-needed (PRN) basis

Answer: C. The nurse must first assess the postoperative esophagectomy patient with bright red NG tube drainage. The presence of blood in NG drainage is an unexpected finding 2 days after esophagectomy and requires immediate investigation.The young adult scheduled for a CT scan, the adult with GERD, and the older adult with an ileus are all stable and do not require the nurse's immediate attention.

A client is having an arthroscopy of the left knee and has just been moved to the surgical holding area. Which statement by the nurse properly identifies the client while the nurse checks the identification label? A) "Are you Mr. Smith?" B) "Good morning, Mr. Smith." C) "What is your name, and when were you born?" D) "What surgery are you having today?"

Answer: C. The nurse must verify the client's identity with two types of identifiers, such as name and birthdate. This practice prevents errors by drowsy or confused clients.When asked to verify his or her name, or respond to a greeting, the client may respond inappropriately if he or she is anxious or sedated. Asking the client about his or her surgery does help with identification. However, it is really done to ascertain that the client's perception of the procedure, the operative permit, and the operative schedule are the same.

A preoperative client wears a hearing aid and is extremely hard of hearing without it. What does the nurse do to help reduce this client's anxiety? A) Actively listen to this client's concerns. B) Allow the client to wear the hearing aid to surgery. C) Ask if the client may wear the hearing aid until anesthesia is given. D) Explain that it is hospital policy to remove a hearing aid before surgery.

Answer: C. The nurse needs to ask if the client can wear the hearing aid to the operating room (OR). In some facilities, clients may wear eyeglasses and hearing aids until after anesthesia induction.Listening isn't always enough; more intervention is needed. Sending the client to the OR with the hearing aid without checking first is inappropriate. The OR staff may have a different policy, considering that the hearing aid may get lost. Telling the client that a policy precludes the client's needs is not therapeutic.

A patient returns to the unit after having an exploratory abdominal laparotomy. How does the nurse position this patient after the patient is situated in bed? A) High Fowler's. B) Lateral Sims' (side-lying) C) Semi-Fowler's. D) Supine.

Answer: C. The nurse places the postoperative abdominal laparotomy patient in the semi-Fowler's position in bed. The patient is maintained in this position to facilitate the drainage of peritoneal contents into the lower region of the abdominal cavity after an abdominal laparotomy. This position also helps increase lung expansion.High-Fowler's position would be too high for the patient postoperatively. It would place strain on the abdominal incision(s), and, if the patient was still drowsy from anesthesia, this position would not enhance the patient's ability to rest. Sims' position does not promote drainage to the lower abdomen. The supine position does not facilitate drainage to the abdomen or increased lung expansion. The patient would be more likely to develop complications (wound drainage stasis and atelectasis) in the supine position.

A patient at risk for colorectal cancer asks the nurse, "Can you tell me some foods to include in my diet so that I can reduce my chances of getting this disease?" Which dietary selection does the nurse suggest? A) Steak with pasta. B) Spaghetti with tomato sauce C) Steamed broccoli with turkey D) Tuna salad with wheat crackers

Answer: C. The nurse suggests steamed broccoli and turkey to the patient who wants to know what foods to include in his/her diet to reduce the chance of getting colorectal cancer.Animal fat from red meats is carcinogenic, and pasta is high in refined carbohydrates, which are known to contribute to colon cancer. Spaghetti and wheat crackers also contain large amounts of refined carbohydrates.

An older malnourished client who is taking digoxin (Lanoxin), ranitidine (Zantac), and potassium chloride elixir (Kay Ciel) develops a severe case of diarrhea. What does the nurse suspect is a possible cause? A) Digoxin (Lanoxin) B) Gastritis C) Potassium chloride (Kay Ciel) D) Ranitidine (Zantac)

Answer: C. The nurse suspects that potassium chloride may be the possible cause. In some cases, Potassium chloride (Kay Ciel) may cause diarrhea. This may be the result of liquid medications such as elixirs and suspensions that have a very high osmolality.Diarrhea is not a frequent side effect of digoxin or ranitidine (Zantac). Gastritis does not cause diarrhea, and the other signs and symptoms of gastritis are not mentioned in this scenario.

What teaching does the home health nurse give the family of a client with hepatitis C to prevent the spread of the infection? A) The client must not consume alcohol. B) Avoid sharing the bathroom with the client. C) Members of the household must not share toothbrushes. D) Drink only bottled water and avoid ice.

Answer: C. The nurse teaches the family of a client with Hepatitis C that toothbrushes, razors, towels, and any other items may spread blood and body fluids and must not be shared.The client should not consume alcohol, but abstention will not prevent spread of the virus. The client may share a bathroom if he or she is continent. To prevent hepatitis A when traveling to foreign countries, bottled water should be consumed and ice made from tap water needs to be avoided.

A preoperative client smokes a pack of cigarettes a day. What is the nurse's teaching priority for the best physical outcomes? A) Instruct the client to quit smoking. B) Teach about the dangers of tobacco. C) Teach the importance of incentive spirometry. D) Tell the client that smoking increases postoperative complications.

Answer: C. The nurse would first teach the importance of incentive spirometry. Incentive spirometry is good for lung hygiene and it encourages deep breathing.The nurse can suggest quitting or advice about the dangers of tobacco, but it is not therapeutic to instruct it at this time. Telling the client that smoking causes increased complications is not helpful or therapeutic just prior to surgery.

The nurse is assessing a client who has come to the emergency department with acute abdominal pain. The client is very thin and the nurse observes visible peristaltic movements when inspecting the abdomen. What does the nurse suspect? A) Acute diarrhea. B) Aortic aneurysm. C) Intestinal obstruction. D) pancreatitis.

Answer: C. The nurse would suspect an intestinal obstruction related to peristaltic movements. Peristaltic movements are rarely seen except in thin clients. This needs to be reported to the HCP.Acute diarrhea does not cause visible peristaltic movements. Aortic aneurysm may cause a bulging, pulsatile mass. Pancreatitis is characterized by severe pain.

A client diagnosed with acalculous cholecystitis asks the nurse how the gallbladder inflammation developed when there is no history of gallstones. What is the nurse's best response? A) "This may be an indication that you are developing sepsis." B) "The gallstones are present, but have become fibrotic and contracted." C) "This type of gallbladder inflammation is associated with hypovolemia." D) "This may be an indication of pancreatic disease."

Answer: C. The nurse's best response about acalculous cholecystitis is that "This type of gallbladder inflammation is associated with hypovolemia."Although this type of gallbladder inflammation is associated with sepsis, it is not an indicator that sepsis is developing. Fibrotic and contracted gallstones are associated with chronic cholecystitis and this scenario states that there is no history of gallstones. The presence of acalculous cholecystitis is not an indicator that pancreatic disease has developed.

A client has been discharged to home after being hospitalized with an acute episode of pancreatitis. The client, who is an alcoholic, is unwilling to participate in Alcoholics Anonymous (AA), and the client's spouse expresses frustration to the home health nurse regarding the client's refusal. What is the nurse's best response? A) "Your spouse will sign up for the meetings only when he is ready to deal with his problem." B) "Keep mentioning the AA meetings to your spouse on a regular basis." C) "I'll get you some information on the support group Al-Anon." D) "Tell me more about your frustration with your spouse's refusal to participate in AA."

Answer: C. The nurse's best response involves putting the client's spouse in contact with an Al-Anon support group. This action may help with the spouse's frustration and help both to cope with the situation.Telling the spouse that the client will sign up for AA meetings when the client is ready and telling the spouse to keep mentioning AA do not address the spouse's frustration with the client's refusal to participate in AA. Encouraging the spouse to say more about his or her frustration may allow the spouse to vent frustration, but it does not offer any options or solutions.

The nurse finds a patient vomiting coffee-ground emesis. On assessment, the patient has a blood pressure of 100/74 mm Hg, is acutely confused, and has a weak and thready pulse. Which intervention is the nurse's first priority? A) Administering a histamine2 (H2) antagonist B) Initiating enteral nutrition C) Administering intravenous (IV) fluids D) Administering antianxiety medication

Answer: C. The nurse's first priority is to administer intravenous (IV) fluids. Administering IV fluids is necessary to treat the hypovolemia caused by acute gastrointestinal (GI) bleeding.Administration of an H2 antagonist will not treat the basic problem, which is upper GI bleeding. Enteral nutrition will not be part of the treatment plan for acute GI bleeding. Administration of antianxiety medication will not treat the acute confusion. The patient's change in mental status is due to hypovolemia caused by acute GI bleeding.

The nurse is teaching a patient how to prevent recurrent chronic gastritis symptoms before discharge. Which statement by the patient demonstrates a correct understanding of the nurse's instruction? A) "It is okay to continue to drink coffee in the morning when I get to work." B) "I will need to take vitamin B12 shots for the rest of my life." C) "I should avoid alcohol and tobacco." D) "I should eat small meals about six times a day."

Answer: C. The patient's statement that he/she needs to avoid alcohol and tobacco shows that the patient correctly understands the nurse's instructions. The patient with chronic gastritis should avoid alcohol and tobacco.The patient also needs to eliminate caffeine from the diet. The patient will need to take vitamin B12 shots only if he/she has pernicious anemia. The patient would also not eat six small meals daily. This practice may actually stimulate gastric acid secretion.

The home health nurse is teaching a patient about the care of a new colostomy. Which patient statement demonstrates a correct understanding of the instructions? A) "A dark or purplish-looking stoma is normal and would not concern me." B) "If the skin around the stoma is red or scratched, it will heal soon." C) "I need to check for leakage underneath my colostomy." D) "I need to strive for a very tight fit when applying the barrier around the stoma."

Answer: C. The patient's statement, "I need to check for leakage underneath my colostomy" shows that the patient correctly understands the instructions about how to care for a new colostomy. The pouch system must be checked frequently for evidence of leakage to prevent excoriation.A purplish stoma is indicative of ischemia and necrosis. Redness or scratched skin around the stoma must be reported to prevent it from beginning to break down. An overly tight fit may lead to necrosis of the stoma.

A patient with an inoperable esophageal tumor is receiving swallowing therapy. Which task does the home health nurse delegate to an experienced home health aide? A) Teaching family members how to determine whether the patient is obtaining adequate nutrition B) Assessing lung sounds for possible aspiration when the patient is swallowing clear liquids C) Reminding the patient to use the chin-tuck technique each time the patient attempts to swallow D) Instructing family members about symptoms that may indicate a need to call the primary health care provider

Answer: C. The role of a home health aide when caring for a patient with swallowing difficulty includes reinforcement of previously taught swallowing techniques.Teaching and providing instructions to family members are not within the scope of practice of a home health aide and would be done by the home health nurse. Likewise, assessing lung sounds is part of the nursing process and would be done by the nurse.

During a preoperative assessment, which statement by a client requires further investigation by the nurse to assess surgical risks? A) "I am taking vitamins." B) "I drink a glass of wine a night." C) "I had a heart attack 4 months ago." D) "I quit smoking 10 years ago."

Answer: C. The statement by the client that he or she had a heart attack 4 months ago requires further investigation. Cardiac problems increase surgical risks, and the risk for a myocardial infarction during surgery is higher in clients who have heart problems.The type of vitamins the client takes should be assessed, but this is not the highest risk. Moderate alcohol consumption is not considered high-risk behavior. A past history of smoking should be noted, but current or more recent smoking is of greater concern.

A client receiving total parenteral nutrition (TPN) exhibits symptoms of congestive heart failure (CHF) and pulmonary edema. Which complication of TPN is the client most likely experiencing? A) Calcium imbalance B) Fluid volume deficit C) Fluid volume overload D) Potassium imbalance

Answer: C. This client is most likely experiencing fluid volume overload. CHF and pulmonary edema are symptoms of this condition.Calcium imbalance, fluid volume deficit, and potassium imbalance do not manifest with CHF and pulmonary edema.

If sterile gauze falls to the ground and hits the front of the surgeon's gown on the way down, what does the nurse do to ensure proper infection control? A) Helps the surgeon change the gown. B) Picks the gauze up with a pair of sterile gloves. C) Picks the gauze up without touching the surgeon. D) Sprays an antimicrobial on the surgeon's gown.

Answer: C. To ensure proper infection control, the nurse picks up the gauze without touching the surgeon. The surgeon is sterile, but the gauze is now nonsterile and must be removed and counted.A sterile gauze touching a sterile gown does not require a gown change. Sterile gloves are not needed to pick up the gauze. An antimicrobial spray is inappropriate in this situation.

The nurse is teaching a client with gallbladder disease about diet modification. Which meal does the nurse suggest to the client? A) Steak and French fries. B) Fried chicken and mashed potatoes C) Turkey sandwich on wheat bread. D) Sausage and scrambled eggs

Answer: C. Turkey is an appropriate low-fat selection for this client. High fiber, from the wheat bread, also helps reduce the risk. Typically, diets high in fat, high in calories, low in fiber, and high in refined white carbohydrates place clients at higher risk for developing gallstones.Steak, French fries, fried chicken and mashed potatoes, and sausage are too fatty. Eggs are too high in cholesterol for a client with gallbladder disease.

The nurse manager in a long-term care facility plans nutritional assessments of all residents. Which nutritional assessment activity does the nurse delegate to unlicensed assistive personnel (UAP) at the facility? A) Assessing residents' abilities to swallow B) Determining residents' functional status C) Measuring the daily food and fluid intake of residents D) Screening a portion of the residents with the Mini Nutritional Assessment

Answer: C. UAP education includes measurement of clients' oral intake. This skill does not require specific clinical judgment.Assessing swallowing ability, determining functional status of residents, and screening with the Mini Nutritional Assessment all require clinical and assessment skills of a licensed professional nurse.

A home health patient has had severe diarrhea for the past 24 hours. Which nursing action does the RN delegate to the home health aide (unlicensed assistive personnel [UAP]) who assists the patient with self-care? A) Instructing the patient about the use of electrolyte-containing oral rehydration products B) Administering loperamide (Imodium) 4 mg from the patient's medicine cabinet C) Checking and reporting the patient's heart rate and blood pressure in lying, sitting, and standing positions D) Teaching the patient how to clean the perineal area after each loose stool

Answer: C. The RN delegates to the UAP a home health patient with severe diarrhea who needs checking and reporting of the patient's heart rate and blood pressure in lying, sitting, and standing positions. Obtaining the patient's blood pressure and heart rate is included in the education of home health aides and other UAPs.Patient teaching and medication administration are complex skills that would be performed by licensed nurses who have the education and scope of practice needed to safely implement these actions.

A patient is being evaluated in the emergency department (ED) for a possible small bowel obstruction. Which signs/symptoms does the nurse expect to assess? A) Cramping intermittently, metabolic acidosis, and minimal vomiting B) Intermittent lower abdominal cramping, obstipation, and metabolic alkalosis C) Metabolic acidosis, upper abdominal distention, and intermittent cramping D) Upper abdominal distention, metabolic alkalosis, and a great amount of vomiting

Answer: D. A small bowel obstruction is characterized by upper or epigastric abdominal distention, metabolic alkalosis, and a great amount of vomiting.Intermittent lower abdominal cramping and metabolic acidosis are all symptoms of a large bowel obstruction.

A nurse is monitoring a client who received succinylcholine during a surgical procedure. Which of the following actions should the nruse take if the client develops manifestations of malignant hypertermia? A) Give IV atropine. B) Institute seizure precautions. C) Remove endotracheal tube. D) Administer dantrolene.

Answer: D. A) A client who has malignant hyperthermia should manifest tachycardia. Therefore, administering atropine is not recommended because tachycardia is an adverse effect of atropine. B) Seizures are not a manifestation of malignant hyperthermia. C) A client who has malignant hyperthermia should have the endotracheal tube maintained for airway management during the crisis. D) The nurse should administer dantrolene by IV bolus at 2 to 5 mg/kg to reverse the manifestations for a client who has malignant hyperthermia.

A nurse is caring for a client who has GERD and a new prescription for metoclopramide. The nurse should plan to monitor for which of the following adverse effects? A) Hypersalivation. B) Hearing loss. C) Thrombocytopenia. D) Ataxia.

Answer: D. A) An adverse effect of metoclopramide is xerostomia (dry mouth), not hypersalivation. B) Gentamicin and many other medications are ototoxic. However, metoclopramide does not cause hearing loss. C) An adverse effect of low-molecular-weight heparins, such as enoxaparin, is thrombocytopenia. However, thrombocytopenia is not an adverse effect of metoclopramide. D) The nurse should plan to monitor the client for extrapyramidal symptoms, such as ataxia, and should report any of these findings to the provider.

A nurse is providing preoperative teaching to a client who is scheduled for a gastrectomy in 1 week. The client is anxious about the upcoming surgery. Which of the following actions should the nurse take? A) Reassure the client that the surgery will go fine. B) Change the topic of discussion. C) Sympathize with the client's feelings. D) Provide concise, factual information.

Answer: D. A) Giving the client false reassurance is a nontherapeutic communication technique because it discourages continued communication and does not address the client's anxiety. B) Changing the topic shows a lack of empathy for the client and blocks further communication, which can increase the client's anxiety. C) Sympathizing with the client's feelings is nontherapeutic because the nurse over-identifies with the client and loses objectivity, which can impair judgment and prevent problem solving. D) Providing concise, factual information allows for open communication and gives the nurse the opportunity to dispel the client's fears.

A nurse is assessing a client who has a duodenal ulcer. Which of the following findings should the nurse expect? A) The client states that the pain is in the upper epigastrium. B) The client is malnourished. C) The client states that ingesting food intensifies the pain. D) The client reports that pain occurs during the night.

Answer: D. A) Pain associated with a duodenal ulcer is typically located below or to the right of the epigastrium. B) Typically, the client who has a duodenal ulcer is not malnourished. C) Typically, the client who has a duodenal ulcer reports that ingesting food diminishes his pain level. D) Pain associated with a duodenal ulcer occurs when the stomach is empty, which is typically 1.5 to 3 hr after meals and during the night.

A nurse is providing discharge teaching for an older adult client who has mild diverticulitis. Which of the following statements by the client indicates an udnerstanding of the teaching? A) "I will use a mild laxative every day." B) "I will remain active by working in my garden every day." C) "I may experience right lower quadrant pain." D) "I should eat foods that are low in fiber."

Answer: D. A) The client who has diverticulitis should avoid laxatives, which increase intestinal motility and can exacerbate the adverse effects of diverticulitis. B) The client who has diverticulitis should refrain from any activity that increases intra-abdominal pressure because this can result in the perforation of the diverticula. C) Left lower quadrant pain is an expected finding in a client who has diverticulitis. D) The nurse should instruct the client who has diverticulitis to follow a low-fiber diet. When the inflammation subsides, the client should consume foods that are high in fiber.

A nurse is assessing a client who is 2 hr postoperative following an appendectomy. Which of the following findings should the nurse report to the provider? A) WBC 9,000 mm3. B) A 2 cm x 2 cm (0.79 in x 0.79 in) area of bloody drainage on the dressing C) Temperature of 36.5° C (97.7° F) D) Urine output of 20 mL/hr

Answer: D. A) The client's WBC is within the expected reference range; therefore, the nurse does not need to report this finding to the provider. However, the nurse should report a WBC below or above the expected reference range. B) The nurse should expect the client to have a small amount of bloody drainage on the dressing following an appendectomy. However, the nurse should report an increased amount of bloody and/or purulent drainage. C) The client's temperature is within the expected reference range. However, the nurse should report a temperature lower than 36° C (98.5° F), which is an indicator of hypothermia, or a temperature higher than the expected reference range, which is an indicator of an infection. D) The nurse should notify the provider if the client's urine output is less than 30 mL/hr. Decreased output can indicate hypovolemia and decreased perfusion of the kidneys.

A nurse is providing teaching to a client who is schedued to have a mastectomy with reconstructive surgery. Which of the following statements by the client indicates an understanding of the teaching? A) "I should wait a week after surgery to start my hand-strengthening exercises." B) "I should wait to take my pain medication until after I have completed my range-of-motion exercises." C) "I will be able to lift up an object that weighs 10 pounds 2 weeks after my surgery." D) "I will be able to shower after the doctor removes the drain."

Answer: D. A) The nurse should encourage the client to begin hand-strengthening exercises, such as squeezing a ball, the first day after surgery. B) If the client experiences pain while doing range-of-motion exercises, she should take pain medication 30 min prior to exercise. C) The client should avoid lifting objects that weigh 2.26 to 4.54 kg (5 to 10 lb) for at least 4 weeks after surgery. D) A client who had a mastectomy with reconstructive surgery can shower after the provider removes the drain.

A nurse is providing teaching for a client who has cirrhosis and a new prescription for lactulose. The nurse should include which of the following instructions in the teaching? A) Notify the provider if bloating occurs. B) Restrict carbohydrates in the diet. C) Limit oral fluid intake to 1,000 mL per day of clear liquids. D) Expect to have two to three soft stools per day.

Answer: D. A) The nurse should instruct the client that bloating, flatulence, and belching are adverse effects of lactulose and that it is not necessary to notify the provider about these adverse effects. B) The nurse should instruct the client to follow a diet that is high in carbohydrates and protein because clients who have cirrhosis are at risk for malnutrition. C) Dehydration can result from increased stool frequency. The nurse should instruct the client to maintain an adequate fluid intake to offset the dehydrating effects of the medication. D) The purpose of administering lactulose is to promote the excretion of ammonia in the stool. The nurse should instruct the client to take the medication every day and inform the client that two to three bowel movements every day is the treatment goal.

A nurse is teaching a client how to prepare for a colonoscopy. Which of the following instructions should the nurse include in the teaching? A) Drink the oral liquid preparation for bowel cleansing slowly. B) Drink full liquids for breakfast the day of the procedure, and then take nothing by mouth for 2 hr prior to the procedure. C) Begin drinking the oral liquid preparation for bowel cleansing on the morning of the procedure. D) Drink clear liquids for 24 hr prior to the procedure, and then take nothing by mouth for 6 hr before the procedure.

Answer: D. A) The nurse should instruct the client to drink the oral liquid preparation quickly to prevent nausea. B) The nurse should instruct the client to take nothing by mouth except water for 4 to 6 hr prior to the procedure. C) The nurse should instruct the client to begin drinking the oral liquid preparation the day before the colonoscopy to ensure adequate time for bowel cleansing. D) The nurse should instruct the client to drink clear liquids for 24 hr prior to the colonoscopy to promote adequate bowel cleansing. Maintaining NPO status for 4 to 6 hr prior to the colonoscopy preserves the bowel's cleansed state.

A nurse is caring for a client who is postoperative following abdominal surgery. Which of the following nursing interventions should the nurse perform to prevent respiratory complications? A) Instruct the client to exhale into the incentive spirometer every 1 to 2 hr. B) Reposition the client every 8 hr for the first 48 hr. C) Minimize the amount of pain medication the client receives to prevent sedation. D) Advise the client to splint the surgical incision when coughing and deep breathing.

Answer: D. A) The nurse should instruct the client to inhale deeply from the incentive spirometer and hold his breath to expand the lungs. B) The nurse should reposition the client at least every 2 hr until the client is ambulating frequently. C) The nurse should administer pain medication as needed to allow the client to increase depth of breathing. An increased pain level increases the risk of altered gas exchange and can lead to respiratory complications. D) Splinting the incision supports the surgical site and decreases pain during coughing and deep breathing.

A nurse is providing discharge instructions for a client who is postoperative following abdominal surgery. Which of the following client statements indicates an understanding of the teaching? A) "I should avoid taking over-the-counter pain medication if my pain is not severe." B) "I will have an increase in yellow-colored drainage from my incision for 2 weeks." C) "I will remain on bed rest until my follow-up appointment with my doctor." D) "I will eat foods that are high in protein and vitamin C during my recovery."

Answer: D. A) The nurse should instruct the client to use over-the-counter analgesics for mild to moderate pain unless contraindicated by the provider. B) The nurse should instruct the client to report an increase in serous drainage after the first few days following surgery. An increase in drainage can indicate infection. C) The nurse should instruct the client to increase activity gradually to reduce the risk for infection and development of deep-vein thrombosis. D) The nurse should instruct the client to increase intake of foods with protein and vitamin C to promote wound healing.

A nurse is assessing a client who is 2 days postoperative following a total prostatectomy. The nurse notes that the client's right calf is red, edematous, and warm to the touch. Which of the following actions should the nurse take? A) Gently massage the client's right calf. B) Administer testosterone to the client. C) Apply an ice pack to the client's right calf. D) Elevate the client's right extremity.

Answer: D. A) The nurses should avoid massaging the client's right calf because it could dislodge the thrombus and result in injury. B) This client does not require testosterone. The nurse should use anticoagulants to treat deep-vein thrombosis. C) Applying an ice pack to the client's right calf can result in injury because it causes vasoconstriction. The nurse should apply warm, moist packs to the client's right calf. D) These findings suggest the client has deep-vein thrombosis. The nurse should keep the client's right extremity elevated to promote venous return.

Following paracentesis, during which 2500 mL of fluid was removed, which assessment finding is most important to communicate to the health care provider (HCP)? A) The dressing has a 2-cm area of serous drainage. B) The client's platelet count is 135,000/mm3 (135 × 109/L). C) The client's albumin level is 2.8 g/dL (28 g/L). D) The client's heart rate is 122 beats/min.

Answer: D. After a paracentesis with 2500 ml of fluid removed, the assessment finding of the client's heart rate is the most important finding to communicate to the HCP. Rapid removal of fluid may cause symptoms of shock, including tachycardia, and are especially associated with hypotension.A small amount of serous fluid may leak, so the dressing would be reinforced. Platelets will be checked before the procedure. These are slightly low, but this is not a cause for concern. An albumin level of 2.8 g/dL (28 g/L) is an expected finding for a client with cirrhosis and is not life threatening.

Which client on the medical-surgical unit does the charge nurse assign to the LPN/LVN? A) A 28-year-old with morbid obesity who had bariatric surgery today B) A 30-year-old recently admitted with severe diarrhea and Clostridium difficile infection C) A 36-year-old whose family needs instruction about how to use a gastric feeding tube D) A 39-year-old with a jejunal feeding tube who needs elemental feedings administered

Answer: D. An elemental feeding is a diet that proposes the ingestion of liquid nutrients in an easily assimilated form. LPN/LVN education includes administration of elemental tube feedings and associated client care and monitoring.Initial assessment of a postoperative client, a new admission, and client and family teaching all require RN education and scope of practice.

Which nursing care activity for a malnourished client does the nurse safely delegate to unlicensed assistive personnel (UAP)? A) Completing the Mini Nutritional Assessment B) Determining body mass index (BMI) C) Estimating body fat using skinfold measurements D) Measuring current height and weight

Answer: D. Determining height and weight is the only activity that the nurse can safely delegated to UAP.The nurse is responsible for completing the Mini Nutritional Assessment, determining the client's BMI, and estimating body fat using skinfold measurements.

A client with acute cholecystitis is admitted to the medical-surgical unit. Which nursing activity associated with the client's care will be appropriate for the nurse to delegate to unlicensed assistive personnel (UAP)? A) Assessing dietary risk factors for cholecystitis B) Checking for bowel sounds and distention C) Determining precipitating factors for abdominal pain D) Obtaining the admission weight, height, and vital signs

Answer: D. Obtaining admission height, weight, and vital signs is included in the education for UAPs and usually is included in the job description for these staff members.Assessing for risk factors, checking bowel sounds, and determining precipitating factors for abdominal pain require assessment skills. Assessment skills require broader education and are within the scope of practice of licensed nursing staff and not UAPs.

Which set of assessment findings indicates to the nurse that a client may have acute pancreatitis? A) Absence of jaundice, pain of gradual onset B) Absence of jaundice, pain in right abdominal quadrant C) Presence of jaundice, pain worsening when sitting up D) Presence of jaundice, pain worsening when lying supine

Answer: D. Pain that worsens when lying supine and the presence of jaundice are the only assessment findings indicative of acute pancreatitis.Pain associated with acute pancreatitis usually has an abrupt onset, is located in the mid-epigastric or upper left quadrant, and lessens with sitting up. Also, jaundice is present.

Which statement by a nursing student indicates a need for further teaching about operating room (OR) surgical attire? A) "I must cover my facial hair." B) "I don't need a sterile gown to be in the OR." C) "If I go into the OR, I must wear a protective mask." D) "My scrubs will be sterile."

Answer: D. Scrub attire is provided by the hospital and is clean, not sterile.All members of the surgical team must cover their hair, including any facial hair. Team members who are not scrubbed (e.g., anesthesia provider, student nurse) are not required to be sterile and may wear cover scrub jackets that are snapped or buttoned closed to prevent shedding of organisms from bare arms. Everyone who enters an OR in which a sterile field is present must wear a mask.

A 21-year-old with a stab wound to the abdomen has come to the emergency department (ED). Once stabilized, the patient is admitted to the medical-surgical unit. What does the admitting nurse do first for this patient? A) Administer pain medication. B) Assess skin temperature and color. C) Check on the amount of urine output. D) Take vital signs.

Answer: D. The admitting nurse needs to first take the vital signs of a patient who was just transferred from the ED with a stab wound to the abdomen. Assessment of vital signs must be done first to determine the adequacy of the airway and circulation. Vital signs initially reveal the most about the patient's condition.The patient would not be medicated for pain until his or her alertness level is determined. Skin temperature and color are not specifically indicative of the patient's overall condition. If the patient is in shock, urine output will be scant and will not be an accurate assessment variable.

Which staff member will be best for the nurse manager to assign to update standard nursing care plans and policies for care of the client in the operating room (OR)? A) Surgical technologist with 10 years of experience in the OR at this hospital. B) Certified registered nurse first assistant (CRNFA) who has worked for 5 years in the ORs of multiple hospitals. C) Holding room RN who has worked in the hospital holding room for longer than 15 years. D) Circulating RN who has been employed in the hospital OR for 7 years

Answer: D. The circulating RN is the best staff member for the nurse manager to assign. This nurse has the experience and background to write OR policy, has been employed in the hospital for 7 years, and is aware of hospital policy and procedures.A surgical technologist does not have the background to write policy for nurses. A CRNFA has worked in multiple hospitals but does not have a work history with this specific hospital to be aware of the unit policy. A holding room or preoperative or postoperative care nurse would not be the choice to write OR policy.

Based on nutritional screening findings and assessments, which client will be the preferred candidate for surgical treatment for obesity? A) Man with a body mass index (BMI) of 40, weight 75% above ideal body weight B) Man with a BMI of 41, weight 80% above ideal body weight C) Woman with a BMI of 38, weight 50% above ideal body weight D) Woman with a BMI of 42, weight 100% above ideal body weight

Answer: D. The client who will be most successful with surgical intervention is the client with a BMI of 40 or more and a weight 100% above the ideal body weight.The other clients do not have a high enough BMI-to-weight ratio to be considered for surgical intervention.

The charge nurse for a hospital operating room is making client assignments for the day. Which client is most appropriate to assign to the least-experienced circulating nurse? A) The 20-year-old client who has a ruptured appendix and is having an emergency appendectomy B) The 28-year-old client with a fractured femur who is having an open reduction and internal fixation C) The 45-year-old client with coronary artery disease who is having coronary artery bypass grafting D) The 52-year-old client with stage I breast cancer who is having a tunneled central venous catheter placed

Answer: D. The client with stage I breast cancer who is having a tunneled central venous catheter placed is the most stable client among all scheduled procedures. This assignment would be appropriate for the beginning nurse or one with less experience.The client who has a ruptured appendix is less stable and at high risk for infection/sepsis; a more experienced nurse is required. The client with a fractured femur is at high risk for clotting, infection, and aspiration owing to the surgery; a more experienced nurse would be better. The client with coronary artery disease is having high-risk surgery with risk for multiple complications and requires an experienced operating room nurse.

The nurse is educating a client who is about to undergo cardiac surgery with general anesthesia. Which statement by the client indicates the need for further instruction? A) "I will wake up with a tube in my throat." B) "I will have a bandage on my chest." C) "My family will not be able to see me right away." D) "Pain medication will take away my pain."

Answer: D. The client's statement that, "Pain medication will take away my pain," indicates the need for further instruction. Pain medication will reduce pain, but will not take it away completely.The client statement about waking up with a tube in the throat is accurate, because the client will be intubated. Following heart surgery, a dressing is placed on the chest. The client will not be able to see family immediately because he or she will go to recovery first.

Which statement by a client with cirrhosis indicates that further instruction is needed about the disease? A) "Cirrhosis is a chronic disease that has scarred my liver." B) "The scars on my liver create problems with blood circulation." C) "Because of the scars on my liver, blood clotting and blood pressure are affected." D) "My liver is scarred, but the cells can regenerate themselves and repair the damage."

Answer: D. The client's statement that, although his liver is scarred, the cells can regenerate and repair the damage indicates that further instruction is needed. Although cells and tissues will attempt to regenerate, destroyed liver cells will result in permanent scarring and irreparable damage.Cirrhosis is a chronic condition that leaves scars on the liver. Permanent scars form in response to attempts by the cells to regenerate and create problems in blood circulation moving through the liver. Liver scarring will create problems with blood clotting, cholesterol levels, and blood pressure, as well as with the metabolism of drugs and toxins.

After a colonoscopy, a client reports severe abdominal pain. The nurse obtains these data: temperature 100.2°F (37.9°C), pulse 122 beats/min, blood pressure 100/45 mm Hg, respirations 44 breaths/min, and O2 saturation 89%. Which request from the health care provider does the nurse implement first? A) Give cefazolin (Ancef) 500 mg IV. B) Infuse normal saline at 200 mL/hr. C) Give morphine sulfate 2 mg IV. D) Provide oxygen at 6 L/min per nasal cannula.

Answer: D. The first request the nurse complies with is to place the client on oxygen. This is the most immediate concern because it involves the client's respiratory status. Based on the data given, the client may be experiencing complications of colonoscopy such as bleeding or perforation.An antibiotic request is important but is not the first priority. Fluid supplementation is important, but the client's oxygen saturation level places the client's respiratory status as the priority. The client's need for analgesia should be delayed until respiratory status is addressed. Morphine depresses respiratory status and therefore might not be the right choice for this client.

A health care worker believes that he may have been exposed to hepatitis A. Which intervention is the highest priority to prevent him from developing the disease? A) Requesting vaccination for hepatitis A B) Using a needleless system in daily work C) Getting the three-part hepatitis B vaccine D) Requesting an injection of immunoglobulin

Answer: D. The highest priority intervention to help prevent the health care worker from developing Hepatitis A after exposure to the disease is requesting the administration of immunoglobulin, antibodies to hepatitis A.The vaccine for hepatitis A will take several weeks to stimulate the development of antibodies. Passive immunity in the form of immunoglobulin is needed. Implementing a needleless system and getting the three-part vaccine may prevent the development of hepatitis B, not hepatitis A.

A patient with gastric cancer is scheduled to undergo surgery to remove the tumor once 5 pounds (2.3 kg) of body weight has been regained. The patient is not drinking the vanilla-flavored enteral supplements that have been prescribed. Which is the highest priority nursing intervention for this patient? A) Explain to the patient the importance of drinking the enteral supplements prescribed. B) Ask the patient's family to try to persuade the patient to drink the supplements. C) Inform the patient that a nasogastric tube may be necessary if he or she fails to comply. D) Ask the patient if a change in flavor would make the supplement more palatable.

Answer: D. The highest priority nursing intervention for this patient is to ask the patient if a change in flavor would make the supplement more palatable. This action helps show that the nurse is attempting to determine why the patient is not drinking the supplements. Many patients don't like certain supplement flavors.The nurse would not assume that the patient does not understand the importance of drinking the supplements or that the patient requires persuasion to drink the supplements. The problem may be entirely different. Telling the patient that a nasogastric tube may be necessary could be construed as threatening the patient.

How does the home care nurse best modify the client's home environment to manage side effects of lactulose (Evalose)? A) Provides small frequent meals for the client B) Suggests taking daily potassium supplements C) Elevates the head of the bed in high-Fowler's position D) Requests a bedside commode for the client

Answer: D. The home care nurse best modifies the client's home environment to manage side effects of lactulose by making a bedside commode available to the client. Lactulose therapy increases the frequency of stools. A bedside commode is especially necessary if the client has difficulty reaching the toilet.Small frequent meals and elevating the head of the bed will not have any effect on the side effects of lactulose. Although lactulose produces excessive stools and could potentially result in loss of potassium, it is inappropriate for the nurse to suggest that the client take potassium supplements.

What is the mechanism of action for the chemotherapeutic drug cetuximab (Erbitux)? A) It destroys the cancer's cell wall, which will kill the cell. B) It decreases blood flow to rapidly dividing cancer cells. C) It stimulates the body's immune system and stunts cancer growth. D) It blocks factors that promote cancer cell growth.

Answer: D. The mechanism of action for the chemotherapeutic drug cetuximab is that it blocks factors that promote cancer cell growth. Cetuximab, a monoclonal antibody, may be given for advanced disease. This drug works by binding to a protein (epidermal growth factor receptor) to slow cell growth.Cetuximab does not destroy the cancer's cell walls and does not stimulate the body's immune system or stunt cancer growth in that manner. Cetuximab also does not decrease blood flow to rapidly dividing cancer cells.

An older adult client needs additional dietary protein, but refuses to drink the prescribed liquid protein supplements. Which nursing intervention is most effective in increasing the client's protein intake? A) Administering the liquid supplement with routine medications B) Giving a glucose polymer modular supplement C) Keeping a food and fluid intake diary for at least 3 days D) Providing protein modular supplements in the form of puddings

Answer: D. The most effective intervention to increase the client's protein intake is to provide protein modular supplements in the form of puddings. This would increase the client's protein intake in a format other than a liquid supplement.Administering the liquid supplement with routine medications will not be effective because the client has already refused to drink the supplements. Glucose polymer modular supplements will increase the client's calorie intake but not protein intake. A food and fluid diary will provide information about the client's typical intake pattern, but will not increase protein intake.

A client who has undergone a bariatric surgical procedure is recuperating after surgery. Which nursing intervention most effectively prevents injury to the client who is being re-positioned postoperatively? A) Administering pain medication B) Making sure not to move the client's nasogastric (NG) tube C) Monitoring skinfold areas and keeping them clean and dry D) Using a weight-rated extra-wide bed for the client

Answer: D. The most effective way to reposition a post-operative bariatric client and prevent injury is to use a special weight-related extra wide bed. This will allow adequate room for re-positioning the client comfortably without causing the bed rails to touch his or her body, causing pressure and injury.Pain medication and monitoring skinfold areas will not prevent injury to the client that might occur during repositioning. Not moving the client's NG tube will prevent disruption of the suture line, but will not prevent repositioning injuries.

The nurse asks a client with liver disease to raise the arms to shoulder level and dorsiflex the hands. A few moments later, the hand begins to flap upward and downward. How does the nurse correctly document this in the medical record? A) Positive Babinski's sign. B) Hyperreflexia. C) Kehr's sign. D) Asterixis

Answer: D. The nurse documents asterixis when the client's dorsiflexed hands begin to flap upward and downward when outstretched for a few moments. Liver flap or asterixis is related to increased serum ammonia levels.Babinski's sign is positive when, as the sole of the foot is stroked, the great toe points up and the toes fan out. Hyperreflexia refers to deep tendon reflexes that are overactive. Kehr's sign is reflected by increased abdominal pain, exaggerated by deep breathing, and referred to the right shoulder.

The nurse reviews a medication history for a patient newly diagnosed with peptic ulcer disease (PUD) who has a history of using ibuprofen (Advil) frequently for chronic knee pain. The nurse anticipates that the primary health care provider will request which medication for this patient? A) Bismuth subsalicylate (Pepto-Bismol) B) Magnesium hydroxide (Maalox) C) Metronidazole (Flagyl) D) Misoprostol (Cytotec)

Answer: D. The nurse expects that the primary health care provider will request that Misoprostol be given to the patient. Misoprostol is a prostaglandin analogue that protects against nonsteroidal anti-inflammatory drug (NSAID)-induced ulcers.Bismuth subsalicylate is an antidiarrheal drug that contains salicylates, which can cause bleeding and would be avoided in patients who have PUD. Magnesium hydroxide is an antacid that may be used to neutralize stomach secretions but is not used specifically to help prevent NSAID-induced ulcers. Metronidazole is an antimicrobial agent used to treat Helicobacter pylori infection.

An RN receives the change-of-shift report about these four clients. Which client does the nurse assess first? A) A 30-year-old admitted 2 hours ago with malnutrition associated with malabsorption syndrome B) A 45-year-old who had gastric bypass surgery and is reporting severe incisional pain C) A 50-year-old receiving total parenteral nutrition (TPN) with a blood glucose (BG) level of 300 mg/dL (16.7 mmol/L) D) A 75-year-old with dementia who is receiving nasogastric feedings and has a respiratory rate of 38 breaths/min

Answer: D. The nurse first assesses the client with dementia who has a respiratory rate of 38 breaths/min. This client needs immediate respiratory assessment and interventions. Aspiration is a major complication in clients receiving tube feedings, especially in clients with an altered level of consciousness.The client with malnutrition associated with malabsorption syndrome, the client with incisional pain from gastric bypass surgery, and the client receiving TPN with a BG of 300 mg/dL (16.7 mmol/L) all need assessments and/or interventions by the RN, but maintaining respiratory function in the client with tachypnea is the highest priority.

The nurse is reviewing orders for a patient with possible esophageal trauma after a car crash. Which request does the nurse implement first? A) Give total parenteral nutrition (TPN) through a central venous catheter. B) Administer cefazolin (Kefzol) 1 g intravenously. C) Obtain a computed tomography (CT) scan of the chest and abdomen. D) Keep the patient nothing by mouth (NPO) to prevent further leakage of esophageal contents.

Answer: D. The nurse first implements the request to keep the patient NPO, because patients with possible esophageal tears need to be NPO until diagnostic testing is completed. Leakage of anything taken orally into the sterile mediastinum could occur. In addition, esophageal rest is maintained for about 10 days after esophageal trauma to allow time for mucosal healing.TPN is prescribed to provide calories and protein for wound healing. Although TPN is important, it is not a priority for the nurse to implement first. Antibiotics may be requested to prevent possible infection, but this is not the priority. A CT of the chest and abdomen will be needed but is not the nurse's initial action.

When providing discharge teaching to a client with cirrhosis, it is essential for the nurse to emphasize avoidance of which of these? A) Vitamin K-containing products B) Potassium-sparing diuretics C) Nonabsorbable antibiotics D) Nonsteroidal anti-inflammatory drugs (NSAIDs)

Answer: D. The nurse must emphasize avoidance of NSAIDs when providing discharge teaching to a client with cirrhosis. The client with cirrhosis has an increased risk of hemorrhage. Clients who have cirrhosis must not take NSAIDs because they may predispose to bleeding.Products containing vitamin K can decrease bleeding, so it is not necessary to restrict this in the diet. Potassium-sparing diuretics are used to reduce ascites. Nonabsorbable antibiotics are used to decrease ammonia levels.

The nurse obtains assessment data on a client who had bariatric surgery today. Which finding does the nurse report to the surgeon immediately? A) Bowel sounds are not audible in all quadrants. B) Client's skin under the panniculus is excoriated. C) The client reports pain when being repositioned. D) Urine output total is 15 mL for the past 2 hours.

Answer: D. The nurse reports a urine output total of 15 mL for the past two hours. Normal urine output needs to be at least 30 mL per hour. Oliguria (scant urine output) may indicate severe postoperative complications such as anastomotic leaks or acute kidney failure.Inaudible bowel sounds would typically require intervention, but on the day of surgery, bowel sounds will probably be absent normally for some time. The other findings, excoriated skin under the panniculus and subjective reports of pain, may require nursing interventions, but do not require an immediate report to the surgeon.

A patient with malabsorption syndrome asks the nurse, "What did I do to cause this disorder to develop?" How does the nurse respond? A) "An excessive intake of alcohol is associated with it, so your substance abuse could have contributed to its development." B) "It is inherited, so it could run in your family." C) "It might be caused by a virus, so you could have gotten it almost anywhere." D) "There are a variety of things that can cause malabsorption syndrome to occur. You may have a deficiency is certain enzymes, a bacteria or changes in the lining of your intestines."

Answer: D. The nurse responds to the patient with malabsorption syndrome who asks, "What did I do to cause this?", that there are many things that can cause this disorder. Malabsorption is a syndrome associated with a variety of disorders and intestinal surgical procedures. This syndrome can be caused by inflammation, intrinsic disease, or injury to the lining of the intestine.Malabsorption syndrome is not associated with an excessive intake of alcohol. It is not inherited, although a genetic immune defect is present in the related disease, celiac sprue. It is not caused by a virus but can be caused by some bacterias.

A client has been placed on enzyme replacement for treatment of chronic pancreatitis. In teaching the client about this therapy, the nurse advises the client not to mix enzyme preparations with foods containing which element? A) Carbohydrates. B) High fat. C) High fiber. D) Protein.

Answer: D. The nurse tells the client not to mix enzyme preparations with foods containing protein because the enzymes will dissolve the food into a watery substance. Pancreatic-enzyme replacement therapy (PERT) is the standard of care to prevent malnutrition, malabsorption, and excessive weight loss (Chart 59-3). Pancrelipase is usually prescribed in capsule or tablet form and contains varying amounts of amylase, lipase, and protease.No evidence suggests that enzyme preparations should not be mixed with carbohydrates, food with high fat content, and food with high fiber content.

A client with oral carcinoma has a priority problem of risk for airway blockage related to obstruction by the tumor. At the beginning of the shift, which action will the nurse take first? A) Suction the client's oral secretions to clear the airway. B) Place the client on humidified oxygen per nasal cannula. C) Assist the client to an upright position to facilitate breathing. D) Assess the respiratory effort and quantities and types of oral secretions.

Answer: D. The nurse would first assess the client's respiratory effort and quantities and types of oral secretions. Assessment is the first step of the nursing process.Suctioning the client, placing the client on humidified oxygen, and assisting the client to an upright position are not the first steps in the nursing process. These interventions may or may not be necessary if the nurse follows the nursing process.

A client with oral cancer is depressed over the diagnosis and tells the nurse of plans to have a radical neck dissection. What is the nurse's best reaction? A) Listen to the client and then explain that it is normal to feel depressed about the diagnosis. B) Explain the grieving process and listen to what the client has to say. C) Suggest that the client talk with friends and family and seek their support. D) Listen to the client's concerns and feelings and then suggest that the client join a community group of cancer survivors.

Answer: D. The nurse's best reaction is to listen to the client and suggest a community support group of those with similar diagnoses who can offer support to the client.Telling the client that his or her feelings are normal or explaining the grieving process to the client are not helpful or therapeutic; the client needs more guidance. The nurse should not assume that the client's family and friends are an appropriate support group, because this may not be the case.

The nurse is caring for a client recently diagnosed with type 1 diabetes mellitus who has had an episode of acute pancreatitis. The client asks the nurse how he developed diabetes when the disease does not run in the family. What is the nurse's best response? A) "The diabetes could be related to your obesity." B) "Look online for general information about diabetes." C) "Do you consume alcohol on a frequent basis?" D) "Type 1 diabetes can occur when the pancreas is affected or destroyed by disease."

Answer: D. The nurse's best response is to tell the client that type 1 diabetes can occur when the pancreas is affected or destroyed by disease. This is the only response that accurately describes the relationship of the client's diabetes to pancreatic destruction.Type 2, not type 1, diabetes is usually related to obesity. Telling the client to look online for information is inappropriate because some information available online is incorrect at best.Many factors could produce acute pancreatitis other than alcohol consumption.

A male patient's sister was recently diagnosed with colorectal cancer (CRC), and his brother died of CRC 5 years ago. The patient asks the nurse whether he will inherit the disease too. How does the nurse respond? A) "Have you asked your primary health care provider what he or she thinks your chances are?" B) "It is hard to know what can predispose a person to develop a certain disease." C) "No. Just because they both had CRC doesn't mean that you will have it, too." D) "The only way to know whether you are predisposed to CRC is by genetic testing."

Answer: D. The nurse's response to the patient who asks if he will inherit CRC is "the only way to know whether you are predisposed to CRC is by genetic testing." Genetic testing is the only definitive way to determine whether the patient has a predisposition to develop CRC.Asking the patient what the primary health care provider thinks is an evasive response by the nurse and does not address the patient's concerns. A higher incidence of the disease has been noted in families who have a history of CRC. It is not, however, the responsibility of the nurse to engage in genetic counseling. This patient might not be predisposed to developing CRC.

A client has undergone bariatric surgery. Which nursing intervention is the highest priority in preventing dehydration in this client? A) Ambulating the client as quickly as possible after surgery B) Applying an abdominal binder daily when the client is out of bed C) Observing for tachycardia, nausea, diarrhea, and abdominal cramping D) Providing six small feedings daily and offering fluids frequently

Answer: D. The nursing intervention with the highest priority to prevent dehydration in a post-operative bariatric client is small daily feedings and adequate fluids. This will prevent the development of dehydration in this client.Ambulation will prevent pulmonary embolism and other circulatory problems. An abdominal binder will help support the abdomen and may prevent dehiscence of the wound. Observing for tachycardia, nausea, diarrhea, and abdominal cramping will prevent the development of postoperative dumping syndrome. All of these interventions are important, but preventing dehydration is the priority.

The Certified Wound, Ostomy, and Continence Nurse is teaching a patient with colorectal cancer how to care for a newly created colostomy. Which patient statement reflects a correct understanding of the necessary self-management skills? A) "I will have my spouse change the bag for me." B) "If I have any leakage, I'll put a towel over it." C) "I can put aspirin tablets in the pouch in order to reduce odor" D) "I will apply a non-alcoholic skin sealant around the stoma and allow it to dry prior to applying the bag."

Answer: D. The patient statement that reflects a correct understanding of necessary self-management skills to care for a newly created colostomy is, "I will apply a non-alcoholic sealant around the stoma and allow it to dry prior to putting the bag on." Teach the patient and family to apply a skin sealant (preferably without alcohol) and allow it to dry before application of the appliance (colostomy bag) to facilitate less painful removal of the tape or adhesive.It is not realistic that the spouse will always change the patient's bag and does not reflect correct understanding of self-management skills. A towel is not an acceptable or effective way to cope with leakage. Putting an aspirin in the pouch will not reduce odor and can lead to ulcers in the stoma.

An 80-year-old patient with a 2-day history of myalgia, nausea, vomiting, and diarrhea is admitted to the medical-surgical unit with a diagnosis of gastroenteritis. Which primary health care provider request does the nurse implement first? A) Administer acetaminophen (Tylenol) 650 mg rectally. B) Draw blood for a complete blood count and serum electrolytes. C) Obtain a stool specimen for culture and sensitivity. D) Start an IV solution of 5% dextrose in 0.45 normal saline at 125 mL/hr.

Answer: D. The request the nurse implements first is to start an IV solution of 5% dextrose in 0.45 normal saline at 125 mL/hr. Although the dextrose 5% in 0.45% sodium chloride is hypertonic in the IV bag, once it is infused, the glucose is rapidly metabolized and the fluid is really hypotonic. Fluid therapy is the focus of treatment for patients with gastroenteritis. Older patients are at increased risk for the complications of dehydration such as hypovolemia and acute kidney failure.Acetaminophen 650 mg should be administered rectally soon, and blood draws and stool specimen collection would also be implemented soon, but prevention and treatment of dehydration are the priorities for this patient.

A patient is receiving a tube feeding. Which action by the student nurse requires intervention by the supervising nurse? a. Weighing the patient b. Placing food coloring in the tube feeding to assess for aspiration c. Discarding any unused open cans of feeding solution after 24 hours d. Monitoring the patient for the development of diarrhea

B

A patient is requesting moderate sedation for repair of a torn meniscus and has no medical contraindications. How does the nurse respond to this patient's request? a. "Your surgeon will decide if you will receive moderate sedation or general anesthesia." b. "You can discuss your request for moderate sedation with your surgeon and anesthesiologist." c. "Most patients prefer general anesthesia. Can you tell me why you want moderate sedation?" d. "It can be frightening to see surgery done on yourself. You need to think about that."

B

A patient is scheduled for a procedure to place a stent in the biliary tract. For which procedure does the nurse provide patient teaching? a. Esophagogastroduodenoscopy (EGD) b. Endoscopic retrograde cholangiopancrea- tography (ERCP) c. Upper gastrointestinal (GI) series d. Cholangiogram

B

A patient is scheduled for tests to verify the medical diagnosis of cholecystitis. For which diagnostic test does the nurse provide patient teaching? a. Extracorporeal shock wave lithotripsy (ESWL) b. Ultrasonography of the right upper quadrant c. Endoscopic retrograde cholangiopancreatography (ERCP). D. Serum level of aspartate aminotransferase (AST)

B

A patient is scheduled to have several diagnostic tests to verify the medical diagnosis of gastroesophageal reflux disease (GERD). Which diagnostic test is the most accurate method of diagnosing this disorder? a. Esophagogastroduodenoscopy (EGD) b. Ambulatory pH monitoring examination c. Esophageal manometry d. Motility testing

B

During the perioperative period a patient receives surgery on the wrong extremity. To which agency must this occurrence be reported? a. Association of periOperative Registered Nurses (AORN) b. Centers for Medicare and Medicaid Services (CMS) c. The Joint Commission (TJC) d. American Society of Anesthesiologists (ASA)

B

The acute, life-threatening complication of malignant hyperthermia (MH) results from the use of which agents? a. Hypnotics and neuromuscular blocking agents b. Succinylcholine and inhalation agents c. Nitrous oxide and pancuronium for muscle relaxation d. Fentanyl and regional anesthesia for spinal block

B

The morning after a patient's lower leg surgery, the nurse notes that the dressing is wet from drainage. The surgeon has not yet been in to see the patient on rounds. What does the nurse do about the dressing? a. Removes the dressing and puts on a dry, sterile dressing. b. Reinforces the dressing by adding dry, sterile dressing material on top of the existing dressing. c. Applies dry, sterile dressing material directly to the wound and then retapes the original dressing. d. Does nothing to the dressing but calls the surgeon to evaluate the patient immediately.

B

The nurse detects an epigastric mass while assessing a patient with acute pancreatitis. The patient describes epigastric pain that radiates to his back. What does the nurse suspect? a. Liver cirrhosis b. Pancreatic pseudocyst c. Gallstones d. Chronic pancreatitis

B

The nurse has provided instruction for a patient prescribed sucralfate to treat a gastric ulcer. Which statement by the patient indicates that teaching has been effective? a. "This drug will stop the secretion of acid in my stomach." b. "I will take this drug on an empty stomach." c. "I will not be able to take ranitidine (Zantac) with this drug." d. "The main side effect of this drug that I can expect is diarrhea."

B

The nurse identifies which laboratory value as the usual indication of hepatic encephalopathy? a. Elevated sodium level b. Elevated ammonia level c. Increased blood urea nitrogen (BUN) d. Increased clotting time

B

The nurse is assessing a patient receiving total parenteral nutrition (TPN) at 100 mL/hour. The TPN solution has 50 mL left in the bag. The nurse looks for the next bag of TPN, but it is not on the unit. When the pharmacy is called, the nurse is told it will take at least 1 hour for the next bag of TPN solution to be delivered. What does the nurse do? a. Call the health care provider. b. Administer 10% dextrose/water (D/W) until the TPN is available. c. Prepare to treat the patient for hyperglycemia. d. Cap the TPN line until the next TPN solution is available.

B

The nurse is assessing a patient with liver trauma and finds that the patient is confused, with a blood pressure of 86/50 mm Hg, heart rate of 128/minute, and cool, clammy skin. What does the nurse suspect? a. Septic shock b. Liver hemorrhage c. Liver cancer d. GI bleeding

B

The nurse is caring for three patients who have undergone bariatric surgery. Which activity is most appropriate for the nurse to delegate to the unlicensed assistive personnel (UAP)? a. Give analgesics about 1 hour before mealtimes. b. Document the percentage of food eaten at mealtimes. c. Assess the patient's food preferences. d. Teach the patient about portion control.

B

The nurse is providing discharge instructions to a patient who is malnourished and will be taking iron supplements at home. Which statement by the patient indicates a correct understanding of the instructions? a. "These supplements may cause me to have diarrhea." b. "I will take these supplements before or with my meals." c. "I will limit my fiber intake from now on." d. "I will limit my fluid intake from now on."

B

The nurse is providing teaching for a patient with an anal fissure as a complication of Crohn's disease (CD). Which statement by the patient indicates the need for further teaching? a. "I will use warm sitz baths." B. "A diet that is low in bulk-producing agents is best for me." C. "Hydrocortisone cream may be helpful to decrease discomfort." D. "Topical anti-inflammatory agents will help if I am uncomfortable."

B

The nurse is teaching a male patient about the 2015 Dietary Guidelines for Americans. Which statement by the patient indicates a need for additional teaching? a. "I'll follow a healthy eating pattern across my life span." b. "I'll limit my alcohol intake to 5 drinks per day." c. "I'll increase my intake of fruits and vegetables." d. "I'll limit added sugar and salt in my diet."

B

The nurse is teaching a patient with cirrhosis about nutrition therapy. Which statement by the patient indicates teaching has been effective? a. "I will only use table salt with my dinner meal." b. "I will read the sodium content labels on all food and beverages." c. "I will avoid the use of vinegar." d. "I will not take vitamin supplements."

B

The nurse is teaching incisional care to a patient who is being discharged after abdominal surgery. Which priority instruction must the nurse include? a. Do not rub or touch the incision site. b. Practice proper handwashing. c. Clean the incision site two times a day with soap and water. d. Splint the incision site as often as needed for comfort.

B

The nurse providing care for a patient with a bowel obstruction notes that the patient has started passing flatus and had a small bowel movement. What has occurred with this patient? a. Blockage is complete. b. Peristalsis has returned. c. Peritonitis has occurred. d. The patient is rehydrated

B

The patient has been diagnosed with acute appendicitis. Based on this diagnosis, which intervention does the nurse perform? a. Start a bowel cleansing program. b. Prepare the patient for surgery. c. Apply a heating pad to the lower abdomen. d. Assess the patient's knowledge about dietary modifications.

B

The patient is scheduled for same-day surgery for an uncomplicated cholecystectomy. Which surgical approach will most likely be used? a. Simple b. Minimally invasive c. Open d. Radical

B

The patient tells the nurse that he is fearful of developing stomach cancer, which caused his father's death. Which foods should the nurse teach the patient to avoid? a. Foods that cause reflux b. Pickled and processed foods c. Large and heavy meals d. Heavily spiced foods

B

The patient with gastroenteritis due to infection with the norovirus asks the nurse how this illness occurred. Which statement by the patient indicates correct understanding of the nurse's teaching? a. "I got this infection from being around my grandchildren when they had respiratory illnesses." b. "It is likely that I got this illness from either contaminated water or food." c. "I may have gotten sick when I was travelling last month." d. "It's really important that everything I eat is cooked until it is well done."

B

The postoperative patient has a Penrose drain in place. Which action does the nurse take to prevent skin irritation, wound contamination, and infection? a. Keeps a sterile safety pin in place at the end of the drain b. Places absorbent pads under and around the exposed drain c. Uses minimal tape; when tape is needed, uses hypoallergenic tape d. Shortens the drain by pulling it out a short distance and trimming off the excess external portion

B

The preoperative patient tells the nurse that she is afraid that she may experience a reaction if she must receive blood during or after her surgery. What is the nurse's best response to the patient's concern? a. "The likelihood that you will need a blood transfusion for your surgery is minimal, so do not worry about this." b. "You could donate some of your own blood (autologous donation) a few weeks before your surgery." c. "With today's technology and procedures, it is very unlikely that you would have a reaction to donated blood." d. "The nursing staff follows strict proce- dures to prevent such an event from ever happening."

B

The unlicensed assistive personnel (UAP) reports to the nurse that a patient being fed experienced coughing and choking when swallowing. The patient states that "It feels like the food is stuck in my throat." What does the nurse suspect is happening with this patient? a. The patient is aspirating food. b. The patient is having dysphagia. c. The UAP is feeding the patient too quickly. d. The patient has an airway obstruction.

B

What dietary advice does the nurse give a patient to prevent constipation while maintaining good nutrition? a. "Have a cup of coffee every morning with your breakfast." b. "Drink eight glasses of water and eats lots of fiber." c. "Get at least 30 minutes of exercise every day." d. "Consume a protein source with each meal."

B

When teaching a patient about pernicious anemia, which statement does the nurse include? A. "Patients with pernicious anemia are not able to digest fats." B. "Pernicious anemia results from a deficiency of vitamin B12." C. "All patients with gastrointestinal bleeding will eventually develop pernicious anemia." D. "Oral iron supplements are an effective treatment for pernicious anemia."

B

When the nurse assesses a patient after abdominal surgery, assessment reveals diminished, hypoactive bowel sounds. What is the nurse's best action? a. Notify the surgeon immediately. b. Document the finding and continue to monitor. c. Place an NG (nasogastric) tube. d. Obtain a stat abdominal x-ray.

B

Which diagnostic test is the most accurate in verifying a diagnosis of acute pancreatitis? a. Trypsin b. Lipase c. Alkaline phosphatase d. Alanine aminotransferase

B

Which instruction would the nurse be sure to give to the unlicensed assistive personnel (UAP) who will be assisting a patient with an esophageal tumor to eat? a. Feed the patient as fast as you can because there are three more patients who will need help. b. Position the patient in a high Fowler's position before feedings. c. Always suction the patient between bites to avoid aspiration. d. Remind the patient to cough and deep- breathe between bites of food

B

Which intervention takes priority when a patient is emerging and recovering from general anesthesia? a. Check vital signs every 5 minutes. b. Be prepared to suction the patient. c. Increase the rate of IV fluid administration. d. Check the patient's pupil sizes.

B

Which is the most commonly reported symptom associated with peptic ulcer disease (PUD)? a. Rebound pain b. Indigestion c. Bleeding d. Diarrhea

B

Which lifestyle adjustment may a patient have to make to best control gastroesophageal reflux disease (GERD)? a. Sleep in the Trendelenburg position. b. Attain and maintain ideal body weight. c. Wear snug-fitting belts and waistbands. d. Engage in strenuous exercise such as weightlifting.

B

Which nursing interventions are appropriate during stage 2 of anesthesia? a. Prepare for and assist in treatment of cardiovascular and/or pulmonary arrest. Document in record. b. Shield patient from extra noise and physical stimuli. Protect the patient's extremities. Assist anesthesia personnel as needed. Stay with patient. c. Close operating room doors and control traffic in and out of room. Position patient securely with safety belts. Maintain minimal discussion in operating room. d. Assist anesthesia personnel with intubation of patient. Place the patient in position for surgery. Prep the patient's skin in area of operative site.

B

Which statement is true about the medical treatment of ulcerative colitis (UC)? a. Infliximab is approved as a first-line therapy. b. Immunomodulators are not thought to be effective; however, in combination with steroids, they may offer a synergistic effect. c. When a therapeutic level of glucocorticoids is reached, the dosage of the drug stays the same to maintain the therapeutic effect. d. The method of action for the aminosalicy- lates is interruption of the pain pathway.

B

Which therapy may be used as a cure for patients who have small localized tumors? a. Chemotherapy b. Photodynamic therapy c. Nutrition therapy d. Radiation therapy

B

Which type of oral cavity tumor appears as a red, velvety lesion on the tongue, palate, floor of the mouth, or mandibular mucosa? a. Leukoplakia b. Erythroplakia c. Basal cell carcinoma d. Kaposi's sarcoma

B

What nursing care does a patient with a nasogastric (NG) tube require? Select all that apply. a. Assess proper placement at least every 12 hours. b. Keep patient in a semi-Fowler's position. c. Confirm NG tube placement by x-ray if it is repositioned. d. Monitor contents of the NG tube. e. Irrigate the tube with 30 mL of normal saline as ordered. f. Instruct the patient that feelings of nausea are due to tube placement.

B,C,D,E

Which interventions does the nurse expect to implement when caring for a patient with diverticulitis? Select all that apply. A. Laxative and enemas as ordered b. IV fluids to prevent dehydration c. Broad-spectrum antibiotics d. Teaching the patient to refrain from lifting or straining e. Keeping the patient NPO if symptoms are severe f. Administering diuretics to prevent fluid overload

B,C,D,E

The nurse is teaching a patient with IBS about complementary and alternative therapies for the disease. Which patient statements indicate that teaching has been effective? Select all that apply. a. "Hydrotherapy may help decrease symptoms." b. "Probiotics can help decrease bacteria and decrease my IBS symptoms." c. "Peppermint oil has been used to expel gas and relax spastic intestinal muscles." d. "Regular exercise will help decrease stress and lead to regular bowel movements." e. "Ginkgo can be used for abdominal discomfort and to expel gas." f. "Meditation may help decrease stress and help with GI symptoms."

B,C,D,F

Which statements about body mass index (BMI) measurements are accurate? Select all that apply. a. It is a measure of nutritional status that varies according to frame size. b. It is based on a formula using height and weight. c. Health risks are associated with BMIs . (greater than) 25. d. It indirectly estimates total fat scores. e. There are no health risks associated with a low BMI. f. Older adults should have a BMI between 23 and 27.

B,C,D,F

The nurse on the medical-surgical unit is caring for a postoperative patient. Which assessment criteria indicate to the nurse that the patient is experiencing respiratory difficulty? Select all that apply. a. The patient's oxygen saturation drops from 98% to 94%. b. The patient is using accessory muscles to breathe. c. The patient makes a high-pitched crowing sound when breathing. d. The patient's blood pressure drops from 120/80 to 110/78 mm Hg. e. The patient's respiratory rate is 29/min. f. The patient's urine output drops from 50 mL/hr to 30 mL/hr

B,C,E

The patient with liver cancer will be discharged with a tunneled ascites drain. What statements by the patient indicate an understanding of the purpose of this device? Select all that apply. a. "I will have this drain until I am able to get the tumor removed." B. "I will not remove more than 2000 mL of fluid at a time." C. "The drain will make breathing more comfortable for me after some fluid is removed." D. "After I drain off the extra fluid, I can remove the drain." E. "This drain will be useful to remove fluid from my belly when there is too much." F. "I will flush the tunneled ascites drain twice a day with normal saline."

B,C,E

Which are examples of primary stomatitis? Select all that apply. a. Chemotherapy-induced stomatitis b. Aphthous stomatitis c. Herpes simplex stomatitis d. Bacterial stomatitis e. Traumatic ulcers f. Viral stomatitis

B,C,E

Which characteristics are consistent with bulimia nervosa? Select all that apply. a. It is self-induced starvation. b. There are episodes of binge eating. c. Binge eating is followed by purging. d. It is most often seen in older adults. e. It is most often seen in teens and young adults. F. It results from a strong fear of becoming fat

B,C,E

Which statements about Kupffer cells are true? Select all that apply. a. They are located in the epithelial cell layer lining in the GI tract. B. They are found in the liver. C. They phagocytize harmful bacteria. D. They are part of the substance that aids in the absorption of vitamin B12. E. They are part of the body's reticuloendo- thelial system. F. They aid in the metabolism of proteins.

B,C,E

A patient who is 2 days postoperative for abdominal surgery states, "I coughed and heard something pop." The nurse's immediate assessment reveals an opened incision with a portion of large intestine protruding. Which statements apply to this clinical situation? Select all that apply. a. Incision dehiscence has occurred. b. This is an emergency. c. The wound must be kept moist with normal saline-soaked sterile dressings. d. This is an urgent situation. e. Incision evisceration has occurred. f. A nasogastric (NG) tube may be ordered to decompress the stomach.

B,C,E,F

The nurse discovers that a patient has a recent change in bowel habits. What important information must the nurse gather from this patient? Select all that apply. a. Color and appearance of urine b. Occurrence of diarrhea or constipation c. Presence of abdominal distention or gas d. Intentional weight gain e. Occurrence of bloody or tarry stools f. Current pattern of bowel movements

B,C,E,F

The nurse is teaching a patient with gastroesophageal reflux disease (GERD) about lifestyle changes. Which key points would the nurse include? Select all that apply. a. Consume 4-6 large meals per day. b. Limit or eliminate alcohol and tobacco. c. Eat slowly and chew food thoroughly. d. Elevate the head of your bed 3-5 inches using wooden blocks. e. Do not wear restrictive clothing. f. Reduce or eliminate spicy foods that cause increased gastric acid.

B,C,E,F

The nurse on the surgical unit is expecting to admit the patient who has had an appendectomy with abscess. What does the nurse anticipate care for this patient will include? Select all that apply. A. Clear liquids b. Wound drains c. IV antibiotics d. Nonsteroidal anti-inflammatory drugs (NSAIDs) for pain control e. Nasogastric (NG) tube care f. Prescribed opioid pain drugs

B,C,E,F

The patient who had an ileostomy asks the nurse about how to choose the best ostomy pouching system. Which guidelines best describe an effective system? Select all that apply. a. The adhesive barrier lasts 1-2 days. b. The system protects the patient's skin. c. The pouch system contains the drainage and reduces odor. d. The ostomy pouch system is relatively inexpensive. e. The pouch remains securely attached to the skin for a dependable period of time. f. A large pouch is best because it holds more stools.

B,C,E,F

Which are common manifestations in a 28-year-old patient with dehydration secondary to gastroenteritis? Select all that apply. a. Peripheral edema b. Elevated temperature c. Dry mucous membranes d. Hypertension e. Oliguria f. Poor skin turgor

B,C,E,F

Which factors may lead to an anesthetic overdose in a patient? Select all that apply. a. Amount of anesthesia retained by fat cells b. Older age of patient c. Slowed metabolism and drug elimination d. An uncooperative patient e. Liver or kidney disease f. Drugs that alter metabolism

B,C,E,F

Which interventions prevent or minimize the risk factors in patients at risk for aspiration? Select all that apply. a. Requesting medications in pill form b. Providing liquids with a thickening agent c. Positioning the patient upright at 90 degrees d. Keeping the head of bed elevated for 5 minutes after eating e. Keeping suction equipment nearby f. Feeding the patient small bites

B,C,E,F

Which signs and symptoms suggest to the nurse that a patient is experiencing a small bowel obstruction? Select all that apply. a. Intermittent lower abdominal cramping b. Epigastric abdominal distention c. Nausea with profuse vomiting d. Metabolic acidosis e. Fluid and electrolyte imbalances f. Pain with visible peristaltic waves

B,C,E,F

Which characteristics pertain to Crohn's disease (CD)? Select all that apply. a. It begins in the rectum and proceeds in a continuous manner toward the cecum. b. Fistulas commonly develop. c. There are five to six soft, loose, nonbloody stools per day. d. There is an increased risk of colon cancer. E. Some patients experience extraintestinal manifestations such as migratory polyarthritis, ankylosing spondylitis, and erythema nodosum. F. There is a cobblestone appearance of the internal intestine.

B,C,F

Which statements about hepatitis are accurate? Select all that apply. A. Hepatitis D is the leading cause of cirrhosis and liver failure in the U.S. b. Hepatitis A is spread through the fecal-oral route. C. Hepatitis B can be transmitted through unprotected sexual intercourse. D. Hepatitis carriers have chronic obvious signs of hepatitis B. e. Hepatitis C is transmitted by casual contact or intimate household contact. F. Hepatitis D only occurs with hepatitis B to cause viral replication

B,C,F

The pancreas performs which functions? Select all that apply. a. Breaks down amino acids b. Secretes enzymes for digestion from the exocrine part of the organ c. Breaks down fatty acids and triglycerides d. Produces glucagon from the endocrine part of the organ e. Produces enzymes that digest carbohydrates, fats, and proteins f. Detoxifies potentially harmful compounds

B,D,E

Which statements about gastritis are accurate? Select all that apply. A. The diagnosis of gastritis is made solely on clinical symptoms. B. The onset of infection with Helicobacter pylori can result in acute gastritis. C. Long-term use of acetaminophen is a high-risk factor for acute gastritis. D. Atrophic gastritis is a form of chronic gastritis that is seen most in older adults. E. Type B chronic gastritis affects the glands in the antrum but may affect all the stomach. F. Type A chronic gastritis involves erosion in the fundus of the stomach.

B,D,E

Which statements best describe the preopera- tive period? Select all that apply. A. It begins when the patient makes the appointment with the surgeon to discuss the need for surgery. B. It ends at the time of transfer to the surgical suite. C. It is a time during which the patient's need for surgery is established. D. It begins when the patient is scheduled for surgery. E. It is a time during which the patient receives testing and education related to impending surgery. F. It is a time when patients and families receive discharge instructions.

B,D,E

Which nursing care actions should the nurse delegate to the unlicensed assistive personnel (UAP) for an older patient with a bowel obstruction? Select all that apply. a. Administer analgesics as needed. b. Provide mouth care every 2 hours. c. Assess abdomen for distention. d. Teach the patient about surgical procedures. e. Provide the patient with a few ice chips. f. Record intake and output.

B,E,F

Which statements about obesity are accurate? Select all that apply. a. Waist-to-hip ratio (WHR) is a strong predictor of colon cancer. b. Obesity is the second-leading cause of preventable deaths in the United States. c. Genetics have been found to have no role in obesity. d. Drug therapy is the first-line treatment for obesity. e. Waist circumference is a stronger predictor of coronary artery disease than is BMI. f. Obese adults weigh at least 20% above the upper limit of the normal range for ideal body weight.

B,E,F

A 76-year-old patient is having a bilateral cataract removal. What is the correct classification for this surgery? A. Major. B. Cosmetic. C. Elective. D. Emergent

C

A patient is scheduled for multiple tests to evaluate an oral tumor. The patient asks the nurse which of the tests is the best to deter- mine if the tumor is cancerous. What is the nurse's best response? a. "All of the tests need to be looked at together because no single test can tell if you have cancer." b. "Magnetic resonance imaging is the only diagnostic test that will need to be done." c. "Biopsy is the definitive method for diagnosing oral cancer." d. "An aqueous solution of toluidine blue 1% can be applied to the oral lesion. If the lesion is malignant it will not absorb the solution."

C

A patient with acute pancreatitis is at risk for the development of paralytic (adynamic) ileus. Which action provides the nurse with the best indication of bowel function? a. Observing contents of the nasogastric drainage b. Weighing the patient every day at the same time c. Asking the patient if he or she has passed flatus or had a stool d. Obtaining a computed tomography (CT) scan of the abdomen with contrast medium

C

A preoperative patient is scheduled for surgery at 7:30 am. At 0600, the patient's vital signs are BP 90/60, HR 110 and irregular, respirations 22/minute, and oral temperature 100.9oF (38.3 oC). The patient's oxygen saturation is 92% and he has a productive cough. What is the nurse's priority action at this time? a. Administer acetaminophen (Tylenol) with just a sip of water. b. Recheck the vital signs at 7:00 am. c. Call and notify the surgeon immediately. d. Have the patient cough and take some deep breaths

C

An older adult patient is admitted with an upper GI bleed. Which finding does the nurse expect to assess in the patient? a. Decreased pulse b. Increased hemoglobin and hematocrit c. Dizziness d. Increased blood pressure

C

During abdominal assessment, the nurse detects a loud bruit near midline. What must the nurse do? a. Measure the circumference of the patient's abdomen just under the diaphragm. b. Check the patient's record for a history of stomach ulcers. c. Avoid palpation or percussion of the abdomen. d. Ask the patient about nausea and gastric reflux.

C

The daughter of a patient with cholelithiasis has heard that there is a genetic disposition for cho- lelithiasis. The daughter asks the nurse about the risk factors. How does the nurse respond? a. "There is no evidence that first-degree rela- tives have an increased risk for this disease." b. "Cholelithiasis is seen more frequently in patients who are underweight." c. "Hormone replacement therapy has been as- sociated with increased risk for cholelithiasis." d. "Patients with diabetes mellitus are at increased risk for cholelithiasis."

C

The fluid shift that occurs in peritonitis may result in which of the following events? a. Intracellular fluid moving into the perito- neal cavity b. Significant increase in circulatory volume c. Decreased circulatory volume and hypovolemic shock d. Increased bowel motility caused by increased fluid volume

C

The nurse has instructed a patient in the recovery phase of acute pancreatitis about diet therapy. Which statement by the patient indicates that teaching has been successful? a. "I will eat the usual three meals a day that I am used to." b. "I am eating tacos for my first meal back home." c. "I will avoid eating chocolate and drinking coffee." d. "I will limit the amount of protein in my diet."

C

The nurse is assessing a patient with acute cholecystitis whose abdominal pain is severe. The patient is pale, is diaphoretic, and de- scribes extreme fatigue. Vital signs are: heart rate of 118/minute, BP 95/70, respirations 32/min, temperature 101°F (38.33°C). What is the nurse's priority action at this time? a. Instruct the unlicensed assistive personnel (UAP) to reposition the patient for comfort. b. Auscultate the patient's abdomen in all four quadrants. c. Notify the patient's health care provider. d. Administer the ordered opioid analgesic.

C

The nurse is assessing a patient with massive ascites. What related complication must the nurse monitor for with this patient? a. Bleeding due to fragile, thin-walled veins b. Hematemesis due to absence of clotting factors c. Increased ascites due to sodium and water retention d. Bruising due to low platelet count

C

The nurse is monitoring a patient after endos- copy. Vital signs are stable, and the patient tells the nurse that he is very thirsty. What is the nurse's best action? a. Administer a small amount of ice chips only. b. Give the patient small sips of water through a straw. c. Check to see if the patient's gag reflex has returned. d. Keep the patient NPO for at least 4 hours.

C

The nurse is performing an abdominal assess- ment on a patient. For which finding does the nurse alert the health care provider immediately? a. Borborygmus b. Blumberg's sign c. Bulging, pulsating mass d. Cullen's sign

C

The nurse is performing an abdominal assessment on a patient suspected of having an abdominal hernia. The nurse auscultates the abdomen and determines the absence of bowel sounds. What does the nurse suspect in this patient? a. Peritonitis b. IBS c. Obstruction and strangulation d. Low intraabdominal pressure

C

The nurse transfers a patient to the postanes- thesia care unit (PACU) with an incision and drainage of an abscess in the right groin under general anesthesia. Blood pressure is 80/47 mm Hg, heart rate 117/min in sinus tachycardia, respiratory rate 28/min, pulse oximetry reading 93% on oxygen at 3 L per nasal cannula, and temp 101.3oF (38.5°C) . The Jackson- Pratt drain has 70 mL of a cream-colored output. Normal saline is infusing at 150 mL/hr. The surgeon orders a bolus of 500 mL IV nor- mal saline over 1 hour, two sets of blood cul- tures, and culture drainage from the Jackson- Pratt drain. The patient's history includes vulvar cancer with a needle biopsy of the right groin, hypertension treated with lisinopril 5 mg PO daily, and no known drug allergies. The patient is designated as a full code. Using the Situation, Background, Assessment, Rec- ommendation (SBAR) charting format, which information should be included in assessment? a. Nurse transfers patient to the PACU with an incision and drainage of an abscess in the right groin with general anesthesia. b. Surgeon sends orders to bolus the patient with 500 mL normal saline over an hour, draw two sets of blood cultures, and send a culture of drainage from the Jackson- Pratt drain. c. Blood pressure is 80/47 mm Hg, heart rate 117/min in sinus tachycardia, respirations 28/min, pulse oximetry 93% on O2 at 3 L nasal cannula, and temp 101.3°F (38.5°C); Jackson-Pratt drain with 70 mL cream- colored output. d. Patient had a right groin abscess. History of vulvar cancer. Needle biopsy of right groin completed 1 week ago. History of hypertension treated with lisinopril (Zestril) 5 mg. No known drug allergies. Full code.

C

The nurse would teach the patient about what preventive measure for diverticular disease? a. Excluding whole-grain breads from the diet b. Avoiding fresh apples, broccoli, and lettuce c. Taking bulk agents such as psyllium hydrophilic mucilloid d. Taking routine anticholinergics to reduce bowel spasms

C

The patient comes to the emergency depart- ment (ED) with severe abdominal pain in the midepigastric area. The patient states that the pain began suddenly, is continuous, radiates to his back, and is worst when he lies flat on his back. What condition does the nurse suspect? a. Acute cholecystitis b. Pancreatic cancer c. Acute pancreatitis d. Pancreatic pseudocyst

C

The patient who had a liver transplant develops a heart rate of 134/minute, temperature of 102°F (38.8oC), jaundiced skin, and right upper quadrant pain. What does the nurse suspect? a. Liver infection b. Hypovolemic shock c. Liver transplant rejection d. Liver trauma from the transplant surgery

C

The patient who needs a liver transplant asks the nurse where the livers come from. What is the nurse's best response? a. "Most commonly they come from family members." B. "Often they are harvested from cadavers." C. "Trauma victims are where most donor livers come from." D. "It is best if the liver comes from a blood relative."

C

The patient with a gastric ulcer suddenly develops sharp constant epigastric pain that spreads over the entire abdomen. What complication has the patient most likely developed? a. Hemorrhage b. Gastric erosion c. Perforation d. Gastric cancer

C

The patient with gastroesophageal reflux disease (GERD) describes painful swallowing. Which symptom does the nurse identify? a. Dyspepsia b. Regurgitation c. Odynophagia d. Dysphagia

C

The patient with malabsorption syndrome is experiencing liquid stools. Which drug does the nurse expect will be prescribed? a. Dicyclomine hydrochloride b. Docusate c. Atropine sulfate d. Penicillin

C

The postanesthesia care unit (PACU) nurse is caring for a postoperative patient. The patient's oxygen saturation drops from 98% to 88%. What is the nurse's priority action? a. Call the anesthesia provider. b. Call the surgeon. c. Call the Rapid Response Team. d. Call the respiratory therapist.

C

The unlicensed assistive personnel (UAP) is car- ing for a patient undergoing radiation therapy to the neck. Which action by the UAP requires intervention by the supervising nurse? a. Reminding the patient to avoid sun exposure when he goes home b. Shaving the patient with an electric razor c. Using of alcohol-based aftershave lotion d. Using gentle nondeodorant soap to wash the patient

C

The unlicensed assistive personnel (UAP) is providing care to a patient with stomatitis. Which intervention by the UAP illustrates correct care for this patient? a. Using a hard-bristled toothbrush to thoroughly clean the oral cavity b. Rinsing the mouth with a commercial mouthwash. C. Using a warm saline, hydrogen peroxide, or sodium bicarbonate solution to rinse the mouth. D. Rinsing the mouth frequently with cold tap water and vinegar solution.

C

What is one of the main advantages of cholecystectomy by the natural orifice transluminal endoscopic surgery (NOTES) procedure? a. Very small visible incisions b. Jackson-Pratt drain removes excess fluid c. No visible incision lines d. Resumption of normal activities the day of surgery

C

What is the most common and serious complication after a Whipple procedure? a. Diabetes mellitus b. Wound infection c. Fistula development d. Bowel obstruction

C

What is the most common symptom of esophageal cancer reported by patients? a. Productive cough b. Reflux especially at night c. Difficulty with swallowing d. Shortness of breath

C

Which activity does the nurse tell the patient to avoid after surgery for a hernia repair? a. Ambulating b. Turning c. Coughing d. Deep-breathing

C

Which definition is appropriate for local anesthesia? a. Injection of anesthetic agent into or around a nerve or group of nerves, resulting in blocked sensation and motor impulse transmission. b. Injection of the anesthetic agent into the epidural space; the spinal cord areas are never entered. c. Injection of an anesthetic agent directly into the tissue around an incision, wound, or lesion. d. Injection of anesthetic agent that blocks multiple peripheral nerves and reduces sensation in a specific body region.

C

Which description best defines intussuscep- tion of the intestine? a. Twisting of the intestine b. Fecal constipation or impaction c. Telescoping of a segment of the intestine within itself d. Adhesions forming scar tissue

C

Which drug may the surgeon allow the patient to take prior to surgery? A. Daily vitamin. B. Stool softener. C. Antiseizure drug. D. Daily baby aspirin

C

Which is the top priority for nurses during the perioperative period? a. Patient teaching b. Patient diagnostic testing c. Patient safety d. Patient care documentation

C

Which manifestation(s) would the nurse expect to see in a patient with a vitamin D deficit? a. Swollen, bleeding gums b. Hepatomegaly c. Osteomalacia, bone pain, rickets d. Xerosis of conjunctiva

C

Which oral cavity lesion is associated with progression of human immunodeficiency virus (HIV) to acquired immunodeficiency syndrome (AIDS)? a. Erythroplakia b. Squamous cell carcinoma c. Oral hairy leukoplakia d. Basal cell carcinoma

C

Which sign/symptom is a patient who had an AP resection instructed to report to the health care provider immediately? a. Serosanguineous drainage from the wound b. Sensations of having a bowel movement c. Constant perineal odor and pain d. Occasional perineal pain and itching

C

Which type of stoma will a patient with diverticulitis most likely have postoperatively? a. Ileostomy b. Jejunostomy c. Colostomy d. Cecostomy

C

A feeling of fullness, cramping, and passage of flatus can be expected for several hours after the test, and a small amount of blood may be in the first stool after the test if a biopsy speci- men is taken or a polypectomy is performed. Vital signs should be checked every 15 minutes, the patient should be monitored for signs of perforation or hemorrhage, and excessive bleed- ing should be reported immediately. Which procedure is this follow-up care describing? a. Esophagogastroduodenoscopy (EGD) b. Enteroscopy c. Small bowel series d. Colonoscopy

D

A patient arrives at the postanesthesia care unit (PACU), and the nurse notes a respiratory rate of 10 with sternal retractions. The report from the anesthesia provider indicates that the patient received fentanyl during surgery. What is the nurse's best first action? a. Monitor the patient for effects of anes- thetic for at least 1 hour. B. Closely monitor vital signs and pulse oximetry readings until the patient is responsive. C. Administer oxygen as ordered, monitoring pulse oximetry. D. Maintain an open airway through positioning and suction if needed.

D

A patient comes to the clinic after having bar- iatric surgery and says, "After I eat, I feel really funny. My heart races, I feel nauseated, and my abdomen cramps up. I even have diarrhea." What does the nurse suspect is happening with this patient? A. Hyperglycemia b. Intestinal obstruction c. Peritonitis d. Dumpting syndrome

D

A patient comes to the emergency department (ED) reporting rapid onset of epigastric pain with nausea and vomiting. The patient says the pain is worse than any heartburn he has had, and that he has not had an appetite for the past day. What does the nurse suspect this patient has? a. Peritonitis b. H. pylori infection c. Duodenal ulcer d. Acute gastritis

D

A patient develops respiratory distress after having a left total hip replacement. The patient develops labored breathing, and a pulse oxim- etry reading is 83% on 2 L oxygen via nasal cannula. Which intervention is appropriate for the nurse to delegate to unlicensed assistive personnel (UAP)? A. Assess change in patient's respiratory status. B. Order necessary medications to be administered. C. Insert oral airway to maintain open airway. D. Check the patient's vital signs.

D

A patient experiences malignant hyperthermia (MH) immediately after induction of anesthesia. What is the nurse anesthetist's priority action? a. Administer IV dantrolene sodium (Dantrium) 2-3 mg/kg. b. Apply a cooling blanket over the torso. c. Assess arterial blood gases (ABGs) and serum chemistries. d. Stop all inhalation anesthetic agents and succinylcholine

D

A patient returns to the unit following a total proctocolectomy with a permanent ileostomy. The nurse understands that which organs were removed during this procedure? a. All of the small intestine b. Distal colon and rectum c. Colon, rectum, and anus d. Colon, rectum, and anus, with surgical closure of the anus

D

A patient with Crohn's disease (CD) has a fistula. Which assessment finding indicates possible dehydration? a. Weight gain of 2 pounds in one day b. Abdominal pain c. Foul-smelling urine d. Decreased urinary output

D

A patient with acute gastritis is receiving treatment to block and buffer gastric acid secretions to relieve pain. Which drug does the nurse identify as an antisecretory agent (proton pump inhibitor)? a. Sucralfate b. Ranitidine c. Mylanta d. Omeprazole

D

As part of the routine treatment plan for a patient with bacterial gastroenteritis, which drugs does the nurse anticipate the patient will most likely be prescribed? a. Anticholinergics b. Antiemetics c. Antiperistaltic drugs d. Antibiotics

D

Disseminated intravascular coagulation (DIC) is a complication of pancreatitis. What pathophysiology leads to this complication? a. Hypovolemia b. Peritoneal irritation and seepage of pancreatic enzymes c. Disruption of alveolar-capillary membrane d. Consumption of clotting factors and formation of microthrombi

D

Following an esophagogastroduodenoscopy, the nurse should instruct the patient in which activity? a. Avoid all strenuous activity for 2 weeks. b. Maintain high-fat, low-protein diet after procedure. c. Use incentive spirometry for 24 hours after procedure. d. Avoid driving for 12 hours postprocedure.

D

Patients with cirrhosis are susceptible to bleeding and easy bruising because there is a decrease in the production of bile in the liver, preventing the absorption of which vitamin? a. Vitamin A b. Vitamin D c. Vitamin E d. Vitamin K

D

The health care provider has assessed a patient's abdomen and found rebound tender- ness on deep palpation. What does the nurse recognize? a. Steatorrhea b. Eructation c. Biliary colic d. Blumberg's sign

D

The nurse is assessing an older adult patient with abdominal pain. Assessment findings include generalized abdominal pain with rigidity, nausea and vomiting, temperature 101.2°F (38.4°C), heart rate 122/minute, and chills. The patient is also somewhat confused and does not know where he is. What does the nurse suspect with this patient? A. Crohn's disease b. Ulcerative colitis c. Diverticulitis d. Peritonitis

D

The nurse is caring for a patient after bariatric surgery. What is the nursing priority for postoperative care for this patient? a. Nutritional intake b. Pain management c. Prevention of infection d. Airway management

D

The nurse is caring for a patient who vomited coffee-ground emesis. Where does the nurse suspect the patient is bleeding? a. Colon b. Rectum c. Small intestine d. Upper GI system

D

The nurse is providing care for a patient receiving total enteral nutrition (TEN). The patient has severe diarrhea with a foul odor. What does the nurse suspect? a. Dehydration b. Excessive residual c. Infection around the feeding tube d. Infection with Clostridium difficile

D

The nurse is teaching a patient about colostomy care. Which information does the nurse include in the teaching plan? A. The stoma will enlarge within 6-8 weeks of surgery. b. Use a moisturizing soap to cleanse the area around the stoma. c. Place the colostomy bag on the skin when the skin sealant is still damp. d. An antifungal cream or powder can be used if a fungal rash develops

D

The nurse is teaching a patient being discharged home about taking prescribed medications that include sucralfate. Which statement by the patient indicates teaching has been effective? a. "The main side effect of sucralfate is diarrhea." b. "I will take sucralfate with meals." c. "I will take sucralfate along with the antacid medication I take." d. "Sucralfate works to heal my ulcer."

D

The nurse who is assessing a patient with portal-systemic encephalopathy finds that the patient has fetor hepaticus, a positive Babinski's sign, and seizures, but no asterixis. The nurse identifies the patient as being in which stage of portal-systemic encephalopathy? a. Stage I prodromal b. Stage II impending c. Stage III stuporous d. Stage IV comatose

D

The patient comes to the Urgent Care Unit and describes symptoms of diarrhea, abdominal pain, and low-grade fever. She states she has constant abdominal pain in the right lower quadrant and has lost 25 pounds in the past month. What diagnosis does the nurse suspect? a. Ulcerative colitis b. Diverticulitis c. Peritonitis d. Crohn's disease

D

The patient has an abdominal hernia with a sac that can be returned into the abdominal cavity by gentle pressure. Which type of hernia does the nurse recognize? a. Incisional b. Irreducible c. Indirect inguinal d. Reducible

D

The patient in the OR holding area tells the nurse that his surgery is for the right foot. The patient's chart states that the surgery is for his left foot. What is the nurse's best action? a. Do nothing because the patient is confused after receiving premedications. b. Make a note about this in the nursing notes of the patient's chart. c. Call the nurse anesthetist to check whether the chart or patient is correct. d. Notify the surgeon immediately before the patient goes into the OR about this discrepancy.

D

The patient who just had a liver transplant develops oozing around two IV sites as well as some new bruising. What is the nurse's best first action? a. Apply pressure to the IV sites. b. Measure the size of the bruises. c. Document these findings as the only action. d. Notify the surgeon immediately.

D

The patient with IBS reports abdominal distention and feeling bloated to the nurse. The patient states she had a bowel movement that morning. What drug treatment does the nurse expect the health care provider to order? a. Loperamide b. Psyllium hydrophilic mucilloid c. Lubiprostone d. Rifaximin

D

The patient with cirrhosis is prescribed furosemide 60 mg orally each morning. Which patient care concept is at risk for this patient? A. Comfort. B. Cellular regulation. C. Immunity. D. Fluid and electrolyte balance.

D

The patient with malabsorption syndrome is prescribed IV fluids of one liter 0.9 saline to infuse over 8 hours for hydration. At what rate does the nurse set the IV pump in mL/hr to infuse this fluid as ordered? a. 100 mL/hr b. 110 mL/hr c. 120 mL/hr d. 125 mL/hr

D

The postoperative patient who had esophageal surgery has an NG tube in place. What intervention should the nurse delegate to the unlicensed assistive personnel (UAP)? a. Check the NG tube for proper placement. b. Teach the patient about the purpose of the NG tube. c. Assess the patient's lungs for the presence of abnormal breath sounds. d. Provide the patient with thorough mouth and nasal care every 2-4 hours.

D

The surgical team understands that time is crucial in recognizing and treating a malignant hyperthermia (MH) crisis. Once recognized, what is the treatment of choice? a. Danazol gluconate b. Phenytoin sodium c. Diazepam sulfate d. Dantrolene sodium

D

Total enteral nutrition (TEN) is contraindicated for which patient? a. Older adult receiving chemotherapy b. Patient who has had a stroke and has dysphagia c. Patient who has had extensive jaw and mouth surgery d. Patient with intestinal obstruction that has progressed to diffuse peritonitis

D

What diagnostic test best identifies a hiatal hernia? a. Esophagogastroduodenoscopy (EGD) b. 24-hour ambulatory pH monitoring c. Esophageal manometry d. Barium swallow study with fluoroscopy

D

What is the most common cause for Laennec's cirrhosis? a. Hepatitis C virus (HPC) b. Chronic biliary obstruction c. Autoimmune disorders d. Chronic alcoholism

D

What statement is true about the Candida albicans form of stomatitis? a. It is a common type of primary stomatitis. b. Candida albicans is a bacterial infection. c. This infection is uncommon in patients who are immunocompromised. d. Patients on steroid therapy often experi- ence this infection.

D

Which description illustrates the beginning of the postoperative period? a. Completion of the surgical procedure and arousal of the patient from anesthesia in the operating room (OR) b. Providing care before, during, and after surgery c. Closure of the patient's surgical incision with sutures d. Completion of the surgical procedure and transfer of the patient to the postanesthesia care unit (PACU)

D

Which diagnostic test does the nurse expect the health care provider to order to visually examine a patient's liver, gallbladder, bile ducts, and pancreas to identify the cause and location of an obstruction? a. Esophagogastroduodenoscopy (EGD) b. Upper GI radiographic series c. Percutaneous transhepatic cholangiography (PTC) d. Endoscopic retrograde cholangiopancreatography (ERCP)

D

Which duties are within the scope of practice of the circulating nurse in the operative setting? a. Manages the patient's care while the patient is in this area and initiates documentation on a perioperative nursing record b. Sets up the sterile field; assists with the draping of the patient; and hands sterile supplies, equipment, and instruments to the surgeon c. Assumes responsibility for the surgical procedure and any surgical judgments about the patient d. Coordinates, oversees, and participates in the patient's nursing care while the patient is in the operating room

D

Which intervention for postsurgical care of a patient is correct? a. When positioning the patient, use the knee gatch of the bed to bend the knees and relieve pressure. b. Gently massage the lower legs and calves to promote venous blood return to the heart. c. Encourage bedrest for 3 days after surgery to prevent complications. d. Teach the patient to splint the surgical wound for support and comfort when getting out of bed.

D

Which operative procedure includes excision of a segment of the mandible with the oral lesion, and radical neck dissection? a. Oropharyngeal resection b. Glossectomy c. Mandibulectomy d. Commando procedure

D

Which procedure uses energy waves to heat cancer cells and kill them? a. Cryotherapy b. Selective internal radiation therapy (SIRT) c. Hepatic artery embolization d. Radiofrequency ablation (RFA)

D

Which statement about the care of a patient with a Jackson-Pratt (JP) drain after a traditional cholecystectomy is true? A. The patient is maintained in the prone position. B. When the patient is allowed to eat, the JP drain is clamped continuously. C. The JP drain is irrigated every hour for the first 24 hours. D. Serosanguineous drainage stained with bile is expected for 24 hours.

D

Which statement about the use of antacids in the treatment of gastric ulcers is true? a. Antacids should be administered with meals. b. Patients should take calcium carbonate. c. The patient should take antacids on an empty stomach. d. Avoid using antacids with phenytoin.

D

Which statement describes the correct method of testing the pH of gastrointestinal (GI) contents at the bedside? a. The tube is in the stomach if the pH reading is 8.0. b. Before aspirating the GI contents, flush the tube with 10 mL of air. c. If the patient takes certain medications such as H2 blockers, the pH of the stomach is usually 2.0. d. Wait at least 1 hour after drug administra- tion before assessing the pH of GI contents.

D

Which statement is true about the drug rabeprazole for treatment of gastroesophageal reflux disease (GERD)? a. It is rapidly released into the body after it is administered. b. The tablets are large and may be crushed if the patient has difficulty swallowing them. c. It is a histamine receptor antagonist. d. If once-a-day dosing does not control symptoms, it may be taken twice a day.

D

Which type of diet has been implicated in the formation of diverticula? a. High-fat diet b. Low-protein diet c. High-cholesterol diet d. Low-fiber diet

D

Which type of drug is used to treat acute severe biliary pain? a. Acetaminophen b. Nonsteroidal antiinflammatory drugs (NSAIDs) c. Antiemetics d. Opioids

D

Which type of nonsteroidal anti-inflammatory (NSAID) drug is less likely to cause mucosal damage to the stomach? a. Ibuprofen b. Aspirin c. Acetaminophen d. Celecoxib

D

he health care team determines a patient's readiness for discharge from the postanesthe- sia care unit (PACU) by noting a postanesthe- sia recovery score of at least 10. After deter- mining that all criteria have been met, the patient is discharged to the hospital unit or home. Review the patient profiles after 1 hour in the PACU listed below. Which patient should the nurse expect to be discharged from the PACU first? a. 10-year-old female, tonsillectomy, general anesthesia. Duration of surgery 30 minutes. Immediate response to voice. Alert to place and person. Able to move all extremities. Respirations even, deep, rate of 20. Vital signs (VS) are within normal limits. IV solution is D5RL. Has voided on bedpan. Eating ice chips. Complaining of sore throat. b. 55-year-old male, repair of fractured lower left leg. General anesthesia. Duration of surgery 1 hour, 30 minutes. Drowsy, but responds to voice. Nausea and vomiting twice in PACU. No urge to void at this time. IV infusing D5NS. Pedal pulses noted in both lower extremities. VS: temperature 98.6° F (37oC); pulse 130 beats/min; respiratory rate 24/min; blood pressure 124/76 mm Hg. c. 24-year-old male, reconstruction of facial scar. General anesthesia. Duration of surgery 2 hours. Sleeping, groans to voice command. VS are within normal limits. Respirations 10 breaths/min. No urge to void. IV of D5RL infusing. Complains of pain in surgical area. d. 42-year-old female, colonoscopy. IV conscious sedation. Awake and alert. Up to bathroom to void. IV discontinued. Resting quietly in chair. VS are within normal limits.

D

_____: A stoma created from the abdominal wall into the stomach.

Gastrostomy

_____: A reversible loss of consciousness induced by inhibiting neuronal impulses in the central nervous system.

General anesthesia

_____: The "hunger hormone" that is secreted in the stomach; increases in a fasting state and decreases after a meal.

Ghrelin

_____: A form of hepatitis that is caused by the hepatitis B virus (HBV), which is shed in the body fluids of infected people and asymptomatic carriers. It is spread through unprotected sexual intercourse with an infected partner, needle sharing, blood transfusions, and other modes. Symptoms usually occur within 25 to 180 days of exposure and include nausea, fever, fatigue, joint pain, and jaundice. Most adults who get hepatitis B recover, clear the virus from their bodies, and develop immunity; however, up to 10% of patients with the disease do not develop immunity and become carriers.

Hepatitis B

_____: Hepatitis that is caused by the hepatitis C virus (HCV). Transmission is blood to blood, most commonly by needle sharing or needlestick injury with contaminated blood. The rate of sexual transmission is very low; it is not spread by casual contact and is rarely transmitted from mother to fetus. The average incubation period is 7 weeks. Most people are asymptomatic and are not diagnosed until long after the initial exposure when an abnormality is detected during a routine laboratory evaluation or when symptoms of liver impairment appear. Hepatitis C causes chronic inflammation in the liver that eventually causes the hepatocytes to scar and may progress to cirrhosis.

Hepatitis C

_____: Hepatitis E virus (HEV) was originally identified by its association with waterborne epidemics of hepatitis in the Indian subcontinent. Since then, it has occurred in epidemics in Asia, Africa, the Middle East, Mexico, and Central and South America, typically after heavy rains and flooding. In the United States, hepatitis E has been found only in travelers returning from endemic areas. The virus is transmitted via the fecal-oral route, and the clinical course resembles that of hepatitis A. HEV has an incubation period of 15 to 64 days. There is no evidence at this time of a chronic form of hepatitis E.

Hepatitis E

_____: Extreme body wasting and malnutrition that develop from an imbalance between food intake and energy use.

Cachexia

_____: Inflammation of the gallbladder usually following and created by obstruction of the cystic duct by a stone (calculus).

Calculous cholecystits

Which assessment finding indicates neurologic function deterioration in a patient with stage II cirrhosis? a. Fetor hepaticus b. Asterixis c. Palmar erythema d. Icterus

B

_____: A device, usually a pad made with firm material, that is held in place over the hernia with a belt to keep the abdominal contents from protruding into the hernial sac.

Truss

Which diagnostic test measures urinary excretion of vitamin B12 for diagnosis of pernicious anemia and other malabsorption syndromes? a. Bile acid breath test b. Schilling test c. Hydrogen breath test d. D-xylose absorption test

B

_____: The application of small "O" bands around the base of esophageal varices to cut off their blood supply

Endoscopic variceal ligation (EVL) (banding)

_____: Bowel sounds, especially loud gurgling sounds, resulting from hypermotility of the bowel.

Borborygmus (borborygmi)

The nurse is auscultating a patient's abdomen. Which technique should the nurse use? a. Place the diaphragm of the stethoscope lightly on the abdominal wall. b. Place the bell of the stethoscope lightly on the abdominal wall. c. Hold the diaphragm of the stethoscope firmly on the abdominal wall. d. Hold the bell of the stethoscope firmly on the abdominal wall.

A

_____: In pain, the type of intense pain that feels as if it is going through the body.

Boring

Which statement by the new graduate nurse about nutritional assessment requires clarifica- tion by the student's mentor? a. "A complete nutritional assessment must be done for every patient admitted to the hospital." b. "A nutritional screening must be completed on every patient admitted to the hospital." c. "An intentional weight loss of 10% within a 6-month period should be evaluated." d. "Measurement of height and weight are part of the nutritional assessment."

A

Which statement by the student nurse indi- cates the need for a better understanding of the care of patients with oral cavity problems? a. "I will use lemon-glycerin swabs to clean the patient's mouth." b. "The patient should eat a soft, bland diet." c. "Dentures should be removed." d. "Gauze may be used for oral care."

A

Following an endoscopic retrograde cholangi- opancreatography (ERCP), the nurse would include which postprocedure follow-up care interventions in the plan of care? Select all that apply. a. Observe for cholangitis, perforation, sepsis, and pancreatitis. b. Assess for nausea and vomiting. c. Tell the patient to report any abdominal pain. d. Assess for return of gag reflex. e. Provide ice chips to moisten throat. f. Insert a nasogastric tube (NG) for excessive nausea

A,B,C,D

A patient receiving intravenous fat emulsions should be monitored closely for which mani- festations of fat overload syndrome? Select all that apply. a. Increased triglycerides b. Clotting problems c. Fever d. Multisystem organ failure e. Excessive weight gain f. Infection

A,B,C,D,F

By which actions do drugs used to treat gastroesophageal reflux disease (GERD) help to decrease the pain and discomfort the patient experiences? Select all that apply. a. Inhibition of gastric acid production b. Blocking of pain sensation in the CNS c. Accelerating gastric emptying d. Decreasing lower esophageal sphincter pressure e. Protecting the gastric mucosa f. Destroying H. pylori bacteria

A,C,E

_____: A group of drugs that inhibit gastric acid secretion by blocking the effects of histamine on parietal cell receptors in the stomach.

H2-receptor antagonists

A hospitalized client reports lack of appetite to the nurse. What nursing intervention is appropriate to encourage nutrition intake? (Select all that apply.) A) Assess the client's level of pain and provide interventions as necessary. B) Remove objects from sight in the client's room such as bedpans or urinals at mealtimes. C) Provide a quiet environment during meal times. D) Provide washcloth and towel for washing hands before each meal. E) Encourage the client to eat quickly to get the entire meal consumed. F) Provide food to the client only when hunger is reported.

ANS: A, B, C, D. To encourage client nutrition intake, assess pain and provide interventions as necessary. Pain can discourage a client from wishing to eat. Toileting equipment should be removed from sight. A pleasant, quiet environment can encourage intake. A client benefits from being clean prior to touching food. The client should be given plenty of time to eat at a comfortable pace, and frequent, small amounts of intake should be offered since this can benefit the client who does not wish to eat large meals, or feels overwhelmed at mealtime.

_____: A foul odor of the mouth.

Halitosis

_____: A term that refers to a patient who goes to the hospital or physician's office for treatment and returns home on the same day.

Ambulatory

_____: An enzyme that converts starch and glycogen into simple sugars; found most commonly in saliva and pancreatic fluids.

Amylase

_____: Pain in a joint.

Arthralgia

The nurse is caring for a postoperative patient after esophageal surgery. On assessment, the nurse discovers that the patient's temperature is 101°F (38.3°C), heart rate is 120/minute, and respiratory rate is 32/minute. Lung sounds include bilateral crackles. What is the nurse's priority first action? a. Raise the head of the patient's bed. b. Call the Rapid Response Team. c. Apply oxygen at 2 L per nasal cannula. d. Administer IV normal saline at 75 mL/hour

B

_____: The donation of a patient's own blood before scheduled surgery for use, if needed, during the surgery to eliminate transfusion reactions and reduce the risk of bloodborne disease.

Autologous donation

A patient with breast cancer is scheduled for a left mastectomy. The patient has informed the surgeon and nurse that she is a Jehovah's Witness and does not want any blood transfusions. In preparation for intraoperative care of this patient, what measures does the nurse take? Select all that apply. a. Obtain two units of packed red blood cells, typed and cross-matched. b. Make provider aware of patient's request for no blood transfusions. c. Ensure autotransfusion device is in place intraoperatively. d. Ensure patient has a medical necessity order for emergency blood transfusion. e. Inform the patient of potential risks if blood transfusion is not given. f. Tell the patient that in case of emergency she may receive blood to save her life.

B,C,

A 49-year-old patient is in the postanesthesia care unit (PACU) following a frontal craniot- omy for repair of a ruptured cerebral aneu- rysm. The nurse assesses that the patient's eyes open on verbal stimulation. Pupils are equal and reactive to light, and diameter is 3 mm. The patient's hand grasps are equal and strong. The patient is able to state name correctly. The patient has had one episode of nausea and vomiting. Incision edges are dry and approxi- mated with sutures. Lung sounds are slightly diminished on auscultation, and the nurse observes the patient is using abdominal accessory muscles to breathe. Which body systems has the nurse assessed? Select all that apply. a. Cardiovascular b. Gastrointestinal c. Neurologic d. Integumentary e. Respiratory f. Renal/urinary

B,C,D,E

In which situations is regional anesthesia used instead of general anesthesia? Select all that apply. A. For an endoscopy or cardiac catheterization b. In patients who have had an adverse reaction to general anesthesia c. In some cases when pain management after surgery is enhanced by regional anesthesia d. In patients with serious medical problems e. When the patient has a preference and a choice is possible f. When the patient is having surgery of the head, neck, upper torso, and abdomen

B,C,D,E

The nurse is performing an admission assess- ment on a morbidly obese patient. Which common complications of obesity does the nurse assess for? Select all that apply. A. Type 1 diabetes mellitus b. Metabolic syndrome c. Urinary incontinence d. Gout e. Early osteoarthritis f. Decreased wound healing

B,C,D,E,F

After a patient has undergone a radical neck dissection, what is the priority nursing intervention? a. Manage the patient's pain. b. Maintain fluid and electrolyte balance. c. Maintain the patient's airway. d. Enhance the patient's ability to communicate.

C

The nurse is providing discharge instructions to the family of an older female patient who was admitted for failure to thrive. The patient has a history of osteoarthritis, stroke, and dementia. What information does the nurse include to promote nutritional intake? Select all that apply. a. Do not allow her to get out of bed while eating. b. Withhold analgesics prior to meals. c. Be sure she has her glasses and hearing aid on. d. Encourage self-feeding as much as possible. e. Keep environmental noise to a minimum. f. Ask the patient about food likes and dislikes

C,D,E,F

Which signs/symptoms are considered post- operative complications? Select all that apply. A. Sedation b. Pain at the surgical site c. Pulmonary embolism d. Hypothermia e. Wound evisceration f. Postoperative ileus

C,D,E,F

Laboratory values for a patient with acute pancreatitis may show which abnormal findings? Select all that apply. A. Increased hemoglobin. B. Decreased serum amylase. C. Increased serum lipase. D. Decreased urine nitrates. E. Increased serum amylase. F. Increased cholesterol

C,E,F

Which are true statements about caring for a patient with a truss? Select all that apply. a. A surgical binder holds the truss in place. b. The truss is removed only for bathing. c. The truss is only used after the hernia has been reduced by the health care provider. d. The truss is applied before the hernia is reduced to decrease pain. e. Powder should be applied to the skin under the truss daily. f. The patient should apply the truss on waking.

C,E,F

_____: The surgical removal of the gallbladder.

Cholecystectomy

_____: Surgical anastomosis of the common bile duct with the jejunum.

Choledochojejunostomy

_____: The presence of gallstones.

Cholelithiasis

_____: The liquid formed when food is transformed during the digestion process in the gastrointestinal tract.

Chyme

_____: A surgical procedure that may be performed in patients with an esophageal tumor when the tumor involves the stomach or the stomach is otherwise unsuitable for anastomosis. In colon interposition, a section of right or left colon is removed and brought up into the thorax to substitute for the esophagus.

Colon interposition

_____: Surgery performed for colorectal cancer in which the tumor and regional lymph nodes are removed.

Colon resection

_____: Mnemonic for combined neck dissection, mandibulectomy, and oropharyngeal resection—a procedure in which the surgeon removes a segment of the mandible with the oral lesion and performs a radical neck dissection.

Commando procedure (Combined neck dissection, mandibulectomy, oropharyngealresection)

Which intervention applies to the nursing care of an older patient with heart failure and hypovolemia related to an intestinal obstruction? a. Provide frequent mouth care with lemon-glycerin swabs. b. Offer ice chips to suck on before surgery. c. Offer a small glass of water. D. Assess for crackles in the lungs

D

Which intervention does the nurse delegate to the unlicensed assistive personnel (UAP) to promote nutritional intake for an older patient? a. Feed the patient even if he/she is able to self-feed. b. Assess which foods the patient likes to eat. c. Administer analgesic medication before meals. d. Assist the patient to sit up in a chair for meals.

D

Which is a key feature in advanced gastric cancer? A. Feeling of fullness b. Indigestion c. Epigastric, back, or retrosternal pain d. Progressive weight loss

D

Which laboratory finding does the nurse expect may occur with a diagnosis of appendicitis? A. Decreased hematocrit and hemoglobin b. Increased coagulation time c. Decreased potassium d. Increased WBC count

D

Which laboratory test is a sensitive indicator of protein deficiency in a malnourished patient? a. Cholesterol b. Total lymphocyte count (TLC) c. Serum albumin d. Prealbumin

D

_____: A form of cirrhosis in which liver function is significantly impaired with obvious manifestations of liver failure.

Decompensated cirrhosis

_____: A partial or complete separation of the outer layers of a wound, sometimes described as a "splitting open" of the wound.

Dehiscence

_____: A constellation of vasomotor symptoms that typically occur within 30 minutes after eating; believed to occur as a result of the rapid emptying of gastric contents into the small intestine, which shifts fluid into the gut and causes abdominal distention. Early manifestations include vertigo, tachycardia, syncope, sweating, pallor, and palpitations.

Dumping syndrome

_____: Recovery from anesthesia.

Emergence

_____: A tube that allows viewing and manipulation of internal body areas.

Endoscope

_____: Ulceration.

Erosion

_____: Caused by the accumulation of fats in and around the hepatic cells. It may be caused by alcohol abuse or other factors.

Fatty liver (steatosis)

_____: An abnormal opening between two adjacent organs or structures.

Fistula

_____: A sac formed from the peritoneum that contains a portion of the intestine or omentum. The hernia pushes downward at an angle into the inguinal canal. In males, indirect inguinal hernias can become large and often descend into the scrotum.

Indirect inguinal hernia

_____: Proteins produced primarily by white blood cells that assist in the inflammatory and immune responses of the body (e.g., tumor necrosis factor, interleukins).

Inflammatory cytokines

_____: A patient who is admitted to a hospital.

Inpatient

_____: The practice of injecting gas or air into a cavity before surgery to separate organs and improve visualization.

Insufflation

_____: A hemorrhoid that is located above the anal sphincter and cannot be seen on inspection of the perineal area.

Internal hemorrhoids

_____: Pressure contained within the abdominal cavity.

Intra-abdominal pressure

_____: During surgery.

Intraoperative

_____: A hernia that cannot be reduced or placed back into the abdominal cavity; requires immediate surgical evaluation.

Irreducible hernia

_____: A vegetarian diet pattern in which milk, cheese, and dairy foods are eaten but meat, fish, poultry, and eggs are avoided.

Lacto-vegetarian

_____: The inability to convert lactose (found in milk and dairy products) to glucose and galactose in the body.

Lactose intolerance

_____: A minimally invasive procedure in which the surgeon makes several small incisions near the umbilicus through which a small endoscope is placed to examine the abdomen; direct examination of the pelvic cavity through an endoscope.

Laparoscopy

_____: White, patchy lesions on a mucous membrane.

Leukoplakia

_____: A tube that is inserted through a nostril and into the small intestine.

Nasoduodenal tube (NDT)

_____: A tube that is inserted through a nostril and into the stomach for liquid feeding or for withdrawing gastric contents.

Nasogastric (NG) tube

_____: Inflammation of the pancreas that is characterized by diffusely bleeding pancreatic tissue with fibrosis and tissue death. This form affects about 20% of patients with pancreatitis.

Necrotizing hemorrhagic pancreatitis (NHP)

_____: A screening by the health care provider that includes visual inspection, measured height and weight, weight history, usual eating habits, ability to chew and swallow, and any recent changes in appetite or food intake. The screening is a way to determine which patients need more extensive nutritional assessment.

Nutrition screening

_____: An increase in body weight at least 20% above the upper limit of the normal range for ideal body weight, with an excess amount of body fat; in an adult, a body mass index greater than 30.

Obesity

_____: Jaundice caused by an impediment to the flow of bile from the liver to the duodenum; may be caused by edema of the ducts or gallstones.

Obstructive jaundice

_____: Pain on swallowing.

Odynophagia

_____: An increase in body weight for height compared with a reference standard (e.g., the Metropolitan Life height and weight tables) or 10% greater than ideal body weight. However, this weight may not reflect excess body fat, which in an adult is a body mass index of 25 to 30.

Overweight

_____: The impairment of gastric mucosal defenses so that they no longer protect the epithelium from the effects of acid and pepsin.

Peptic ulcer disease (PUD)

_____: A stoma created from the abdominal wall into the stomach for insertion of a short feeding tube.

Percutaneous endoscopic gastrostomy (PEG)

_____: Pinpoint red spots on the mucous membranes, palate, conjunctivae, or skin.

Petechiae

_____: A complication that can occur in patients with portal hypertension, with or without esophageal varices. Slow gastric mucosal bleeding may result in chronic slow blood loss, occult positive stools, and anemia.

Portal hypertensive gastropathy

_____: A clinical disorder seen in hepatic failure and cirrhosis; it is manifested by neurologic symptoms and is characterized by an altered level of consciousness, impaired thinking processes, and neuromuscular disturbances. Also called hepatic encephalopathy and hepatic coma.

Portal-systemic encephalopathy (hepatic encephalopathy) (PSE)

_____: The occurrence of the clinical manifestations of biliary tract disease following cholecystectomy; caused by residual or recurring calculi, inflammation, or stricture of the common bile duct.

Postcholecystectomy syndrome (PCS)

_____: An unpleasant itching sensation.

Pruritis

_____: Amount of feeding that remains in the stomach after enteral nutrition.

Residuals

_____: A hormone produced by fat cells that creates resistance to insulin activity.

Resistin

_____: Going against the normal direction of flow.

Retrograde

_____: Having a bloody appearance.

Sanguineous

_____: Systemic disease associated with sepsis; the presence of pathogens in the blood.

Septicemia

_____: Having a serum-like appearance, or yellow color.

Serous

_____: An agent that stimulates the flow of saliva.

Sialagogue

_____: Total nutritional intake orally or intravenously with commercially prepared products (either total enteral nutrition or total parenteral nutrition).

Specialized nutrition support (SNS)

_____: The sheath of muscle fibers surrounding the papillary opening of the duodenum.

Sphincter of Oddi

_____: A procedure for opening a sphincter.

Sphincterectomy

_____: Surgical removal of the spleen.

Splenectomy

_____: Enlargement of the spleen.

Splenomegaly

_____: Bacterial infection of the abdominal peritoneum caused by ascites; often seen in patients with cirrhosis of the liver.

Spontaneous bacterial peritonitis (SBP)

_____: Straining, especially painful straining to defecate.

Tenesmus

_____: Provision of intensive nutritional support for an extended time; delivered to the patient through access to central veins, usually the subclavian or internal jugular veins.

Total parenteral nutrition (TPN)

_____: A type of gastritis that affects the glands of the antrum and may involve the entire stomach.

Type B gastritis

_____: Protrusion of the intestine at the umbilicus; can be congenital or acquired. Congenital umbilical hernias appear in infancy. Acquired umbilical hernias directly result from increased intra-abdominal pressure and are most commonly seen in obese people.

Umbilical hernia

_____: A noninvasive alternative to the colonoscopy procedure. A scanner is used to view the colon.

Virtual colonoscopy

_____: Proteins such as albumin that circulate in the bloodstream and may be produced by the liver.

Visceral proteins

_____: Reflex salivary hypersecretion that occurs in response to reflux in the patient with gastroesophageal reflux disease.

Water brash


Related study sets

Chapter 23 Abdomen Book: Fetal Hips

View Set

Chapter 19: Processes and Stages of Labor & Birth

View Set

Rhetorical Theory: Quiz 2, 3, 4, 5 &6

View Set

CH 22 Nursing Management of the Postpartum Woman at Risk; CH 23 Nursing Care of the Newborn With Special Needs; CH 24 Nursing Care of the Newborn at Risk PrepU (Developmental)

View Set

Chapter 20: Trauma and Surgical Management

View Set

UNIT #2: Regulation of Investment Adviser Representative

View Set